Инфоурок Математика Другие методич. материалыПЗ по специальности Технология машиностроения,1 курс(2016-2017 уч.г.)

ПЗ по специальности Технология машиностроения,1 курс(2016-2017 уч.г.)

Скачать материал

Выберите документ из архива для просмотра:

Выбранный для просмотра документ МЕТОД.УК. ПР-ТЕХМАШ,1 КУРС..docx

ГБПОУ СПО (ССУЗ) «Чебаркульский профессиональный техникум»












Методические указания

к выполнению практических работ обучающихся

по дисциплине Математика: алгебра и начала математического анализа; геометрия.

для специальности 15.02.08. Технология машиностроения.

























2016 г.



Рассмотрено и одобрено на заседании ПЦК

Протокол № ___ от___________20__г.

Председатель ПЦК ________________________


Составлены в соответствии с программой дисциплины Математика: алгебра и начала математического анализа; геометрия.

для специальности

15.02.08. Технология машиностроения.








Составитель: Зайцева С.Е., преподаватель













































Пояснительная записка

Цель методических указаний: оказание помощи обучающимся в выполнении практических работ по дисциплине Математика.

Настоящие методические указания содержат работы, которые позволят обучающимся самостоятельно овладеть профессиональными знаниями и умениями, опытом творческой деятельности при решении проблем учебного и профессионального уровня и направлены на формирование следующих компетенций:

ОК 1. Понимать сущность и социальную значимость своей будущей

профессии, проявлять к ней устойчивый интерес.

ОК 2. Организовывать собственную деятельность, выбирать типовые

методы и способы выполнения профессиональных задач, оценивать их

эффективность и качество.

ОК 3. Принимать решения в стандартных и нестандартных

ситуациях и нести за них ответственность.

ОК 4. Осуществлять поиск и использование информации,

необходимой для эффективного выполнения профессиональных задач,

профессионального и личностного развития.

ОК 5. Использовать информационно-коммуникационные технологии

в профессиональной деятельности.

ОК 6. Работать в коллективе и команде, эффективно общаться

с коллегами, руководством, потребителями.

ОК 7. Брать на себя ответственность за работу членов команды

(подчиненных), результат выполнения заданий.

ОК 8. Самостоятельно определять задачи профессионального и

личностного развития, заниматься самообразованием, осознанно

планировать повышение квалификации.

ОК 9. Ориентироваться в условиях частой смены технологий

в профессиональной деятельности.

ОК 10. Исполнять воинскую обязанность, в том числе с применением

полученных профессиональных знаний (для юношей).

В результате выполнения практических работ по дисциплине Математика обучающиеся должны:

уметь:

    • Выполнять арифметические действия, сочетая устные и письменные приемы, применение вычислительных устройств;

    • Решать уравнения и неравенства (линейные, квадратные);

    • Вычислять значения числовых и буквенных выражений, осуществляя необходимые подстановки и преобразования;

    • Находить значения корня натуральной степени, степени с рациональным показателем, логарифма, используя при необходимости вычислительные устройства;

    • Проводить по формулам и правилам преобразования буквенных выражений, включающих степени, радикалы, логарифмы, тригонометрические функции;

    • Строить графики изученных функций;

    • Решать показательные и логарифмические уравнения и неравенства; простейшие иррациональные и тригонометрические уравнения;

    • Вычислять производные и первообразные элементарных функций, используя справочные материалы;

    • Исследовать в простейших случаях функции на монотонность, находить наибольшие и наименьшие значения функций; строить графики многочленов с использованием аппарата математического анализа;

    • Распознавать на чертежах и моделях пространственные формы; соотносить объекты с их описаниями, изображениями;

    • Описывать взаимное расположение прямых и плоскостей в пространстве ;

    • Изображать основные многогранники и круглые тела;

    • Решать планиметрические и простейшие стереометрические задачи на нахождение геометрических величин (длин, углов, площадей, объемов);

    • Использовать при решении стереометрических задач планиметрические факты и методы;

    • Решать простейшие комбинаторные задачи методом перебора, а также с использованием формул;

    • Вычислять в простейших случаях вероятности событий на основе подсчета числа исходов;

    • Использовать приобретенные знания и умения для анализа числовых данных, представленных в виде таблиц, диаграмм;

знать:

    • Выполнение арифметических действий, сочетая устные и письменные приемы, применение вычислительных устройств;

    • Решение уравнений и неравенств (линейных, квадратных);

    • Вычисление значений числовых и буквенных выражений, осуществляя необходимые подстановки и преобразования;

    • Нахождение значений корня натуральной степени, степени с рациональным показателем, логарифма, используя при необходимости вычислительные устройства;

    • Преобразования буквенных выражений, включающих степени, радикалы, логарифмы, тригонометрические функции;

    • Построение графиков изученных функций;

    • Решение показательных и логарифмических уравнений и неравенств; простейших иррациональных и тригонометрических уравнений;

    • Вычисление производных и первообразных элементарных функций, используя справочные материалы;

    • Исследование в простейших случаях функций на монотонность, нахождение наибольших и наименьших значений функций; построение графиков многочленов с использованием аппарата математического анализа;

    • Вычисление в простейших случаях площадей и объемов с использованием определенного интеграла;

    • Распознавание на чертежах и моделях пространственных форм; соотношение объектов с их описанием, изображением;

    • Описание взаимного расположения прямых и плоскостей в пространстве ;

    • Изображение основных многогранников и круглых тел ;

    • Решение планиметрических и простейших стереометрических задач на нахождение геометрических величин (длин, углов, площадей, объемов);

    • Использование при решении стереометрических задач планиметрических фактов и методов;

  • Решение простейших комбинаторных задач методом перебора, а также с использованием формул;

  • Вычисление в простейших случаях вероятности событий на основе подсчета числа исходов;

  • Использование приобретенных знаний и умений для анализа числовых данных, представленных в виде таблиц, диаграмм;

  • Значение математической науки для решения задач, возникающих в теории и практике; широту и в то же время ограниченность применения математических методов к анализу и исследованию процессов и явлений в природе и обществе;

  • Значение практики и вопросов, возникающих в самой математике для формирования и развития математической науки; историю развития понятия числа, создания математического анализа, возникновения и развития геометрии;

  • Универсальный характер законов логики математических рассуждений, их применимость во всех областях человеческой деятельности;

Критерии оценки:

Оценка «5» выставляется если : выполнено задание из части 1) и 2),выборочно из части 3).

Оценка «4» выставляется , если : выполнено задание из части 2),выборочно из части 3).

Оценка «3» выставляется, если : выполнено задание из части 1),выборочно из части 2).

Указания к выполнению работы:

В работах есть следующие части:

1) опорный конспект (обучающиеся должны прочитать его и кратко законспектировать );

2) задачи-расчет (обучающиеся должны переписать и заполнить пропуски);

3) -4) задачи для самостоятельного решения.

В некоторых работах есть дополнительное задание (тест, кроссворд), обучающиеся могут выбирать задания.


Требования к оформлению самостоятельной работы:

Расчетные задания должны быть выполнены в рабочей тетради №3.

Учебно-методическое и информационное обеспечение: приложение №1.

































Списки практических работ по специальности:

15.02.08. Технология машиностроения.

Списки ПР

Сроки выполнения

ПЗ № 1. Признаки взаимного расположения прямых. Угол между прямыми. Взаимное расположение прямых и плоскостей.

1 семестр, 3 неделя

ПЗ № 2. Перпендикуляр и наклонная к плоскости. Угол между прямой и плоскостью. Теоремы о взаимном расположении прямой и плоскости. Теорема о трех перпендикулярах.

1 семестр, 4 неделя

ПЗ № 3. Признаки и свойства параллельных и перпендикулярных плоскостей. Расстояние от точки до плоскости, от прямой до плоскости, расстояние между плоскостями, между скрещивающимися прямыми, между произвольными фигурами в пространстве.

1 семестр, 4 неделя

ПЗ № 4. Параллельное проектирование и его свойства. Теорема о площади ортогональной проекции многоугольника. Взаимное расположение пространственных фигур.

1 семестр, 5 неделя

ПЗ № 5. Различные виды многогранников. Их изображения. Сечения, развертки многогранников. Площадь поверхности.

1 семестр, 7 неделя

ПЗ № 6. Виды симметрий в пространстве. Симметрия тел вращения и многогранников.

1 семестр, 9 неделя

ПЗ № 7. Вычисление площадей и объемов.

1 семестр, 10 неделя

ПЗ № 8. Векторы. Действия с векторами. Декартова система координат в пространстве. Уравнение окружности, сферы, плоскости.

1 семестр, 12 неделя

ПЗ № 9. Расстояние между точками. Действия с векторами, заданными координатами.

1 семестр, 12 неделя

ПЗ № 10. Скалярное произведение векторов. Векторное уравнение прямой и плоскости. Использование векторов при доказательстве теорем стереометрии.

1 семестр, 13 неделя

ПЗ № 11. История развития комбинаторики, теории вероятностей и статистики и их роль в различных сферах человеческой жизнедеятельности. Правила комбинаторики. Решение комбинаторных задач.

1 семестр, 14 неделя

ПЗ № 12. Размещения, сочетания и перестановки. Бином Ньютона и треугольник Паскаля. Прикладные задачи.

1 семестр, 15 неделя

ПЗ № 13. Классическое определение вероятности, свойства вероятностей, теорема о сумме вероятностей.

1 семестр, 15 неделя

ПЗ № 14. Вычисление вероятностей. Прикладные задачи. Представление числовых данных. Прикладные задачи.

1 семестр, 16 неделя

ПЗ № 15. Арифметические действия над числами, нахождение приближенных значений величин и погрешностей вычислений (абсолютной и относительной), сравнение числовых выражений.

1 семестр, 18 неделя

ПЗ № 16. Вычисление и сравнение корней. Выполнение расчетов с радикалами.

1 семестр, 19 неделя

ПЗ № 17. Нахождение значений степеней с рациональными показателями. Сравнение степеней.

1 семестр, 19 неделя


ПЗ № 18. Преобразования выражений, содержащих степени. Решение прикладных задач.

1 семестр, 20 неделя

ПЗ № 19. Нахождение значений логарифма по произвольному основанию. Переход от одного основания к другому.

1 семестр, 21 неделя

ПЗ № 20. Вычисление и сравнение логарифмов. Логарифмирование и потенцирование выражений. Приближенные вычисления и решения прикладных задач.

1 семестр, 21 неделя

ПЗ № 21. Радианный метод измерения углов вращения и связь с градусной мерой.

1 семестр, 23 неделя

ПЗ № 22. Основные тригонометрические тождества, формулы сложения, удвоения, преобразование суммы тригонометрических функций в произведение, преобразование произведения тригонометрических функций в сумму.

1 семестр, 24 неделя

ПЗ № 23. Простейшие тригонометрические уравнения и неравенства.

1 семестр, 25 неделя

ПЗ № 24. Обратные тригонометрические функции: арксинус, арккосинус, арктангенс.

1 семестр, 25 неделя

ПЗ № 25. Примеры зависимостей между переменными в реальных процессах из смежных дисциплин. Определение функций.

1 семестр, 26 неделя

ПЗ № 26. Построение и чтение графиков функций. Исследование функции.

1 семестр, 27 неделя

ПЗ № 27. Свойства линейной, квадратичной, кусочно-линейной и дробно-линейной функций.

1 семестр, 28 неделя

ПЗ № 28. Непрерывные и периодические функции. Свойства и графики синуса, косинуса, тангенса и котангенса.

1 семестр, 29 неделя

ПЗ № 29. Обратные функции и их графики. Обратные тригонометрические функции.

2 семестр, 29 неделя

ПЗ № 30. Преобразования графика функции. Гармонические колебания. Прикладные задачи.

2 семестр, 30 неделя

ПЗ № 31. Показательные, логарифмические, тригонометрические уравнения и неравенства.

2 семестр, 30 неделя

ПЗ № 32. Решение иррациональных уравнений.

2 семестр, 31 неделя

ПЗ № 33. Решение показательных уравнений.

2 семестр, 32 неделя

ПЗ № 34. Решение логарифмических уравнений.

2 семестр, 32 неделя

ПЗ № 35. Корни уравнений. Равносильность уравнений. Преобразование уравнений. Основные приемы решения уравнений.

2 семестр, 33 неделя

ПЗ № 36. Решение систем уравнений. Использование свойств и графиков функций для решения уравнений и неравенств.

2 семестр, 34 неделя

ПЗ № 37. Числовая последовательность, способы ее задания, вычисления членов последовательности. Предел последовательности. Бесконечно убывающая геометрическая прогрессия.

2 семестр, 35 неделя

ПЗ № 38. Производная: механический и геометрический смысл производной. Уравнение касательной в общем виде.

2 семестр, 36 неделя

ПЗ № 39. Правила и формулы дифференцирования, таблица производных элементарных функций.

2 семестр, 36 неделя

ПЗ № 40. Исследование функции с помощью производной.

2 семестр, 37 неделя

ПЗ № 41. Нахождение наибольшего, наименьшего значения и экстремальных значений функции.

2 семестр, 37 неделя

ПЗ № 42. Интеграл и первообразная. Теорема Ньютона—Лейбница.

2 семестр, 39 неделя

ПЗ № 43. Применение интеграла к вычислению физических величин и площадей.

2 семестр, 39 неделя

Всего практ.работ : 43

86 ч.









ПРИЛОЖЕНИЕ №1


Основные учебники :


  1. Алгебра и начала анализа 10-11 кл. Ш.А.Алимов и др. М., «Просвещение», 2009 г.

  2. Геометрия 10-11 кл. Л.С. Атанасян. М., «Просвещение»,2011 г.


Дополнительные учебники :



  1. Алгебра и начала математического анализа 10-11 кл. Колмогоров А.Н. и др. М., «Просвещение»,2009г.

  2. Алгебра и начала математического анализа 10-11 кл. Башмаков М.И. М., «Дрофа»,2009г.

  3. Геометрия 10-11 кл. Потоскуев Е.В., Звавич Л.И. М., «Просвещение», 2009г.




Интернет-ссылки для ВСР.

Алгебра:

  1. http://math-prosto.ru/?page=pages/library-math/alimov-10-11.php

  2. http://nashol.com/2012102467590/algebra-i-nachala-matematicheskogo-analiza-10-11-klass-alimov-sh-a-kolyagin-u-m-2012.html

  3. http://nsportal.ru/shkola/algebra/library/algebra-i-nachala-analiza-10-11-klass-po-uchebniku-sha-alimova-i-dr

  4. http://nashol.com/2014021575799/algebra-i-nachalo-matematicheskogo-analiza-10-klass-muravin-g-k-2013.html

  5. http://elkniga.ucoz.ru/load/multimedijnye_posobija/matematika/multimedijnoe_posobie_po_matematike_uroki_algebry_kirilla_i_mefodija_10_11_klass/14-1-0-15

Геометрия:

  1. http://nashol.com/knigi-po-matematike/#po_godam_2012

  2. http://nashol.com/2011102361137/geometriya-uchebnik-10-11-klass-atanasyan-l-s-butuzov-v-f-kadomcev-s-b-2009.html

  3. http://4book.org/uchebniki-rossiya/10-klass/62-geometriya-uchebnik-dlya-10-11-klassov-atanasyan-l-s-i-dr

  4. http://neovit.net/edu/math1.htm

  5. http://elkniga.ucoz.ru/publ/uchebniki/10_klass/geometrija_atanasjan_l_s_uchebnik_dlja_10_11_klassa_obshheobrazovatelnykh_uchrezhdenij/98-1-0-311





И любые другие аналогичные из интернета по разделам «Алгебра и начала анализа», «Геометрия».







Просмотрено: 0%
Просмотрено: 0%
Скачать материал
Скачать материал "ПЗ по специальности Технология машиностроения,1 курс(2016-2017 уч.г.)"

Методические разработки к Вашему уроку:

Получите новую специальность за 3 месяца

Дефектоскопист

Получите профессию

Копирайтер

за 6 месяцев

Пройти курс

Рабочие листы
к вашим урокам

Скачать

Выбранный для просмотра документ ПЗ№1.docx

ПЗ № 1. Признаки взаимного расположения прямых. Угол между прямыми. Взаимное расположение прямых и плоскостей.hello_html_m4f96c18b.jpg

Задание:

1)А)Перепишите и заполните пропуски:

  1. Найти точки пересечения прямых и плоскостей.

а) АВ ∩ АD = А , б) АА1 ∩ А1В1 = …, в) СС1∩ВС=…,

г) (АА1 D1D) ∩ (АА1 В1В) = АА1 ,

д) ( АВСD ) ∩ ( СDD1С1 ) = …,

е) ( АВСD ) ∩ ( АВD1С1 ) = …,

  1. Определить взаимное расположение прямых: вставить знак

( ||, ∩, · )

а) АВ и А1В1, б) А1D и АВ, в) DС1 и АВ ,

г) ВС и В1С1 , д) В1В и ВС, е) АD и ВС .

  1. а) Точка М не лежит в плоскости ромба АВСD. Докажите, что прямая АВ параллельна плоскости DМС. hello_html_m63271b33.jpg

Доказательство: АВ || СD , СDDМС,=> АВ|| DМС

б) АВСD- трапеция, МК- средняя линия трапеции. Докажите, что прямая МК параллельна плоскости α, в которой лежит основание трапеции АD и не лежит ВС.

Доказательство:

МК || АВ, АВ  α, => МК ... α hello_html_4d3b9131.jpg

в) В параллелограмме АОВК сторона ОВ параллельна прямой m , а АО и m –скрещивающиеся прямые. Найти угол между скрещивающимися прямыми , если один из углов параллелограмма равен а) 22°, б) 34°,в) 110°,г) 140°.

Решение: а) φ = 22 °,б) φ = …, в) φ = 180° – 110° =70 °,

г ) φ = 180 °– 140°= …

  1. Точка С лежит на отрезке АВ. Через точку А проведена плоскость α, а через точки В и С- параллельные прямые , пересекающие эту плоскость соответственно в точках В1 и С1. Найти длину отрезка СС1, если а) ВВ1 = 10 см ; б) АС : СВ = 3 : 2 и ВВ1 = 10 см. hello_html_bd13706.jpg

Решение:

а) СС1 = ВВ1 : 2 = 10 : 2 = …

б) СС1 = ВВ1 · 3: (3 + 2) = 10 · 3: (3 + 2) =…

5) Дано: ABCD - параллелограмм; а || ВС; а  (ABCD) (рис. 1).hello_html_m456621d.jpg

Доказать: а и CD - скрещивающиеся.

Найти: угол между а и CD, если BCD = 50°.

Решение:

I. 1) Так как а || ВС, то проведем через них плоскость α.

2) D  α, так как иначе DC  α, то есть α совпала бы с плоскостью ABCD и а  (ABCD), что противоречит условию.

3) Тогда DC  α в точке С  а;

4) Вывод: по теореме а и CD - скрещивающиеся.

II. Проведем через точку С прямую, параллельную прямой а. Это будет прямая СВ. Значит, угол между а к СВ равен углу между прямыми СВ и CD, то есть BCD = ...°. (Ответ: 50°.)

В) Пример 1. Дано: а || α, b α, а || b, с - секущая, 2 1= 30°,

Найти:1, 2.hello_html_69440865.png

Решение: Углы 1 и 2 внутренние односторонние,

их сумма равна 180°, т. е.  l + 2 = 180°. (1)

Обозначим градусную меру угла 1 через х.

По условию 2 х = 30°, или 2 = 30° + x.

Подставим в равенство (1) значения углов 1 и 2, получим 
х + 30° + х = 180°, 2х = 150°,

Решая это уравнение, получим х = …°, т. е. 

1 = 75°, a 2 = 180° 75° = …°.

Ответ: 1 = 75°, a 2 = 105°.

Пример 2. Две параллельные прямые, одна из которых лежит в плоскости α, пересечены третьей. Известно, что сумма двух внутренних накрест лежащих углов равна 150°. Чему равны эти углы и остальные шесть?hello_html_2ed53250.png

Дано: а || α, bα, а || b, с - секущая, l + 2 = 150°,

Найти:1, 2,3,4,5,6,7,8.

Решение: Углы 1 и 2 внутренние накрест лежащие, следовательно, они равны.

Сумма этих углов по условию задачи равна 150°, тогда 1 = 2 = 150° : 2 = ...

Найдем остальные углы . 1 = 3 = 75° и 2 = 7 = 75° (вертикальные). Углы 4 и 5, 6 и 8 равны как вертикальные, a 5 = 6 как внутренние накрест лежащие. Все перечисленные углы 4, 5, 6 и 8 равны между собой , так как 4 + 3 = 180°,

то 4 = 180° 3 = 180 ° – 75 ° = ...

Получили четыре угла по 75°, четыре угла по 105°.

Ответ:1 = 2 = 3 = 7 = 75°, 4 = 5 = 6 = 8 = 105°.hello_html_m40fa69e3.jpg

Пример 3. Дано: а || α, bα, а || b, с - секущая, 1 = 150°,

Найти: 2,3,4,5,6,7,8.

Решение: 3 = 1 = 150°(верт.), 3 = 5 = 150°(н.леж.),

5 = 7 = 150°(верт.), 1 + 2 = 180°(смежные),

2 = 180° 1 = 180° 150° = …°,

2 = 4 = 30°(верт.), 4 = 6 = 30°(н.леж.), 6 = 8 = 30°(верт.).

Ответ: 3 = 5 = 7 = 150°, 2 = 4 = 6 = 8 = 30°.

Пример 4. Дано: а || α, bα, а || b, с - секущая, 4 = 70°,

Найти: 1,2,3, ,5,6,7,8.

Решение: 2 = 4 = …°(верт.), 4 = 6 = 70°(н.леж.),

6 = 8 = …°(верт.), 4 + 3 = 180°(смежные),

3 = 180°4 = 180° 70° = …°,

3 = 1 = 110°(верт.), 3 = 5 = …°(н.леж.), 5 = 7 = …°(верт.).

Ответ: 1 = 3 = 5 = 7 = 110°, 2 = 6 = 8 = 70°.

Пример 5.Плоскости α и β пересекаются по прямой АВ. Прямая а || α, а || β. Докажите, что а || АВ.

Док-во: Через точку А проведем АМ || α. Так как а || α, а || β, то АМ α, АМ β. Таким образом, αβ =АМ, т.е. она совпадает с АВ. Следовательно, АВ || а.

Пример 6. Дано: АВСD- прямоугольник, М ( АВСD), (СВМ) = α. Докажите, что АD || α.

Док-во: АD || ВС, ВС α. Следовательно, АD || α.

Пример 7. Дано: AC || BD, AC ∩ α = A; BD ∩ α = B. AC = 8 cm, BD = 6 см, AB = 4 см (рис.). Доказать: CD ∩ α = E.Найти: BE. 

Решение: 1) Проведем плоскость (ACDB), если CD || АВ, то ACDB - параллелограмм, то есть АС = BD, но это противоречит условию, значит, CDAB = Е.hello_html_4fbb0191.jpg

Рассмотрим ΔАСЕ и ΔBDE. CAE = DBE, АСЕ = BDE - как соответственные при параллельных прямых, значит,

ΔEDB ~ ΔЕСА (по 3 углам) следовательно,

= , то есть ,  

BE = 12 (см).

Ответ: BE = 12 см.

Пример 8. Через основание AD трапеции ABCD проведена плоскость α. ВС α. Докажите, что прямая, проходящая через середины сторон АВ и CD, параллельна плоскости α. рис.

hello_html_m20ed3981.jpg

Дано: ABCD - трапеция; AD α, СВ α; АК = КВ, CN = ND (рис.).

Доказать: KN || α.

Доказательство:1. KN - средняя линия трапеции, значит KN || AD.

2. KN || AD , AD α, KN || α (по теореме о параллельности прямой и плоскости).

С)1)Построить таблицу:

hello_html_m7f53789a.png

2)Приведите примеры взаимного расположения прямых в пространстве из окружающего мира

Параллельные

Пересекающиеся

Скрещивающиеся

Лампы дневного света

циркуль

Башенный кран

Батареи отопления

Перекрёсток дорог

Вертолёт, самолёт

Ножки стола

стрелки часов

антенна

.

.

.

.

.

.


3) Тест «Прямые в пространстве. Параллельность прямых, прямой и плоскости»

1.Прямая а, параллельная прямой b, пересекает плоскость α. Прямая с параллельна прямой b, тогда:

а) прямые а и с пересекаются; б) прямая с лежит в плоскости α;

в) прямые а ис скрещиваются; г) прямые а и с параллельны.

2. Каким может быть взаимное расположение прямых а и b, если через прямую а можно провести плоскость, параллельную прямой b?

а) скрещиваются или пересекаются;

б) скрещиваются или параллельны;                      

в) только скрещиваются;

г) только параллельны.

3. Прямые а  и  в лежат в параллельных плоскостях, следовательно эти прямые                             а)скрещиваются или пересекаются; б) скрещиваются или параллельны;                    

в) только скрещиваются; г) только параллельны.

4. Каким может быть взаимное расположение двух прямых, если обе они параллельны одной плоскости?

а) только параллельны; б) все случаи взаимного расположения;

в) только скрещиваются; г) только пересекаются.

5. Прямая а параллельна плоскости α. Какое из следующих утверждений верно?

а) Прямая а параллельна любой прямой, лежащей в плоскости α;

б) прямая а не пересекает ни одну прямую, лежащую в плоскости α; 

в) прямая а скрещивается со всеми прямыми плоскости α;

г) прямая а имеет общую точку с плоскостью  .

4) Задание - отгадайте зашифрованное слово - две прямые, находящиеся в разных плоскостях.

Вопросы:

1. Раздел геометрии, изучающий свойства фигур в пространстве (12 букв).

2.Утверждение, не требующее доказательства.

3. Простейшая фигура планиметрии и стереометрии (6 букв).

4. Раздел геометрии, изучающий свойства фигур на плоскости (11 букв).

5. Защитное приспособление воина в виде круга, овала, прямоугольника.

6. Теорема, задающая свойства предметов.

7. Направленный отрезок (6 букв).

8. Планиметрия - плоскость, стереометрия -…

9. Женская одежда в форме трапеции (4 буквы).

10. Точка, принадлежащая обеим прямым.

11. Какую форму имеют гробницы фараонов в Египте? (8 букв)

12. Какую форму имеет кирпич? (14 букв)

13. Одна из основных фигур стереометрии.

14. Она может быть прямой, кривой, ломаной.


hello_html_m30062b56.pnghello_html_m4f96c18b.jpg

2) Решите задачи(по примерам):

А)

  1. АВСDА1В1С1D1- параллелепипед.

Найти точки пересечения прямых и плоскостей.

а) DС ∩ D1D = ? , б) С1В1 ∩ А1В1 = ?,

в) (СС1 В1В) ∩ (А1В1С1D1) = ? , г) ( А А1 D1D ) ∩ ( DD1С1С ) = ?

  1. Определить взаимное расположение прямых:hello_html_33135d75.jpg

а) АС и А1С1, б) С1D и DВ, в) DВ1 и АС ,

г) ВА и В1А1 , д) В1D и А1В1, е) В1D и АС .

  1. а) Докажите, что прямая АВ параллельна плоскости α, если АВСD параллелограмм, (А1В1 С D )= α и

А1В1 С D-трапеция.

б) Докажите, что прямая МК параллельна плоскости α, в которой лежит основание АВ треугольника АВС ,
а МК- средняя линия треугольника АВС .
hello_html_4d3b9131.jpg

в) В параллелограмме АОВК сторона ОВ параллельна прямой m , а АО и m –скрещивающиеся прямые. Найти угол между скрещивающимися прямыми , если один из углов параллелограмма равен 117°.

  1. Точка С лежит на отрезке АВ. Через точку А проведена плоскость α, а через точки В и С- параллельные прямые , пересекающие эту плоскость соответственно в точках В1 и С1. hello_html_bd13706.jpg

Найти длину отрезка СС1 ,если

а) ВВ1 = 8 см; б) АС : СВ = 3 : 2 и ВВ1 = 30 см.

В)

  1. Дано: а || α, bα, а || b, с - секущая, 2 1= 40°,Найти: 1, 2.

  2. Дано: а || α, bα, а || b, с - секущая, l + 2 = 150°,Найти: 1, 2, 3, 4, 5, 6, 7, 8.

  3. Дано: а || α, bα, а || b, с - секущая, 1 = 145°,Найти: 2, 3, 4, 5, 6, 7, 8.

  4. Дано: а || α, bα, а || b, с - секущая, 4 = 50°,Найти: 1, 2, 3, , 5, 6, 7, 8.

  5. Плоскости α и β пересекаются по прямой МС. Прямая а || α, а || β. Докажите, что а || МС.

  6. Дано: АВСD- прямоугольник, К ( АВСD), (СВК) = α. Докажите, что АD || α.

  7. Дано: AC || BD, AC ∩ α = A; BD ∩ α = B. AC = 10 см, BD = 8 см, AB = 2 см.

Доказать: CD ∩ α = E.Найти: BEhello_html_m41773e40.jpg

  1. Дано: ABCD - трапеция; AD α, АЕ = ЕВ, CF = FD (рис. ). Доказать: EF || α.

3) Решите задачи:

А)

  1. Дано: ΔABC, AC α, AD = DB, BE = EC. Доказать: DE || α. (рис.) рис.  hello_html_m7f9b8754.jpg

  2. Дан ΔВСЕ. Плоскость, параллельная прямой СЕ, пересекает BE в точке Е1, а ВС - в точке С1. Найдите ВС1, если С1Е1 : СЕ = 3 : 5, ВС = 30 см.

  3. Дано: А, В, С, D; В (ACD). Е, F, М, К- середины сторон АВ, ВС, CD, AD; AC = 8 см, BD = 14 см. Доказать: EFMK - параллелограмм. Найти: P(EFMK).  

  4. Дано: ΔАВК, М (АВК); E.D- точки пересечения медиан ΔМВК и ΔАВМ; АК = 24 см.

Доказать: ADEK - трапеция. Найти: DE

  1. Дано: AC || BD, AC ∩ α = A; BD ∩ α = B. AC = 12 см, BD = 10 см, AB = 3 см.

Доказать: CD ∩ α = E.Найти: BEhello_html_m4c348d2f.png

В)

1)Дана пирамида ABCS

Укажите:

1.Прямые, которые лежат в плоскости BSC

2. Прямые, пересекающие плоскость АВС

А С

2) Дан куб ABCDA1B1C1D1

  1. Рёбра, которые лежат на прямых, параллельных hello_html_m7b6c3827.png

ребру АА1

  1. Рёбра, которые лежат на прямых, пересекающих

ребро АА1

  1. Прямые, которые скрещиваются с прямой АА1

3) Дана пирамида ABCD Укажите: hello_html_m21257e9f.png

1.плоскости, в которых лежат прямые РЕ, МК, DB, АВ, ЕС;

2.точки пересечения прямой DK с плоскостью ABC, прямой СЕ с плоскостью ADB;

3. точки, лежащие в плоскостях ADB и DBC;

4.прямые, по которым пересекаются плоскости ABC и DCB, ABD и CDA, PDC и ABC.


С)

1.Дан куб ABCDA1B1C1D1. Точка М– середина ребра B1C1,

 N– серединаC1D1,K– середина DC,О– точка пересечения диагоналей основания ABCD.

Укажите взаимное расположение между следующими прямыми:

а) АA1 и 1; б)A1C1 и B1D1; в)A1C1 и C1D1; г) A1М и CC1; д)A1D и DC1; е)A1C1 и BD;

ё) A1C и АС; ж)A1B и D1С; з)A1C и ВB1; и) A1D и АВ; й)A1М и ВС; к)A1М и ВК;

л) C1К и B1N; м)C1О и AB1; н)A1О и B1D.

2. Дан тетраэдр ABCD. Точка К– середина ребра AD,L– середина DB,М– середина АС,

N– середина ВС. Определите взаимное расположение прямых и плоскостей.

а) DB и AMN; б)MN и ABC; в) КС и DMN;

г) MN и ABD; д)KL и DMN; е)LN и KML;

ё) CL и ADN; ж)LN и DMK.

3.Точка М не лежит в плоскости ромба ABCD. На отрезке ВМ выбрана точка F тaк,

что MF: FB = 1 : 3.

а)    Постройте точку К - точку пересечения прямой МС с плоскостью AFD.

б)    Найдите FK, если AD = 16 см.

4. 1)Пользуясь данным рисунком, назовите: а) четыре точки, лежащие в плоскости SAB; б) плоскость, в которой лежит прямая MN; в) прямую, по которой пересекаются плоскостиSAC и SBC.hello_html_m79d99f9a.jpg

2) Точка С - общая точка плоскости α и β. Прямая проходит через точку С. Верно ли, что плоскости аир пересекаются по прямой с? Ответ объясните.

3) Через прямую а и точку А можно провести две различные плоскости. Каково взаимное расположение прямой а и точки А? Ответ объясните.

4) Пользуясь данным рисунком, назовите: а) четыре точки, лежащие в плоскости ABC; б) плоскость, в которой лежит прямая KN; в) прямую, по которой пересекаются плоскости SAC и САВ.

5) Плоскости α и β имеют три общие точки. Верно ли, что эти плоскости совпадают? Ответ объясните.

6) Через А, В и С можно провести две различные плоскости. Каково взаимное расположение точек А, В и С? Ответ объясните. 

5.Дан прямоугольник АВСД, О - точка пересечения его диагоналей. Известно, что точки А, В, О лежат в плоскости α. Докажите, что точки С и Д также лежат в плоскости α. Вычислите площадь прямоугольника, если АС = 8 (см), AOB = 60°.

6.  Прямые а и b пересекаются в точке О, А  α, В  b, Р  АВ. Докажите, что прямые а и b и точка Р лежат в одной плоскости.

D)

  1. Дано: а || α, bα, а || b, с - секущая, 2 1= 50°,Найти: 1, 2.

  2. Две параллельные прямые, одна из которых лежит в плоскости α, пересечены третьей. Известно, что сумма двух внутренних накрест лежащих углов равна 110°. Чему равны эти углы и остальные шесть?

  3. Дано: а || α, bα, а || b, с - секущая, 1 = 135°,Найти: 2, 3, 4, 5, 6, 7, 8.

  4. Дано: а || α, bα, а || b, с - секущая, 4 = 20°,Найти: 1, 2, 3, , 5, 6, 7, 8.

  5. Плоскости α и β пересекаются по прямой МК. Прямая а || α, а || β. Докажите, что а || МК.

  6. Дано: ОРТЕ- прямоугольник, К ( ОРТЕ), (РТК) = α. Докажите, что ОЕ || α.

  7. Дано: АВСD- трапеция, М ( АВСD), (СВМ) = α. Докажите, что АD || α.

  8. Дано: АВСD- трапеция, АВ || α, С α, MN – средняя линия трапеции. Докажите, что MN || α.

  9. Дано: АВСD- параллелограмм, К ( АВСD), (АВК) = α. Докажите, что СD || α.hello_html_m477473c3.png

  10. Дано: А α, В α, С α, М – середина АС, К – середина ВС. Докажите, что MК || α.

  11. hello_html_me6b84c4.png


  1. Плоскость α проходит через середины боковых сторон АВ и CD трапеции ABCD- точки M и N. а) Докажите, что AD׀׀α.
    б) Найдите ВС, если
    AD=10 см, MN=8см.

  2. Плоскость α проходит через основание АD трапеции ABCD. Точки M и Nсередины боковых сторон трапеции. а) Докажите, что MN׀׀α.

б) Найдите AD, если BC = 4 см, MN = 6см.hello_html_m3e0c4470.jpg

  1. По готовому рисунку: а) докажите, что: KMEF;
    б) найдите
    KM , если EF=8 см.

  2. Дан ΔВСЕ. Плоскость, параллельная прямой СЕ, пересекает BE
    в точке Е1,а ВС - в точке С1. Найдите ВС1, если С1Е1 : СЕ = 3 : 8, ВС = 28 см.

  3. Через основание AD трапеции ABCD проведена плоскость α. ВС α. Докажите, что прямая, проходящая через середины сторон АВ и CD,
    параллельна плоскости α.
    hello_html_m1a1d6694.jpg

  4. Дано: А, В, С, D; В (ACD). Е, F, М, К- середины сторон АВ, ВС, CD, AD;
    AC = 6 см, BD = 8 см. Доказать: EFMK - параллелограмм. Найти: P(EFMK).  

  5. Дано: ΔАВК, М (АВК); E.D- точки пересечения медиан ΔМВК и ΔАВМ;
    АК = 14 см. Доказать: ADEK - трапеция. Найти: DE

Просмотрено: 0%
Просмотрено: 0%
Скачать материал
Скачать материал "ПЗ по специальности Технология машиностроения,1 курс(2016-2017 уч.г.)"

Получите профессию

Бухгалтер

за 6 месяцев

Пройти курс

Рабочие листы
к вашим урокам

Скачать

Выбранный для просмотра документ ПЗ№10.docx

ПЗ № 10. Скалярное произведение векторов. Векторное уравнение прямой и плоскости. Использование векторов при доказательстве теорем стереометрии.

Задание:

1) Опорный конспект.

hello_html_m4c520382.jpg

hello_html_3a89eddb.png

hello_html_m5533c7c2.jpg


hello_html_ma8301e3.jpg

2) Перепишите и заполните пропуски:
1. м.диктант.

2. Дано

hello_html_m28d7893e.gif{-hello_html_m10ab8df5.gif; -hello_html_m10ab8df5.gif; -2}, hello_html_m355018cf.gif{hello_html_m10ab8df5.gif/2; hello_html_m10ab8df5.gif/2; 1}

Найдите угол между векторами hello_html_m28d7893e.gif и hello_html_m355018cf.gif.

Решение:

1) hello_html_m5f0379c8.gif. hello_html_49d81ced.gif.

2) hello_html_m506bc32c.gif– 2 ∙ 1 = –1 – 1 – 2 = –4 .

3) cos α =hello_html_22bfaa40.gif= –1 hello_html_70dfa0d8.gif α = ...°.

Ответ: 180°.

3.Дано: ,,. Выяснить: а) и, б) , .

Решение:

а)

, значит угол ….

б) = ++ = ,

в) == ++ = , угол ...

Ответ: а) тупой, б) острый, в) прямой.

4. а)Дано: ^=^=60º,

Вычислить: (+

Решение:

(+ = +

= )=1 =2=…

2)= )=2=4=…

3) (+ =+ = 1+2=…

Ответ: 3.

б)Дано: 3-5+ , -5

Найти: (- 2-2)

Решение:

1) , , 2 , , ,2

2)- 2 , -2

3)(- 2-2)=3+(-7)=…

Ответ: 28

5. ABCD - тетраэдр, =90º, AB=BD=2, BC=1.

а) Вычислите косинус угла между прямой, проходящей через середины ребер AD и BC,и прямой AC.

Решение:1). =


2) А (2, 0, 0) , В (0, 0, 0) , С (0, 1, 0) , D (0, 0, 2)

Т.к. М – середина [AD], то , т.е.

М (1, 0, 1), аналогично N (0, , 0)

,



=

= . Ответ:

б) Вычислить синус угла между прямой MN и плоскостью грани ABD.


Решение:

- нормаль.

, .

=

Ответ:

6.Составить параметрические уравнения прямой, проходящей через точку M0 (1; 2; 0) и параллельной вектору s = (2; 0; −3).

Решение:





x = m t + x0

Запишем параметрические уравнения прямой: y = n t + y0





z = p t + z0


В нашем случае направляющий вектор прямой равен s = (2;0; −3), т.е. m = 2 , n = 0 и p = −3 .

Прямая проходит через точку M0 (1; 2; 0), поэтому

x0=1 , y0 = 2 z0 = 0 .

Подставим значения m = 2 ,

n = 0 , p = −3 , x0 =1 , y0 = 2

z0 = 0 в

параметрические уравнения прямой


x = m t + x0

x = 2 t +1

x = 2 t +1


y = n t

+ y0

y = 0 t + 2 ,




z = p t


z = − 3 t +0




Заметим, что прямая лежит в плоскости y = 2 , перпендикулярной оси

ординат. Поэтому прямая также перпендикулярна этой оси.

7. Найти уравнение плоскости, проходящей через точки A(1; 0; 2), B (2 ; 1; 1) и

параллельной оси OY .

Решение. В уравнении плоскости отсутствует переменная y , так как плоскость параллельна оси OY . Следовательно, уравнение плоскости, проходящей через

точку A(1; 0; 2), имеет вид

a ( x x0 ) + b ( y y0 ) + c ( z z 0 )= 0 a (x x0 )+ 0 ( y y0 ) + c ( z z 0 )= 0

a ( x 1) + c ( z 2 )= 0

Подставим в это уравнение координаты второй точки B (2 ; 1; 1). Получаем:

a ( x 1) + c ( z 2 )= 0 a ( 2 1) + c (1 2 )= 0 , a c = 0

Заметим, что один из неизвестных коэффициентов можно принять равным любому ненулевому числу (что повлияет только на изменение длины нормали плоскости). Например, полагаем a =1 , тогда из равенства ac = 0 находим c =1 . Подставим эти значения в уравнение искомой плоскости

a ( x 1) + c ( z 2 )= 0 x 1 + z 2 = 0 , x + z = 3

Ответ. x + z = 3 .

8. а)Если одна из двух параллельных прямых перпендикулярна к третьей, то и другая прямая перпендикулярна к этой прямой

a  b

a c


bc

…………, bc.


б) Если одна из двух параллельных прямых перпендикулярна к плоскости, то и другая прямая перпендикулярна к этой плоскости

a  b

a 

b

Пусть х- произвольная прямая плоскости , тогда


a  а…х …

)…..)=

b…х b.

в)

hello_html_1d9831e5.png

9.


hello_html_35a224f7.gif

10.


hello_html_13920dcf.png




3)Решить задачи :


1.Вычислите скалярное произведение векторов и , если

2. Дан куб АBCDA1B1C1D1. Найдите угол между прямыми AD1 и ВМ, где М – середина ребра DD1.

3. При движении прямая а отображается на прямую а1, а плоскость α – на плоскость α1. Докажите, что если а║α, то а1║ α1.

4.а) Найти косинус угла между прямыми MN и DC;

б) Найти синус угла между прямой MN и плоскостью BDC.

  1. Дан квадрат ABCD. Найдите угол между векторами и .

  2. Найдите скалярный квадрат вектора = 7.

  3. Найдите скалярное произведение если = 60˚.

  4. Вычислите скалярное произведение векторов если

  5. ABCDA1B1C1D1 - куб, ребро которого равно 1. Найдите скалярное произведение

векторов и .

  1. Вычислите угол между прямыми АВ и CD, если А(; 1; 0), С( 0; 2; 0 ),

В(0; 0; 2), D().

11. Найдите координаты вектора hello_html_1dc502d9.gif, если А (5; –1; 3), В (2; –2; 4).

12. Даны векторы hello_html_54ee4636.gif(3; 1; –2) и hello_html_m9b9f52c.gif(1; 4; –3). Найдите hello_html_5b70e5.gif.

13. Изобразите систему координат Oxyz и постройте точку А (1; –2; –4). Найдите расстояния от этой точки до координатных плоскостей.

  1. Дан квадрат ABCD. Найдите угол между векторами и .

  2. Найдите скалярный квадрат вектора = 5+2.

  3. Найдите скалярное произведение если = 150˚.

  4. Вычислите скалярное произведение векторов если - 3 + и = 4 - .

  5. ABCDA1B1C1D1 - куб, ребро которого равно 2. Найдите скалярное произведение

векторов и .

  1. Вычислите угол между прямыми АВ и CD, если А(1; 1 ; 5 ), С(8 ; 5 ; 5 ),

В(4; 7; 5), D(5).

20. Доказать, что прямая, проведенная через середины оснований трапеции, проходит через точку пресечения продолжений боковых сторон.

21. Точка С середина отрезка АВ, а О – произвольная точка плоскости. Доказать, что

22.

hello_html_68aa67d4.pnghello_html_68aa67d4.png



Просмотрено: 0%
Просмотрено: 0%
Скачать материал
Скачать материал "ПЗ по специальности Технология машиностроения,1 курс(2016-2017 уч.г.)"

Получите профессию

Няня

за 6 месяцев

Пройти курс

Рабочие листы
к вашим урокам

Скачать

Выбранный для просмотра документ ПЗ№11,12.docx

ПЗ № 11. История развития комбинаторики, теории вероятностей и статистики и их роль в различных сферах человеческой жизнедеятельности. Правила комбинаторики. Решение комбинаторных задач.

1)Опорный конспект.

Человеку часто приходится иметь дело с задачами, в которых нужно подсчитать число всех возможных способов расположения некоторых предметов или число всех возможных способов осуществления некоторого действия.

В повседневной жизни нередко перед нами возникают проблемы, которые имеют не одно, а несколько различных вариантов решения. Чтобы сделать правильный выбор, очень важно не упустить ни один из них. Для этого надо осуществить перебор всех возможных вариантов или хотя бы подсчитать их число. Такого рода задачи называют комбинаторными.

Комбинаторика – ветвь математики, изучающая комбинации и перестановки предметов, казалось, долгое время лежала вне основного русла развития математики и ее приложений. На протяжении двух с половиной столетий основную роль в изучении природы играл математический анализ. Процессы, имевшие атомистическую природу, заменялись непрерывными, чтобы можно было применить к ним развитый аппарат математики. Положение коренным образом изменилось после создания быстродействующих вычислительных машин, компьютеров. С их помощью стало возможным делать переборы, ранее требовавшие сотен и тысяч лет. В эпоху расцвета дискретной математики изменилась и роль древнейшей области дискретной математики – комбинаторики. Из области, интересовавшей большей частью составителей занимательных задач и находившей основные применения в кодировании и расшифровке древних письменностей, она превратилась в область, находящуюся на магистральном пути развития науки. Стали выходить журналы по комбинаторике, печататься книги, посвященные этой науке.

В нынешнее время комбинаторика имеет огромное значение в различных областях науки и сферы. С комбинаторными величинами приходится иметь дело представителям многих специальностей: ученому – химику, биологу, конструктору, диспетчеру и т.п.

Ещё первобытный вождь понимал, что у десятка охотников вероятность поразить копьём зубра гораздо больше, чем у одного. Поэтому и охотились тогда коллективно.

Позднее, с опытом, человек всё чаще стал взвешивать случайные события, классифицировать их исходы как невозможные, возможные и достоверные. Он заметил, что случайностями не так уж редко управляют объективные закономерности.

Наиболее интересные для начинающих задачи комбинаторики и теории вероятностей возникли в области азартных игр. Этому, по-видимому, способствовало наличие монеты или игральной кости.

Одним из первых занялся подсчетом числа различных комбинаций при игре в кости итальянский математик Тарталья. Он составил таблицу, показывавшую, сколькими способами могут выпасть р костей. Однако при этом не учитывалось, что одна и та же сумма очков может быть получена разными способами.

Со временем появились различные игры (нарды, карты, шашки, шахматы и т.д.). В каждой из этих игр приходилось рассматривать различные сочетания фигур, и выигрывал тот, кто их лучше изучил, знал выигрышные комбинации и умел избегать проигрышных. Не только азартные игры давали пищу для комбинаторных размышлений математиков. Еще с давних пор дипломаты, стремясь к тайне переписки, изобретали сложные шифры, а секретные службы других государств пытались эти шифры разгадать. Стали применять шифры, основанные на комбинаторных принципах, например, на различных перестановках букв с использованием ключевых слов и т. д.

Как раздел математики комбинаторика возникла в XVI веке, так как для решения вероятностных задач необходимо было подсчитать число различных комбинаций элементов. Первые научные исследования по комбинаторике принадлежат итальянским ученым Дж. Кардано, Н. Тарталье (ок. 1499-1557 гг.), Г. Галилею (1564-1642 гг.) Дальнейшее развитие комбинаторики связано с трудами Б. Паскаля (1623 – 1662 гг.) и П. Ферма (1601 – 1665 гг.) по теории азартных игр. Позднее крупный вклад в развитие комбинаторных методов был сделан Г. Лейбницем (1646 – 1716 гг.), Я. Бернулли (1654 – 1705 гг.) и Л. Эйлером (1707 – 1783 гг.). В их работах были даны определения основных понятий комбинаторики, развиты первые комбинаторные методы и указаны их применения, а также прослежена связь комбинаторики с исчислением вероятностей. Именно комбинаторика послужила фундаментальной основой началам теории вероятностей. Немецкий учёный Г.Лейбниц в своей работе «Об искусстве комбинаторики», опубликованной в 1666 г., впервые выделил комбинаторику как самостоятельный раздел математики. Он также впервые ввел термин «комбинаторика».


Комбинаторика в различных областях жизнедеятельности человека.

Комбинаторика в литературе

В басне Ивана Андреевича Крылова «Квартет»: «проказница Мартышка, Осёл, Козёл да косолапый Мишка» устроили любопытный эксперимент, они исследовали влияние взаимного расположения музыкантов на качество исполнения.

Математика на шахматной доске и в играх.

Профессиональный интерес математиков к шахматам проявился довольно давно и был связан с двумя направлениями: математической логикой и комбинаторикой. Первое — рассмотрение игры с точки зрения построения ее формальной модели, удобной для логического анализа на основе действующих соревновательных правил. Второе — исследование конкретных позиций или их классов в игре для достижения определенных результатов, например матовой позиции за определенное число ходов. Последнее направление породило множество изящных логико-вычислительных проблем. Некоторые из них и по сей день предлагаются на различных математических и программистских олимпиадах, а также для развлечения на досуге.

Необыкновенно популярна головоломка - кубик Рубика, изобретенный в 1975 г. преподавателем архитектуры из Будапешта Эрне Рубиком для развития пространственного воображения у студентов. Задача поиска оптимального (по числу ходов) алгоритма сбора кубика Рубика является самой сложной и не решенной пока математической задачей.

Пароли и коды в нашей жизни.

Вся наша жизнь состоит из множества разнообразных программ. Чтобы запустить ту или иную программу нужно ввести соответствующий верный пароль.

В качестве кода в зависимости от рода программы могут выступать всевозможные цифры, слова или комбинации слов, поведение или действие, и так далее...

Когда мы узнаем что-то новое, развиваемся, к нам приходит жизненный опыт, он то, как раз и есть ничто иное, как набор всевозможных паролей, комбинаций. Ведь опытный человек всегда найдет лучшее решение в конкретной ситуации, потому – что он располагает большей комбинацией паролей.

Мебельная комбинаторика.

Мебельная комбинаторика позволяет рассматривать различные варианты комплектации предметов мебели и выбирать из них наилучшее, комфортнее и практичнее.

hello_html_m22e78f5d.jpg

Задачи на перебор возможных вариантов

Задачи на перебор возможных вариантов решаются следующим образом: выдвигается некая гипотеза, которая подтверждается или опровергается в ходе дальнейших рассуждений. Гипотезы выдвигаются до тех пор, пока не найдется единственно правильный вариант. Решение задач сопровождается записью рассуждений и там, где это необходимо для наглядности, графической иллюстрацией. В рассуждениях выдвижение гипотезы начинается со слова «Пусть». Если при рассмотрении какой-то гипотезы получен ответ на вопрос задачи, надо обязательно проверить и другие варианты для подтверждения, того, что найденное решение является единственным истинным.

1. Сколько двузначных чисел можно составить из цифр 1, 4 и 7? 

1

4

7

1

4

7

Перебором:

Ответ: 9 чисел.

2. Сколько чётных двузначных чисел можно составить из цифр 0, 1, 2, 4, 5, 9? 

0

2

4

1

10

12

14

2

20

22

24

4

40

42

44

5

50

52

54

9

90

92

94

5 * 3 = … чётных двузначных чисел

3. На завтрак Вова может выбрать плюшку, бутерброд, пряник или кекс, а запить их он может кофе, соком или кефиром. Из скольких вариантов завтрака Вова может выбрать?

Плюшка

Бутерброд

Пряник

Кекс

Кофе

Плюшка
Кофе

Бутерброд
Кофе

Пряник
Кофе

Кекс
Кофе

Сок

Плюшка
Сок

Бутерброд
Сок

Пряник
Сок

Кекс
Сок

Кефир

Плюшка
Кефир

Бутерброд
Кефир

Пряник
Кефир

Кекс
Кефир

  1. * 4 = 12 – вариантов завтрака/

4. В футбольном турнире участвуют несколько команд. Оказалось, что все они для трусов и футболок использовали белый, красный, синий и зеленый цвета, причем были представлены все возможные варианты. Сколько команд участвовали в турнире?
Решение. Трусы могут быть белого, красного, синего или зеленого цвета, т.е. существует 4 варианта. Каждый из этих вариантов имеет 4 варианта цвета майки.
4 х 4 = 16. Ответ: 16 команд.

2)Решить задание:

  1. Запишите все двузначные числа, в записи которых используются только цифры 3, 5, 7, 9. Сколько двузначных чисел можно записать, если использовать при записи числа каждую цифру только один раз?

  2. В четверг в первом классе должно быть три урока: русский язык, математика и физкультура. Сколько различных вариантов расписания можно составить на этот день?
    Указание: Перебирая варианты введите обозначения:
    Р – русский язык, М – математика, Ф – физкультура.

  3. Саша выбрал в библиотеке 5 книг, но одновременно можно взять только две книги. Сколько вариантов выбора двух книг есть у Саши?

  4. Школьники из Волгограда собрались на каникулы поехать в Москву, посетив по дороге Нижний Новгород. Сколькими различными способами могут ребята осуществить свое путешествие, если из Волгограда в Нижний Новгород можно отправиться на теплоходе ли поезде, а из Нижнего Новгорода в Москву – на самолете, теплоходе, поезде или автобусе?

  5. Девять школьников, сдавая экзамены по математике и английскому языку, получили отметки «4» и «5». Можно ли утверждать, что по крайней мере двое из них получили по каждому предмету одинаковые отметки?

  6. Сколько существует двузначных чисел, у которых первая цифра больше второй?

  7. Несколько стран в качестве символа своего государства решили использовать флаг в виде четырех вертикальных полос, одинаковых по ширине, но разных по цвету: белый, синий, красный, зеленый. У каждой страны свой, отличный от других, флаг.
    а) Сколько всего стран могут использовать такую символику?
    б) Сколько всего стран могут использовать такую символику с верхней белой полосой?
    в) Сколько всего стран могут использовать такую символику с нижней зеленой полосой?
    г) Сколько всего стран могут использовать такую символику с синей и красной полосами, расположенными рядом?

  8. За свои рисунки ученик получил две положительные оценки. Какими они могут быть? Сколько вариантов?

  9. Сколько флагов можно составить из трех разных цветов, если имеются полосы синего, белого, красного цветов?

  10. В понедельник в пятом классе 5 уроков. Сколькими способами можно составить расписание на понедельник?

  11. Из десяти учащихся надо выбрать старосту, физорга и культорга. Сколькими способами это можно сделать?

  12. Сколькими способами можно переставить 5 различных геометрических фигур?

  13. Пять человек обменялись друг с другом фотографиями. Сколько всего фотографий было?

  14. На плоскости отмечены 6 точек. Каждые две точки соединили отрезком. Сколько получилось отрезков?

  15. Сколькими способами можно расставить на полке 4 различные книги?

  16. Алфавит племени тумба-юмба состоит из букв А, У, С. Словом является любая последовательность из 4 букв. Сколько слов в языке этого племени?

  17. «Проказница Мартышка, Осел, Козел и косолапый Мишка затеяли сыграть квартет». Сколькими способами они могут распределить четыре имеющихся у них инструмента?

  18. а)Сколько двузначных чисел можно составить из цифр 1,4,5,7, используя в записи числа каждую из них не более одного раза?

б) На завтрак Вова может выбрать бутерброд, пряник или кекс, а запить их он может кофе, соком или кефиром. Из скольких вариантов завтрака Вова может выбирать?

  1. Сколькими способами можно представлять друг с другом цифры 1, 2, 3, 4?

  2. За столом пять мест. Сколькими способами можно расставить пятерых гостей?

  3. Путешественник из пункта А в пункт С может попасть, доехав до промежуточного пункта В по одной из трёх существующих автомагистралей, а из В в С доехать либо поездом, либо на такси. Сколько существует различных маршрутов между пунктами А и С?

  4. Стас решил пойти на новогодний карнавал в костюме мушкетёра. В ателье проката ему предложили на выбор различные по фасону и цвету предметы: 3 пары брюк, 4 камзола, 3 шляпы, 2 пары сапог. Сколько различных карнавальных костюмов можно составить из этих предметов?

  5. Олеся, Оксана и Юля купили билеты на концерт симфонического оркестра на 1, 2 и 3-е места первого ряда. Сколько существует способов размещения девочек на эти места?

  6. Сергей, Игорь и Миша могут занять 1-е, 2-е и 3-е призовые места в соревнованиях по шахматам. Перечислить всевозможные последовательности из имён мальчиков, где порядковый номер в последовательности соответствует занятому мальчиком месту в соревнованиях. Подсчитать их количество.

  7. В школьной столовой имеются помидоры, огурцы и лук. Сколько различных салатов можно приготовить, если в каждый из них должны входить в равных долях 2 различных вида овощей? Записать все сочетания овощей в составленных салатах.

  8. Девятиклассники Миша, Дима, Антон и Саша побежали на перемене к теннисному столу, за которым уже шла игра. Сколькими способами подбежавшие к столу четверо девятиклассников могут занять очередь для игры в настольный теннис?

  9. Сколько различных смешанных пар для игры в теннис можно образовать из восьми юношей и шести девушек?

  10. Для подарков первоклассникам закупили книги пяти разных авторов и игрушки шести разных видов. Сколько различных подарков можно составить, если в каждый должна входить одна книга и одна игрушка?

  11. В финальном забеге на 100 м участвуют Иванов, Громов и Орлов. Назовите возможные варианты распределения призовых мест.

  12. В кружок бального танца записались Петя, Коля, Витя, Олег, Таня, Оля, Наташа, Света. Какие танцевальные пары девочки и мальчика могут образоваться?

  13. Запишите все возможные варианты расписания пяти уроков на день из предметов: математика, русский язык, история, английский язык, физкультура, причем математика должна быть вторым уроком.

  14. Маша, Оля, Вера, Ира, Андрей, Миша и Игорь готовились стать ведущими на Новогоднем празднике. Назовите возможные варианты, если ведущими могут быть только одна девочка и один мальчик.

  15. 6 учеников сдают зачет по математике. Сколькими способами их можно расположить в списке?

  16. Сколько четных двузначных чисел можно составить из цифр 0, 2, 3, 4, 6, 7?

ПЗ № 12. Размещения, сочетания и перестановки. Бином Ньютона и треугольник Паскаля. Прикладные задачи.

1)Опорный конспект.

hello_html_m3c3aae1b.jpg

Понятие бинома Ньютона
Биномом Ньютона называют разложение вида:
hello_html_3109c0eb.gif
Но, строго говоря, всю формулу нельзя назвать биномом, так как «бином» переводится как «двучлен». Кроме того, формула разложения была известна еще до Ньютона, Исаак Ньютон распространил это разложение на случай n<0 и n – дробного.
Цель изучения бинома Ньютона – упрощение вычислительных действий.
Компоненты формулы «бином Ньютона»:правая часть формулы – разложение бинома;
hello_html_m7f7221a0.gif


hello_html_m3030b558.gif – биномиальные коэффициенты, их можно получить с помощью треугольника Паскаля (пользуясь операцией сложения).


Практическая значимость треугольника Паскаля заключается в том, что с его помощью можно запросто восстанавливать по памяти не только известные формулы квадратов суммы и разности, но и формулы куба суммы (разности), четвертой степени и выше.
Например, четвертая строчка треугольника как раз наглядно демонстрирует биномиальные коэффициенты для бинома четвертой степени:
hello_html_4d3e168.gif
Альтернатива треугольнику Паскаля:

  1. перемножить почленно четыре скобки:
    hello_html_c44bb7f.gif;

  1. вспомнить разложение бинома Ньютона четвертой степени:
    hello_html_m32884135.gif
    общий член разложения бинома n-й степени: 
    hello_html_m6cb3ccf5.gif,
    где Т – член разложения; 
    hello_html_m5cd3cd0b.gif – порядковый номер члена разложения.
    Свойства бинома и биномиальных коэффициентов

  1. hello_html_m40d1b2c3.gif

  2. Число всех членов разложения на единицу больше показателя степени бинома, то есть равно hello_html_m38da65e0.gif

  3. Сумма показателей степеней a и b каждого члена разложения равна показателю степени бинома, то есть n. Доказательство: Рассмотрим hello_html_m5cd3cd0b.gif-й член разложения: hello_html_583ec35b.gif
    Сумма показателей степеней 
    a и bhello_html_me119c4e.gifЧ.т.д

  1. Биномиальные коэффициенты членов разложения, равноотстоящих от концов разложения, равны между собой: hello_html_m6c57fe34.gif (правило симметрии)

  2. Сумма биномиальных коэффициентов всех членов разложения равна hello_html_29daa9a6.gif
    hello_html_3bb7339e.gif
     
    hello_html_5a27b673.gifhello_html_m61d3eb50.gif

  1. Сумма биномиальных коэффициентов, стоящих на нечетных местах, равна сумме биномиальных коэффициентов, стоящих на четных местах и равнаhello_html_m4cf34de5.gif


hello_html_m63a4870a.gif

  1. Правило Паскаля: hello_html_2b56e16.gif

  1. Любой биномиальный коэффициент, начиная со второго, равен произведению предшествующего биномиального коэффициента и дроби hello_html_2d89af74.gif,hello_html_m714b47b0.gif
    Типовые задачи по теме «Бином Ньютона»
    К типовым (стандартным) заданиям по данной теме можно отнести задачи на вычисление, среди которых:

    1. Найти член (номер члена) разложения бинома

    2. Вывести бином по известным членам разложения (по известной сумме)


Вычислить сумму биномиальных коэффициентов разложения бинома
и другие.
Продемонстрируем на примерах (их решение несложное, поэтому большинство предлагаем решить самостоятельно).
Пример .В биномиальном разложении hello_html_7d2a897c.gif найти член разложения, не содержащий х
Решение:
hello_html_m34d61411.gif
Так как в разложении мы ищем член не содержащий 
х, то hello_html_1568eb8e.gif
Тогда 
hello_html_m660919.gif
Ответ: 
hello_html_44d6f621.gif
Задачи, сводящиеся к использованию формулы бинома Ньютона 
(нестандартные задачи по теме «Бином Ньютона»)
К нестандартным заданиям по данной теме можно отнести такие, в которых нет явного намека на необходимость использования бинома. Однако в итоге, решение сводится к нему и выглядит очень интересным.
Пример1. Доказать, что для любых hello_html_44cfd3a4.gif и для любых hello_html_66e6eb8a.gif верно неравенство Бернулли:

hello_html_m19bbb27c.gif . Доказательство: Пусть hello_html_m761619fc.gif
Так как 
hello_html_44cfd3a4.gif, то hello_html_35376388.gif
Переформулируем требование: Доказать, что 
hello_html_m1fdc89ff.gif, где hello_html_m6810b779.gif

hello_html_6388f0a4.gif
Так как 
hello_html_m4f82866c.gif, значит в разложении как минимум три члена разложения, тогда:
hello_html_5b8534f5.gif
Это означает, что 
hello_html_m19bbb27c.gifЧ.т.д.
Пример 2 . Доказать, что при любом натуральном n число hello_html_1f8d1336.gif делится на 9

Доказательство:hello_html_1dac8434.gif
hello_html_m2a8a54c5.gifЧ.т.д.
Пример 3.Решить уравнение hello_html_m10b576fe.gif

Решение: Осуществим замену: hello_html_22b9cd2c.gif
Тогда уравнение перепишем: 
hello_html_6f72da60.gif
Применим формулу бинома к левой части уравнения: 
hello_html_7c84c5a2.gif
В итоге 
hello_html_m45e4118a.gif
Ответ: 
hello_html_7054713a.gif

2)Перепишите и заполните пропуски:

Пример 1. а) Сколько трехзначных чисел можно записать, используя цифры 1,3,6,7,9, если каждая их них может быть использована в записи только один раз?

Решение: по формуле получаем: способов.

Ответ:60.

б) Из 20 учащихся надо выбрать старосту, его заместителя и редактора газеты. Сколькими способами это можно сделать?

Решение: по формуле получаем: способов.

Ответ:6840.

в)Сколькими способами четверо юношей могут пригласить четырех из шести девушек на танец?

Решение: Два юноши не могут одновременно пригласить одну и ту же девушку. И варианты,

при которых одни и те же девушки танцуют с разными юношами, считаются разными, поэтому:

Ответ: 360.

Пример 2. а)Сколькими способами можно представлять друг с другом цифры 1, 2, 3, 4?

Решение: Р4 = 4!= = …

Ответ: 24.

б)За столом пять мест. Сколькими способами можно расставить пятерых гостей?

Решение: Р5 = 5! =

Ответ:120.

в)Сколькими способами можно выложить в ряд красный, черный, синий и зеленый шарики?

Решение: На первое место можно поставить любой из четырех шариков (4 способа), на

второе – любой из трех оставшихся (3 способа), на третье место – любой из

оставшихся двух (2 способа), на четвертое место – оставшийся последний шар.

Всего 4 · 3 · 2 · 1 = 24 способа. Р4 = 4! = 1 · 2 · 3 · 4 = 24. Ответ: 24 способа.

г)Сколькими способами можно переставить буквы слова «ананас»?

Решение: всего букв 6. Из них одинаковы n1«а»=3, n2«н»=2, n3«с»=1. Следовательно, число различных перестановок равно

Ответ:60.

Пример 3. а) Из 15 членов туристической группы надо выбрать трех дежурных. Сколькими способами можно сделать этот выбор?

Решение: каждый выбор отличается от другого хотя бы одним дежурным. Значит, здесь речь идет о сочетаниях из 15 элементов по 3. Следовательно, по формуле получаем

Ответ:455.

б) На полке стоит 12 книг: англо-русский словарь и 11 художественных произведений на английском языке. Сколькими способами читатель может выбрать 3 книги, если :

1) словарь нужен ему обязательно; 2) словарь ему не нужен?

Решение:

1) 2)

Ответ: 1) 55,2) 165.

в) Учащимся дали список из 10 книг, которые рекомендуется прочитать во время каникул. Сколькими способами ученик может выбрать из них 6 книг?


Решение: Выбор 6 из 10 без учёта порядка: способов.

Ответ: 210 способов.

г) Сколько трехкнопочных комбинаций существует на кодовом замке (все три кнопки нажимаются одновременно), если на нем всего 10 цифр.

Решение: Так как кнопки нажимаются одновременно, то выбор этих трех кнопок – сочетание. Отсюда возможно вариантов.

Ответ:120.

Пример 4. а) Имеются 10 различных книг, три из которых – справочники. Сколькими способами

можно расставить эти книги на полке так, чтобы все справочники стояли рядом?

Решение: Т.к. в справочники должны стоять рядом, то будем рассматривать их как одну книгу. Тогда на полке надо расставить 10 – 3 + 1= … книг. Это можно сделать P8 способами. Для каждой из полученных комбинаций можно сделать P3 перестановок справочников.

Поэтому число способов расположения книг на полке равно произведению:

P8 · P3 = 8! · 3! = 40320 · 6 = ...

Ответ: 241920.

б) Сколько всего существует результатов опыта, заключающегося в подбрасывании двух одинаковых игральных костей?

Решение: Формула числа сочетаний из m элементов по n элементов с повторениями имеет вид:

,
Ответ: 21.

Пример 5. Сколько различных перестановок можно образовать из букв слова «задача»?

Решение: Образовать какую – либо перестановку из букв слова «задача» - это значит на шесть занумерованных мест, каким – нибудь образом поставить одну букву «з», одну букву «д», одну букву «ч» и три буквы «а». Если буквы «з», «д» и «ч» как – то поставлены, то остальные места заполняются буквами «а». Но сколькими способами можно поставить три различные буквы на шесть мест? Очевидно, что число способов равно числу всех трёхэлементных упорядоченных подмножеств шестиэлементного множества, т.е. равно

А63 = 6 ∙5 ∙4 = …

Ответ: 120 различных перестановок.

Пример 6. а) Сколькими способами можно разместить на полке 5 книг?

Решение: Задача сводится к подсчёту числа перестановок из пяти элементов:

Р5 = 5! = 1 ∙2 ∙3 ∙4 ∙5 = … способов.

Ответ: 120 способов

б) Сколькими способами могут быть расставлены 8 участниц финального забега на 8-ми беговых дорожках?

Решение: Р8 = 8!= 1 ∙2 ∙3 ∙4 ∙5 ∙ 6 ∙7 ∙8 = 40320 способов.

Ответ: 40320 способов забега.

Пример 7. В чемпионате страны по футболу (высшая лига) участвуют 18 команд, причём каждые две команды встречаются между собой 2 раза. Сколько матчей играется в течение сезона?

Решение: В первом круге состоится столько матчей, сколько существует двухэлементных подмножеств у множества, содержащего 18 элементов, т.е. их число равно С182. По формуле (4) получаем

18∙17

С218 = 2 =153.

Во втором круге играется столько же матчей, поэтому в течение сезона состоится … встреч.

Ответ: 306 встреч.

Пример 8. Олеся, Оксана и Юля купили билеты на концерт симфонического оркестра на 1, 2 и 3-е места первого ряда. Сколько существует способов размещения девочек на эти места?

Решение: Количество различных способов равно числу перестановок из 3 элементов:

Р3 = 3! = 1×2×3 = …способов.

Ответ: 6 способов.

Пример 9. Сколькими способами можно составить флаг, состоящий из трех горизонтальных полос различных цветов, если имеется материал пяти цветов?

Решение. Искомое число трехполосных флагов:

hello_html_469d74c0.gif

Пример 10. Сколькими способами можно в игре “Спортлото” выбрать 5 номеров из 36?

Искомое число способов
hello_html_4df0ad63.gif


3)Решить задание ( по примерам):

  1. а) Сколько трехзначных чисел можно записать, используя цифры 1,2,4,6,7,9, если каждая их них может быть использована в записи только один раз?

б) Из 15 учащихся надо выбрать старосту, его заместителя и редактора газеты. Сколькими способами это можно сделать?

в)Сколькими способами четверо юношей могут пригласить четырех из пяти девушек на танец?

  1. а)Сколькими способами можно представлять друг с другом цифры 1, 2, 3, 4,5?

б)За столом семь мест. Сколькими способами можно расставить семерых гостей?

в)Сколькими способами можно выложить в ряд красный, черный, синий, белый и зеленый шарики?

г)Сколькими способами можно переставить буквы слова «Миссисипи»?

  1. а) Из 25 членов туристической группы надо выбрать трех дежурных. Сколькими способами можно сделать этот выбор?

б) На полке стоит 15 книг: англо-русский словарь и 14 художественных произведений на английском языке. Сколькими способами читатель может выбрать 3 книги, если :
1) словарь нужен ему обязательно; 2) словарь ему не нужен?

в) Учащимся дали список из 10 книг, которые рекомендуется прочитать во время каникул. Сколькими способами ученик может выбрать из них 7 книг?

г) Сколько четырехкнопочных комбинаций существует на кодовом замке (все три кнопки нажимаются одновременно), если на нем всего 10 цифр.

  1. а)Сколько двузначных чисел можно составить из цифр 1,4,5,7, используя в записи числа каждую из них не более одного раза?

б) На завтрак Вова может выбрать бутерброд, пряник или кекс, а запить их он может кофе, соком или кефиром. Из скольких вариантов завтрака Вова может выбирать?

  1. а)Имеются 10 различных книг, 6 из которых – справочники. Сколькими способами

можно расставить эти книги на полке так, чтобы все справочники стояли рядом?
б)
Сколько всего существует результатов опыта, заключающегося в подбрасывании трех одинаковых игральных костей?

4)Решить задание:
Подсчет числа размещений, перестановок, сочетаний.

  1. «Вороне где-то Бог послал кусочек сыра», колбасы, хлеба и шоколада. «На ель Ворона взгромоздясь, позавтракать совсем уж было собралась, да призадумалась»: если есть кусочки по очереди, то из скольких вариантов придется выбирать?

  2. Сколькими способами можно из 25 учащихся выбрать 5 для участия в школьном марафоне?

  3. Сколькими способами могут быть распределены золотая и серебряная медали по итогам первенства по футболу, если число команд 12?

  4. В классе 7 человек успешно занимаются математикой. Сколькими способами можно выбрать из них двоих для участия в математической олимпиаде?

  5. Из 12 солдат нужно в разведку послать 5. Сколькими способами это можно сделать?

  6. Сколько пятизначных чисел можно составить, используя только цифры 3 и 5?

  7. «Проказница Мартышка, Осел, Козел и косолапый Мишка затеяли сыграть квартет». Сколькими способами они могут распределить четыре имеющихся у них инструмента?

  8. «Проказница Мартышка, Осел, Козел и косолапый Мишка затеяли сыграть квартет». На складе 12 музыкальных инструментов. Мишке поручили принести со склада 8 любых инструментов. Сколько вариантов выбора есть у мишки?

  9. Гера, Афина и Афродита попросили Париса не только назвать самую красивую из них, но и указать, кто «на втором и третьем местах». Сколько есть вариантов ответа?

  10. В магазине «Филателия» продается 8 различных наборов марок, посвященных «Дню Победы». Сколькими способами можно сформировать из них 3 набора?

  11. В классе 27 учеников, из которых нужно выбрать троих: первый ученик должен решить задачу, второй – сходить за мелом, третий – пойти дежурить в столовую. Сколькими способами это можно сделать?

  12. Сколькими способами можно из 6 человек составить комиссию, состоящую из двух человек?

  13. В соревновании участвуют 10 человек. Сколькими способами могут распределиться между

  14. Сколькими способами можно переставить 5 различных геометрических фигур?

  15. Сколькими способами можно выбрать гласную и согласную буквы из слова «здание»?

  16. За свои рисунки ученик получил две положительные оценки. Какими они могут быть? Сколько вариантов?

  17. В соревновании участвуют 10 человек. Сколькими способами могут распределиться между ними места?

  18. У одного человека 7 книг по математике, а у второго – 9. Сколькими способами они могут обменять друг у друга две книги на две книги.

  19. Имеется пять различных стульев и семь рулонов обивочной ткани различных цветов. Сколькими способами можно осуществить обивку стульев.

  20. На ферме есть 20 овец и 24 свиньи. Сколькими способами можно выбрать одну овцу и одну свинью? Если такой выбор уже сделан, сколькими способами можно сделать его еще раз?

  21. Сколько существует четных пятизначных чисел, начинающихся нечетной цифрой?

  22. В 9 классе учатся 7 учащихся, в 10 - 9 учащихся, а в 11 - 8 учащихся. Для работы на пришкольном участке надо выделить двух учащихся из 9 класса, трех – из 10, и одного – из 11 . Сколько существует способов выбора учащихся для работы на пришкольном участке?

  23. Сколько наборов из семи пирожных можно составить, если в продаже имеется четыре сорта пирожных?

  24. На памятные сувениры в «Поле Чудес» спонсоры предлагают кофеварки, утюги, телефонные аппараты, духи. Сколькими способами 9 участников игры могут получить эти сувениры? Сколькими способами могут быть выбраны 9 предметов для участников игры?

  25. Сколькими способами можно выложить в ряд красный, черный, синий и зеленый шарики?

  26. Сколько экзаменационных комиссий, состоящих из 7 членов, можно образовать из 14 преподавателей?

  27. Сколькими способами могут быть расставлены 6 участниц финального забега на 6-ми беговых дорожках?

  28. Сколькими способами можно разместить на полке 4 книг?

  29. Сколько различных двузначных чисел можно составить из цифр 1, 2, 3, 4, 5, 6, 7, 8 при условии, что в записи числа каждая цифра используется только один раз?

  30. Из слова РОТ перестановкой букв можно получить еще такие слова: ТОР, ОРТ, ОТР, ТРО, РТО. Их называют анаграммами. Сколько анаграмм можно составить из слова ЛОГАРИФМ?





















Просмотрено: 0%
Просмотрено: 0%
Скачать материал
Скачать материал "ПЗ по специальности Технология машиностроения,1 курс(2016-2017 уч.г.)"

Получите профессию

Интернет-маркетолог

за 6 месяцев

Пройти курс

Рабочие листы
к вашим урокам

Скачать

Выбранный для просмотра документ ПЗ№13,14.docx

ПЗ № 13. Классическое определение вероятности, свойства вероятностей, теорема о сумме вероятностей.

1)Опорный конспект.

При классическом способе подсчета вероятность события А вычисляется по формуле: Р(А)=m/n, где:

все элементарные исходы равновозможны, т.е. ни один из них не является более возможным, чем другой;m – число элементарных исходов испытания, благоприятствующих появлению события А;

n – общее число всех возможных элементарных исходов испытания.

Отметим основные свойства вероятности.
1. Вероятность любого события заключена между нулем и единицей:hello_html_m54c11b95.png

2. Вероятность достоверного события I, т. е. такого события, которое при испытании обязательно произойдет, равна единице: hello_html_m74a48814.png

3. Вероятность невозможного события О, т. е. события, которое в результате испытания не может произойти, равна нулю: hello_html_m15aeba51.png

4. Сумма вероятностей двух противоположных событий А и А, т. е. таких событий, что появление одного из них исключает появление другого, равна единице: hello_html_m2bca0899.png

Вероятностью события А называют отношение числа m исходов испытаний, благоприятствующих наступлению события А, к общему числу n всех равновозможных несовместных исходов: Р(А)=m/n.

Условной вероятностью события А (или вероятностью события А при условии, что наступило событие В), называется число РВ(А) = Р(АВ)/Р(В), где А и В – два случайных события одного и того же испытания.

Суммой конечного числа событий называется событие, состоящее в наступлении хотя бы одного из них. Сумма двух событий обозначается А+В.

Правила сложения вероятностей:

правило сложения вероятностей совместных событий А и В:
Р(А+В) = Р(А)+Р(В)-Р(АВ), где Р(А) – вероятность события А, Р(В) – вероятность события В, Р(А+В) – вероятность появления хотя бы одного из двух событий, Р(АВ)- вероятность совместного появления двух событий.

правило сложения вероятностей несовместных событий А и В:
Р(А+В) = Р(А)+Р(В), где Р(А) – вероятность события А, Р(В) – вероятность события В.

2) Перепишите и заполните пропуски:
Пример 1. a)В партии из 100 деталей имеется 5 бракованных. Определить вероятность того, что, взятая наугад, деталь окажется стандартной.

Решение: А: взятая наугад деталь оказалась стандартной.

Число исходов, благоприятствующих наступлению события А, равно 95.Поэтому вероятность события равна P(A) = m/ n = 95/100 = … .hello_html_m27618eb7.gif Ответ: 0,95.

б) Из пяти букв разрезной азбуки составлено слово «книга». Ребенок, не умеющий читать, рассыпал эти буквы, а затем собрал их в произвольном порядке. Найти вероятность того, что у него снова получится слово «книга».

Решение: А: из рассыпанных букв сложится слово «книга»

Число всех возможных исходов равно n = Pn = 5! = 120.

Число исходов, благоприятствующих событию А равно m =1.

Вероятность события А равна P(A) = m/ n = 1/120 = … .hello_html_m27618eb7.gif 

Ответ: 0,0083.

Пример 2.a) В коробке лежат 8 зеленых, 7 синих и 15 красных карандашей. Вычислить вероятность того, что взятый наугад карандаш будет, синим или зеленым.

Решение: А: взяли синий карандаш, В: взяли зеленый карандаш, С: взяли синий или зеленый карандаш. Событие С равно сумме событий А и В: С = А + В

Вероятность события А равна P(A) = m/ n = 7/30. 

Вероятность события В равна P(B) = m/ n = 8/30. 

Вероятность события С равна P(C) = P(A) = 7/30 8/30 = 15/30 = ... Ответ: 0,5. б) В урне лежат шары, двузначные номера которых составлены из цифр 1,2,3,4,5. Какова вероятность вынуть шар с номером 15? Решение: А: вынут шар с номером 15.

Число всех возможных исходов равно n =

Число исходов, благоприятствующих событию А, m = 1.

Вероятность события А равна P(A) = m/ n = 1/20 = … .

Ответ: 0,05.

Пример 3.a) Набирая номер телефона, абонент забыл последние две цифры и, помня лишь, что эти цифры различны, набрал их наугад. Найти вероятность того, что набраны нужные цифры.

Решение: А: абонент наугад набрал нужные цифры.

Число всех возможных исходов равно n =

Число исходов, благоприятствующих событию А, m = 1

Вероятность события А равна P(A) = m/ n = 1/90 = .... 

Ответ: 0,011.

б) Устройство содержит два независимо работающих элемента. Вероятности отказа элементов равны соответственно 0,05 и 0,08. Найти вероятности отказа устройства, если для этого достаточно, чтобы отказал хотя бы один элемент. 

Решение: Пусть событие А — «устройство не работает», В1 — «отказал первый элемент», 

В2 — « отказал второй элемент». Событие А соответствует тому, что может отказать один из «цементов либо оба элемента. События  В1 и В2  независимы в совокупности, поэтому:

q1 = 10,05 = 0,95,   q2 = 10,08 = 0,92. P(A) = 1 q1q2= 10,950,92 = 10,874 = ...

Ответ:  0,126.

Пример 4. a)Вероятность хотя бы одного попадания в цель при четырех выстрелах равна 0,9984. Найти вероятность попадания в цель при одном выстреле.

Решение: Пусть p - вероятность попадания в цель при одном выстреле. Введем событие 

X = {при четырех выстрелах есть хотя бы одно попадание} и противоположное ему событие  

= {при четырех выстрелах нет ни одного попадания}.

Вероятность события  равна P(  ) = (1p)4, тогда вероятность события Х равна 

P(X) =1P(  ) = 1 (1p)4. По условию эта вероятность равна 0,9984, откуда получаем уравнение относительно p: 1 (1p)4 = 0,9984, (1p)4 = 0,0016, (1p) = 0,2, p = ...

Таким образом, вероятность попадания в цель при одном выстреле равна 0,8.

Ответ: 0,8.

б)На полке в случайном порядке расставлено 40 книг, среди которых находится трехтомник Пушкина. Найти вероятность того, что эти тома стоят в порядке возрастания номера слева направо, но не обязательно рядом.

Решение: Используем классическое определение вероятности: P = m/n, где n- число всех равновозможных элементарных исходов, m - число элементарных исходов, благоприятствующих осуществлению события  A = (Тома стоят в порядке возрастания номера слева направо, но не обязательно рядом). n= 40⋅39⋅38 =59280, так как первый том можно поставить на любое из 40 мест, второй - на любое из 39 мест и третий - на любое из оставшихся 38 мест.

А число m= 40! / (37! 3!) = (40⋅39⋅38) : (1⋅2⋅3) = ...

Тогда искомая вероятность P(A)= m/n = 9880/59280 = 1/6. Ответ: 1/6.

Пример 5. а)В коробке имеется 250 лампочек, из них 100 по 90Вт, 50 - по 60Вт, 50 - по 25Вт и 50 – по 15Вт. Определить вероятность того, что мощность любой наугад взятой лампочки не превысит 60Вт.

Решение: 1. Рассматриваем следующие события: А = {мощность лампочки равна 90Вт}, вероятность Р(А) = 100/250 = 0,4; В = {мощность лампочки равна 60Вт}; С = {мощность лампочки равна 25Вт}; D = {мощность лампочки равна 15Вт}.

2. События А, В, С, D образуют полную систему, так как все они несовместны и одно из них обязательно наступит в данном опыте (выборе лампочки). Вероятность наступления одного из них есть достоверное событие, тогда Р (А)Р (В)Р (С)Р (D) = 1.

3. События {мощность лампочки не более 60Вт} (т.е. меньше или равна 60Вт), и {мощность лампочки более 60Вт} (в данном случае – 90Вт) являются противоположными. По свойству противоположных чисел Р (В)Р (С)Р (D) = 1Р (А).

4. Учитывая, что Р (В)Р (С)Р (D) = Р (ВСD), получим

Р (В СD) = 1Р (А) = 10,4 = ... Ответ: 0,6. 

б) Вероятность поражения цели первым стрелком при одном выстреле равна 0,7, а вторым стрелком – 0,9. Найти вероятность того, что 

1) цель будет поражена только одним стрелком; 2) цель будет поражена хотя бы одним стрелком.

Решение: 1. Рассматриваем следующие события:
А
1 = {первый стрелок поражает цель}, Р (А1) = 0,7 из условия задачи;
1 = {первый стрелок промахнулся}, при этом Р (А1)Р (1) = 1, поскольку А1 и А̄1 – противоположные события. Отсюда Р (1) = 10,7 = …;
А
2 = {второй стрелок поражает цель}, Р (А2) = 0,9 из условия задачи;
2 = {второй стрелок промахнулся}, при этом Р (2) = 10,9 = …

2. Событие А={цель поражена только одним стрелком} означает, что наступило одно из двух несовместных событий: либо А12, либо 1А2.
По правилу сложения вероятностей Р (А) = Р (А12) + Р (1А2).По правилу умножения вероятностей независимых событий:
Р (А12) = Р (А1)Р (2) = 0,70,1= 0,07; Р (1А2) = Р (1)Р (А2) = 0,30,9 = ...
Тогда Р (А)= Р (А12) Р (1А2) = 0,070,27 = ...

3. Событие B ={цель поражена хотя бы одним стрелком} означает, что либо цель поразил первый стрелок, либо цель поразил второй стрелок, либо цель поразили оба стрелка.

Событие = {цель не поражена ни одним стрелком} является противоположным событию В, а значит Р(В) = 1Р ().
Событие B̄ означает одновременное появление независимых событий 1 и 2, следовательно Р () = Р (12) = Р (1)Р (2) = 0,30,1 = 0,03. Тогда Р (В) = 1Р () = 10,03 = ...

Ответ: 1) 0,34; 2) 0,97.

3.Решить задачи ( по примерам):

  1. a)В партии из 100 деталей имеется 3 бракованных. Определить вероятность того, что, взятая наугад, деталь окажется стандартной.

б) Из 4 букв разрезной азбуки составлено слово «мама». Ребенок, не умеющий читать, рассыпал эти буквы, а затем собрал их в произвольном порядке. Найти вероятность того, что у него снова получится слово «мама».

  1. a)В коробке лежат 5 зеленых, 3 синих и 12 красных карандашей. Вычислить вероятность того, что взятый наугад карандаш будет, синим или зеленым.

б) В урне лежат шары, двузначные номера которых составлены из цифр 1,2,3,4. Какова вероятность вынуть шар с номером 123?

  1. a)Набирая номер телефона, абонент забыл последние 3 цифры и, помня лишь, что эти цифры различны, набрал их наугад. Найти вероятность того, что набраны нужные цифры.

б) Устройство содержит два независимо работающих элемента. Вероятности отказа элементов равны соответственно 0,04 и 0,09. Найти вероятности отказа устройства, если для этого достаточно, чтобы отказал хотя бы один элемент. 

  1. a)Вероятность хотя бы одного попадания в цель при четырех выстрелах равна 0,9919. Найти вероятность попадания в цель при одном выстреле.

б)На полке в случайном порядке расставлено 21 книга, среди которых находится трехтомник Пушкина. Найти вероятность того, что эти тома стоят в порядке возрастания номера слева направо, но не обязательно рядом.

  1. a)В коробке имеется 200 лампочек, из них 60 по 90Вт, 60 - по 60Вт, 40 - по 25Вт и 40 – по 15Вт. Определить вероятность того, что мощность любой наугад взятой лампочки не превысит 60Вт.

б) Вероятность поражения цели первым стрелком при одном выстреле равна 0,4, а вторым стрелком – 0,7. Найти вероятность того, что 

1) цель будет поражена только одним стрелком;2) цель будет поражена хотя бы одним стрелком.

  1. Экспедиция издательства отправила газеты в три почтовых отделения. Вероятность своевременной доставки газет в первое отделение равна 0,95, во второе – 0,9, в третье – 0,8. Найти вероятность того, что, хотя бы одно отделение получит газеты с опозданием.

  2. Автомобилист проезжает два поста дорожно-патрульной службы. Вероятность того, что его остановят на первом посту, равна 0,4, на втором – 0,1. Найти вероятность того, что автомобилиста остановят хотя бы на одном посту.

  3. Производится наблюдение за группой, состоящей из четырех однородных объектов. Каждый из них за время наблюдения может быть обнаружен или не обнаружен. Рассматриваются события:

D — обнаружено ровно два объекта; Е — обнаружено ровно три объекта;

F — обнаружены все четыре объекта. Указать, в чем состоит событие: D + Е + F

  1. В урне a – белых, b – черных, c – красных шаров. Вероятность какого события определяется формулой /

  2. Найти вероятность того, что при извлечении наугад одного шара из корзины, в которой находятся 2 белых, 3 зеленых и 5 красных шаров, извлеченный шар окажется зеленым.

4.Решить задачи :

  1. В урне находится 10 шаров, из них 6 белых и 4 черных шара. Вынули из урны 2 шара. Какова вероятность того, что оба шара - белые?

  2. В секретном замке на общей оси 4 диска, каждый из которых разделен на 5 секторов, на которых написаны различные цифры. Замок открывается, если диски установлены так, что цифры на них составляют определенное четырехзначное число. Найти вероятность того, что при произвольной установке дисков замок будет открыт.

  3. Батарея, состоящая из 10 орудий, ведет огонь по 15 кораблям неприятеля. Найти вероятность того, что все орудия стреляют: а) по одной цели; б) по разным целям/

  4. В партии из 10 деталей 7 стандартных. Найти вероятность того, что среди шести взятых наудачу деталей 4 стандартных.

  5. Набирая номер телефона, абонент забыл последние 3 цифры и, помня лишь, что эти цифры различны, набрал их на удачу. Найти вероятность того, что набраны нужные цифры.

  6. В ящике 50 годных и 16 дефектных деталей. Сборщик наудачу достает 8 деталей. Найти вероятность того, что среди них: а) нет дефектных; б) 3 дефектных.

  7. В группе из 30 студентов на контрольной работе 6 студентов получили «5», 10 студентов – «4», 9 студентов – «3», остальные – «2». Найти вероятность того, что 3 студента, вызванные к доске, получили по контрольной работе «2».

  8. В почтовом отделении имеются открытки 6 видов. Какова вероятность того, что среди 4 проданных открыток все открытки различны?

  9. Выбирают наугад число от 1 до 100. Определить вероятность того, что в этом числе не окажется цифры 3.

  10. По условию лотереи «Спортлото 5 из 36» участник, угадавший 4 цифры из 5, получает второй приз. Найдите вероятность такого выигрыша.

  11. Набирая номер телефона, абонент забыл одну цифру и набрал ее наудачу. Найти вероятность того, что набрана нужная цифра.

  12. В кармане 3 пятикопеечные монеты и 7 десятикопеечных монет. Наугад берется одна за другой две монеты. Вторая оказалась десятикопеечной. Определить вероятность того, что и первая десятикопеечная.

  13. Набирая номер телефона, абонент забыл последние две цифры и, помня лишь, что эти цифры различны, набрал их наудачу. Найти вероятность того, что набраны нужные цифры.

  14. Собрание, состоящее из 30 человек, среди которых 8 женщин, выбирает делегацию из 3 человек. Найти вероятность того, что в делегацию войдет одна женщина.

  15. Брошены две игральные кости. Найти вероятность того, что: а) сумма выпавших очков не превосходит семи; б) на обеих костях выпадет одинаковое число очков; в) произведение выпавших очков делится на 4; г) хотя бы на одной кости выпадет 6.

  16. Из 20 сбербанков 10 расположены за чертой города. Для обследования случайным образом отобрано 5 сбербанков. Какова вероятность того, что среди отобранных окажется 3 сбербанка в черте города?

  17. 8 шахматистов, среди которых 3 гроссмейстера, путем жеребьевки делятся на две подгруппы по 4 человека. Какова вероятность того, что два гроссмейстера попадут в одну подгруппу?

  18. В группе 10 юношей и 10 девушек. Для дежурства на вечере путем жеребьевки выделяют 5 человек. Какова вероятность того, что в число дежурных войдут: а) 5 юношей; б) 2 юноши и 3 девушки.

  19. Для постановки танца хореограф выбирает 8 человек. Определить вероятность того, что из выбранных можно составить 4 пары, если в танцевальной студии занимается 12 девочек и 8 мальчиков.

  20. Лотерея выпущена на общую сумму 1000000 рулей. Цена одного билета 50 рублей. Ценные выигрыши падают на каждый десятый билет. Определить вероятность выигрыша при покупке: а) одного билета; б) двух билетов.



ПЗ № 14. Вычисление вероятностей. Прикладные задачи. Представление числовых данных. Прикладные задачи.

1)Опорный конспект.

Определение . Пусть событие А может произойти только совместно с одним из событий Н1, Н2,…, Нп, образующих полную группу несовместных событий. Тогда события Н1, Н2,…, Нп называются гипотезами

Теорема . Вероятность события А, наступающего совместно с гипотезами Н1, Н2,…, Нп, равна: где p(Hi) – вероятность i- й гипотезы, а p(A/Hi) – вероятность события А при условии реализации этой гипотезы. Формула (3.1) носит название формулы полной вероятности.

Пример. Имеются три одинаковые урны с шарами. В первой из них 3 белых и 4 черных шара, во второй – 2 белых и 5 черных, в третьей – 10 черных шаров. Из случайно выбран-ной урны наудачу вынут шар. Найти вероятность того, что он белый.

Решение. Будем считать гипотезами Н1, Н2 и Н3 выбор урны с соответствующим номером. Так как по условию задачи все гипотезы равновозможны, то Найдем условную вероятность А при реализации каждой гипотезы: Тогда

Формула Байеса (теорема гипотез).

Пусть известен результат опыта, а именно то, что произошло событие А. Этот факт может изменить априорные (то есть известные до опыта) вероятности гипотез. Например, в предыдущем примере извлечение из урны белого шара говорит о том, что этой урной не могла быть третья, в которой нет белых шаров, то есть р (Н3/А) = 0. Для переоценки вероятностей гипотез при известном результате опыта используется формула Байеса:

Пример. После двух выстрелов двух стрелков, вероятности попаданий которых равны 0,6 и 0,7, в мишени оказалась одна пробоина. Найти вероятность того, что попал первый стрелок.

Решение. Пусть событие А – одно попадание при двух выстрелах, а гипотезы: Н1 – первый попал, а второй промахнулся, Н2 – первый промахнулся, а второй попал, Н3 – оба попали, Н4 – оба промахнулись. Вероятности гипотез: р(Н1) = 0,6·0,3 = 0,18, р(Н2) = 0,4·0,7 = 0,28, р(Н3) = 0,6·0,7 = 0,42, р(Н4) = 0,4·0,3 = 0,12. Тогда р(А/Н1) = р(А/Н2) = 1, р(А/Н3) = р(А/Н4) = 0. Следовательно, полная вероятность р(А) = 0,18·1 + + 0,28·1 + 0,42·0 + 0,12·0 = 0,46. Применяя формулу Байеса, получим Закон распределения дискретной случайной величины.
Как известно, случайной величиной называется переменная величина, которая может принимать те или иные значения в зависимости от случая. Случайные величины обозначают заглавными буквами латинского алфавита (X, Y, Z), а их значения – соответствующими строчными буквами (x, y, z). Случайные величины делятся на прерывные (дискретные) и непрерывные.
Дискретной случайной величиной называется случайная величина, принимающая лишь конечное или бесконечное (счетное) множество значений с определенными ненулевыми вероятностями.
Функция, связывающая значения случайной величины с соответствующими им вероятностями, называется
 законом распределения дискретной случайной величины. Закон распределения может быть задан одним из следующих способов.

  1. Закон распределения может быть задан таблицей:

Значения xi

x1

x2

x3

...

xn

Вероятности pi

p1

p2

p3

...

pn


События X = xi (i = 1, 2, 3,…,n) являются несовместными и единственно возможными, т.е. они образуют полную систему событий. Поэтому сумма их вероятностей равна единице: р123+…+рn = ∑pi =1

2. Закон распределения может быть задан аналитически (формулой) P(X = xi) = ϕ(xi). Например:а) с помощью биномиального распределения: Pn(X=k) = Сnk pk qn-k, 0<р<1, k = 0, 1, 2, …, n;

б) с помощью распределения Пуассона: hello_html_mc1ab25c.jpg

где λ>0, k = 0, 1, 2, … .

в) с помощью функции распределения F(x), определяющей для каждого значения x вероятность того, что случайная величина X примет значение, меньшее x, т.е. F(x) = P(X < x).

- свойства функции F(x)

3. Закон распределения может быть задан графически – многоугольником (полигоном) распределения

Основные числовые характеристики дискретной случайной величины:

Mатематическое ожидание (среднее значение) дискретной случайной величины M(X)=Σ xipi.
Для биномиального распределения M(X)=np, для распределения Пуассона M(X)=λ

Дисперсия дискретной случайной величины D(X)= M[X–M(X)]2 или D(X) = M(X2)−[M(X)]2. Разность X–M(X) называют отклонением случайной величины от ее математического ожидания.
Для биномиального распределения D(X)=npq, для распределения Пуассона D(X)=λ

Среднее квадратическое отклонение (стандартное отклонение) σ(X)=√D(X).

Задача 1.

Найти математическое ожидание числа очков, выпадающих при бросании игральной кости.

Решение. Случайная величина X числа очков принимает значения 1, 2, 3, 4, 5, 6. Вероятность того, что выпадет одно из данных значений равна 1/6. Закон распределения представим в виде таблицы:


Значения xi

1

2

3

4

5

6

Вероятности pi

1/6

1/6

1/6

1/6

1/6

1/6

Найдем математическое ожидание величины Х:

М(Х) = 1*1/6 + 2*1/6 + 3*1/6 + 4*1/6 + 5*1/6 + 6*1/6 = (1+2+3+4+5+6)/6 = 21/6 = 3,5

Задача 2.

Выпущено 1000 лотерейных билетов: на 5 из них выпадает выигрыш в сумме 500 рублей, на 10 – выигрыш в 100 рублей, на 20 – выигрыш в 50 рублей, на 50 – выигрыш в 10 рублей. Определить закон распределения вероятностей случайной величины X – выигрыша на один билет.

Решение. По условию задачи возможны следующие значения случайной величины X: 0, 10, 50, 100 и 500.

Число билетов без выигрыша равно 1000 – (5+10+20+50) = 915, тогда P(X=0) = 915/1000 = 0,915.

Аналогично находим все другие вероятности: P(X=0) = 50/1000=0,05, P(X=50) = 20/1000=0,02, P(X=100) = 10/1000=0,01, P(X=500) = 5/1000=0,005. Полученный закон представим в виде таблицы:

Значения xi

0

10

50

100

500

Вероятности pi

0,915

0,05

0,02

0,01

0,005


2) Перепишите и заполните пропуски:

Пример 1. В пирамиде 5 винтовок, три из которых снабжены оптическим прицелом. Вероятность того, что стрелок поразит мишень при выстреле из винтовки с оптическим прицелом, равна 0,95; для винтовки без оптического прицела эта вероятность равна 0,7. Найти вероятность того, что мишень будет поражена, если стрелок производит один выстрел из наудачу взятой винтовки.

Решение: по классическому определению:    – вероятности  выбора винтовки с оптическим и без оптического прицела соответственно.

По условию,  – вероятности попадания в мишень из соответствующих типов винтовок.

По формуле полной вероятности:
– вероятность того, что стрелок поразит мишень из наугад выбранной винтовки.

Ответ: 0,85.

Пример 2. Число грузовых машин, проезжающих мимо бензоколонки, относится к числу проезжающих там же легковых машин как 3:2. Вероятность того, что будет заправляться грузовая машина, равна 0,1; для легковой машины эта вероятность равна 0,2. На заправку подъехала машина. Найдите вероятность того, что подъехавшая машина грузовая.
Решение. Пусть событие А состоит в том, что проезжающая машина остановилась на заправку, а гипотезы Н1 и H2 соответственно означают, что проезжающая машина грузовая или легковая. Нам нужно найти вероятность Р(Н1/А). Из условия следует, что Р(Н1) = 0,6, Р(Н2) = 0,4, Р(А/Н1) = 0,1 и Р(А/Н2) = 0,2. Сначала по формуле полной вероятности находим Р(А) = 0,6 * 0,1 + 0,4 * 0,2 = ... Далее применяем формулу Байеса:
Р(Н1/А)= (Р(А/Н1)*Р(Н1))/Р(А)= (0,1*0,6)/0,14 = 6/14= … ≈ 0,4286.

Пример 3. На склад поступило 2 партии изделий: первая – 4000 штук, вторая – 6000 штук. Средний процент нестандартных изделий в первой партии составляет 20%, а во второй – 10%. Наудачу взятое со склада изделие оказалось стандартным. Найти вероятность того, что оно: а) из первой партии, б) из второй партии.

Первая часть решения состоит в использовании формулы полной вероятности. Иными словами, вычисления проводятся в предположении, что испытание ещё не произведено и событие «изделие оказалось стандартным» пока не наступило.

Рассмотрим две гипотезы:
 – наудачу взятое изделие будет из 1-й партии;
 – наудачу взятое изделие будет из 2-й партии.

Всего: 4000 + 6000 = … изделий на складе. По классическому определению:
. Контроль:
 

Рассмотрим зависимое событие: hello_html_m518765a0.gif – наудачу взятое со склада изделие будет стандартным.

В первой партии 100% – 20% = 80% стандартных изделий, поэтому:  – вероятность того, что наудачу взятое на складе изделие будет стандартным при условии, что оно принадлежит 1-й партии.

Аналогично, во второй партии 100% – 10% = 90% стандартных изделий и- вероятность того, что наудачу взятое на складе изделие будет стандартным при условии, что оно принадлежит 2-й партии.

По формуле полной вероятности:
 – вероятность того, что наудачу взятое на складе изделие будет стандартным.

Часть вторая. Пусть наудачу взятое со склада изделие оказалось стандартным. Эта фраза прямо прописана в условии, и она констатирует тот факт, что событие hello_html_m518765a0.gif произошло.

По формулам Байеса:

а)   – вероятность того, что выбранное стандартное изделие принадлежит 1-й партии;

б)  – вероятность того, что выбранное стандартное изделие принадлежит 2-й партии.

После переоценки гипотезы, , разумеется, по-прежнему образуют полную группу:
 
(проверка)

Ответ:a) б)   


Пример 4. Выпущено 1000 лотерейных билетов: на 5 из них выпадает выигрыш в сумме 500 рублей, на 10 – выигрыш в 100 рублей, на 20 – выигрыш в 50 рублей, на 50 – выигрыш в 10 рублей. Определить закон распределения вероятностей случайной величины X – выигрыша на один билет.

Решение: По условию задачи возможны следующие значения случайной величины X:

0, 10, 50, 100 и 500.

Число билетов без выигрыша равно 1000 – (5+10+20+50) = …, тогда P(X=0) = 915/1000 = ...

Аналогично находим все другие вероятности: P(X=0) = 50/1000=…, P(X=50) = 20/1000=…,

P(X=100) = 10/1000=…, P(X=500) = 5/1000=... Полученный закон представим в виде таблицы:


Вероятности pi

0,915

0,05

0,02

0,01

0,005


Пример 5. a)Закон распределения случайной величины X имеет вид:

X

1

0

1

2

P

0,1

0,2

0,3

0,4

Вычислить Dx   и Ϭx .

Решение: Найдем вначале математическое ожидание случайной величины X:

Mx = .

Вычислим дисперсию Dx :Dx = .

Тогда среднее квадратическое отклонение: Ϭx = .

Ответ: Dx = 1, Ϭx = 1.

б) Закон распределения случайной величины X имеет вид:

X

0

1

2

P

0,1

0,2

x

Найти x. Составить функцию распределения. Вычислить: P{X > 0,7} , Mx , Dx и Ϭx .

Решение. Согласно условию нормировки имеем уравнение: 0,1   Отсюда x = 0,7 . Далее, воспользовавшись рядом распределения, найдем:

P{X > 0,7} = P {X = 1}P{X = 2} = 0,2 0,7 = …; Mx =

Dx = ; Ϭx = .

Ответ: x = 0,7 ; P{X > 0,7} = 0, 9; Mx Dx ; Ϭx

Пример 6. a)Известно, что случайная величина X, принимающая два значения  x1 = 2 и x2 = 3 , имеет математическое ожидание, равное 2,2. Построить ряд распределения случайной величины X, найти дисперсию, среднее квадратическое отклонение и составить функцию распределения.

Решение. Пусть P{X = 2} = p . Тогда, согласно условию нормировки,P{X = 3} = 1  . Используя определение математического ожидания, получим Mx = 2p . Имеем уравнение 3 , откуда находим p = 0,8 . Ряд распределения имеет вид:

X

2

3

P

0,8

0,2

Теперь вычислим дисперсию и среднее квадратическое отклонение:

Dx = ; Ϭx =  .

Согласно определению функция распределения имеет вид

Fx(x) =

Ответ: Dx ; Ϭx =   Fx(x) =

б) Возможные значения случайной величины X таковы: x1 = 2 , x2 = 3, x3 = 3 . Известно, что Mx = 2,3 ,α2 = 5,9 . Найти вероятности, соответствующие возможным значениям X, и записать ряд распределения.

Решение. Ряд распределения, с учетом возможных значений случайной величины X, будет выглядеть следующим образом:

X

1

2

3

P

p1

p2

p3

Найдем вероятности p1 , p2 и p3, соответствующие возможным значениям X.

По условию Mx = 2,3 , поэтому имеем первое уравнение, связывающее p1p2 и p3 :

 . Аналогично из условия α2 = 5,9   получим второе уравнение:

 . Третье уравнение возникает из условия нормировки:

p1 p2 p3 = 1. Итак, имеем систему:


Ответ: ряд распределения имеет вид

X

1

2

3

P

0,2

0,3

0,5

3)Решить задачи :

  1. На склад поступают телефоны трех заводов, причем доля телефонов первого завода составляет 25%, второго - 60%, третьего - 15%. Известно также, что средний процент бракованных телефонов для первой фабрики составляет 2%, второй - 4%, третьей - 1%. Найти вероятность того, что наудачу выбранный телефон бракованный.

  2. В офисе: 4 ноутбука изготовлены компанией A, 6- компанией B, 8 -компанией  C и 2 - компанией D . Гарантии, что ноутбуки этих компаний будут работать в течение гарантийного срока без ремонта составляют 70%, 80%, 85%, и 55% для каждой из них. Найти вероятность того, что выбранный ноутбук будет работать без ремонта в течение гарантийного срока.

  3. Экспортно-импортная фирма собирается заключить контракт на поставку оборудования в одну из развивающихся стран. Если основной конкурент фирмы не станет одновременно претендовать на заключение контракта, то вероятность получения контракта оценивается в 0,45; в противном случае — в 0,25. По оценкам экспертов компании вероятность того, что конкурент выдвинет свои предложения по заключению контракта, равна 0,40. Чему равна вероятность заключения контракта для этой фирмы?

  4. Каждому из 3 первоклассников - Пете, Коле и Мише - предложили одинаковое количество загадок. Петя отгадывает в среднем 3 загадки из 4. Коля 5 из 6. Миша 9 из 10. Наугад выбранный школьник не отгадал загадку. Какова вероятность того, что это был Коля?

  5. На каждые 100 электрических ламп завода «А» в среднем приходится 83 стандартных, завода «В» - 63 стандартных. В магазин поступает 70% лампочек с завода «А» и 30% - с завода «В». Купленная лампочка оказалась стандартной. Найти вероятность того, что лампочка изготовлена на заводе «А».

  6. В студенческой группе 70% - юноши. 20% юношей и 60% девушек имеют сотовый телефон. После занятий в аудитории был найден кем-то забытый телефон. Найти вероятность того, что он принадлежал юноше

  7. Сотрудники отдела маркетинга полагают, что в ближайшее время ожидается рост спроса на продукцию фирмы. Вероятность этого они оценивают в 80%. Консультационная фирма, занимающаяся прогнозом рыночной ситуации, подтвердила предположение о росте спроса. Положительные прогнозы консультационной фирмы сбываются с вероятностью 95%, а отрицательные – с вероятностью 99%. Какова вероятность того, что положительный прогноз сбудется? 

  8. Имеются три одинаковые урны. В первой урне находятся 4 белых и 7 черных шаров, во второй – только белые и в третьей – только черные шары. Наудачу выбирается одна урна и из неё наугад извлекается шар. Какова вероятность того, что этот шар чёрный?

  9. Двигатель работает в трёх режимах: нормальном, форсированном и на холостом ходу. В режиме холостого хода вероятность его выхода из строя равна 0,05, при нормальном режиме работы – 0,1, а при форсированном – 0,7. 70% времени двигатель работает в нормальном режиме, а 20% – в форсированном. Какова вероятность выхода из строя двигателя во время работы?

  10. В магазине три холодильника в которых заканчивается мороженое. В первом 4 белых  и 6 шоколадных, во втором - 2 белых и 8 шоколадных, в третьем - 3 белых и 7 шоколадных. Наугад выбирают холодильник и вынимают из него мороженое. Определить: a) вероятность того, что оно белое, б) вероятность того, что мороженное извлекли из второго холодильника.

  11. Число грузовых машин, проезжающих мимо бензоколонки, относится к числу проезжающих там же легковых машин как 3:2. Вероятность того, что будет заправляться грузовая машина, равна 0,2; для легковой машины эта вероятность равна 0,3. На заправку подъехала машина. Найдите вероятность того, что подъехавшая машина грузовая.

  12. На склад поступают телефоны трех заводов, причем доля телефонов первого завода составляет 25%, второго - 60%, третьего - 15%. Известно также, что средний процент бракованных телефонов для первой фабрики составляет 2%, второй - 4%, третьей - 1%. Найти вероятность того, что:  а) наугад взятый телефон окажется с браком; б) телефон изготовлен на первом заводе, если он бракованный;  в) на каком заводе скорее был изготовлен телефон, если он сделан качественно ?

  13. Выпущено 200 лотерейных билетов: на 5 из них выпадает выигрыш в сумме 500 рублей, на 10 – выигрыш в 100 рублей, на 20 – выигрыш в 50 рублей, на 40 – выигрыш в 10 рублей. Определить закон распределения вероятностей случайной величины X – выигрыша на один билет.

  14. a)Закон распределения случайной величины X имеет вид:

X

1

0

1

2

P

0,1

0,15

0,3

0,45

Вычислить Dx и Ϭx . б) Закон распределения случайной величины X имеет вид:

X

0

1

2

P

0,2

0,3

x

Найти x. Составить функцию распределения. Вычислить: P{X > 0,7} , Mx , Dx и Ϭx .

  1. a)Известно, что случайная величина X, принимающая два значения  x1 = 2 и x2 = 3 , имеет математическое ожидание, равное 2,4. Построить ряд распределения случайной величины X, найти дисперсию, среднее квадратическое отклонение и составить функцию распределения.

б) Возможные значения случайной величины X таковы: x1 = 2 , x2 = 3, x3 = 3 . Известно, что Mx = 2,5 ,α2 = 6,7 . Найти вероятности, соответствующие возможным значениям X, и записать ряд распределения.

  1. Вероятность того, что телевизор имеет скрытые дефекты, равна 0,2. На склад поступило 20 телевизоров. Какое событие вероятнее: что в этой партии имеется два телевизора со скрытыми дефектами или три?

  2. В работе телефонной станции происходят в среднем 3 сбоя в час. Определить вероятность хотя бы одного сбоя за 1 час.

  3. В честь национального праздника состоялся массовый забег на дистанцию 10 км. В забеге приняли участие 250 человек. Обычно в забегах такого типа из каждых десяти участников 8 доходят до финиша. Какова вероятность того, что до финиша дойдут 200 человек?

  4. Радиотелеграфная станция передает цифровой текст. В силу наличия помех каждая цифра независимо от других может быть неправильно принята с вероятностью 0,01. Найдите вероятность того, что в принятом тексте, содержащем 1100 цифр, будет менее 20 ошибок.

  5. Предприятие имеет проблемы с поставками сырья. Вероятность того, что в каждом отдельном месяце предприятие будет полностью обеспечено сырьем, равна 0,9. Какова вероятность того, что за полугодовой период предприятие будет полностью обеспечено сырьем:

а) ровно в трех месяцах? б) не менее чем в двух месяцах?

  1. В работе телефонной станции происходят в среднем 3 сбоя в час. Определить вероятность пяти сбоев за 2 часа.

  2. Радиотелеграфная станция передает цифровой текст. В силу наличия помех каждая цифра независимо от других может быть неправильно принята с вероятностью 0,01. Найдите вероятность того, что в принятом тексте, содержащем 1100 цифр, будет 15 ошибок.

  3. В честь национального праздника состоялся массовый забег на дистанцию 10 км. В забеге приняли участие 250 человек. Обычно в забегах такого типа из каждых десяти участников 8 доходят до финиша. Какова вероятность того, что до финиша дойдут от 180 до 220 человек?

  4. Всхожесть семян данного растения составляет 90%. Найти вероятность того, что из 800 посеянных семян взойдёт не менее 700.













Просмотрено: 0%
Просмотрено: 0%
Скачать материал
Скачать материал "ПЗ по специальности Технология машиностроения,1 курс(2016-2017 уч.г.)"

Получите профессию

HR-менеджер

за 6 месяцев

Пройти курс

Рабочие листы
к вашим урокам

Скачать

Выбранный для просмотра документ ПЗ№15.docx

ПЗ № 15. Арифметические действия над числами, нахождение приближенных значений величин и погрешностей вычислений (абсолютной и относительной), сравнение числовых выражений.

1)Опорный конспект.

В процессе вычислений весьма часто приходится иметь дело с приближенными числами. Пусть А точное значение некоторой величины, называемое в дальнейшем точным числом А. Под приближенным значением величины А, или приближенным числам, называется число а, заменяющее точное значение величины А. Если а < А, то а называется приближенным значением числа А по недостатку. Если а > А, — то по избытку. Например, 3,14 является приближенным значением числа π по недостатку, а 3,15 — по избытку. Для характеристики степени точности данного приближения пользуются понятием погрешности или ошибки.

Погрешностью Δа приближенного числа а называется разность вида

Δа = А — а, (1.1)

где А — соответствующее точное число.

Определение. Абсолютной погрешностью А приближенного числа а называется абсолютная величина погрешности этого числа

Δ = |А — а|. (1.2)

В силу того, что точное число А, как правило, неизвестно, то пользуются понятием предельной абсолютной погрешности.

Определение. Предельной абсолютной погрешностью Δa приближенного числа а называется число, не меньшее абсолютной погрешности этого числа, т. е.

Δa ≥ Δ. (1.3)

Из (1.3) имеем

Δa |А — а|,

следовательно,

а - Δa £ А £ а + Δa, (1.4)

т. е. а - Δa является приближением числа А по недостатку, а а + Δa — приближением числа А по избытку. Формулу (1.4) кратко записывают в виде А = а ± Δa.

На практике под точностью измерений обычно понимают предельную абсолютную погрешность. Например, если расстояние между двумя пунктами, равное S = 900 м, получено с точностью до 0,5 м, то точное значение величины S заключено в границах 899,5 м £ S £ 900,5 м.

Введение абсолютной или предельной абсолютной погрешностей совершенно недостаточно для характеристики степени точности приближенных чисел. Существенным показателем точности приближенных чисел является их относительная погрешность.

Определение. Относительной погрешностью δ приближенного числа а называется отношение абсолютной погрешности Δ этого числа к модулю соответствующего точного числа А (А ¹ 0)

. (1.5)

Определение. Предельной относительной погрешностью приближенного числа а называется число δа не меньшее относительной погрешности этого числа, т. е.

δаδ. (1.6)

Из (1.6) имеем Δ £ |А|δа

Следовательно, можно считать, что предельная абсолютная погрешность числа а равна

Δа £ |А|δа. (1.7)

Если принять А » а, то формула (1.7) примет вид

Δа £ |а|δа. (1.8)

Следовательно, точное число А лежит в следующих границах:

а(1 - δа) £ А £ а(1 + δа).

Формула (1.8) позволяет определять предельную абсолютную погрешность по заданной предельной относительной погрешности и наоборот.

Пример 1. Округляя точные числа hello_html_m518765a0.gif до трех значащих цифр, определить абсолютную hello_html_m120caa96.gif и относительнуюhello_html_7030a9e7.gif погрешности полученных приближенных чисел.

Дано: hello_html_m600c8ad6.gif hello_html_7d22ad7.gif . Найти: hello_html_m171d5a2f.gif

Решение:

hello_html_5972f99.gif- приближенное значение числа A

Абсолютная погрешность: hello_html_5dc3a3d6.gif

Относительная погрешность: hello_html_79e9a63b.gif

Ответ: hello_html_c7ee83f.gif; hello_html_m69af9963.gif

Пример 2. Определить абсолютную погрешность приближенных чисел hello_html_31311e32.gif по их относительной погрешности hello_html_7030a9e7.gif.

Дано: hello_html_5716c6a.gif hello_html_m652ddba.gif. Найти: hello_html_m120caa96.gif

Решение:

Абсолютная погрешность: hello_html_m116fefef.gif

Ответ: hello_html_7262f7c7.gif

Пример 3 . При измерении длины с точностью до 5 м получено hello_html_m6a0990d6.gif км, а при определении другой длины с точностью до 0.5 см, получено hello_html_m4a535580.gif метров. Какое измерение по своему качеству лучше?

Дано: hello_html_m1623fc6.gifКм, hello_html_m56eb8e11.gifМ, hello_html_mb27ab20.gifМ, hello_html_m175c9b8b.gifСм

Сравнить: hello_html_50409580.gif и hello_html_17d9b7cc.gif

Решение: Итак, по 1-му измерению, результат hello_html_m1623fc6.gifКм = hello_html_m2ee8bf0.gifМ с точностью до hello_html_mb27ab20.gifМ (hello_html_m2cafe90a.gif - абсолютная погрешность величины hello_html_m6a0990d6.gif).

Тогда относительная погрешность: hello_html_293aab59.gif%

По 2-му измерению, результат hello_html_m56eb8e11.gifКм с точностью до hello_html_m175c9b8b.gifСм =hello_html_3b05be7.gifМ (hello_html_7cc15c1e.gif - абсолютная погрешность величины hello_html_m4a535580.gif).

Тогда относительная погрешность: hello_html_m587113e.gif%

Так как hello_html_5928e3a2.gif, то измерение hello_html_m4a535580.gif можно считать по качеству лучше, чем hello_html_m6a0990d6.gif.

Ответ: измерение hello_html_m4a535580.gif по качеству лучше, чем hello_html_m6a0990d6.gif.

2)Решить задание:

А)Вычислить:

  1. а) (– 2,35 – 4,65) · 5 : (16,9 – 2,9),
    б) (7,63 + (– 5,13)) · 0,4 : (3,17 + 6,83),

  2. а) 30,3 · (124,9 – (48,96 : 6,8 + 36,04) : 9,2),

б) 73, 2 · 48, 3 – 37,4 · (166,02 + 219,38) : 1,64,

  1. а) 3,44 : 0,4 + 24,56 , б) 684 · 245 – 675 · 246,

  2. а) (93 · 7 + 141) : 72 , б) 7091 + 9663 – (243916 + 75446) : 527 : 3,

в) (15,964 · 5,2 – 12) · 0,1 , г) (96,6 + 98,6) : 6,4 · 1,2 – 0,2,

  1. а) ((27,12+ 43,08) · 0,007 – 0,0314) · 100,
    б) 1,53 · 54 – 0,42 · (512 – 491,2) + 1,116,

в) (867000 : 2125 – 396,4) · 2,15,

  1. а) 51,6 + (70,2 – 4,4 · (73,73 : 7,3)) · 1,6,
    б) 18,305 : 0,7 – 0,0368 : 0,4 + 0,492 : 1,2,

в) (0,6739 + 1,4261) · 557, 55 : (16,7 · 2,9 – 42,13),

г) 702,3 – (59 – 389,64 : 6,8) · (59,3 – 5,64 : 9,4),

  1. а) 316219 – (27090 : 43 +16422 : 119), б) 565,3 – 465,3 : ((1,25 + 5,8) · (55,8 – 49,2)),

в) 74 : 100 – 0,4 : 10 + 17,8 : 1000, г) 0,35 · 10 + 0,0237 · 100 – 0,00087 · 1000,

  1. а) 0,7 : 0,1 + 0,0474 : 0,01 – 0,00174 : 0,001, б) 12,3 + 7,7 · 187,2 : 4,5 : 6,4 – 3,4,

в) 10,1 + 9,9 · 107,1 : 3,5 · 6,8 – 4,85, г) 37 · 0,01 – 0,2 · 0,1 + 8,9 · 0,001.

  1. Найди значения выражений:
    а) (18370+23679):7, 156-96:(12:4):2, б) (800035 784942)∙6,

в) 98560:7 ,83216:4, 8656:4 ,91620:4, г) 73170:9 ,3726:9 ,91728:9, 705355:5.

  1. Найди значения выражений:
    а) (10283+16789):9, 5∙(125+75):20+80, б) (200496 134597)∙2,

в) 54663:7, 80395:5, 6543:9, 860073:3, г) 1836:4,7542:9, 3906:6, 9150:3,

д)795 ·504 248.952:492,

  1. Реши примеры на деление:

114595 : 215 =

200064 : 384 =

404758 : 922 =

5370 : 358 =

396204 : 548 =

263082 : 978 =

181116 : 387 =

118956 : 276 =

115419 : 487 =

140070 : 435 =

223925 : 689 =

420210 : 435 =


  1. а)1098 + (1453 – 564) · 176  + 195 539– 352 004,

б)30257 · 8 + 7 280400 · 5 5 897 · 6 3504: 8.

В)

  1. Найдите значение выражения:

а) 48 – 29 + 37 – 19;  б) 156 + 228 – 193 – 66; в) 39 • 45 : 65 • 2;  г) 1024 : 128 • 15 : 10; 
д) 245 : 7 – 224 : 16 + 35 • 11;  е) 322 : 23 • 70 – 161 • 9 : 69; 

  1. а) 315 : (162 + 12 • 24 - 11 • 39) + 558 : 31;  б) (24 • 7 - 377 : 29) • (2378 : 58 – 38); 
    в) (120 + 16 • 7) • 240 : (300 – 5 • 44);  г) (372 + 118 • 6) : (38 • 35 – 34 • 37) - 12; 
    д) 3124 : (3 • 504 – 4 • 307) + 10 403 : 101;  е) 15 + (12 322 : (24 + 37) – 12 • 15) : (35 • 2 – 59).

  2. Измените порядок действий на основании свойств сложения, вычитания и умножения для удобства вычислений:

а) 348 + 54 + 46;                      г) 54 • 2 • 50; 
б) 543 + 89 – 43;                       д) 34 • 8 + 66 • 8; 
в) 427 – 33 – 67;                        е) 135 • 12 – 35 • 12.

  1. Выполните действия по схеме .

hello_html_m7f78617e.jpg

  1. Найдите частное:

а) 1 989 680 : 187;                            в) 9 018 009 : 1001; 
б) 572 163 : 709;                               г) 533 368 000 : 83 600.

С) ТЕСТ.


Найти абсолютную погрешность приближения числа 3,9 числом 4

1) 0,9; 2) – 0,9; 3) 0,1; 4) – 0,1

  1. Какое из чисел является более точным приближением числа 3,464 ?

1) 3

2) 3,4

3) 3,6

4) 4

  1. Записать оценку величины n в виде двойного неравенства, если n = 0,385 0,001

1) 0,384 0,386

2) 0,3860,384

3) 0,384 0,386

4) 0,384

  1. Округлить число 734,256 до десятых

1) 734,2 2) 734,3 3) 730 4) 734,26

  1. Найти относительную погрешность приближения числа числом 0,14

  1. Записать число 0,00018 в стандартном виде

  1. Найти значение выражения ( 2,5 . 103 ) : ( 5 . 10 – 2 )


D)

1.Округляя точные числа hello_html_m518765a0.gif до трех значащих цифр, определить абсолютную hello_html_m120caa96.gif и относительнуюhello_html_7030a9e7.gif погрешности полученных приближенных чисел.

Дано: А= 0,1766 , hello_html_7d22ad7.gif . Найти: hello_html_m171d5a2f.gif.

2. Определить абсолютную погрешность приближенных чисел hello_html_31311e32.gif по их относительной погрешности hello_html_7030a9e7.gif.

Дано: а =\ 4,782, hello_html_m652ddba.gif. Найти: hello_html_m120caa96.gif

3.Вычислить:

а)

б)


в)

hello_html_1598fcc6.gif

г)

hello_html_346bb90f.gif

д)

hello_html_2f57140e.gif




Просмотрено: 0%
Просмотрено: 0%
Скачать материал
Скачать материал "ПЗ по специальности Технология машиностроения,1 курс(2016-2017 уч.г.)"

Получите профессию

Секретарь-администратор

за 6 месяцев

Пройти курс

Рабочие листы
к вашим урокам

Скачать

Выбранный для просмотра документ ПЗ№16.docx

ПЗ № 16. Вычисление и сравнение корней. Выполнение расчетов с радикалами.

Задание:

1) Перепишите и заполните пропуски:
А)Пример 1. Вычислите: . Решение: .

Ответ: 1.

Пример 2. Вычислите: .

Решение: .

Ответ: 0,2.

Пример 3. Упростите выражение: .

Решение: .

Ответ: 3.

Пример 4. Вычислите: а) б)

в) г)д)

е) ж)

з)

Ответ: а) 6, б) hello_html_15da1ba3.gif8, в) 3,5; г) 40, д) 2, е) 10, ж) 1,25; з) 250.

Пример 5. Выполнить действия: Решение: Ответ:

Пример 6. Решите уравнения:

Решение: Ответ: а) х = 4, б) х = 2.

Пример 7. а) Вынести множитель из-под знака корня: ,

внесите множитель под знак корня: .

Ответ: а) б) .

Пример 8. Сравните числа .

.

Пример 9.: Вычислите:

Чтобы выполнить вычисление, нужно преобразовать числитель, для этого во второй скобке представим составные числа в виде простых:

Получаем:

После преобразований получаем дробь:

Несложно заметить в полученном выражении формулу разности квадратов, свернем ее:


Пример 10. а)Вычислите: .

Сначала вычислим внутренний корень:


После преобразования получили выражение:


б) упростить выражение: .

Выделяем полный квадрат:

Получаем:

Комментарий: число  отрицательное, имеем право раскрыть модуль.

В)

  1. Упростите иррациональные выражения:

1. 2. 3.

4.,5.,6. ,

7. ,8.

  1. Вычислите:

а)

б)

  1. Найдите значение выражения:

а)

б)

в)

  1. Вычислите:

а) , б) ,

в) , г)

д)

е)

ж)

  1. Найти значение выражения:

  2. Упростите выражение:


  1. Разложите на множители:

а) , б) ,

в) ,

  1. Сократите дробь:

а)

б)

в)

2)Решить задание ( по примерам):

  1. Вычислите: .

  2. Вычислите: .

  3. Упростите выражение: .

  4. Вычислите:

а) б) в) г)д)

е) ж) з)

  1. Выполнить действия:

  2. Решите уравнения:

  3. а) Вынести множитель из-под знака корня: ,

внесите множитель под знак корня: .

3)Решить задание :

  1. Вычислите: а) ,б) ,в) ,г) ,д) ,

е) ,ж) , з) .

  1. Решите уравнения: а) , б) , в) , г)

  2. Упростите выражение: .

  3. Вычислите: а) б) в)

  4. а) вынести множитель из-под знака корня:

б) внесите множитель под знак корня:

  1. Вычислите а) ,б) ,в) г) ,д) .

  2. Упростите выражение


  1. Найдите значения выражения при у = 16.

  2. Найдите значение выражения:

  3. Сократите дробь: .

  4. Найдите значения выражения при а = 4, b = 5.

  5. Найдите значение выражения при р = 49.

  6. Упростите выражение .

  7. Упростите выражение .

  8. Вычислите

  9. Упростите выражение .

  10. Найдите значение выражения:

  11. Найдите значение выражения: .

  12. Упростите выражение .

  13. Упростите выражение


Просмотрено: 0%
Просмотрено: 0%
Скачать материал
Скачать материал "ПЗ по специальности Технология машиностроения,1 курс(2016-2017 уч.г.)"

Получите профессию

Секретарь-администратор

за 6 месяцев

Пройти курс

Рабочие листы
к вашим урокам

Скачать

Выбранный для просмотра документ ПЗ№17,18.docx

ПЗ № 17. Нахождение значений степеней с рациональными показателями. Сравнение степеней.

Задание:

1)Перепишите:

Определение. Степенью числа с рациональным показателем , где m-целое число, а n-натуральное (), называется число . Итак, по определению .

Свойства степени с рациональным показателем,

где r,s-рациональные числа, ,.





Замечание. При рациональная степень числа а не определяется.

Пример 1.

Пример 2. Сравните числа .

.

Пример 3.

Пример 4.

Пример 5.

Пример 6. Упростите выражение:

Решение:

=======

==27

2)Решить задание:

  1. Найдите значение числового выражения .

  2. Сравните числа .

  3. Вычислите: а) ; б) ; в) ; г) .



  1. Упростите выражение:

а) ; б); в) ; г) ; д) .

  1. Представьте выражение в виде степени и найдите его значение при у = 8.

  2. Сократите дробь: а) ; б).

  3. Вычислите:

  1. Вычислите:

  1. Данные     выражения    представить  в  виде    степеней   с одинаковыми показателями и сравнить их по величине:

1) 42 и 28;  2)  273  и  96 ; 3)  1252 и 253; 4) 4300 и 3400;   5) — 1/8   и   (—1/32)3;  6) ( — 6/7 )4  и (36/49)6;

  1. Найдите значение выражения .

3)Решить задание (тест):

А)Часть 1.

1. Представьте выражение в виде степени с основанием a. Ответ: ________________

2. Какое из данных выражений не равно ?

А. Б. В. Г.

3. Найдите значение выражения при m =

А. -16 Б. В. Г. 16

4. Решите уравнение: . Ответ: ________________

5. Разложите на множители:

А. ()() Б. ()() В. ()() Г. ()()

6. Найдите значение выражения А. 60 Б. 30 В. 12 Г.

Часть 2.

7. Упростите выражение: .


  1. Вычислите.

а) [а) 15 б) 30 в) 60 г) 10] б) [а) 1,5 б) в) 2,5 г) 0,4]

в) ( [а) - б) в) г) ] г) 10∙10∙10 [а)10 б)10 в) 10 г)10]

д) (5∙ [а)20 б) в)9 г)-20] е) 4: 2[а)2 б) 2 в)4 г) 4]

ж) 8 - 8 [а) 0 б) 1 в) 4 г) 1] з) [а)6 б) 246 в) 6 г)108]

и) [а) 24 б) 12 в) г)1] к) [а) б) 5 в) 13 г) 4]


В)

1 вариант.

А1. Вычислите .

1) 2; 2) 3; 3) 9; 4) .

А2. Вычислите .

1) 2; 2) 4; 3) 2; 4) 4.

А3. Упростите выражение

1) ; 2) ; 3) а; 4) .

А4. Вычислите

1) 0,25; 2) 0,5; 3) 0,15; 4) 5.

А5. Найдите значения выражения при у = 18.

1) 9(4+3); 2) ; 3) 4+3; 4) 9.

А.6. Упростите выражение

1) 2) 3) 4)

А7. Найдите значение выражения: .


1) 12; 2) 6; 3) 3; 4) –3.


А8. Найдите значение выражения: .

1) ; 2) 1,2; 3) ; 4) .

А9. Найдите значение выражения:

1) 4; 2) 9; 3) 5; 4) 5.


А10. Сократите дробь:

1) а; 2) ; 3) ; 4) а+1.


2 вариант.

А1. Вычислите .

1) 5; 2) 4; 3) 25; 4) .

А2. Вычислите .

1) 2; 2) 4; 3)2; 4) 4.


А3. Упростите выражение

1) ; 2) ; 3) а; 4) .

А4. Вычислите

1) 0,09; 2) 0,03; 3) 0,3; 4) 3.

А5. Найдите значения выражения при а = 4, b = 5.

1) ; 2) 2; 3) 0; 4) .

А.6. Упростите выражение .

1) 2) 3) 4)

А7. Найдите значение выражения: .


1) 45; 2) 5; 3) 3; 4) –45.

А8. Найдите значение выражения: .

1) 5,5; 2) 2; 3) ; 4) .

А9. Найдите значение выражения:

1) 4; 2) 25; 3) 9; 4) 16.

А10. Найдите значение выражения при р = 49.

1) 49; 2) ; 3) ; 4) 7.




ПЗ № 18. Преобразования выражений, содержащих степени. Решение прикладных задач.

Задание:

1) Перепишите:

hello_html_m7ca5a23c.png

hello_html_mea6a4e4.png

hello_html_1f9e2b77.png

2) Решение теста по теме «Степень с рациональным, действительным показателем, ее свойства».

( заполните пропуски и запишите номер правильного ответа)

А1. Вычислите .

1) 2; 2) 4; 3) 9; 4) .

А2. Вычислите .

1) 2; 2) 4; 3) 5; 4) 4.

А3. Упростите выражение

1) ; 2) ; 3) а2; 4) .


А4. Вычислите 1) 0,25; 2) 0,3; 3) 0,15; 4) 5.

А5. Найдите значения выражения при у = 16.

1) 9(4+3); 2) ; 3) 4+3; 4) .

А.6. Упростите выражение

1) 2) 3) 4)

А7. Найдите значение выражения: .


1) 12; 2) 6; 3) 8; 4) –3.


А8. Найдите значение выражения: .

1) ; 2)1,4; 3) ; 4)

А9. Найдите значение выражения:

1) 4; 2) 9; 3) 45; 4) 5.

А10. Сократите дробь:

1) 0,5а; 2) ; 3) ; 4) а+1.

А11. Вычислите . 1) 9; 2) 4; 3) 25; 4) .

А12. Вычислите .

1) 2; 2) 4; 3)6; 4) 4.

А13. Упростите выражение

1) ; 2) ; 3) а; 4) .

А14. Вычислите

1) 0,09; 2) 0,03; 3) 0,8; 4) 3.

А15. Найдите значения выражения

при а = 4, b = 5.

1) ; 2) 2; 3) 0; 4) .

3)Решить задание :

  1. Упростите иррациональные выражения:

1. ,2. , 3. , 4. ,5. ,

6. ,7. ,8.

  1. Упростите выражения: а) б) в)


  1. Разложите на множители:

а) ,б) ,в) ,г) ,д)

е) ,ж) ,з) ,и) ,к)

  1. Вычислите:

1.

2.

3.

4.

5.

6.

7.

  1. Найдите значение выражения:

а) б) в) г)

д) е) ж)

  1. Сократите дробь:

а) б)

в) г)


д)


  1. Вычислите:

а) , б) , в) ,г)

д) е)
ж) з) и)

  1. Упростите выражение:

а) 60,7/ 60,3; б) к -5,3∙4к 0,1; в) а7/6-5/6;

г) (а3/2: а-7/2)∙а3;д) 4с2/7+ 3(с1/7 )2 ; е) (4∙ 4-2а) -2.

  1. Вычислите:

а) б) в)

г) д) е)

ж) з)

  1. Найти значение выражения

hello_html_6f3d862e.png

  1. Найти значение выражения

hello_html_ma7e682b.png

  1. Упростите выражение.

hello_html_1432db50.png




Просмотрено: 0%
Просмотрено: 0%
Скачать материал
Скачать материал "ПЗ по специальности Технология машиностроения,1 курс(2016-2017 уч.г.)"

Получите профессию

Бухгалтер

за 6 месяцев

Пройти курс

Рабочие листы
к вашим урокам

Скачать

Выбранный для просмотра документ ПЗ№19,20.docx

ПЗ № 19. Нахождение значений логарифма по произвольному основанию. Переход от одного основания к другому.

Задание:

1)Перепишите:

1.Вычислим пример по формуле

2. Вычислим пример по формуле

3. Вычислим пример по формуле


4.Вычислим пример по формуле


5.Найдите значение выражения:

6. Найдите значение выражения:

7. Найдите значение выражения:
8. Найдите значение выражения:

9. Найдите значение выражения:


10. Найдите значение выражения:


11. Найдите значение выражения:

12. Найдите значение выражения:

13. Найдите значение выражения:

14. Найдите значение выражения:

15. Найдите значение выражения: .

16. Найдите  , если


2) Перепишите и заполните пропуски:

1) 2) 3)

4) 5) 6) 7)

8) 9) 10)

11) 12)

13)

14)

15) 16)

17) 18)

19)

20)

21)

22)

23)

24)

25) 26)

27)

28)


30)

hello_html_m6da07957.png


3)Решить задание :Найдите значение выражения:





ПЗ № 20. Вычисление и сравнение логарифмов. Логарифмирование и потенцирование выражений. Приближенные вычисления и решения прикладных задач.

Задание:

1) Перепишите и заполните пропуски:
Пример 1. log3 9 = 2, так как 32 = 9, log5 25 = 2, так как 52 = 25, log3 81 = 4, так как 34 = 81,

Ответ: 2,2,4.

Пример 2. Вычислите : а) log2 16, б) log3 3, в) , г) , д) log2 2 log3 81, е) log12 2 + log12 72, ж) log5 75 – log5 3.
Решение:
а) log2 16 = 4, б) log3 3 = …, в) = 16, г) = = …,

д) log2 2 log3 81= 1· 4 = …, е) log12 2 + log12 72 = log12 (2 ·72) = log12 144 = …,

ж) log5 75 – log5 3= log5 (75:3) = log5 25 = …

Ответ: а) 4, б) 1, в) 16, г) 8, д) 4, е) 2, ж) 2.

Пример 3. Найдите х, если logx 36 = 2 и log2 x = – 2.

Решение: logx 36 = 2, х2 = 36, х = log2 x = - 2, х = 2 -2 = 1 / 4 = …

Ответ: 0,25
Пример 4. Вычислите: а) , б) , в) .

Решение: а) = log2 16=…, б) = 5 · = 5 · 3 = … ,

в) = = 17 = 1296 – 17 = …

Ответ: – 4, 15, 1279.

Пример 5. Упростите выражение :

а) ;

б)

;

в) ;

Ответ: 24, 7, 2.

2) Опорный конспект.

Вспомним основные приемы сравнения положительных чисел а и b.


1) Составить разность a-b и сравнить ее с нулем.


2) Составить частное a:b и сравнить его с единицей.

Свойства логарифмической функции или свойства логарифмов чисел применяются при сравнении логарифмов чисел:

Монотонность логарифмической фунции; Свойства: 0 1 2

loga X1 < loga X2, если a >1

loga X1 > loga X2, если 0 < а < 1

Пример 1. Сравнить и ,

a = 15 >1, > .

Пример 2. Сравнить и ,


знак меняется , <


Пример 3. Сравнить и .

Решение:

значит, значит,

Известными способами сравнить можно не все числа. Рассмотрим следующий пример. Основания разные, логарифмируемые числа разные. Также, как и при сравнении иррациональных чисел, применяется метод «оценки» или сравнение с каким-нибудь «хорошим» числом.

Пример 4. Сравнить и .

, а, значит, .

Пример 5. Сравнить и .


Основное логарифмическое тождество

,

Логарифм произведения — это сумма логарифмов

,

Логарифм частного — это разность логарифмов

,

Свойства степени логарифмируемого числа и основания логарифма

Показатель степени логарифмируемого числа  

Показатель степени основания логарифма,

в частности если m = n, мы получаем формулу: ,

например:

Переход к новому основанию, частности, если c = b, то , и тогда: .


Логарифмирование – это нахождение логарифмов заданных чисел или выражений.

                                                              b
Пример: Найдем логарифм x = a2 · — .
                                                             c

Решение.

Последовательно воспользуемся сразу всеми тремя основными свойствами логарифмов, которые изложены выше (логарифм произведения, логарифм частного и логарифм степени):
                  
    b
lg x = lg (a2 · —) = lg a2 + lg b lg c = 2lg a + lg b lg c.
                      c

Потенцирование – это нахождение чисел или выражений по данному логарифму числа (выражения).

Потенцировать – значит освобождаться от значков логарифмов в процессе решения логарифмического выражения.

Пример: Ответ:

Десятичный логарифм — логарифм с основанием 10, который обозначается как lg.

lg100=2log10100=2, так как 102=100 .

Натуральный логарифм — логарифм с основанием e, обозначается ln.


hello_html_5eccaba.gif

hello_html_m34eaff33.gif


Задача 2hello_html_m74ce2fe.gif


hello_html_615d882b.gif

Задача 3

По данным газеты «Зори» от 12 апреля 2011 года из доклада П. Е. Шишкина население в городе Старый Оскол за один год увеличилось с 256100 человек до 257135 человек. Через сколько лет население этого города увеличится в 1,5 раза?
Решение. Для решения этой задачи применим формулу сложных процентов: A=a(1+p/100)x. Примем население города, которое было, за а=256100тогда А=257135-это население, которое стало, х -неизвестно. р=((257135256100)/257135)100≈0,4%

Сделав подстановку в формулуполучим 256100∙1,5=256100(1+0,4/100) x

Чтобы решить это показательное уравнение прологарифмируем его.

xlg 1,004=lg1,5, откуда x =lg 1,5 /lg1,004

Найдя по таблице lg1,5 и lg1,004 , получим x=0,18/0,002≈90.

Ответ : примерно через 90 лет.

Задача 4

Высота над уровнем моря вычисляется по формуле h=(8000/0,4343)lg(p/p),

где p0 =760 мм рт.ст., р - давление на высоте h м.
Давление в городе Старый Оскол на 15 апреля 2011 года равно 738 мм рт. ст. Вычислим, на какой высоте находится наш город.

Решение. Найдем высоту, на которой находится наш город:
h=(8000/0,4343)lg(760/738) ≈235 м

Ответ: 235м.

Задача 5

В начальный момент времени было 8 бактерий. Через 2 часа после помещения бактерий в питательную среду, их число возросло до 100. Через сколько времени с момента размещения в питательную среду следует ожидать появления 500 бактерий?

Решение.

Для решения данной задачи, необходимо вспомнить понятия скорости и ускорения.

Было -8

Стало- 100

hello_html_7da3953.gif

1 изменение:


=> hello_html_m73c4f0ec.gif конечное значение скорости распространения бактерий при первом изменении - hello_html_32205d0c.gif

Было -8

Стало- 500

hello_html_m5e8cc156.gif

2 изменение:


=> hello_html_m73c4f0ec.gifконечное значение скорости распространения бактерий при втором изменении - hello_html_83a73c1.gif.

Составим формулу для ускорения, учитывая, что начальная скорость hello_html_40273d1b.gif (т.е. было -8, стало -8):

hello_html_m2617e0a1.gif hello_html_m71fb25b1.gif

hello_html_bdff533.gif hello_html_m1c3702f.gif

Т.к. ускорение постоянно => hello_html_2e81f73.gif =>

hello_html_m7ee7891c.gif

Перейдем к натуральному основанию логарифмов, для того, чтобы можно было воспользоваться табличными значениями:

hello_html_25013b08.gif

hello_html_m264b8706.gif

Ответ: приблизительно 3часа 15 минут.

3)А)Решить задание ( по примерам):

  1. Вычислите а) log3 27, б) log4 1,в) log1/2 4,

  2. Вычислите а) log2 32, б) log3 9, в) , г) ,

д) log3 3· log2 8, е) lg 5 + lg 2, ж) log3 15 – log3 5.

  1. Найдите х, если log2 4 = x и log6 x = 2.

  2. Вычислить а) б)

в) .

  1. а)

б)

в)

г)

д)

е)

Б) 1)log9 81 ; 2) 3) log3 1; 4) log5 5 ; 5)

6) 7) log2 log3 9; 8) lg100; 9) 92 log9 5 ; 10) log64 8

Ответы: вставьте номер задания

задания


2

7

10



5



1

ответ

2

4

1

0,5

1

0

-2

25

-1

2


В) 1.Сравнить и ,

2. Сравнить и

3. Сравнить и .

4. Прологарифмируйте по основанию 10 (a > 0,b > 0), x = a2b3.

5. Прологарифмируйте по основанию 3 (a > 0,b > 0), x = 7a3b.

6.Найдите x, если

7. Найдите x, если .

С) Задания для практической работы:

hello_html_3b2eb585.gif

1-2.




hello_html_m41b85ba0.gif


3-4.

hello_html_3139472f.gif







5.Для обогрева помещения, температура в котором равна Тп = 200С, через радиатор отопления, пропускают горячую воду температурой  Тв = 1000С. Расход проходящей через трубу воды m = 0,2 кг/с. Проходя по трубе расстояние x (м), вода охлаждается до температуры Т0С, при чём

hello_html_m119072aa.gif, где с = 4200Дж/кг*С — теплоемкость воды

hello_html_m1c48a4a9.gif = 42 Вт/м * 0С— коэффициент теплообмена , = 1,4 — постоянная.

До какой температуры (в градусах Цельсия) охладится вода, если длина трубы 28 м?

4)Решить задание :

1. Вычислите (по свойству степени):

1) , 2) , 3) , 4) , 5) ,

6) , 7) , 8) ,9) , 10) , 11) ,

12) , 13) , 14) , 15) , 16) .

2. Вычислите (по основному лог. тождеству):

1) , 2) , 3) , 4) , 5) , 6) ; 7) ,

8) , 9) , 10) , 11) , 12) , 13) ,

14) ,15) , 16) , 17) , 18) , 19) , 20) .

3. Вычислите: а),б) , в) ,

г), д).

4. Упростите выражение :

а) ;б) ,в);

5.Вычислить логарифмы: log381,ln e, lg1000, log7343,ln7,29, lg0,001.

6.Вычислить логарифмы: log432 + log42, log552, log2(8128), log654 + log64, log3108 – log34.

7.Вычислить логарифмы:

8.

9. Вычислите: 1) ; 2) ; 3) ; 4) ; 5) ;

6) , 7) , 8) .

10. Выразите данный логарифм через натуральный и вычислите на микрокалькуляторе с точностью до 0,01 : 1) ; 2) .

Просмотрено: 0%
Просмотрено: 0%
Скачать материал
Скачать материал "ПЗ по специальности Технология машиностроения,1 курс(2016-2017 уч.г.)"

Получите профессию

Интернет-маркетолог

за 6 месяцев

Пройти курс

Рабочие листы
к вашим урокам

Скачать

Выбранный для просмотра документ ПЗ№2.docx


 ПЗ № 2. Перпендикуляр и наклонная к плоскости. Угол между прямой и плоскостью. Теоремы о взаимном расположении прямой и плоскости. Теорема о трех перпендикулярах.

Задание:

1) A)а) Записать по рисунку:

  • какой отрезок является перпендикуляр, наклонная, проекция наклонной,

  • угол между наклонной и плоскостью α.

АС - …, АВ - …, СВ – …, АВ2 = ВС2 + АС2.

- угол между наклонной и плоскостью α.

б)Перепишите и заполните пропуски:

Пример 1. Из точки, не принадлежащей данной плоскости , проведены к ней
две наклонные, равные 10см и 18см. Сумма длин их проекций на

плоскость равна 16см. Найти проекцию каждой наклонной.(рис.1)

Дано: ОС - перпендикуляр, АС и ВС - наклонные, АО и ОВ – их проекции, рис.1

АС = 10 см, СВ = 18 см, АО + ОВ = 16 см,

Найти: АО, ОВ

Решение: АС = 10, СВ = 18, АО + ОВ = 16, АО = х, ОВ = 16 х,

АС2 АО2 = ВС2 – ОВ2 , 102 х2 = 182 – (16 х)2, 100 х2 = 324 – 256 + 32 х х2 ,

32 х = 32, х = … , АО = 1, ОВ = 16 – 1 = .... Ответ: 1 и 15 см.

Пример 2. Из точки к плоскости проведены две наклонные. Одна из них длиной 12см наклонена к плоскости под углом 60°, проекция другой на эту плоскость равна 6 см.

Найти длину этой наклонной.

Дано: ОС - перпендикуляр, АС и ВС - наклонные, АО и ОВ – их проекции,

СА = 12 см , САО = 60°, ОВ = 6 см ,

Найти: СВ

Решение: Δ АОС- прямоугольный, АСО = 90 ° 60 ° = 30°, АО = СА : 2 = 12: 2 = … ,

СО2 = СА2 –АО2 = 122 – 62 = 144 – 36 = … ,

СВ2 = СО2 + ОВ2 = 108 + (6 )2 = 108 + 36 6 = 108 + 216 = … , СВ = … см. Ответ: 18 см.

Пример 3. Из точки С к данной плоскости  проведены перпендикуляр СО = 6см и две наклонные. Каждая из наклонных образует с плоскостью  угол 60°. Угол между наклонными 120°. Найти расстояние между основаниями наклонных.

Дано: ОС - перпендикуляр, АС и ВС - наклонные, АО и ОВ – их проекции,

СО = 6см, САО = СВО = 60°, АСВ = 120°,

Найти: АВ
Решение: sin САО = СО : АС, АС = ВС = СО : sin САО = 6: sin60 ° = 6 : = 12 : = 4 ,

Δ АВС – равнобедренный, АВ2 = АС2 + ВС2 – 2АС ВС cos АСВ =

= (4)2 + (4)2 – 24 cos 120° = 16 3 + 16 3 - 216 3( – ) = 48 + 48 + 48 = … ,

АВ = … см. Ответ: АВ = 12 см.

Пример 4. Из точки С к данной плоскости проведены перпендикуляр СО и две наклонные СВ и АС. ОВ= 4,САО = 30°, СВО = 60°, а угол между наклонными 90°. Найти расстояние между основаниями наклонных.

Дано: ОС - перпендикуляр, АС и ВС - наклонные, АО и ОВ – их проекции,

ОВ= 4,САО = 30°, СВО = 60°, АСВ = 90°,

Найти: АВ

Решение: ΔСОВ – прямоугольный, СВО = 60°, ОСВ = 90 ° - 60 ° = 30 °,

ВС= 2 ОВ = 24 = … , СО2 = ВС2 – ОВ2 = 82 – 42 = 64 – 16 = … , СО = = 4,

АС = 2 СО = 24 = … , ΔАСВ - прямоугольный, АВ2 = АС2 + ВС2 = (8)2 + 82 =

= 64 3 + 64 = … , АВ = … см. Ответ: АВ = 16 см.
Пример 5. Диагонали квадрата АВСD пересекаются в точке О.


Из точки О проведён к плоскости квадрата перпендикуляр ОМ. Найти расстояние от точки М до

стороны ВС, если AD = 6см, ОМ = 4см. (рис.2)

Дано: АВСD - квадрат, ОМ - перпендикуляр,
О - точка пересечения диагоналей квадрата,

МК - расстояние от точки М до стороны ВС, AD = 6см, ОМ = 4см.

Найти: МК

Решение: ОК = АВ : 2 = AD : 2 = 6 : 2 = … , ΔМОК - прямоугольный, Рис.2

МК2 = ОМ2 + ОК2 = 42 + 32 = 16 + 9 = … , МК = ... Ответ: МК = 5 см.
Пример 6. Прямые АВ, АС и AD попарно перпендикулярны . Найдите отрезок CD, если: АВ = 3 см, ВС = 7 см, AD = 1,5 см;

Дано: АВ, АС и AD попарно перпендикулярны, АВ = 3 см, ВС = 7 см, AD = 1,5 см; Найти: CD

Решение: Δ САВ – прямоугольный, АС2 = СВ2 – АВ2, АС2 = 72 – 32 = 49 – 9 = … ,

Δ САD – прямоугольный, СD2 = АС2 + АD2, СD2 = 40 + 1,52 = 40 + 2,25 = … ,

СD = … см. Ответ: СD = 6,5 см.hello_html_2b8f3f4.png

Пример 7.

hello_html_2b8f3f4.png

Решение:

hello_html_2b8f3f4.png

Пример 8. Дан параллелепипед АВСDА1В1С1D1 , у которого основание квадрат. hello_html_75ed7ad2.jpg

Докажите, что а) СD В1С1 , б) С1D1 АD .

Доказательство: а) СD || A1B1, A1B1 В1С1 СD В1С1 ( по лемме),

б) С1D1 || ВС , ВС АD С1D1 АD ( по лемме) .

B) Тест на опознание.

  1. Перпендикуляр от точки до прямой – это …

  2. Перпендикуляр к плоскости – это …

  3. Если прямая перпендикулярна к прямой плоскости, то …

  4. Чтобы прямая была перпендикулярна к плоскости, необходимо, чтобы она была …

  5. Наклонная к плоскости – это прямая …

  6. Если наклонные, проведенные к плоскости из одной точки, равны, то равны и …

  7. Если проекции наклонных, проведенных к плоскости из одной точки, равны, то равны и …

  8. Если прямая плоскости перпендикулярна к наклонной., то …

  9. Если прямая перпендикулярна к проекции наклонной, то …

  10. Если наклонная перпендикулярна к прямой плоскости, то …

  11. Если проекция наклонной перпендикулярна к прямой плоскости, то …

Какие из формулировок относятся к теореме о трех перпендикулярах?

  1. Если прямая, пересекающая плоскость, перпендикулярна к плоскости, то она перпендикулярна ко всем ее прямым.

  2. Если прямая плоскости перпендикулярна к наклонной, то она перпендикулярна и ее проекции.

  3. Если прямая, пересекающая плоскость, перпендикулярна двум пересекающимся прямым плоскости, то она перпендикулярна и самой плоскости.

  4. Прямая плоскости, перпендикулярная проекции наклонной, перпендикулярна и самой наклонной.

C) Тест на различение.

  1. Какие из прямых MA, MD или MC перпендикулярны AD?

  2. Какие из прямых AM, NM или DM перпендикулярны AD?

  3. Какие из прямых MA или MC перпендикулярны AC?

  4. Какие из прямых MA, MN или MC перпендикулярны AC?

1. 2.















3. 4.

Тест на различение.


Какие из прямых перпендикулярны к прямой AD?

  1. Какие из прямых: AB1; DB1; CB1 перпендикулярны к AD или DC?

  2. Какие из прямых: AB1; DB1; CB1 перпендикулярны к AD или DC? :

  3. Какие из прямых: AB1 или DC1 перпендикулярны к AD

  4. Как построить перпендикуляр из точки пространства к прямой плоскости?










2)Решить задачи ( по примерам):

  1. Из точки, не принадлежащей данной плоскости, проведены к ней две наклонные,
    равные 20 см и 36 см. Сумма длин их проекций на плоскость равна 32 см.
    Найти проекцию каждой наклонной.

  2. Из точки к плоскости проведены две наклонные. Одна из них длиной 24 см наклонена к плоскости под углом 60°, проекция другой на эту плоскость равна 12 см. Найти длину этой наклонной.

  3. Из точки С к данной плоскости проведены перпендикуляр СО = 12 см и две наклонные. Каждая из наклонных образует с плоскостью угол 60°. Угол между наклонными 120°.
    Найти расстояние между основаниями наклонных.

  4. Из точки С к данной плоскости  проведены перпендикуляр СО и две наклонные СВ и АС.
    ОВ= 8,
    САО = 30°, СВО = 60°, а угол между наклонными 90°. Найти расстояние между основаниями наклонных.

  5. Диагонали квадрата АВСD пересекаются в точке О. Из точки О проведён к плоскости квадрата перпендикуляр ОМ. Найти расстояние от точки М до стороны ВС, если AD = 12 см, ОМ = 8 см.

  6. Прямые АВ, АС и AD попарно перпендикулярны.
    Найдите отрезок CD, если: АВ = 6 см, ВС = 14 см, AD = 3 см;

  7. Дан параллелепипед АВСDА1В1С1D1 , у которого основание квадрат.
    Докажите, что
    а) С1D1 ВС, б) СD А1D1 .

3)Решить задачи :

  1. Дано: АС - перпендикуляр, АВ - наклонная,
    а)
    АВ = 10 см, ВС = 6 см, АС = ?, б) АС = 12 см, ВС = 5 см, АВ = ? (Указание:АВ2 = ВС2 + АС2 )

  2. Дано: Δ АВС – равнобедренный, АК(АВС), АК = 12 см, АВ = АС = 5 см, ВС = 6 см,
    КМ
    ВС. Найти: КМ, АМ.

(Указание: АВ = АС => КВ = КС => Δ СКВ – равнобедренный, КМ ВС => ВМ- медиана,

ВМ = МС = ВС : 2, КС2 = АК2 + АС2 , КМ2 = КС2 - МС2 , АМ2 = АС2 - МС2 )

  1. Дано: АО - перпендикуляр, АВ и АС - наклонные, АВ = АС, ОАВ = ВАС = 60°,
    АО = 2,5 см.
    Найти: ВС. (Указание: Δ ВАС – равносторонний, ВС = АВ = АС = 2АО)

  2. Телефонная проволока длиной 15 м протянута от телефонного столба, где она прикреплена на высоте 8 м от поверхности земли, к дому, где ее прикрепили на высоте 20 м. Найдите расстояние между домом и столбом, предполагая, что проволока не провисает.

Дано: AB = 15 м, АС = 8 м, BD = 20 м, Найти: CD.

(Указание: Δ BKА – прямоугольный, АK2 = AB2 - BK 2)

  1. Дан куб АВСDА1В1С1D1 . Найдите следующие двугранные углы: а) АВ В1С , б) АDD1В,
    в) А
    1ВВ1К, где К- середина А1D1.

  2. Из вершины равностороннего треугольника АВС проведен перпендикуляр АК к плоскости треугольника. Найдите длину АК, если ВС = 3 см, КС = 3 см.

  3. Найдите тангенс угла между диагональю куба и плоскостью одной из его граней.

  4. Дано: ABCD квадрат; AM - прямая; АМ  (ABCD); АС  BD = О. Доказать: a) BD  (АМО); б) МО  BD.

  5. Прямые АВ и CD перпендикулярны некоторой плоскости и пересекают ее в точках В и D соответственно. Найдите AС, если АВ = 9, CD = 15, BD = 8.

  6. Отрезок МН пересекает некоторую плоскость в точке К. Через концы отрезка проведены прямые HP и ME, перпендикулярные плоскости и пересекающие ее в точках Р и Е. Найдите РЕ, если HP = 4 см, НК = 5 см, ME = 12 см.

  7. Треугольник ABC правильный, точка О - его центр. Прямая ОМ перпендикулярна плоскости AВС. Докажите, что МА = MB = МС. Найдите МА,если АВ = 6 см, МО = 2 см.hello_html_m754188c5.jpg

  8. ABCD - квадрат . АЕ - перпендикулярно плоскости квадрата, К  BE. Найти: (ВС; АК).

  9. ABCD прямоугольник. Отрезок АЕ перпендикулярен к плоскости ABC. ЕВ = 15, ЕС = 24, ED = 20. Докажите, что треугольник EDC прямоугольный, и найдите АЕ.

  10. Точка А принадлежит окружности, АК - перпендикуляр к ее плоскости, АК = 1 см, АВ - диаметр, ВС — хорда окружности, составляющая с АВ угол 45°. Радиус окружности равен 2 см. Докажите, что треугольник КСВ прямоугольный, и найдите КС.

  11. Дано: α; АВ - отрезок;AB α = O,AD α; BC α; AD α = D,BC α = C, AD = 6 см, BC = 2 см, OC = 1,5 см. Найти: АВ.

  12. Прямые АВ, АС и AD попарно перпендикулярны . Найдите отрезок CD, если: АВ = 6 см, ВС = 14 см, AD = 3 см;

  13. Через точки А и В проведены прямые, перпендикулярные плоскости α, пересекающие ее в точках С и D соответственно. Найдите расстояние между точками А и В, если АС = 8 см, BD = 20 см, CD = 5см и отрезок АВ не пересекает плоскость α.

  14. Телефонная проволока длиной 26 м протянута от телефонного столба, где она прикреплена на высоте 6 м от поверхности земли, к дому, где ее прикрепили на высоте 30 м. Найдите расстояние между домом и столбом, предполагая, что проволока не провисает.

  15. К плоскости треугольника из центра, вписанной в него окружности радиуса 1 м восставлен перпендикуляр длиной 2,4 м. Найдите расстояние от конца этого перпендикуляра до сторон треугольника.

  16. Дано: ΔАВС; АВ = АС = ВС; CD  (ABC); AM = MB, DM = 17, CD = 8. Найти: SΔADB.


Просмотрено: 0%
Просмотрено: 0%
Скачать материал
Скачать материал "ПЗ по специальности Технология машиностроения,1 курс(2016-2017 уч.г.)"

Получите профессию

Копирайтер

за 6 месяцев

Пройти курс

Рабочие листы
к вашим урокам

Скачать

Выбранный для просмотра документ ПЗ№21.docx

ПЗ № 21. Радианный метод измерения углов вращения и связь с градусной мерой.


Задание:

1)Перепишите и заполните пропуски:
Пример 1. Найти радианную меру угла, выраженного в градусах: а) α = 40°, б) α = 120°, в) α = 150°.

Решение:

Ответ:

Пример 2. Найти градусную меру угла, выраженного в радианах:

Решение:

Ответ:

Пример 3. Вычислите:

Решение:



Ответ:

Пример 4. Вычислите:

Решение:



Ответ:

Пример 5.

Решение:



Ответ:

2)Решить задание ( по примерам):

  1. Найти радианную меру угла, выраженного в градусах: а) α = 75°, б) α = 32°, в) α = 140°.

  2. Найти градусную меру угла, выраженного в радианах:

  3. Вычислите:

  4. Вычислите:

3)Решить задание :

  1. Вычислите:

  2. Радианная мера двух углов треугольника равна   и  . Найдите градусную меру каждого из углов треугольника. 

  3. Выразите в градусной мере величину угла: .

  4. Выразите величину угла в радианах: .

  5. Найдите знак произведения, используя правило знаков по четвертям:

  6. Вычислите значение выражения:

  7. Вычислите:

  8. Вычислите:

  9. Найдите знак произведения:

  10. Вычислить значения и ,если α =120°.

  11. Вычислите значение тригонометрических функций: sin π/3;cos 7π/6;tg π;sin π/4;tg 2π/3;ctg π/2.

  12. Найдите радианную меру углов треугольника, если их величины относятся как 2:3:4.

  13. Может ли косинус быть равным:  

  14. Может ли синус быть равным:

  15. Вычислите:

  16. Вычислите :

  17. Известно, что Вычислите значение выражения:

  18. Известно, что . Вычислите значение выражения:

  19. Вычислите : .

  20. Известно, что . Вычислите: .

  21. Известно, что Вычислите:

  22. Найдите значение выражения: 5sin²3х – 6,если cos²3х = 0,6.

  23. Найдите значение выражения: 5sin²4х – 6,если cos²4х = 0,8.

  24. Известно, что . Вычислите: .

  25. Известно, что Вычислите:

Просмотрено: 0%
Просмотрено: 0%
Скачать материал
Скачать материал "ПЗ по специальности Технология машиностроения,1 курс(2016-2017 уч.г.)"

Получите профессию

Секретарь-администратор

за 6 месяцев

Пройти курс

Рабочие листы
к вашим урокам

Скачать

Выбранный для просмотра документ ПЗ№22.docx

ПЗ № 22. Основные тригонометрические тождества, формулы сложения, удвоения, преобразование суммы тригонометрических функций в произведение, преобразование произведения тригонометрических функций в сумму.

Задание:

1) Опорный конспект.

Основные тригонометрические тождества:

Формулы сложения:



Формулы двойного аргумента:

Формулы преобразования сумм или разности в произведение.



Формулы преобразования произведения тригонометрических функций в сумму:

2)Перепишите и заполните пропуски:
Пример 1.Вычислить : а) cos 18° cos 12° sin 18° sin 12°; б) cos 107° cos 17°sin 107° sin 17°;

в) sin 17° cos 13° sin 13° cos 17°; г) sin 43° cos 13° sin 13° cos 43°;

д) , е) .

Решение: а) cos 18° cos 12° sin 18° sin 12° = cos(18°12°) = cos 30° = …,

б) cos 107° cos 17° sin 107° sin 17° = cos(107°17°) = cos 90° = …,

в) sin 17° cos 13° sin 13° cos 17° = sin(17°13°) = sin 30° = …,

г) sin 43° cos 13° sin 13° cos 43° = sin(43°13°) = sin 30° = …,

д) = tg (9°51°) = tg 60° = …, е) = tg (65°20°) = tg 45° = … .

Ответ: а); б) 0; в) 0,5; г) 0,5; д) ; е) 1.

Пример 2.Вычислить : а) cos π /7 cos /21 sin π/ 7sin /21;

б) sin π /3 cos π /12  cos π /3sin π /12; в) .

Решение: а) cos π /7 cos /21 sin π /7sin /21 = cos /7 4π /21) = cos (3π /21 4π /21) =

= cos /21 = cos π /3 = …,

б) sin π /3 cosπ /12 cos π /3 sin π /12 = sin /3 π /12) = sin (4π /12π /12) = sin /12 =

= sin π /4 = …,

в) = tg (π /7 4π /21) = tg π /3 = …

Ответ: а) 0,5; б) /2; в).

Пример 3. Упростить: а) cos α cos 3α sinα sin3α; б) sin 2α cos α cos 2α sin α;

в) sin α cos 3α cos α sin 3α; г) .

Решение: а) cos α cos 3α sinα sin3α = cos (α 3α) = cosα;

б) sin 2α cos α cos 2α sin α = sin (2α α) = sin α;

в) sin α cos 3α cos α sin 3α = sin (αα) = sinα; г) = tg (x 3x) = tgx.

Ответ: а) cos 4α; б) sin α; в) sin 4α; г) tg 4x.

Пример 4. Упростить : а) cos α cos β sin α sin β, если α = 42 °, β = 18 °;

б) cos(x y) cos(x + y) + sin(x y) sin(x + y).

Решение: а) cos α cos β sin α sin β = cos (α β) = cos (42 ° 18 °) = cos 60 ° = …,

б) cos(x y) cos(x + y) + sin(x y) sin(x + y) = cos ((x  y) – (x + y)) = cos (–2y) = cos 2y.

Ответ: а) 0,5; б) cos 2y .

Пример 5. Упростить выражение:

Решение: Ответ: 1.

Пример 6. Вычислите: cos630°– sin1470°– сtg1125°.

Решение: cos630°– sin1470°– сtg1125° = cos(360° + 270°)– sin(4360° + 30°)сtg(3360 ° + 45°) =

= cos270°– sin30°– сtg45° = 0 – 0,5 – 1= … Ответ: – 1,5.

Пример7. Вычислить

Решение: Ответ:

Пример 8. Вычислить

Решение: Ответ:

Пример 9. Вычислить sin2α, если sinαcosα =

Решение: Возведем обе части равенства в квадрат: (sinαcosα)2 = ,

sin2α – 2sinαcosα + cos2α = , 2sinαcosα = – 1, 2sinαcosα = , sin2α = …

Ответ: .

Пример 10. Вычислить sin2α, если sinα = 0,6,

Решение: sin2α = 2sinα cosα . Т.к. ,то cosα < 0,

cos α =

sin2α = 2() () = ...

Ответ: 0,96.

Пример 11. Вычислить sinα/2, cosα/2, tgα/2, ctgα/2, если cosα = 0,8,

Решение: cos2 α/2 = (1 + cosα) : 2 = 1,8 : 2 = 0,9, cosα/2 = .

sin2 α/2 = (1 cosα) : 2 = 0,2 : 2 = 0,1, sinα/2 = .

tgα/2 = sinα/2 : cosα/2 = 0,33 : 0,95 = 33/95, ctgα/2 = cosα/2 : sinα/2 = 0,95 : 0,33 = 95/33.

Ответ: 0,33; 0,95;33/95; 95/33.

Пример 12. Пусть Найдем sin2, cos2, tg2.

Решение:


Ответ:

Пример 13. Докажите справедливость равенства cos 5π/ 12 + cos7π/ 12 =0 .

Решение:  cos 5π/ 12 + cos 7π/ 12 = 2cos π/ 2 cos( − π /12 ) = 20cos( − π/ 12 ) = … , Ответ:0.

Пример 14. Вычислить hello_html_39d0c6cc.gif

Решение: sin 70° + sin 10° = 2sin 40° cos30° = sin 40°.

cos70° − cos10° =  − 2 sin 40° sin 30° =  −  sin 40°.

hello_html_39d0c6cc.gif= sin 40° : (−  sin 40° ) =  −  .

Ответ: −  .

Пример 15. Вычислить cos11 π/12 – cos 5 π/12 .

Решение: cos11 π/12 – cos 5 π/12 = − 2sin(11 π/12 + 5 π/12) : 2 sin(11 π/12−5 π/12) : 2 = = − 2 sin 8π/12 sin 3π/12 = − 2 sin 2π/3 sin π/4 = − 2 = .

Ответ:.

Пример 16. Вычислить sin7 π/12 – sin π/12 .

Решение: sin7 π/12 – sin π/12 = 2 sin(7 π/12 − π/12) : 2 cos(7 π/12+ π/12) : 2 =

= 2 sin π/4 sin π/3 = 2 1/2 =

Ответ: .

Пример 17. Вычислим значение выражения .


Пример 18. Преобразуйте в сумму произведение Ответ.

2)Решить задание ( по примерам):

  1. Вычислить : а) cos 38° cos 22° sin 38° sin 22°; б) cos 55° cos 10°sin 55° sin 10°;

в) sin 47° cos 13° sin 13° cos 47°; г) sin 103° cos 13° sin 13° cos 103°;

д) , е) .

  1. Вычислить : а) cos  π /5 cos π /20 sin π/ 5sin π /20;

б) sin π /4 cos π /12  cos π /4sin π /12; в) .

  1. Упростить: а) cos 2α cos 6α – sin 2α sin 6α; б) sin 3α cos α cos 3α sin α;

в) sin 2α cos 3α cos 2α sin 3α; г) .

  1. Упростить : а) cos α cos β sin α sin β, если α = 42 °, β = 48 °;

б) cos(2x y) cos(2x + 3y) + sin(2x y) sin(2x + 3y).

  1. Упростить выражение:

  2. Вычислите: cos450°– sin750°сtg765°.

  3. Вычислить

  4. Вычислить

  5. Вычислить sin2α, если sinαcosα = 1/4.

  6. Вычислить cos 2α, если sinα = 0,8,

  7. Вычислить sinα/2, cosα/2, tgα/2, ctgα/2, если sinα = 0,8,

  8. Пусть Найдем sin2, cos2, tg2.

  9. Преобразовать в произведение .

  10. Преобразовать в произведение

  11. Преобразуйте в сумму произведение

  12. Вычислим значение выражения

3)Решить задание :

  1. Упростите выражение: sin(3π/2 – αcos(π/2 + α) + sin(2 π – α) + cos(3π/2 + α) + cosα ·sinα.

  2. Найдите cosß, если tgß = 7/24 и ß (π; 3π/2).

  3. Найдите значение выражения: 2sin²2х – 9cos²2х, если cos2х = – 0,9.

  4. Вычислите:3ctg60º· (sin310ºcos70º sin70ºcos310º).

  5. Найдите значение выражения:5 cos(3π/2 + α) , если α = 7π/6.

  6. Найдите значение выражения: 4 + 5tg²х • cos²х, если sinх = 0,4.

  7. Найдите значение выражения:7 cos(π + α) – sin(3π/2 + α), если cosα = 0,6.

  8. Упростить выражение 4⋅(tg(π t) + ctg(π t) + ctg(3π/2 t)) ctg(π t).

  9. Упростите выражение: .

  10. Вычислите:

  1. Упростите выражение: .

  2. Вычислите

  1. Найди значение выражения sin1050°+cos4620°+tg1035°.

  2. Вычислите:

  3. Упростите выражение 

  4. Вычислите:

  5. Упростите выражение:

  6. Вычислите: а) sin810°cos900o + tg585octg l845o + cos l35osin405°;
    б)
    cosl05°sinl95° + sin(-135°);

  7. Найдите значение выражения sin (х + у), если sin х= 9/41; cos у =40/41; х - угол II четверти.

  8. Найдите , если  и.

  9. Найдите значение выражения, если .

  10. Вычислить


  1. Дано: cos х =-12/13; 180 º < х < 270 º. Найти: cos х/2,tg x/2.

  2. Упростите выражение 

  3. Упростите выражение 

  4. Найти значение выражения: 2sin150 cos150.

  5. Найдите sin 2α, cos 2α,tg2α,  если и  .

  6. Найдите 24cos2α, если sinα = - 0,2 .

Просмотрено: 0%
Просмотрено: 0%
Скачать материал
Скачать материал "ПЗ по специальности Технология машиностроения,1 курс(2016-2017 уч.г.)"

Получите профессию

Технолог-калькулятор общественного питания

за 6 месяцев

Пройти курс

Рабочие листы
к вашим урокам

Скачать

Выбранный для просмотра документ ПЗ№23.docx

ПЗ № 23. Простейшие тригонометрические уравнения и неравенства.

Задание:

1)Перепишите и заполните пропуски:
Пример 1.Решите уравнение sin4xcos2x = 0.

Решение: sin4x – cos2x = 0 , 2sin2x cos2x – cos2 x = 0 , сos2x(2sin2x – 1)=0 ,

сos2x=0 или sin2x=1/2 .

2x = π/2 + π k, k , 2x = (1) n π/6 + π n, n Z .

х1= π/…+ π k/2 , k Z, x2 = (1)n π/…+ π n/2 , n Z .

Ответ: х1= π/4+ π k/2 , k Z; x2 = (1)n π/12+ π n/2 , n Z .

Пример 2. Решите уравнение (2 sin x – 1) (tg x) = 0.

Решение: ( 2 sin x – 1) hello_html_m61765ba1.gif (tg x) = 0,

2 sin x – 1= 0 или tg x = 0,

sin x = 1/2 tg x = ,

х1= (–1) n π/… + π n, n Z , х2 = π/… + π k, k .

Ответ: х1= (–1) n π/6 + π n, n Z , х2 = π/3 + π k, k .

Пример 3. Решите уравнение ( ctg x – 1) hello_html_m61765ba1.gif (2sin + 1) = 0.

Решение: ( ctg x – 1) hello_html_m61765ba1.gif (2sin + 1) = 0,

ctg x – 1 = 0 или 2sin + 1 = 0,

ctg x = 1 sin = – 1/2, х/2 = (–1) n +1 π/6 + π n, n Z,

х1 = π/… + π k, k , х2 = (–1) n +1 π/… + 2π n, n Z.

Ответ: х1 = π/4 + π k, k , х2 = (–1) n +1 π/3 + 2π n, n Z.

Пример 4. Решите уравнение . Решение: , cos (4x – 3x) =, cos x =, x = n, nZ. Ответ: x = n, nZ.

Пример 5. Решите уравнение 2cos( х + π/3) = .

Решение: 2cos( х + π/3) = , cos( х + π/3) = , х + π/3 = ± 5π/6+2πn, nZ, x = π/3 ± 5π/6 + 2πn, nZ. x1 = π/3 + 5π/6+2πn, nZ, x1 = π/… +2πn, nZ,

x2 = π/3 5π/6+2πn, nZ, x2 = 7π/… +2πn, nZ.

Ответ: x1 = π/2 +2πn, nZ, x2 = –7π/6 +2πn, nZ.

Пример 6. Решите уравнение sin( 2х + π/2) = 0.

Решение: sin( 2х + π/2) = 0, 2х + π/2 = πn, nZ,2х = – + πn, nZ,х = , nZ.

Ответ: х = = – , nZ

Пример 7. Решите уравнение a) arccos б)arcsin (5x+2) = 0.

Решение: a) arccos cos(arccos ,

б) sin(arcsin (5x+2)) = sin0, 5x + 2 = 0, 5x = – 2,x = …

Ответ: a) 2,5, б) – 0,4.

Пример 8. Решите уравнение arctg (4x – 1) =

Решение: tg(arctg (4x – 1)) = tg 4x – 1 = 1, 4x = 2, x = …

Ответ: 0,5.

Пример 9. Решить неравенство sin(t) 1/2.

Решение: Рисуем единичную окружность. Так как sin(t) по определению - это координата y, отмечаем на оси Оу точку у = 1/2. Проводим через неё прямую, параллельную оси Ох. В местах пересечения прямой с графиком единичной окружности отмечаем точки Pt1 и Pt2. Соединяем двум отрезками начало координат с точками Pt1 и Pt2

Решением данного неравенства будут все точки единичной окружности расположенные выше данных точек. Другими словами решением будет являться дуга l. Теперь необходимо указать условия, при которых произвольная точка будет принадлежать дуге l.hello_html_m203b787d.jpg

Pt1 лежит в правой полуокружности, её ордината равна 1/2,

тогда t1= arcsin(1/2) = π/6. Для описания точки Pt1 можно записать следующую формулу: t2 = π – arcsin(1/2) = 7π/6. В итоге получаем

для t следующее неравенство:/6 t 7π/6, Мы сохраняем знаки неравенств. А так как функция синус функция периодичная, значит решения будут повторяться через каждые 2π. Это условие добавляем к полученному неравенству для t и записываем ответ.

Ответ: /6+2πn t 7π/6 + 2πn, при любом целом n.

Пример 10. Решить неравенство cos(t) <1/2.

Решение: Нарисуем единичную окружность. Так как согласно определению cos(t) это координата х, отмечаем на грфике на оси Ох точку x = 1/2.
Проводим через эту точку прямую, параллельную оси Оу. В местах пересечения прямой с графиком единичной окружности отмечаем точки P
t1 и Pt2. Соединяем двум отрезками начало координат с точками Pt1 и Pt2.
hello_html_mecb7ec3.jpg

Решениями будут все точки единичной окружности, которые принадлежать

дуге l. Найдем точки t1 и t2t1 = arccos(1/2) = π/3 ,

t2 = 2π arccos(1/2) = 2π/3 = 5π/3.

Получили неравенство для t: π/3 < t < 5π/3.

Так как косинус - это функция периодичная, то решения будут повторяться через каждые 2π. Это условие добавляем к полученному неравенству для t и записываем ответ.hello_html_m174f9f6b.jpg

Ответ: π/3+2πn π/3+2πn, для любого целого n.

Пример 11. Решить неравенство tg(t) 1.

Решение: Период тангенса равняется π. Найдем решения, которые принадлежат промежутку (-π/2; π/2) правая полуокружность. Далее воспользовавшись периодичностью тангенса, запишем все решения данного неравенства. Нарисуем единичную окружность и отметим на ней линию тангенсов.

Если t будет являться решение неравенства, то ордината точки Т = tg(t) должна быть меньше или равна 1. Множество таких точек будет составлять луч АТ. Множество точек Pt, которые будут соответствовать точкам этого луча – дуга l. Причем, точка P(-π/2) не принадлежит этой дуге. Найдем условие, при котором некоторая точка Pt будет принадлежать дуге l. t1 = arctg(1) = π/4.  Получаем неравенство /2 t/4. 

Учитывая период тангенса записываем ответ.

Ответ: /2 + πnt /4 + πn, для любого целого n.

Пример 12. Решить неравенство:   sin x > 0.

Решение: В пределах одного оборота единичного радиуса это неравенство справедливо

при 0 < x < hello_html_3fac242e.gif. Теперь необходимо добавить период синуса  2hello_html_3fac242e.gif n :hello_html_m12869a62.png

0 + 2πn х π + 2πn, 2πn х π + 2πn, при любом целом n.hello_html_6e343a75.png

Ответ: 2πn х π+ 2πn, при любом целом n.

Пример 13. а) Решить неравенство:   sin x > 0.5 .

Решение: π/6 + 2πn < х < 5π/6 + 2πn, для любого целого n.

б) Решить неравенство cosх > /2.

Решение: /4 + 2πn х π/4 + 2πn, для любого целого n.( по рис.)

Пример 14. Решить неравенство cos (x/4 – 1) ≤ (/2).

Решение: Обозначим x/4 – 1 = у. Решая неравенство cos у ≤ (/2), находим 
3π/4 + 2πn ≤ у ≤ 5π/4 + 2πn, n Z.

Заменяя у = x/4 – 1, получаем 3π/4 + 2πn ≤ x/4 – 1 ≤ 5π/4 + 2πn, откуда 
1 + 3π/4 + 2πn ≤ x/4 ≤ 1 + 5π/4 + 2πn, 4 + 3π + 8 πn ≤ х ≤ 4 + 5π + 8 πn, n Z.

Ответ: 4 + 3π + 8 πn ≤ х ≤ 4 + 5π + 8 πn, n Z.


2)Решить задание ( по примерам):

  1. Решите уравнение sin4xcos2x = 0.

  2. Решите уравнение (2 sin x ) hello_html_m61765ba1.gif (tg x – ) = 0.

  3. Решите уравнение (ctg x ) hello_html_m61765ba1.gif (2sin +) = 0.

  4. Решите уравнение cos 4xcos3x + sin4xsin3x =

  5. Решите уравнение 2cos(х + π/4) = –

  6. Решите уравнение sin( 2х + π/3) = 0.

  7. Решите уравнение a) arccos б)arcsin (5x + 4) = 0.

  8. Решите уравнение arctg (3x – 1) =

  9. Решить неравенство cos x ≤ - .

  10. Решить неравенство sin x > - .

  11. Решить неравенство sin x ≥ .

  12. Решить неравенство ctg x < - .

  13. Решить неравенство tg x > 1.

  14. Решить неравенство tg x ≤ 1.

  15. Решить неравенство ctg x ≥ - .

  16. Решить неравенство сtg x ≤ 1.

3)Решить задание :

  1. Решите уравнения:

  1. Решите уравнения:

  1. Решите уравнения:

  1. Решите уравнения:

  1. Решите уравнения:

  2. Решите уравнения:

  1. Решите уравнения:

  2. Решите уравнения:

  3. Решите уравнения:


  1. Решите уравнения:

  2. Решите уравнения:

  1. Решите уравнения:

  1. Решите неравенства:

а) ,б) , в) .

  1. Решите неравенства:

а) ;б) ;в) ;

  1. Найдите какой-либо корень уравнения , удовлетворяющий неравенству .

  2. Решите неравенство: .

  3. Решите неравенство:

  1. Решите неравенства:

а) б) в) .

  1. Решите неравенства:

а)

  1. Определите все а, при каждом из которых неравенство

4sinx + 3cosxа имеет хотя бы одно решение.




Просмотрено: 0%
Просмотрено: 0%
Скачать материал
Скачать материал "ПЗ по специальности Технология машиностроения,1 курс(2016-2017 уч.г.)"

Получите профессию

Методист-разработчик онлайн-курсов

за 6 месяцев

Пройти курс

Рабочие листы
к вашим урокам

Скачать

Выбранный для просмотра документ ПЗ№24.docx

ПЗ № 24. Обратные тригонометрические функции: арксинус, арккосинус, арктангенс.

Задание:

1) Перепишите и заполните пропуски:
Для выполнения заданий, связанных с обратными тригонометрическими функциями, нужно, во-первых, четко помнить определения этих понятий:


hello_html_mc5281e4.gif





,


hello_html_m3aca29e1.gif

Пример 1. Вычислить:

а) arccos (cos ), б) cos(arccos 0,4),в) arcsin (sin ),

г) sin(arcsin 0,6), д) sin (arccos 0,6),е)tg(arcsin 0,8).

Решение:

а) arccos (cos ) = , б) cos(arccos 0,4) = …,

в) arcsin (sin ) = …, г) sin(arcsin 0,6) = 0,6,

д) ,

е) .

Ответ:

а) arccos (cos ) = , б) cos(arccos 0,4) = 0,4,в) arcsin (sin ) = ,

г) sin(arcsin 0,6) = 0,6, д) , е) .

Пример 2. Вычислить cos(4arctg 5).

Решение:

Пусть α = arctg5, тогда tg α = 5. Требуется найти cos4α. Вычислим вначале cos2α, используя универсальную подстановку:

.


Тогда получаем, что:

.

Ответ: .

Пример 3. Вычислить  arcsin (sin 12).

Решение:

По условию задачи требуется найти угол, синус которого равен синусу угла в 12 радиан и который принадлежит промежутку   Заметим, что поэтому . Поскольку  , угол 12 - 4π является искомым углом: его синус равен sin 12, и он находится в области возможных значений арксинуса.

Ответ: arcsin (sin12) = 12 - 4π.

Пример 4. Вычислить  

Решение:

Введем два угла:  и . Оба они лежат в первой четверти, значит, все их тригонометрические функции положительны.

Мы знаем, что. Требуется найти синус суммы этих углов, а для этого нужно знать их синусы и косинусы.

Во-первых,

  .

Во-вторых, 

.

Следовательно,

Ответ:  .

Пример 5. Вычислить 

Решение:

Типичная ошибка в данном случае – это сразу же написать в ответ 4. Как мы указывали в предыдущем примере, для использования основных свойств аркфункций необходимо проверить соответствующие ограничения на их аргумент. Мы имеем дело со свойством:

при . Но 4> .

Главное на этом этапе решения не подумать, что указанное выражение не имеет смысла и его нельзя вычислить. Ведь четверку, которая является аргументом тангенса, мы можем уменьшить при помощи вычитания периода тангенса, и это не повлияет на значение выражения. Проделав такие действия, у нас появится шанс уменьшить аргумент так, чтобы он вошел в указанный диапазон.

 , т.к. < 1 поскольку > 3, следовательно, , т.к. .

Ответ: .


Пример 6. Вычислить sin (2 arcsin 0,6).

Решение:


Ответ: 0,96.

 Пример 7. Вычислить arccos xarcsin x = arccos .

Решение:

Учитывая, что arccos = и arcsin x + arccos x = , заменим в уравнении arcsin x выражением – arccos x, получим уравнение

arccos x – (– arccos x)= ,

2arccos x – = , 2arccos x = + = .

arccos x = , x = cos , x= 1/2 = …

Ответ: 0,5.

Пример 8. Решите уравнения:

а) 6arcsin (x2 – 6x+8,5) = π ;

б)  3arcsin2x – 10arcsinx + 3 = 0.    

Решение:

а) 6arcsin (x2 – 6x+8,5) = π  , arcsin(x2 – 6x+8,5) = ,

x2 – 6x+8,5 = 0,5; x2 – 6x+8 = 0,

D = 36 – 418 =….

, .

б) 3arcsin2x – 10arcsinx + 3 = 0.    arcsinx = у,

2 – 10у + 3 = 0, D = 100 – 433 = 64.

- не уд. усл.

.

arcsinx = 0,3, х= sin 0,3

Ответ: а) x1= 4,x2 = 2.б) х= sin 0,3

Пример 9. Вычислить: а) arcsin (-2), б) arccos

Решение:

а) Типичная ошибка в данном случае – это начать выносить минус и что-то упрощать. Первое, что необходимо заметить, это то, что аргумент арксинуса не входит в область определения.

Следовательно, данная запись не имеет значения, и вычислить арксинус нельзя.

б) Стандартная ошибка в данном случае заключается в том, что путают местами значения аргумента и функции и дают ответ 1/2. Это неверно! Конечно, возникает мысль, что в таблице косинусу  соответствует значение 1/2, но в таком случае перепутано то, что вычисляются аркфункции не от углов, а от значений тригонометрических функций. Т.е. arccos 1/2 =, а не arccos  = 1/2.

Кроме того, поскольку мы выяснили, что  является именно аргументом арккосинуса,

то необходимо проверить, чтобы он входил в область определения. Для этого вспомним, что >1,т.е.  , а значит арккосинус не имеет смысла и вычислить его нельзя.

Кстати, например, выражение arccos  имеет смысл, т.к. , но поскольку значение косинуса, равное  не является табличным, то и вычислить арккосинус с помощью таблицы нельзя.

Ответ: Выражения не имеют смысла.

 Пример 10. Вычислить arcсtg х, если известно, что arctg х =  .

Решение:

Вспомним, по какой формуле связаны между собой указанные функции:


И выразим из нее то, что нам нужно: .

Ответ: .

2)А)Решить задание ( по примерам):

  1. Вычислить:

а) arccos (cos ), б) cos(arccos 0,25),в) arcsin (sin ),

г) sin(arcsin 0,45), д) sin (arccos 0,8),е)tg(arcsin 0,6).

  1. Вычислить cos(4arctg 3).

  2. Вычислить  arcsin (sin 6).

  3. Вычислить  

  4. Вычислить 

  5. Вычислить sin (2 arcsin 0,8).

  6. Вычислить arccos x – arcsin x = arccos .

  7. Решите уравнения:

а) 6arcsin (x2 – 7x+12,5) = π ;

б)  3arcsin2x – 11arcsinx + 6 = 0.    

  1. Вычислить: а) arcsin (hello_html_m7cdd195e.gif 4), б) arccos

  2. Вычислить arcсtg х, если известно, что arctg х =  .


Б) Построить таблицы:

Табличные значения обратных тригонометрических функций.

hello_html_4c5b70ac.jpg

hello_html_m6baf672f.jpg

В) Преобразование выражений.

(Перепишите и заполните пропуски)









3)Решить задание ( по примерам):


hello_html_m46bcf841.gif

hello_html_m3156b9af.gif



4)Решить задание :

  1. Вычислите значения: a) cos; б)sin; в) sin

  2. Упростите выражение: а) arctg+ arctg; б) 3arcsin+ arcsin.

  3. Упростите выражение: а) arccos б) - arcsin;

  4. Решите уравнение: а) arcsin x =; б) arcsin x = arcctg x; в) arccos(x+1) = arcctg x; г) 4arccos xhello_html_24574eef.gif4arccos x hello_html_24574eef.gif 4arcsinx+=0;д) arcsin(x +1) + arcos2x =0;

Просмотрено: 0%
Просмотрено: 0%
Скачать материал
Скачать материал "ПЗ по специальности Технология машиностроения,1 курс(2016-2017 уч.г.)"

Получите профессию

Интернет-маркетолог

за 6 месяцев

Пройти курс

Рабочие листы
к вашим урокам

Скачать

Выбранный для просмотра документ ПЗ№25.docx

ПЗ № 25. Примеры зависимостей между переменными в реальных процессах из смежных дисциплин. Определение функций.

Задание:

1) Опорный конспект.

А) Примеры применения различных функций в жизни, технике, природе.

Определение. Числовой функцией с областью определения D называется соответствие, при котором каждому числу x из множества D сопоставляется по некоторому правилу единственное число y, зависящее от x. Принято называтьx независимой переменной или аргументом, а у — зависимой переменной или значением функции.

Записывают указанное соотношение между x и у в общем виде так: у = f (x) или у = F (x) и т. п.

График функции y = f (х) - это множество всех точек плоскости, координаты (х, у) которых удовлетворяют соотношению y = f(x).

Способы задания функции:

  1. аналитический (с помощью формулы);

  2. графический;

  3. табличный;

  4. словесный.

В наши дни без функций невозможно не только рассчитать космические траектории, работу ядерных реакторов, и бег океанской волны и закономерности развития циклона, но и экономично управлять производством, распределением ресурсов, организацией технологичных процессов, прогнозировать течение химических реакций или изменение численности различных взаимосвязанных в природе видов животных и растений, потому что все это – динамические процессы, которые описывает функция.

а) Линейная функция

Функция y = a x + b называется линейной потому, что ее график есть прямая линия. Характеристическим свойством линейной функции является изменение функции пропорционально изменению аргумента. Поэтому с помощью линейной функции описываются пропорциональные зависимости. Например, при равномерном движении с постоянной скоростью v пройденный путь s пропорционален времени t и выражается формулой s = v t, т.е. s – линейная функция t.hello_html_31dd52b1.png

Пример линейной функции дает зависимость между различными шкалами температур. Абсолютная температураТ (по Кельвину) связана с температурой tͦC на шкале Цельсия формулой t = T + 273 ͦ. Другой пример – напряжение в электрической цепи прямо пропорционально силе тока U = IR. Можно много приводить примеров линейных зависимостей в физике, химии. Рассмотрим задачу на линейное расширение тел.

Задача. При температуре 0оС рельс имеет длину l0= 12,5 м. при возрастании температуры происходит тепловое расширение рельса и его длина, выраженная в метрах, меняется по закону l(tо) = l0(1 + hello_html_m5cdf3cb6.giftо),где hello_html_m5cdf3cb6.gif= 1,2 ˖ 10–5коэффициент теплового расширения в градусах Цельсия в минус первой степени, tо – температура (в градусах Цельсия). При какой температуре рельс удлинится на 6 мм. Ответ выразить в градусах Цельсия.

Решение. Выразим из заданной формулы t: .

Заметим, ,

тогда hello_html_53e6f54e.gif

Ответ: 40.

б) Квадратичная функция

Графиком квадратичной функции

является парабола.

Хорошо известно, что траектория камня, брошенного под углом к горизонту, летящего футбольного мяча или артиллерийского снаряда будет параболой (при отсутствии сопротивления воздуха). Однако мало кто знает, что зона достижимости для пущенных нами камней вновь будет параболой. В данном случае мы говорим об огибающей кривой траекторией камней, выпущенных из данной точки под разными углами, но с одной и той же начальной скоростью. Если рассматривать такую огибающую в пространстве, то возникнет поверхность, образованная вращением этой параболы вокруг ее оси. Такая поверхность носит название параболоида вращения.

Задача. Выcота над землeй подброшенного вверх мяча меняетcя по закону hello_html_403f5e9b.png, где h — выcота в метрах, t — время в cекундах, прошедшее c момента броcка. Cколько cекунд мяч будет находитьcя на выcоте не менее трeх метров?

Решение. Решим неравенство,

, , t1 = 1,4, t2 = 0,2

0,2t1,4

t = 1,4 - 0,2 = 1,2

Ответ: 1,2

Парабола обладает оптическим свойством: все лучи, исходящие из источника света, находящегося в фокусе параболы, после отражения оказываются направленными параллельно ее оси. Это свойство параболы используется при изготовлении прожекторов, автомобильных фар, карманных фонариков, зеркала которых имеют вид параболоидов вращения. hello_html_m779371d5.png

Б)Функциональные зависимости в повседневной жизни

Пример 1. Рассмотрим деление праздничного торта между гостями. Отчего зависит количество порций?– от числа гостей. А от чего зависит вес порции? – тоже от числа гостей.

В первом случае, чем больше гостей, тем на большее количество порций мы должны разделить торт (рис. 1).hello_html_m9d34e61.png

Здесь наглядно можно представить прямую пропорциональную зависимость.hello_html_m70d6cf06.gif

Во втором случае, чем больше гостей, тем меньше вес порции. Здесь мы видим обратную пропорциональную зависимость (рис. 2).

Пример 2. Мы живём в век информационных технологий. Ежедневно мы получаем массу информации из различных источников: телевидения, радио, газет, журналов, и, конечно, из Интернета. Известно, что объём информации каждые пять лет увеличивается в два раза.hello_html_m64a6d6cd.png

Если построить график зависимости объёма информации от времени, то получим некоторую кривую, которая в математике называется экспонентой и является графиком показательной функции (рис. 3).

Пример 3.На голове человека растут волосы, которые регулярно стригут.hello_html_fde3c9e.png

График полученной зависимости (при условии, что стрижку делают регулярно) похож на функцию дробной части числа, смещённую на aединиц вверх: (рис. 4).


Пример 4. За время обучения в школе каждый год переходим в следующий класс.hello_html_4bc21f47.png

Такая зависимость сходна с функцией целой части числа на ограниченном промежутке (рис. 5).

Пример 5. По графикам оценить

множество значений каждой из

представленных функций.



hello_html_m321966bf.gif

Ответ:

hello_html_5687e039.gif

2)Решить задание:

1)

hello_html_20988e1e.jpg

2) Функция задана формулой f(х) = - 5х – 2. Найдите: а) f(0); б) f(2); в) f(- 3); г) f().

3)Известно, что g(х) = 12 – 4х. Найдите значение х, при котором: а) g(х) = 0; б) g(х) = 6; в) g(х) = - 8.

4)Функция задана формулой f(х) = . Найдите: а) f(0); б) f(2); в) f(- 3); г) f().

5)Постройте график функции у = х +3. При каких значениях х выполняется неравенство ?

6)С помощью формул описано изменение температуры воды в баке (в 0С) как функции времени t (в минутах):

2t + 20, если 0 ≤ t < 40,

p = 100, если 40 ≤ t < 60,

t + 140, если 60 ≤ t ≤ 150.

Найдите: р (20); р (40); р (50); р (60); р (90). Постройте график функции р = f (t). Какой физический смысл имеет рассматриваемый процесс в каждом из промежутков [0;40], [40;60], [60;150]?

7)Известно, что g(х) = 2х – 4. Найдите значение х, при котором: а) g(х) = 0; б) g(х) = 6; в) g(х) = - 8.

8)Зависимость расстояния s (в километрах) велосипедиста до базы от времени его движения t (в часах) задана следующим образом:

2t + 20, если 0 ≤ t < 40,

s = 100, если 40 ≤ t < 60,

t + 140, если 60 ≤ t ≤ 150.

Найдите: s (0); s (1); s (1,4); s (2). Постройте график функции s = f (t) (масштаб по оси t: 1 ед. – 6 клеточек; по оси s: 10 ед. – 4 клеточки). Опишите, как происходило движение велосипедиста.

9) По графикам оценить множество значений каждой из представленных функций.


hello_html_m5e890c1e.gif

10) Садово-огородные процессы тоже можно представить в виде функции и потроить график. К примеру, яблоко росло, зрело, потом его высушили. Постройте эту кусочную функцию.hello_html_m6473b701.gif

11) Графиком проиллюстрировать смысл пословицы «Каково жизнь проживешь, такую славу наживешь».

12) 1. По графику функции у = 2х2 найдите:

а) значение функции, если х = 10; х = – 12;

б) значение аргумента, если у = 4; у = – 5.

2. Постройте график функции у = – 0,5х2

13) Постройте график функции:

.

14) Вычислите координаты точек пересечения параболы и прямой:

.

15) Постройте график функции:

.


Просмотрено: 0%
Просмотрено: 0%
Скачать материал
Скачать материал "ПЗ по специальности Технология машиностроения,1 курс(2016-2017 уч.г.)"

Получите профессию

Бухгалтер

за 6 месяцев

Пройти курс

Рабочие листы
к вашим урокам

Скачать

Выбранный для просмотра документ ПЗ№26,27.docx

ПЗ № 26. Построение и чтение графиков функций. Исследование функции.

Задание:

1)Опорный конспект.

Построение и чтение графиков.

Построение и чтение графиков должно опираться на множество практических навыков.

Приведем некоторые из рекомендованных правил при графической обработке данных и чтении графиков:

- во всякой диаграмме графический образ, как основной элемент, для которого существуют и которому подчинены все остальные элементы, должен быть в центре внимания пользователя;

- композиция диаграммы должна подчиняться правильному соотношению ее частей (согласованию их размеров, толщины, формы и положения);

- график должен быть достаточно четким, но наиболее важные его элементы должны выделяться на общем фоне в соответствии с их значением;

- вертикальную шкалу для кривой независимо от ее назначения желательно выбирать так, чтобы на диаграмме оказалась нулевая отметка;

- нулевые линии шкал для кривой следует резко отграничивать от других координатных линий;

- когда шкала относится к датам, а представляемый период является неполным, лучше не выделять первые и последние ординаты, так как подобная диаграмма не отмечает начало или конец времени;

- для кривых, которые имеют шкалу, изображающую проценты, желательно выделить каким-то образом линию 100% или другие линии, используемые в качестве основы для сравнения, а также обязательно показывать 0%;

- кривые линии диаграммы должны резко отличаться от прямых линий;

- для кривых, характеризующих группы наблюдений, рекомендуется по возможности ясно указывать на диаграмме все кривые, представляющие отдельные наблюдения;

- горизонтальную шкалу для кривых следует писать слева направо, а вертикальную - снизу вверх;

- рекомендуется показывать достаточный минимум координатных линий для облегчения чтения диаграммы;

- при необходимости желательно включать в график цифровые данные или формулы;

- на графиках при резких колебаниях кривых закраска полос неудобна;

- использование изображения линейных величин с помощью площадей или объемов некорректно, т.к. их не удастся правильно истолковать;

- характер координатной сетки должен быть разный в зависимости от назначения графического образа.

- название графика располагают под ним, хотя иногда его можно писать выше диаграммы, если она не предназначена для печати, например, в настенных графиках;

- наименования следует формулировать возможно яснее и полнее. Если требуется, необходимо дополнительно вводить подзаголовки или пояснения;

- общая структура графиков должна предполагать чтение слева направо;

- чтение графика следует начинать с заголовка, сообщающего, какие сведения можно из него получить. Затем надо уяснить строение графического образа и изучить специфику его элементов: шкалу, масштабы, единицы измерения, легенды и т.п., что необходимо для определения сообщаемой информации по частным вопросам. Нужно начинать восприятие графического образа как общего целого, т. е. во взаимоотношениях элементов. Затем надо увидеть выражаемое содержание графика, ясно представлять, чему соответствуют те или иные изменения графического образа.hello_html_m188e6616.png

Примеры графиков.

1)Оценка ущерба из-за загрязнения окружающей среды при производстве электроэнергии (ленточная диаграмма сравнения с логарифмической шкалой).

2)По графику найдите:hello_html_m63b1a32.png

а)Какова область определения функции?

б)Назовите множество значений функции.

в)Назовите нули функции.

г) Назовите точки максимумов функции.

д)Назовите точки минимумов функции.


Ответ:

а) ,б) ,в) – 4, - 2,0,2,4 , г) – 3 ;1, д) – 1 ;3.

3) Построить график функции hello_html_4124e417.gif на основании результатов исследования функции.

Решение:

Для построения графика функции исследуем ее, придерживаясь общей схемы исследования.

  1. Нахождение области определения:

hello_html_m40b31e73.gif.

  1. Определение четности или нечетности:

hello_html_m6a4e5b1f.gif. Функция hello_html_4124e417.gif — четная.

Дальнейшее исследование будем проводить для hello_html_m4066d436.gif.

  1. Область изменения функции.

Если x = 1, то y = 0. Если hello_html_6087da64.gif. Следовательно, hello_html_402daa4e.gif.

hello_html_m2a046132.gif

  1. Пересечение с координатными осями.

Пересечений с осью 0y нет, т.к. x = 0 не входит в область определения функции.

  1. Выделение промежутков монотонности.

Для x1 > x2  ≥ 1 рассмотрим разность:

hello_html_62b3d6c1.gif.

При возрастании значений x от 1 до ∞ значения y возрастают.hello_html_m7fc1e073.gif

  1. Нахождение корней функции и промежутков знакопостоянства.

Если hello_html_191efdd0.gif при x = 1, hello_html_m77f109f8.gif при всех hello_html_156ca2f3.gif.

По результатам исследований строим график функции hello_html_4124e417.gifhello_html_m287b4e73.png

4) Построить графики следующих функций: 

Решение:

  1. Рассмотрим функцию. Она определена на всей оси 0x, четная. Ее график состоит из двух лучей, выходящих из начала координат и направленных по биссектрисам I и II координатных углов.hello_html_4a52d6cc.png

  2. Обратимся к функции hello_html_96be002.gif. Она определена на всей оси 0x, четная. На рисунке приведен ее график, причем он построен из двух половинок y = 2x , при x ≥ 0 и y = 2-x , при x < 0.

5) По графику выполните задание:

hello_html_m46e816e6.pnghello_html_m3316d847.png

hello_html_535ea5db.png

Ответ: 4,4,24;3,8;6;2,5.

6)

hello_html_m2930b060.jpg

7) По графику выполните задание:

hello_html_48efe2fd.png

hello_html_c75446b.pnghello_html_m72dfc30b.pnghello_html_142e8802.pnghello_html_2cc50b8.png

2)Решить задание:

  1. По графику функции найдите:

а)Какова область определения функции?

б)Назовите множество значений функции.

в)Назовите нули функции.

г) Назовите точки максимумов функции.

д)Назовите точки минимумов функции.

  1. Функция задана формулой . Найдите значение , при котором () =0.

  2. Определите при каких значениях существует функция .

  3. Постройте график квадратичной функции и запишите ее свойства (область определения функции, область значений функции, нули функции, промежутки знакопостоянства, промежутки монотонности):

  4. По графику выполните задание:

hello_html_m3575ae9b.pnghello_html_m5d77f20a.png

hello_html_m60d1b10c.pnghello_html_3ec17e75.png

  1. Определите при каких значениях существует функция .hello_html_m18d33162.png

  2. По графику функции найдите:

а) множество значений функции;

б) значение аргумента при у (– 3; 2];

в) промежутки знакопостоянства функции;

г) точки экстремума функции.

  1. Функция задана формулой . Найдите (-5).hello_html_404ccfeb.gif

  2. Функция задана формулой . Найдите (-3).

  3. Функция задана формулой . Найдите значение , при котором () =0.

  4. Постройте график функции у = 2х + 3 .

  5. По графику функции у = найдите:

а) ;б) значение аргумента, если у = 3; у = 1,5.hello_html_mbde74e8.png

  1. Проходит ли график функции у = через точки

А(13; 196 ); В(7; 49); С(–10; 100).

  1. На рисунке изображены графики функций y = f(x) и y = g(x), заданных на промежутке [-9; 8]. Укажите те значения х, при которых выполняется неравенство f(x) < g(x).



ПЗ № 27. Свойства линейной, квадратичной, кусочно-линейной и дробно-линейной функций.

Задание:

1)Опорный конспект.

Переменная величина y называется функцией переменной величины x, если каждому значению x соответствует определенное значение y.

Множество всех тех значений, которые принимает аргумент x функции y = f (x) , называется областью определения этой функции.

Множество всех тех значений, которые принимает сама функция y = f (x) , называется областью значений (изменения) этой функции.

Функция y = f (x)   называется четной, если при всех значений x из области определения этой функции f (- x) = f (x).

Функция y = f (x)   называется нечетной, если при всех значений x из области определения этой функции f (- x) = - f (x).

Функция y = f (x)   называется возрастающей (убывающей)  на данном промежутке, если при произвольных двух различных значениях аргумента, из данного промежутка, большему значению аргумента соответствует большее (меньшее) значение функции.

Функция y = f (x)   называется периодической, с периодом T, где T ≠ 0, если значение функции не изменяется при прибавлении числа T к любому допустимому значению аргумента:

 f ( x +T) = f (x).

Функция y = f (x)   называется ограниченной, если можно указать такое положительное число M, что hello_html_m66ebaed3.gifдля всех значений x из области определения функции. Если же точка M не существует, то функция называется неограниченной.

Графиком функции y = f (x)   называется множество всех точек плоскости,

координаты которых (x, f (x)). Функцию вида y = ax2 + bx + c называют квадратичной. Графиком квадратичной функции является кривая, называемая параболой.

Точку с координатами (- b/2a, - (b2 – 4ac) / 4a) называют вершиной параболы.

Соответствие между элементами двух множеств X и Y, при котором каждому элементу множества X сопоставляется не более одного элемента Y, называется функцией.

Отсюда следует, что понятие функции имеет три главных компонента:

множество X (которое называется областью определения функции);

множество Y (которое называется областью значений функции);

закон соответствия (который иногда называется функциональной зависимостью).

При этом закон соответствия может быть задан любым способом: таблицей, графиком, формулой или как-то иначе, например, при помощи словесного описания.

Если функцию задают формулой, то при этом фактически указывают область определения функции и закон соответствия (область значений функции не указывается явно, так как она устанавливается, исходя из данной формулы).

Областью определения функции, заданной явной аналитической формулой, считают множество всех тех значений аргумента, для которых все указанные в формуле операции выполнимы.

Опишем далее способы построения графиков функций.

1 способ «по точкам». Вытекает из определения графика функции. Он является длинным и недостаточно надежным. Применяется в школьном курсе математики при первоначальном знакомстве с простейшими функциями.

2 способ «путем сдвига графиков основных функций или сдвига осей координат». Чтобы построить график функции y = f (x)  + c можно или график функции y = f (x)   сдвинуть вдоль оси 0y на c единиц в сторону, совпадающую со знаком c , или перенести параллельно ось 0y в сторону, противоположную знаку c.

Чтобы построить график функции y = f (x + b) , можно или график функции y = f (x)   вдоль оси 0x на b единиц в сторону, противоположную знаку b, или перенести параллельно ось 0y в сторону, совпадающую со знаком b.

3 способ, «путем симметричного отображения относительно осей координат». Чтобы построить график функции y = - f (x)   , можно построить изображение, симметричное графику функции y = f (x)   относительно оси абсцисс.

Чтобы построить график функции y = f ( - x) , можно построить изображение, симметричное графику функции y = f (x)   относительно оси ординат.

4 способ, «путем деформирования графиков основных функций». Чтобы построить график функции y =a f (x)   при a > 0, можно график исходной функции растянуть (сжать) вдоль оси ординат, если a > 1 (0 < a < 1).

Чтобы построить график функции y = f (bx)    при b > 0, можно график исходной функции растянуть (сжать) вдоль оси абсцисс, если b > 1 (0 < b < 1).

5 способ. «Способы построения графиков функций, аналитическое выражение которых содержит знак абсолютной величины».

  1. Функция четная. Чтобы построить ее график, достаточно построить для всех неотрицательных значении аргумента график функции y = f (x), а затем достроить его левую часть, симметричную правой относительно оси ординат.

  2. Рассмотри далее, как строить функцию. Можно данную функцию рассматривать как совокупность двух функций:  .

  3. Чтобы построить график функции, достаточно построить график функции y = f (x)  и ту часть графика, которая расположена в нижней полуплоскости, симметрично отразить относительно оси абсцисс.

  4. Вспомним, как строится функция.

Функция  четная. Построить для всех неотрицательных значений аргумента график функции y = f (x)   , затем его симметрично отразить относительно оси ординат, и, наконец, ту часть полученного графика, которая расположена в нижней полуплоскости, симметрично отразить относительно оси абсцисс.

6 способ. «Кусочно-линейная функция».

Графиком кусочно-линейной функции является ломаная линия. Для построения графика находят уравнения звеньев ломаной.

Пример 1. Построить график функции hello_html_m13626efb.png 

Решение:

Дана функция hello_html_m13626efb.png. Преобразуем hello_html_m331da3eb.png

  1. Область определения: hello_html_5b28aadc.gif

  2. Область значений: hello_html_m64d87c78.gif

  3. Четность, нечетность: и ни четная, ни нечетная hello_html_m493e4791.gif

  4. Монотонность: убывает во всей области определения hello_html_m247f5f00.gif.

  5. Пересечение с осями 0x и 0yhello_html_m6dc57d6.gif

  6. Промежутки знакопостоянства:

hello_html_m3b988a6f.gif,hello_html_509010ce.gif.hello_html_m64191655.png

  1. Поведение функции вблизи точек разрыва и при hello_html_m263828a.gif:

hello_html_m64b6c4e7.gif

По результатам решения строим график.

Комментарий. При построении графика функции следует найти точки, в которых он пересекает оси координат, а также выяснить поведение функции при x, стремящемся к hello_html_m4087d48.gif в случае, когда ее область определения не ограничена. Необходимо также исследовать поведение функции вблизи тех точек, в которых она не определена.

Пример 2.Примеры функций.

hello_html_m7e89d1ab.jpghello_html_587b5025.png


2)Решить задание:

  1. С помощью графиков определите, сколько решений имеет система уравнений:

  2. Постройте график функции у = х2 + х – 6.

Используя график, решите неравенство х2 + х – 6 < 0.

  1. Постройте график функции у = 2х2 найдите:

а) значение функции, если х = 10; х = – 12;hello_html_42f57f2d.png

б) значение аргумента, если у = 4; у = – 5.

  1. Постройте график у = найдите:

а) значение y, если х = 10; х = – 8;

б) значение x, если у = 11;у = – 7.

  1. По графику функции у = 0,5х + 3 найдите:

а) значение функции, если х = 10; х = – 12;

б) значение аргумента, если у = 4; у = – 5.

  1. Постройте график функции у = – 2х

  2. Решите графически систему уравнений:

  1. Постройте график функции у = 0,5х2 найдите:а) значение функции, если х = 10; х = – 12;

б) значение аргумента, если у = 4; у = – 5.

  1. Постройте график функции:

.

  1. Постройте график функции и запишите ее свойства (область определения, область значений, нули функции, промежутки знакопостоянства, промежутки монотонности):

  1. Функция задана формулой . Найдите (-3).

  2. Функция задана формулой . Найдите значение , при котором () =0.

  3. Вычислите координаты точек пересечения параболы и прямой:

.

  1. Найдите область значения функции .

  2. Функция задана формулой . Найдите (-5).

  1. Для каждой функции, заданной формулой, укажите график.

1) у = х – 1 2) у = – х + 1 3) у = х2 – 1

hello_html_m7e6c4af8.gifhello_html_6cdb33ee.gifhello_html_65e33a5c.gif

а) б) в)



  1. Постройте график функции: .

  2. Функция задана формулой f(х) = - 2х –12. Найдите: а) f(0); б) f(2); в) f(- 3); г) f().

  3. Известно, что g(х) = 12 – 6х. Найдите значение х, при котором:

а) g(х) = 0; б) g(х) = 6; в) g(х) = - 8.

  1. Функция задана формулой f(х) = . Найдите: а) f(0); б) f(2); в) f(- 3); г) f().

  2. Постройте график функции у = х +2. При каких значениях х выполняется неравенство ?

  3. С помощью формул описано изменение температуры воды в баке (в 0С) как функции времени t (в минутах):

2t + 20, если 0 ≤ t < 40,

p = 100, если 40 ≤ t < 60,

t + 140, если 60 ≤ t ≤ 150.

Найдите: р (20); р (40); р (50); р (60); р (90). Постройте график функции р = f (t). Какой физический смысл имеет рассматриваемый процесс в каждом из промежутков [0;40], [40;60], [60;150]?

  1. Известно, что g(х) = 2х – 6. Найдите значение х, при котором: а) g(х) = 0; б) g(х) = 6; в) g(х) = - 8.

  2. Постройте график функции и запишите ее свойства (область определения, область значений, нули функции, промежутки знакопостоянства, промежутки монотонности):

  1. Постройте график функции у = найдите:

а) значение у, если х = 10; х = – 8; б) значение х, если у = 11; у = – 7.

hello_html_5f6884f1.png


hello_html_5f6884f1.pnghello_html_5f6884f1.png

Просмотрено: 0%
Просмотрено: 0%
Скачать материал
Скачать материал "ПЗ по специальности Технология машиностроения,1 курс(2016-2017 уч.г.)"

Получите профессию

Экскурсовод (гид)

за 6 месяцев

Пройти курс

Рабочие листы
к вашим урокам

Скачать

Выбранный для просмотра документ ПЗ№28,29.docx

ПЗ № 28. Непрерывные и периодические функции. Свойства и графики синуса, косинуса, тангенса и котангенса.

Задание:

1)Перепишите и заполните пропуски (используя графики):

hello_html_2271e06c.gifhello_html_ma7f3704.gif


Пример 1.Определить возрастает или убывает функция: а) y = cos x при ,hello_html_m70210242.png

б) y = sin x при ,

в) у = tg x при .

Решение: а) убывает,…,б) …, возрастает, в) возрастает.

Ответ: а) …, возрастает, б) убывает, …, в) ...

Пример 2.Нацдите х, при котором функция пересекает ось ох: а) y = cos x при ,

б) y = sin x при ,в) у = tg x при .

Решение: а) ,б) ,в) . Ответ: а) ,б) ,в) .

Пример 3. Определить принимает положительные или отрицательные значения функция:

а) y = cos x при ,б) б) y = sin x при ,в) у = tg x при .

Решение: а) положительные при , отрицательные при ,

б) положительные при , отрицательные при ,

в) положительные при , отрицательные при .

Ответ: а) положительные при , отрицательные при ,

б) положительные при , отрицательные при ,

в) положительные при , отрицательные при .

Пример 4.Сравнить а) и , б) и .

Решение: а) <, (0< < < ),б) >.Ответ: : а) <,б) >.

Пример 5.Построить график функции по таблице:

а) y = 2cos x , б) y = 6sinx .

х


0




х


0




у

0

2

0

2

0

у

6

0

6

6

0



Решение:

а) б)

hello_html_mab2a1ac.jpghello_html_m5dd79f1d.jpg

2)Решить задание ( по примерам):

  1. Определить возрастает или убывает функция: а) y = cos x при ,

б) y = sin x при ,в) у = tg x при.

  1. Нацдите х, при котором функция пересекает ось ох: а) y = cos x при ,

б) y = sin x при,в) у = tg x при .

  1. Определить принимает положительные или отрицательные значения функция:

а) y = cos x при ,б) б) y = sin x при ,в) у = tg x при.

  1. Построить график функции по таблице:

а) y = 6cos x . б) y = 4sinx .

x


0




y

0

6

0

6

0

x

0





y

0

4

0

4

0


  1. Сравнить а) и , б) и .

3)Решить задание :

  1. Построить график функции y = ctg x ; запишите свойства этой функции, используя свойства функции y = tg x, и то что эти функции взаимо обратны.

  2. Сравнить числа: а) и, б) tg 2,3 и tg 3, в) и, г) tg 1 и tg 1,5.

  3. Построить график функции по таблице: y = sin 4x .


x

0





y

0

1

0

1

0


  1. Построить график функции по таблице: y = cos 4x.


x

0





y

1

0

1

0

1


  1. Построить график функции по таблице: y = tg 2x .


x

0





y

0

1


1

0


  1. Построить график функции:

а) y = sin 2x , б) y = 2sin x , в) y = cos 2x , г) y = 5sin x , д) y = 4cos x,е) y = 2сtg x .

  1. Построить график функции:

а) y = sin 4x, б) y = cos 4x, в) y = tg 2x, г) y = 2sin x, д) y = 2cos x, е) y = 2tg x.

ПЗ № 29. Обратные функции и их графики. Обратные тригонометрические функции.

Задание:

1)А)Опорный конспект.

Определение обратной функции.

Пусть функция hello_html_m94574d3.gif строго монотонная (возрастающая или убывающая) и непрерывная на области определения hello_html_m5c67524f.gif, область значений этой функции hello_html_m1ceea025.gif, тогда на интервале hello_html_m4d9ff88d.gif определена непрерывная строго монотонная функция hello_html_6b07b91.gif с областью значений hello_html_m2f63b715.gif, которая является обратной дляhello_html_m94574d3.gif.

Другими словами, об обратной функции hello_html_6b07b91.gif для функции hello_html_m94574d3.gif на конкретном промежутке имеет смысл говорить, если на этом интервале hello_html_m94574d3.gif либо возрастает, либо убывает.

Функции f и g называют взаимно обратными.

Пример1. Найти функцию обратную для hello_html_2dd2d5b0.gif.

Решение.

Областью определения и областью значений этой функции является все множество действительных чисел. Выразим x через y (другими словами, решим уравнение hello_html_2dd2d5b0.gif относительно x ).

hello_html_m6133d6d7.gif - это и есть обратная функция, правда здесь y – аргумент, а x – функция этого аргумента. Чтобы не нарушать привычки в обозначениях (это не имеет принципиального значения), переставив буквы x и y , будем писать hello_html_10fbd097.gif.hello_html_4ce77a33.gif

Таким образом, hello_html_2dd2d5b0.gif и hello_html_10fbd097.gif - взаимно обратные функции.

Приведем графическую иллюстрацию взаимно обратных линейных функций.

Очевидно, что графики симметричны относительно прямой y=x (биссектрисы первого и третьего квадрантов).

Пример2. Найти функцию обратную для hello_html_38039a0f.gif.

Решение.

Областью определения этой функции является все множество действительных чисел, областью значений является интервал hello_html_4027ae8.gif. Выразим x через y (другими словами, решим уравнение hello_html_38039a0f.gif относительно x). hello_html_m3d98c9db.gif - это и есть обратная функция. Переставив буквы x и y , имеем hello_html_5b975f78.gif.hello_html_m61499e94.png

Таким образом, hello_html_38039a0f.gif и hello_html_5b975f78.gif - показательная и логарифмическая функции есть взаимно обратные функции на области определения.

График взаимно обратных показательной и логарифмической функций.
Перечислим свойства взаимно обратных функций hello_html_m94574d3.gif и hello_html_6b07b91.gif.

hello_html_m1bccfa33.gif и hello_html_m57c27ae5.gif.

Из первого свойства видно, что область определения функции hello_html_m94574d3.gifсовпадает с областью значений функции hello_html_6b07b91.gif и наоборот.

Графики взаимно обратных функций симметричны относительно прямой y=x.

Если hello_html_m94574d3.gif возрастает, то и hello_html_6b07b91.gif возрастает, если hello_html_m94574d3.gif убывает, то и hello_html_6b07b91.gif убывает.

Примеры нахождения взаимнообратных функций.

1)Для степенной функции hello_html_m3aecfc2f.gif при hello_html_m3f0c2e4.gif обратной является также степенная функция hello_html_2c09c577.gif Если заменить буквы, то получим пару взаимно обратных функций hello_html_m3aecfc2f.gif и hello_html_2c09c577.gif

Графики для положительных а и отрицательных а.
hello_html_2cbbb011.gif

2) Взаимно обратные показательная и логарифмическая функции hello_html_m4ab91685.gif и hello_html_2adc860c.gif, графики.

Подразумеваем, что а положительное и не равное единице число.

Графики для hello_html_m10ba216.gif и для hello_html_6b08bef4.gif
hello_html_3ffd9e74.gif

3) Взаимно обратные тригонометрические и обратные тригонометрические функции.

а)График главной ветви синуса и арксинуса (светлая область).
hello_html_m1494567d.gif hello_html_47303997.pngy = arcsin x

б)График главной ветви косинуса и арккосинуса (светлая область).
hello_html_m21877848.gif hello_html_m5cdd44d0.png y = arccos x


в) График главной ветви тангенса и арктангенса (светлая область).
hello_html_m4f9321da.gif hello_html_m50d8df33.png y = arctg x


г) График главной ветви котангенса и арккотангенса (светлая область).
hello_html_6b553bd4.gif hello_html_67f26bab.png y=arcctgx

Если Вам потребуются обратные функции для ветвей тригонометрических функций, отличных от главных, то соответствующую обратную тригонометрическую функцию нужно будет сдвинуть вдоль оси ординат на необходимое количество периодов.hello_html_m27c17baf.gif

Например, если Вам потребуется обратная функция для ветви тангенса на промежутке hello_html_m9b7177e.gif (эта ветвь получается из главной ветви сдвигом на величину hello_html_7d78f2fc.gifвдоль оси ох ), то ей будет являться ветвь арктангенса, сдвинутая вдоль оси oy на hello_html_7d78f2fc.gif.

Б) Построить таблицы:

Табличные значения обратных тригонометрических функций.

hello_html_4c5b70ac.jpg

hello_html_m6baf672f.jpg

В) Преобразование выражений. (Перепишите и заполните пропуски)





4) Вычислить без калькулятора


2)Решить задание ( по примерам):



hello_html_m46bcf841.gif

3)Решить задание :

В)1. Найдите значение выражения: а) arcsin1; б) arccos; в) arctg(); г)arcctg0.

2. Найдите значение выражения: а) arcsin; б) arccos0; в) arctg; г) arcctg.

3. Построить графики функций:

а) y =2 arcsin x ,б) y = 3arccos x,в) y = 2arctg x



Просмотрено: 0%
Просмотрено: 0%
Скачать материал
Скачать материал "ПЗ по специальности Технология машиностроения,1 курс(2016-2017 уч.г.)"

Получите профессию

Копирайтер

за 6 месяцев

Пройти курс

Рабочие листы
к вашим урокам

Скачать

Выбранный для просмотра документ ПЗ№3.docx

ПЗ № 3. Признаки и свойства параллельных и перпендикулярных плоскостей. Расстояние от точки до плоскости, от прямой до плоскости, расстояние между плоскостями, между скрещивающимися прямыми, между произвольными фигурами в пространстве.

Задание:

1)Перепишите и заполните пропуски:

А)Пример 1. Два отрезка длин а и b упираются концами в две параллельные плоскости. Проекция первого отрезка (длины а) на плоскость равна с. Найдите проекцию второго отрезка, если а = 17 , b = 10, с = 15 см.

Дано: α || β, а = 17 , b = 10, с = 15 см. Найти: х

Решение:

а2 – с2 = b2 – х2, х2 = b2а2 + с2 , х2 = 102 – 172 + 152 =

= 100 – 289 + 225 = …, х = … см.
Ответ: х = 6 см.

Пример 2.

Две параллельные плоскости расстояние между

которыми 2 дм, пересечены прямой, составляющей с каждой из

плоскости угол в 300. Найти длину отрезка этой прямой, заключенной

между плоскостями.

Дано: α || β, АВα = А, АВβ = В, АВС = 30°, АС = 2 дм.

Найти: АВ

Решение: Δ АСВ – прямоугольный, АВС = 30°, АС = 2 дм.

АВ = 2 АС = 2 2 = … дм.
Ответ: АB = 4 дм.

Пример 3. Расстояние между параллельными плоскостями равно 8 см. Отрезок прямой длина которого 17 см расположен между ними так, что его конец принадлежит плоскости. Найти проекцию этого отрезка на другую плоскость.

Дано: α || β, АВα = А, АВβ = В, АВ = 17 см, АС = 8 см.

Найти: ВС

Решение: Δ АСВ – прямоугольный, ВС2 = АВ2 – АС2 = 172 – 82 = 289 – 64 = …, ВС = … см.

Ответ: BС = 15 см.

Пример 4. На параллельных плоскостях α и β, выбрано по паре точек А12 и В12 соответственно так, что прямые А1В1 и А2В2 пересекаются в точке S Вычислите SА1 и SВ2, если А1В1= 6см;
2 = 2,5см; SВ2 : SА2 = 3 : 1 . S

Дано: α || β, А1 А2В1 В2 = S, А1, А2 α, В12 β,

А1В1= 6см; SА2 = 2,5см; SВ2 : SА2 = 3 : 1

Найти: 1, SВ2

Решение: Δ SА1 А2 ~ Δ SВ1В2 , (α || β), SВ2 : SА2 = 3 : 1, SА2 = 2,5см,

2 = 3 2,5 = … см. 1 : SА1 = 3 : 1, А1В1= 6см, SА1 = х ,

( х + 6 ) : х = 3 : 1, 3х = х + 6 , 2х = 6, х = …, SА1 = … см.

Ответ:1 = 3 см, SВ2 = 7,5 см .

Пример 5.

Дано: α || β, а α, bβ, а || b, с - секущая, 1 = 150°,

Найти: 2, 3, 4, 5, 6, 7, 8.

Решение: 3 = 1 = 150°(верт.), 3 = 5 = 150°(н.леж.),hello_html_m40fa69e3.jpg

5 = 7 = 150°(верт.), 1 + 2 = 180°(смежные),

2 = 180° – 1 = 180° – 150° = …°,

2 = 4 = 30°(верт.), 4 = 6 = …°(н.леж.), 6 = 8 = …°(верт.).

Ответ: 3 = 5 = 7 = 150°, 2 = 4 = 6 = 8 = 30°.

В)hello_html_41ca87ca.jpg

Пример 1. Из точек А и В, лежащих в двух перпендикулярных плоскостях, опущены перпендикуляры АС и ВD на прямую пересечения плоскостей. Найдите длину отрезка АВ если:

а) АС = 6 м, ВD = 7 м, СD = 6 м, б) АD = ВС = 5 м, СD = 1 м.

Решение: а) Пусть плоскости α и β перпендикулярны. СD – прямая пересечения плоскостей , тогда АС СВ и ВD АD. Тогда в Δ АСВ: АВ2 = АС2 + ВС2, но из Δ СDВ следует ,что: ВС2 = СD2 + ВD2 , так что АВ2 = АС2 + СD2 + ВD2.

АВ2 = 62 + 72 + 62 = 36 + 49 + 36 = …, АВ = …

б) АВ2 = АС2 + ВС2, но из Δ СDА следует ,что: АС2 = АD2 СD2 ,

так что АВ2 = АD2 СD2 + ВС2. АВ2 = 52 12 + 52 = 251 + 25 = …, АВ = …

Ответ: а) 11 м, б) 7 м.hello_html_552bcaf5.jpg

Пример 2. Точка А находится на расстоянии а = 24 см и b = 10 см от двух перпендикулярных плоскостей α и β. Найдите расстояние от этой точки до прямой пересечения плоскостей.

Решение: Пусть α β и α β = с. Проведем перпендикуляры АВ, АD, АС. Тогда четырехугольник АВСD – прямоугольник. АС2 = а2 + b2 ,

АС –искомое расстояние. ВС - проекция АС на плоскость α, поэтому по теореме о 3 – х перпендикулярах ВС с, ВС β. Так как АD β, то по теореме АD||ВС, а, значит, АD и ВС лежат в одной плоскости.

Итак , АС2 = 242 + 102 = 576 + 100 = … , АС = …hello_html_mf971f4e.jpg

Ответ: АС = 26 см.

Пример 3. Плоскости α и β перпендикулярны. В плоскости α взята точка А, расстояние от которой до прямой с ( линия пересечения плоскостей ) равно 0,5 м. В плоскости β проведена прямая b, параллельная прямой с и отстоящая от нее на 1,2 м. Найдите расстояние от точки А до прямой b.

Решение: Пусть α β , b || с, ВС = 1, АВ = 0,5м , где АВ с и ВС b.

Тогда по теореме о 3 – х перпендикулярах АС b. Так что

АС – искомое расстояние и АС2 = АВ2 + ВС2 = 1,22 + 0,52 = 1,44 + 0 ,25 = …, АС = …

Ответ: АС = 1,3 м.

Пример 4. Перпендикулярные плоскости α и β пересекаются по прямой с. Плоскости α проведена прямая а|| с, в плоскости β – прямая b || с. Найдите расстояние между прямыми а и b , если расстояние между прямыми а и с равно 1,5 м, а между прямыми b и с – 0,8 м. hello_html_m209580fa.jpg

Решение: Возьмем в плоскости α точку А на прямой а. По теореме о 3 – х параллельных прямых получаем, что а || b (так как а || с, b ||с). Проведем

АС с и СВ b. Тогда по теореме о 3 – х перпендикулярах АВ b.

Так что АВ – искомое расстояние и АВ СВ, так как α β
( по условию), из прямоугольного треугольника АВС по теореме Пифагора имеем: АВ
2 = СВ2 + АС2 = 1,52 + 0,82 = 2,25 + 0,64 = … , АВ = …

Ответ: АВ = 1,7 м.

С) Решение теста. Методические рекомендации к выполнению теста:

  1. Прочитать вопрос, ответить на его и записать букву , под которой записан правильный ответ.

  2. Решив задачу, нужно выбрать правильный ответ и записать номер, под которым он записан.

Задание: 1 часть.

1. Расстоянием от точки до плоскости называется

а) длина перпендикуляра, опущенного из точки на эту плоскость.

б) длина перпендикуляра, проведенного из плоскости к этой точке.

в)длина перпендикуляра, проведенного из любой точки одной

плоскости ко второй плоскости, на которой лежит эта точка.

г) расстояние от этой точки до любой из точек лежащих на плоскости.

2. Расстоянием от прямой до параллельной ей плоскости называется

а) длина перпендикуляра, опущенного из произвольной точки на эту плоскость.

б) длина перпендикуляра, опущенного из произвольной точки прямой на эту плоскость.

в) расстояние от точки лежащей на прямой, до любой из точек лежащих на плоскости.

г) длина перпендикуляра, опущенного из произвольной точки плоскости на эту прямую.

3. Найдите расстояние между скрещивающимися прямыми, содержащими

диагональ куба и ребро куба, если ребро куба равно см.

а) см ,б) 1 см, в) 0,5 см, г) 2 см.

4.Если угол между двумя прямыми равен 90°, то эти прямые:

а) пересекаются, б) параллельны, в) скрещиваются, г) перпендикулярны, д) совпадают.


5.Расстояния от точки М до сторон прямоугольного треугольника АВС

(угол С равен 90°) равны. Какое из следующих утверждений верно?

а) плоскости МАВ и АВС перпендикулярны,

б) плоскости МВС и АВС перпендикулярны,

в) плоскости МАС и АВС перпендикулярны,

г) плоскости МАС и МВС перпендикулярны, д) условия в пунктах а - г неверны.

2 часть .

  1. Расстояние от некоторой точки до плоскости квадрата равно 4 см, а до каждой из его вершин – 6 см. Найдите диагональ квадрата. А) 2 см; Б) 5 см; В) 5 см; Г) другой ответ.

2. Найдите расстояние от середины отрезка АВ, пересекающего плоскость α,до плоскости α, если расстояния от точек А и В до плоскости равны соответственно 7 см и 9 см.

А) 8 см; Б) 1 см; В) 4 см; Г) другой ответ.

3.Из вершины равностороннего треугольника АВС проведен перпендикуляр АК к плоскости треугольника. Точка D – середина стороны ВС. Найдите длину АК, если ВС = см, КD = 8 см.

А) 14 см; В) 12 см; В) 7 см; Г) другой ответ.

4.Расстояние от некоторой точки до плоскости прямоугольника равно см, а до всех его вершин – 3 см. Найдите диагональ прямоугольника. А) 4 см; Б) 2 см; В) 5 см; Г) другой ответ.

5. Найдите расстояние от середины отрезка АВ, пересекающего плоскость α ,до плоскости α, если расстояния от точек А и В до плоскости равны соответственно 4 см и 10 см.

А) 7 см; Б) 3 см; В) 2 см; Г) другой ответ.

6. Расстояния от вершин А, В, С параллелограмма ABCD, не пересекающего плоскость α, до плоскости α равны соответственно 19 см, 6 см и 16 см. Найдите расстояние от вершины D до плоскости α .

А) 23 см; Б) 11 см; В) 29 см; Г) другой ответ.

D) Построить таблицу:


Проводим KM α

 (M α). KM = ρ (K; α).

hello_html_m432887ec.gif

SO α.

Проводим KM || SO. 

Тогда KM αи KM = ρ (K; α).

hello_html_m9bde0f8.gif

Проводим через точку K плоскость β α (β пересекает α по AB).

Проводим

KM AB. 

Тогда KM α и KM = ρ (K; α).

hello_html_m7f69f547.gif

Расстояние между прямой и параллельной ей плоскостью

Расстоянием от прямой до параллельной ей плоскости называется расстояние от произвольной точки этой прямой до плоскости.

a || α, A a, ρ (a; α) = ρ (A; α).
Выбираем на прямой a произвольную точку A и находим расстояние от этой точки до плоскости α.

hello_html_41c2f592.gif

Расстояние между параллельными плоскостями

Расстоянием между двумя параллельными плоскостями называется расстояние от произвольной точки одной плоскости до второй плоскости.

β || α, B β, ρ (β; α) = ρ (B; α).
Выбираем в плоскости β произвольную точку B и находим расстояние от этой точки до плоскости α.

hello_html_78b104b6.gif

Расстояние между скрещивающимися прямыми

Общим перпендикуляром к двум скрещивающимся прямым называется отрезок с концами на этих прямых, перпендикулярный каждой из них.

Расстоянием между скрещивающимися прямыми называется длина их общего перпендикуляра. Она равна расстоянию между параллельными плоскостями, которые проходят через эти прямые.

hello_html_m19408d45.gif

AB a, AB b; ρ (a; b) = AB.
Прямые a и b — скрещивающиеся.


2)Решить задачи ( по примерам):

  1. Два отрезка длин а и b упираются концами в две параллельные плоскости. Проекция первого отрезка (длины а) на плоскость равна с. Найдите проекцию второго отрезка, если а = 13 , b = 15, с = 5 см.

  2. Две параллельные плоскости расстояние между которыми 6 дм, пересечены прямой, составляющей с каждой из плоскости угол в 300. Найти длину отрезка этой прямой, заключенной между плоскостями.

  3. Расстояние между параллельными плоскостями равно 10 см. Отрезок прямой длина которого 26 см расположен между ними так, что его конец принадлежит плоскости. Найти проекцию этого отрезка на другую плоскость.

  4. На параллельных плоскостях α и β, выбрано по паре точек А12 и В12 соответственно так, что прямые А1В1 и А2В2 пересекаются в точке S Вычислите SА1 и SВ2, если А1В1= 12см;
    2 = 4,5см; SВ2 : SА2 = 3 : 1.

  5. Дано: α || β, а α, bβ, а || b, с - секущая, 1 = 140°. Найти: 2, 3, 4, 5, 6, 7, 8.

  6. Из точек А и В, лежащих в двух перпендикулярных плоскостях, опущены перпендикуляры АС и ВD на прямую пересечения плоскостей. Найдите длину отрезка АВ если:

а) АС = 3 м, ВD = 4 м, СD = 12 м, б) АD = 4 м, ВС = 7 м, СD = 1 м.

  1. Точка А находится на расстоянии а = 17 см и b = 8 см от двух перпендикулярных плоскостей α и β. Найдите расстояние от этой точки до прямой пересечения плоскостей.

  2. Плоскости α и β перпендикулярны. В плоскости α взята точка А, расстояние от которой до прямой с ( линия пересечения плоскостей ) равно 0,9 м. В плоскости β проведена прямая b, параллельная прямой с и отстоящая от нее на 1,2 м. Найдите расстояние от точки А до прямой b.

  3. Перпендикулярные плоскости α и β пересекаются по прямой с. Плоскости α проведена прямая а|| с, в плоскости β – прямая b || с. Найдите расстояние между прямыми а и b , если расстояние между прямыми а и с равно 4,5 м, а между прямыми b и с – 2,4 м.

3)Решить задачи :

  1. Через точку O, которая находится между параллельными плоскостями α и β, проведены прямые c и d, пересекающие плоскости так, что точки A и B находятся в плоскости α, а точки C и D - в плоскости β ,AB=15 см, DO=29 см и AC=3AO.Вычислить: BD;CD.  

  2. Дан треугольник АВС. Плоскость, параллельная прямой АВ, пересекает сторону АС этого треугольника в точке А1, а сторону ВС в точке В1. Найдите длину отрезка А 1В 1,если АВ = 8 см, АА1 : А1С = 5 : 3.

  3. Сторона правильного треугольника ABC равна 4. Треугольник DBC — равнобедренный
    (DB = DC). Их плоскости взаимно перпендикулярны. Плоскость ADC составляет с плоскостью АВС угол 60°. Найдите площадь треугольника DBC.

  4. В треугольнике ABC синус угла B равен , . отрезок перпендикулярный плоскости данного треугольника. Найдите расстояние от точки A до плоскости (SBC).

  5. В треугольнике ABC сторона . отрезок перпендикулярный плоскости данного треугольника. Найдите расстояние от точки A до плоскости (QBC), если площадь треугольника BQC равна 6,5.

  6. Площадь треугольника ABC равна 3, . отрезок перпендикулярный плоскости данного треугольника. Найдите расстояние от точки A до плоскости (PBC).

  7. В единичном кубе найдите .



Просмотрено: 0%
Просмотрено: 0%
Скачать материал
Скачать материал "ПЗ по специальности Технология машиностроения,1 курс(2016-2017 уч.г.)"

Получите профессию

Менеджер по туризму

за 6 месяцев

Пройти курс

Рабочие листы
к вашим урокам

Скачать

Выбранный для просмотра документ ПЗ№30.docx

ПЗ № 30. Преобразования графика функции. Гармонические колебания. Прикладные задачи.

Задание:

1)Опорный конспект.

1.Преобразования графиков функций — это линейные преобразования функции y = f(x) или её аргумента x к виду y = af(kx + b) + m, а также преобразование с использованием модуля. Зная, как строить графики функции y = f(x), где y = kx + b, y = ax2, y = xn , y=k/x, 

y = sin x, y = cosx, y = tgx, y = ctgx, y=ax,y=logax можно построить график функции y = af(kx + b) + m.


Общий вид функции

Преобразования

y = f(x - b)

Параллельный перенос графика вдоль оси абсцисс на | b | единиц

  • вправо, если b > 0;

  • влево, если b < 0.

y = f(x + b)

  • влево, если b > 0;

  • вправо, если b < 0.

y = f(x) + m

Параллельный перенос графика вдоль оси ординат на | m | единиц

  • вверх, если m > 0,

  • вниз, если m < 0.


Отражение графика

y = f( - x)

Симметричное отражение графика относительно оси ординат.

y = - f(x)

Симметричное отражение графика относительно оси абсцисс.


Сжатие и растяжение графика

y = f(kx)

  • При k > 1 — сжатие графика к оси ординат в k раз,

  • при 0 < k < 1 — растяжение графика от оси ординат в k раз.

y = kf(x)

  • При k > 1 — растяжение графика от оси абсцисс в k раз,

  • при 0 < k < 1 — cжатие графика к оси абсцисс в k раз.


Преобразования графика с модулем

y = | f(x) |

  • При f(x) > 0 — график остаётся без изменений,

  • при f(x) < 0 — график симметрично отражается относительно оси абсцисс.

y = f( | x | )

  • При x≥0— график остаётся без изменений,

  • при x < 0 — график симметрично отражается относительно оси ординат.


hello_html_m1e9c802a.gif

Примеры построения функций.



hello_html_mf8eb205.pnghello_html_4df25cf6.png

hello_html_500049aa.pnghello_html_1aa59faf.png

hello_html_m623ad66f.pnghello_html_m6eb3bff8.png

hello_html_3a70f67b.pnghello_html_m1ba2c529.png

2.Тригонометрические функции используются для описания различных колебательных процессов: колебания груза, подвешенного на пружине, вокруг положения равновесие, закон изменения переменного тока в цепи, колебания маятника, распространение звуковых и цветовых волн и т.д.

Формулы  и  , с помощью которых описываются такие процессы, называются формулами гармонических колебаний. Положительная величина А называется амплитудойколебания, положительная величина w  частотой колебания, величина  начальной фазой колебания. Амплитуда характеризует размах колебания, частота – количество колебаний в единицу времени.

Построение графиков гармонических колебаний (гармоник) , производится в несколько этапов.

Рассмотрим алгоритм построения графика функции  : а) строим график функции  ; б) строим график функции  , сдвигая график функции  на |hello_html_1b4e4442.gif| единиц по осиОХ (если   , то сдвигаем влево, если  , то сдвигаем вправо); в) строим график функции  , сжимая его в w раз к оси OY;

г) строим график функции  , растягивая его в A раз от осиОХ.

Заметим, что функции  и  , описывающие гармонические колебания, являются периодическими с периодом   . Они ограничены сверху и снизу, их наибольшее и наименьшее значения равны  .

Пример 1.Постройте график гармонического колебания  .

Решение. Для этой гармоники амплитуда  , частота –  , начальная фаза –  .

Строим график функции  ; сдвигаем на  единиц по оси ОХ вправо; сжимаем график к оси OY в 2 раза; растягиваем от оси OX в 3 раза (рис. 38).


hello_html_m1f203bb.jpg

Пример 2.Постройте график гармонического колебания  .

Решение. Преобразуем формулу, раскрыв в аргументе косинуса скобки:  . Следовательно, для этой гармоники амплитуда  , частота –  , начальная фаза –  .

Строим график функции  ; сдвигаем график на  единиц по оси ОХ вправо; сжимаем график к оси OY в 2 раза; растягиваем от оси OX в 3 раза (рис. 39).


hello_html_m500cf9e4.jpg

Пример 3.Постройте график гармонического колебания  .

hello_html_41baac58.jpg 

Решение. Эта формула не задает гармоническое колебание, так как  . Применив формулу приведения , преобразуем формулу к виду:  . Следовательно, для этой гармоники амплитуда  , частота –  , начальная фаза –  .

Строим график функции  ; сдвигаем на  единиц по оси ОХ влево; сжимаем график к оси OY в 2 раза; растягиваем от оси OX в 3 раза (рис. 40).

3.

hello_html_m26d06696.gif

hello_html_126ef828.gif



2)Решить задание:

1.

hello_html_60058b54.png

2.

hello_html_m5af30e0a.png

3.

hello_html_m18f76492.png

4. hello_html_m52a0ab8f.png

5. hello_html_m52a0ab8f.png

6. hello_html_m52a0ab8f.png

7.

hello_html_c3deb3c.png

8. Построить графики функций:

hello_html_m25e403ff.png

9. Построить графики функций:1) у = 1/2 sin (3x) – 2, 2) y = 2 3x+1 – 4,

3) y = 2 (x – 1)2 – 3, 4) y = –3 log2(x + 1), .

10.



hello_html_m749d1356.png



















Просмотрено: 0%
Просмотрено: 0%
Скачать материал
Скачать материал "ПЗ по специальности Технология машиностроения,1 курс(2016-2017 уч.г.)"

Получите профессию

Секретарь-администратор

за 6 месяцев

Пройти курс

Рабочие листы
к вашим урокам

Скачать

Выбранный для просмотра документ ПЗ№31.docx

ПЗ № 31. Показательные, логарифмические, тригонометрические уравнения и неравенства.

Задание:

1)Опорный конспект.

Графиком называется множество точек координатной плоскости, у которых значения x и y связаны некоторой зависимостью и каждому значению x соответствует единственное значение y.

Графический способ - один из самых удобных и наглядных способов представления и анализа информации.

На практике довольно часто оказывается полезным графический метод решения уравнений. Он заключается в следующем: для решения уравнений f(x)=0 строят график функции y=f(x) и находят абсциссы точек пересечения графика с осью Оx: эти абсциссы и являются корнями уравнения. Алгоритм решения уравнений графическим способом

Чтобы решить графически уравнение вида f(х) = g(х), нужно:

1.Построить в одной координатной плоскости графики функции:

у = f(х) и у = g(х).

2. Найти точки пересечения этих графиков.

3. Указать абсциссу каждой из этих пересечения.

4. Записать ответ.

Довольно просто решать графически систему уравнений, так как каждое уравнение системы на координатной плоскости представляет какую- то линию.

Построив графики этих уравнений и найдя координаты точек их пересечения (если они существуют), мы получим искомое решение.

Графическое решение неравенств, сводится к отысканию таких точек x, при которых один график лежит выше или ниже другого.

Примеры:

1. Решите уравнение

2. Решите уравнение

3. Решить уравнение hello_html_m49d202a5.gifhello_html_663ba92f.png

Решение: Построим графики функций hello_html_598b2242.gif и y = x


Графики функций не пересекаются, и, значит, уравнение не имеет корней (см. рисунок).

Ответ: корней нет.

4.Найти значение выражения хhello_html_7b9d0882.gif+ уhello_html_7b9d0882.gif,если (х;у) является решением системы уравнений. hello_html_m7beea7c6.gifhello_html_m4aa16645.jpg

Решение:

hello_html_6938f234.gifhello_html_3fee811a.gif-параллельный перенос на 1 единицу влево.

hello_html_3133347a.gif - параллельный перенос на 2 единицы влево.

х= - 1, у=1

х+ у=0.


Ответ: 0.

5. Решите неравенство hello_html_m6d811b21.gif>12 - 1,5х. №6. Решите неравенство hello_html_m342748d5.gif. Oтвет: х>0.

Ответ: х>2. hello_html_m34a0a2e3.jpg

hello_html_m62d29737.jpg

















7. Решить уравнение  sinx + cosx=1. Построим графики функций y=sinx u y=1-cosx.(рисунок 5) Из графика видно, что уравнение имеет 2 решения: х=2πп,где пЄZ и х=π/2+2πk,где kЄZ.


hello_html_6222e497.jpg

8.Решить уравнение: 3x = (х-1) 2 + 3

Решение: применяем функциональный метод решения уравнений:

hello_html_78cdd215.gif

т.к. данная система имеет единственное решение, то методом подбора находим х=1hello_html_1db23044.jpg

Ответ: 1.


9.Решить неравенство: сos x hello_html_m62c2f6f2.gif1 + 3x

Решение:



hello_html_58548c10.gif

Ответ: (hello_html_m293ab4b8.gif ; hello_html_7ea8f9fd.gif).hello_html_m5828edc.png

10. Решить уравнение hello_html_m6e244885.png

В нашем случае функция hello_html_m81b4bc5.pngвозрастает при х>0, а функция y = 3 – x убывает при всех значениях х, в том числе и при х>0, значит, уравнение hello_html_m3f92800e.pngимеет не более одного корня. Заметим, что при х = 2 уравнение обращается в верное равенство, так как hello_html_m47d6d725.png.

Ответ: 2 .

2)Решить задание:

1)Есть ли корень у уравнения и если есть, то положительный он или отрицательный?

а) hello_html_2e3d017c.gif; б) hello_html_m152ef3cd.gif, в) 6х =1/6, г) hello_html_71df238a.gif.hello_html_m1b1b0d93.pnghello_html_m2db28fe1.pnghello_html_66f33c29.png

hello_html_5dceb135.png

2) Решить графическим методом уравнение  .

3) Решите графическим методом уравнения:

а) б).



4)На рисунке изображен график функции y=f(x). Найдите количество целых корней уравнения f(x)= 0. hello_html_m209c52f.gif

1) 1 2) 6 3) 7 4) 8

5) На каком из рисунков изображен график функции ?

1) у 2) у 3) у 4) у



1 1 1



6) График какой функции изображен на рисунке?

1) у = 2х-1,5; 2) у = 2х – 2;

3) у = 2х – 3; 4) у = 2 – 2.

7)График какой функции изображен на рисунке?

hello_html_m352f9896.gif





1) у = sinx; 2) ; 3) ; 4) .

8) На рисунке изображен график функций y

y = f (x) и y = g (x), заданный на промежутке

[-5;6]. Укажите те значения х, для которых

выполняется неравенство g (x) f (x) 1


1) [-5; 0] 2) [-5; 2]

0 1 x

3) [-2; 2] 4) [2; 6]


9) На рисунке изображен график функции y=f(x). Найдите количество целых корней уравнения f(x)= 0.hello_html_3b033243.gif

1) 3 2) 4 3) 2 4) 1



10) На рисунке изображен график функции y=f(x). Найдите количество целых корней уравнения f(x)+2= 0.

hello_html_5555b426.gif

1) 3 2) 5 3) 4 4) 1

11) Решите графическим методом уравнения:

а) , б), в) cos x≤ 1+ 4x, г) 5x = (х - 1) 2 + 5.

12) Решить графическим методом уравнение  .























Просмотрено: 0%
Просмотрено: 0%
Скачать материал
Скачать материал "ПЗ по специальности Технология машиностроения,1 курс(2016-2017 уч.г.)"

Получите профессию

Копирайтер

за 6 месяцев

Пройти курс

Рабочие листы
к вашим урокам

Скачать

Выбранный для просмотра документ ПЗ№32.docx

ПЗ № 32. Решение иррациональных уравнений.

Задание:

1) Перепишите и заполните пропуски:
Пример 1. Решить уравнениеhello_html_3be855b.gif Решение: Уединим радикал hello_html_m5a2fbf9f.gif Это уравнение равносильно системе hello_html_4e1d1e98.gif Решим уравнение (1): hello_html_6861a4c5.gif hello_html_m66272cef.gif hello_html_m211ee85b.gif hello_html_3fa90232.gif х = … Найденное значение hello_html_m727f5169.gif удовлетворяет условиям (2) и (3).

Ответ: –1. Пример 2. а) Найдите корень уравнения = 3 . Решение:

Возведем в квадрат правую и левую части уравнения: )2 = 32, 15 – 2х = 9, –2х = 9 – 15,

2х = – 6, х = ... Сделаем проверку. Для этого подставим число 3 в исходное уравнение: = 3, 3 = 3 – верно.

Ответ: 3.

б) Решить уравнение = .

Решение: = => ˂=> => => х = ...

Ответ: 1. Пример 3. Решить уравнение = х -7 .

Решение: = х -7 => => => => => х = ...

Ответ: 14.

Пример 4. Решите уравнение   = .

Решение:

  = => 7 х + х 2 2 = 2х 5 =>

5 – х = => 25 – 10х + х2 = х2 + 9х – 14 => 2 19х + 39 = 0,

D = (19)2 42 39= 361 – 312 = …, х1= (19 + 7) : 4 = …, х2 = (197) : 4 = …,

Проверка:  

а)  х1= 6,5,   = ,

  = –  неверное равенство.

б) х2 = 3,   – = ,   = , –  верное равенство.

Ответ: 3.

Пример 5. Решить уравнение hello_html_11c97566.gif

Решение:

Возводим в куб обе части уравнения hello_html_m5925937d.gif получим hello_html_36f8101a.gif

Учитывая, что выражение в скобках равно 1 (см. условие), получаем hello_html_m4386db45.gif hello_html_m523bd08c.gif hello_html_73392c.gif

Возводим в куб: hello_html_m30f89b75.gif hello_html_m522b6165.gif hello_html_m3348bac9.gif

Проверкой убеждаемся, что hello_html_m294b9fba.gif и hello_html_3a69ee1b.gif корни уравнения.

Ответ: 80, – 109.


2)Решить задание ( по примерам):

Решить уравнения:

  1. а) . б) .

  2. .

3) Решить задание:

  1. Решить уравнение:.

  2. Решить уравнение:.

  3. Решить уравнение:.

  4. Решить уравнение:.

  5. Решить уравнение: .

  6. Решить уравнение:3.

  7. Решить уравнение: .

  8. Решить уравнение:.

  9. Решить уравнение:.

  10. Решить уравнение:.

  11. Найдите корень уравнения:

  12. Найдите корень уравнения:

  13. Найдите корень уравнения:

Если уравнение имеет более одного корня, укажите меньший из них.

  1. Найдите корень уравнения:

  2. Найдите корень уравнения:

  3. Решить уравнение:

  4. Решить уравнение:

  5. Решить уравнение:

  6. Решить уравнение:

  7. Решить уравнение:

  8. Решить уравнение:

  9. Решить уравнение:

  10. Решить уравнение:

  11. Решить уравнение:

  12. Решить уравнение:























Просмотрено: 0%
Просмотрено: 0%
Скачать материал
Скачать материал "ПЗ по специальности Технология машиностроения,1 курс(2016-2017 уч.г.)"

Получите профессию

Бухгалтер

за 6 месяцев

Пройти курс

Рабочие листы
к вашим урокам

Скачать

Выбранный для просмотра документ ПЗ№33,34.docx

ПЗ № 33. Решение показательных уравнений.

Задание:

1) Перепишите и заполните пропуски:
Пример 1. а)Найдите корень уравнения .

Решение: Чтобы решить это уравнение, вспомним свойства степени и приведем правую и левую части уравнения к степени с основанием 5: ,

Если степени с равными основаниями равны, то равны их показатели. Приравняем показатели степеней: х – 7 = - 3, х = 7 – 3, х = ...

Ответ: 4 .

б)Найдите корень уравнения .

Решение: Представим правую и левую части уравнения в виде степени с основанием ,

Приравняем показатели степеней: – 3 (– 3+ х) = 9, 9 – 3х = …, – 3х = 0, х = ...

Ответ: 0.

Пример 2. Решите уравнение.

Разделим обе части уравнения на : .

Пустьm,m > 0 , тогда 2m2 – 3m – 5 = 0, D = 9 – 42(– 5) = 9 + 40 = …, m1 = (3 + 7) : 4 = …,

m2 = (3 – 7) : 4 = – 4 : 4 = …, – не удовл. условию m > 0 .
Если m = 2,5 , то
Ответ:  – 1.

Пример 3. Решите уравнение 49x 8∙7x + 7 = 0.

Решение: Обозначим получим уравнение относительно у: у2 – 8у + 7 = 0,

D = (– 8)2 41 7= 64 – 28 = …, у1= (8 + 6) : 2 = …, у2 = (8 – 6) : 2 = ... Получим, что и , отсюда х1 = …, х2 = ... Ответ: х1 = 1, х2 = 0.

Пример 4. а)Решить уравнение .
Решение:

Ответ: 3.

б) Решите уравнение 

Решение:

Ответ: 1.

в)Решите уравнение

Решение:

Ответ: 4.

Пример 5. Решите уравнение а) 2х+1 + 2х-1 + 2х = 28, б) 9х – 8∙3х – 9 = 0, в) 8∙4х – 6∙2х + 1 = 0.

Решение:

а) 2х+1 + 2х-1 + 2х = 28, 2х-1 ∙ (22 + 1 + 2) = 28, 2х-1∙7 = 28, 2х-1 = 4, 2х-1 = 22, х – 1 = 2, х = ...

Ответ: 3.

б) 9х – 8∙3х – 9 = 0, (3х)2 – 8∙3х -9 = 0, Обозначим 3х = t, где t >0, тогда t2 – 8t – 9 = 0,

D = (–8)2 41 (–9) = 64 + 36 = …, t 1= (8 + 10) : 2 = …, t 2 = (8 – 10) : 2 = ... . t1 = 9, t2 = – 1, Возвращаемся к замене: 3х = 9, х = …, 3х = – 1, корней нет.

Ответ: 2.

в) 8∙4х – 6∙2х + 1 = 0, 8∙(2х)2 – 6∙2х + 1 = 0, Обозначим 2х = t, где t >0, тогда 8 t2 – 6t + 1 = 0, D = (–6)2 41 8= 36 – 32 = …, t1= (6 + 2) : 16 = …, t2 = (6 – 2) : 16 = ... t1 =, t2 = Возвращаемся к замене: 2х = , х 1= …, 2х = , х 2= ...

Ответ: – 1, – 2.

2)Решить задание ( по примерам):

  1. а) Найдите корень уравнения .б) Найдите корень уравнения: .

  2. Решите уравнение .

  3. Решите уравнение 25x 6∙5x + 5 = 0.

  4. а)Решить уравнение: б)Решите уравнение 

в)Решите уравнение.

  1. Решите уравнение а) 3х+1 + 3х-1 + 3х = 117, б) 16х – 15∙4х – 16 = 0, в) 81х + 6∙9х + 9 = 0.

3)Решить задание :

  1. Решить уравнения: а) , б) в)

  2. Решите уравнение а) 4x 5∙2x + 4 = 0, б) 9x 4∙3x + 3 = 0.

  3. Решите уравнение  

  4. Найдите сумму корней уравнения :.

  5. Если - корень уравнения , то найдите значение выражения .

  6. Найдите произведение корней уравнения .

  7. Решите уравнение .

  8. Решите уравнение .

  9. Решите уравнение

  10. Решите уравнение 4х + 2х – 6 = 0;

  11. Решите уравнение 9х + 3х+1 = 4;

  12. Решите уравнение

  13. Решите уравнение .

  14. Решите уравнение: .

  15. Решите уравнение: 92х+1 – 9 = 72.

  16. Решите уравнение:

  17. Пусть х0 ─ наибольший корень уравнения . Найти 2х0 – 5.

  18. Решите уравнение: 23х+2 + 8х = 0,625.

  19. Пусть х0 ─ наименьший корень уравнения . Найти 3х0 + 2.

  20. Найти сумму корней уравнения: 4х – 40∙2х + 256 = 0.

  21. Решите уравнение:

  22. Решите уравнение :3∙ + 325 ∙ = 0.

  23. Решите уравнения:


  1. Решите уравнения:

  2. Решите уравнение: .

  3. Решите уравнения:


  1. Решите уравнение: .

  2. Решите уравнения:

  3. Найти корень (или сумму корней, если их несколько) уравнения: 7 · 8х+1 + 8х+3 = 71.

  4. Найти корень (или сумму корней, если их несколько) уравнения: 7 = 6 · 7х + 7.

  5. Решите уравнение: .

  6. Решите уравнение: .

ПЗ № 34. Решение логарифмических уравнений.

Задание:

1) Перепишите и заполните пропуски:
Пример 1. Решите уравнение Решение: Используем метод - решение логарифмических уравнений заменой.

ОДЗ: х > 0. Введем замену , чтобы записать исходное уравнение в виде стандартного квадратного уравнения. Тогда уравнение примет вид: у2 – 4у + 4 = 0, ( у – 2)2 = 0, у – 2 = 0, у = ... Вернемся к  х : . Тогда по определению логарифма получаем, что х = 32, х = … - уд.ОДЗ.

Ответ: 9.

Пример 2. Решите уравнение:.  

Решение: Вновь начнем решение с определения области допустимых значений уравнения. Она определяется следующей системой неравенств:


Воспользовавшись правилом сложения логарифмов, переходим к равносильному в области допустимых значений уравнению:

Основания логарифмов одинаковы, поэтому в области допустимых значений можно перейти к следующему квадратному уравнению:

(х + 2) (х + 3) = 1 х , х2 + 6х + 5 = 0, D = (6)2 41 5= 36 – 20 = …,

х1= ( 6 4) : 2 = , х2 = ( 4) : 2 = ...  

Первый корень не входит в область допустимых значений уравнения, второй — входит.

Ответ: x = -1.

Пример 3. Решите уравнение:

Решение: Найдем ОДЗ по определению логарифма. ОДЗ:

.

Перепишем исходное уравнение, используя свойства суммы логарифмов и логарифма степени. Получим следующее уравнение:

Приравняем подлогарифмические выражения:

(3х ) (х) = ,

Найдем корни полученного квадратного уравнения:

D = (92)2 41 () = 8464 + 8436 = …,

х1= (92 + 130) : 6 = 222 : 6 = …, х2 = (92 130) : 6 = .

Учитывая ОДЗ, корнем исходного логарифмического уравнения будет только х = ...

Ответ: х = 37.

Пример 4. Решите уравнение:

Решение: В область допустимых значений входят только те x, при которых выражение, находящееся под знаком логарифма, больше нуля. Эти значения определяются следующей системой неравенств:


С учетом того, что получаем промежуток, определяющий область допустимых значений данного логарифмического уравнения:.

На основании теоремы 1, все условия которой здесь выполнены, переходим к следующему равносильному квадратичному уравнению:

  D = (5)2 41 () = 25 + 56 = …, х1= (5 + 9) : 2 = …, х2 = (5 9) : 2 =

В область допустимых значений входит только первый корень. Ответ: x = 7.

Пример 5. Решите уравнение:

Решение: Используем метод - решение логарифмических уравнений, переходя к одному основанию. ОДЗ: 

К логарифму по основанию x (второе слагаемое) вначале применим свойство логарифма степени, а затем по формуле замены основания логарифма приведем его к основанию 2:



Так как  то


Введем замену  тогда уравнение примет вид: у2 – 5у + 4 = 0.

Найдем корни полученного квадратного уравнения:

D = (5)2 41 = 25 = …, y1= (5 + 3) : 2 = …, y2 = (5 3) : 2 = ...

Вернемся к x, используя определения логарифма:

x = x = …, x = , x = …, Оба значения принадлежат ОДЗ.

Ответ: 16 и 2.

Пример 6. Решите уравнение:

Решение: Область допустимых значений уравнения определяется здесь легко: x > 0.

Используем подстановку: Уравнение принимает вид: 3у2 + 5у = 0,

D = (5)2 43 () = 25 + 24 = …, у1= (5 + 7) : 6 = 1/3, у2 = (5 7) : 6 =

Вернемся к x, используя определения логарифма:

x = , x = , x =... Оба ответа входят в область допустимых значений уравнения, поскольку являются положительными числами.

Ответ: и 4.

2)Решить задание ( по примерам):

  1. Решите уравнение

  2. Решите уравнение .

  3. Решите уравнение

  4. Решите уравнение:

  5. Решите уравнение:

  6. Решите уравнение:

3)Решить задание :

  1. Решите уравнение .

  2. Решите уравнение .

  3. Решите уравнение:

  4. Решите уравнение:

  5. Решите уравнение:

  6. Решите уравнение:

  7. Если - корень уравнения , то найдите значение выражения .

  8. Найдите произведение корней уравнения .

  9. Найдите сумму корней уравнения .

  10. Найдите больший корень уравнения .

  11. Решите уравнение:

  12. Решите уравнение:

  13. Решите уравнение:

  14. Решите уравнение:

  15. Найдите сумму корней уравнения .

  16. Найдите сумму корней уравнения .

  17. Найдите произведение корней уравнения .

  18. Если - корень уравнения , то найдите значение выражения .

  19. Решите уравнение: а) ,б).

  20. Решите уравнение: .

  21. Решите уравнение: .

  22. Решите уравнение:

Просмотрено: 0%
Просмотрено: 0%
Скачать материал
Скачать материал "ПЗ по специальности Технология машиностроения,1 курс(2016-2017 уч.г.)"

Получите профессию

Менеджер по туризму

за 6 месяцев

Пройти курс

Рабочие листы
к вашим урокам

Скачать

Выбранный для просмотра документ ПЗ№35.docx

ПЗ № 35. Корни уравнений. Равносильность уравнений. Преобразование уравнений. Основные приемы решения уравнений.

1)Опорный конспект.

Уравнением с одной переменной x называется выражение f(x) = g(x), содержащее переменную величину x и знак равенства.

Число a называется корнем (или решением) уравнения f(x) = g(x), если при подстановке этого числа в уравнение получается верное числовое равенство.

Решение уравнения – это процесс, состоящий в основном в замене заданного уравнения другим уравнением, ему равносильным. Такая замена называется тождественным преобразованием.

Основные тождественные преобразования:

Замена одного выражения другим, тождественно равным ему. Например,уравнение 

( 3x+ 2 ) 2 = 15x+10 можно заменить следующим равносильным: 9x2 + 12x +4 = 15x + 10

Перенос членов уравнения из одной стороны в другую с обратными знаками. Так, в предыдущем уравнении мы можем перенести все его члены из правой части в левую со знаком « – »: 

9x2 + 12x + – 15x – 10 = 0, после чего получим:9x2 – 3x – 6 = 0 .

Умножение или деление обеих частей уравнения на одно и то же выражение (число), отличное от нуляЭто очень важно, так как новое уравнение может не быть равносильным предыдущему, если выражение, на которое мы умножаем или делим, может быть равно нулю. Уравнение x – 1 = 0 имеет единственный корень x = 1. Умножив обе его части на x – 3 , мы получим уравнение

( x – 1 )( x – 3 ) = 0, у которого два корня: x = 1 и = 3. Последнее значение не является корнем заданного уравнения x – 1 = 0. Это так называемый посторонний корень. И наоборот, деление может привести кпотере корня. Так, если ( x – 1 )( x – 3 ) = 0 является исходным уравнением, то корень x = 3 будет потерян при делении обеих частей уравнения на x – 3 .

Можно возвести обе части уравнения в нечетную степень или извлечьиз обеих частей уравнения корень нечетной степени. Необходимо помнить, что: а) возведение в четную степень может привести к приобретению посторонних корней; б) неправильное извлечение корня четной степени может привести к потере корней.

Уравнение 7= 35 имеет единственный корень x = 5 . Возведя обе части этого уравнения в квадрат, получим уравнение: 49x2 = 1225 , имеющее два корня: = 5 и = – 5. Последнее значение является посторонним корнем.

Неправильное извлечение квадратного корня из обеих частей уравнения 49x 2 = 1225 даёт в результате 7= 35,и мы теряем корень = – 5.Правильное извлечение квадратного корня приводит к
уравнению: | 7| = 35, а следовательно, к двум случаям: 1) 7= 35, тогда x = 5 ;

 2) – 7= 35, тогда x = – 5. Следовательно, при правильном извлечении квадратного корня мы не теряем корней уравнения.

ОДЗ (областью допустимых значений) уравнения называется множество тех значений неизвестной, при которых определены его правая и левая части.

Очевидно, что вне ОДЗ решений не существует, однако не все числа, входящие в ОДЗ, служат решениями уравнения. Уравнение можно решить и не находя ОДЗ. С другой стороны, верно найденное ОДЗ и последующий отбор корней с его помощью не может гарантировать отсутствие ошибок.

Из этих правил следует, что с помощью стандартных приёмов и методов решения уравнений, а именно:

  • преобразования (раскрытие скобок, освобождение от знаменателя, приведение подобных членов, возведение уравнения в нечетную натуральную степень и т. д.),

  • разложения на множители (формально этот приём относится к преобразованиям, но, так как он довольно часто встречается самостоятельно, мы его выделяем особо),

  • введения вспомогательных неизвестных,

уравнение (1) может быть сведено к более простому и, самое главное, равносильному уравнению f1 (x) = g1 (x).

Решение уравнений

1) Если показательное уравнение сводится к виду ax = ab (1) где a> 0 и a ≠1, то оно имеет единственный корень х = b.

2) Иногда, чтобы привести показательное уравнение к виду (1), необходимо в левой части уравнения вынести за скобки общий множитель а х, например:

и т. д.

Или разделить обе части уравнения на выражение, не равное нулю, к примеру:

и т. д..

  1. некоторые показательные уравнения заменой а х = t сводятся к квадратным.

Надо помнить, что t > 0, так как показательная функция не может принимать отрицательные значения.

Чаще всего при решении логарифмического уравнения его приводят к виду

loga (f(x)) = log a (g(x)), тогда f(x) = g(x).

Решив полученное уравнение, следует сделать проверку корней, чтобы исходное уравнение не потеряло смысл.

Примеры: Решить следующие уравнения :

1) 9 х – 73 х = - 12;2) , 3) 4) 81 х – 29 х = 8 3х;

5) log5 (x2 - 10) = log 5 9x; 6) log 7 (x2 + 6x)=1;

7) log 0,6 (x + 3) + log 0,6 (x - 3) = log 0,6 ( 2x - 1) ; 8 ) 25x5x = 625.

Решение:

1) 9 х – 73 х = - 12; 3 х = tt2 - 7t + 12=0t1 = 3,t 2= 43x = 3;3x = 4

x1 = 1; x 2= log 3 4

Ответ. х1 = 1;. x 2= log 3 4.

2) .

Ответ: x1=5; x2= - 1

3) 4x=t

hello_html_1b781d56.png

Ответ. x = log 44,4.

4) 27 х – 29 х = 8 3х; 3x (9x - 23x - 8)=0, ,3x = t > 0t2- 2 t - 8=0t=4 u t= - 2 тогда 3x = 4x = log 34

Ответ. x = log 34.

5) log5 (x2 - 10) = log 5 9x; x2 - 9x - 10=0 x1=10 u x2=-1 x > 0

Ответ. x = 10

6) log 7 (x2 + 6x)=1; (x2 + 6x)=7x2 + 6x - 7=0x1 = - 7 u x2 = 1

Ответ. x1= - 7; x 2= 1.

  1. log 0,6 (x + 3) + log 0,6 (x - 3) = log 0,6 ( 2x - 1) log 0,6 (x + 3) (x - 3) = log 0,6 ( 2x - 1)

х2 – 9 = 2х - 1 х2 – 9 - 2х + 1=0 х 2 – 2х - 8 =0х1 = 4 и х2 = - 2

ОДЗ: х+3 > 0, x – 3 > 0, 2x -1 > 0x > - 3, x > 3,x > 1/2 x > 3 х1 = 4 и х2 = - 2.

Ответ. х1 = 4.

8 ) 25x5x = 625 , 53x = 62553x =543x=4x=4/3. Ответ. х= 4/3.

9)Решить уравнение log2 (x – 5) + log2 (x +2) = 3

Решение: Используем свойство логарифмов. Представим число (3) как логарифм

по основанию 2:

log2( x-5)(x + 2 ) = log2 8 (x-5)(x+2) = 8 x2 – 3x – 10 = 8 .

x2 – 3x - 18 = 0; x1 =- 3; x2 = 6.

Выполнив проверку, убеждаемся, что при x = - 3 log2 (x – 5) и

log2 (x+2) не имеют смысла

Ответ. х = 6.

10) Решить уравнение: .

Решение:

Используя определение логарифма, и учитывая область определения, получим


Ответ: 21.

11) Решить уравнение: .

Решение:


Ответ: 1.

12) Решить уравнение: .

Решение:

Решая полученное квадратное уравнение заменой , находим


Ответ: 2, 8.

2)А)Решить задание :

1.Решить следующие уравнения

1) 4 х+3 + 4 х =260; 2) ; 3) 4) 36 х – 218 х = 8 9х;

5) log3 (x2 + 6) = log3 5x; 6) log12 (x2x)=1; 7) log20,3 (x+1) – 4 log 0,3 (x+1) + 3 =0; 8) 9x3x = 81.

2. Решите уравнение: а) hello_html_m210f189a.gif; б) hello_html_3e7e6ab7.gif; в) hello_html_m1fc53f6a.gif.

3. Решите уравнение: а) hello_html_m46c4ac43.gif; б) hello_html_m62035bd7.gif; в) hello_html_m2653f2da.gif.

4. Решить уравнение: .

5. Решить уравнение: .

6. Решить уравнение: .

7. Решить уравнение: .

8. Решить уравнение: .

9. Решить уравнение: .

10. Решить уравнение: .

В) ТЕСТ.

1.Найдите сумму корней уравнения х3 +2х2 -9х –18 = 0.

1) -2 2) -8 3) 2 4) 8

2. Найдите сумму корней уравнения .

1) 1,5 2) 8 3) 8,5 4) 6,5

3. Решите уравнение .

1) 4 2) 12 3) 2 4) 8

4. Найдите сумму корней уравнения .

1) 1 2) 3 3) 5 4) 6

5. Решите уравнение .


6. Сколько корней имеет уравнение: х4+9х2+4=0.

1) 2 2) ни одного 3) 4 4) 1

7. Решите уравнение .

1) 1 2) 2 3) 3 4) 8

8. Найдите сумму корней уравнения .

1) 2) 3) 15 4)

9. Решите уравнение .

1) 100 2) 1 3) 0,1 4) 10

10. Решите уравнение .


11. Сколько корней имеет уравнение

1) 4 2) 2 3) 1 4) ни одного

12. Укажите промежуток, которому принадлежит корень уравнения

1) (-;0) 2) (0; 5) 3) (5; 50) 4) (50;100).

13. Решите уравнение .

1) 2) 3) 3 4)

14. Найдите сумму корней уравнения .

1) -1,25 2) -3,25 3) -1 4) 1

С) ТЕСТ.

1. Решите уравнение: 2х = 32.

1) 5; 2) 16; 3) 2; 4) 0.

2. Решите уравнение: 3х – 1 = 27.

1) 2; 2) 4; 3) 10; 4) 8.

3. Решите уравнение: log 3 х = 3.

1) 3; 2) 1; 3) 27; 4) –3 .

4. Укажите промежуток, которому принадлежит корень уравнения:

log 5 х + log 5 9 = 3

1) [ – 10; – 15] ; 2) (5; 10); 3) (10; 13); 4) [ 10; 15]

5. Укажите промежуток, которому принадлежит корень уравнения:

х + 1 = 1

1) [ 2; 5] ; 2) (– 2; – 1); 3) [– 1; 1]; 4) [ 1; 2]

6. Решите уравнение: log 7 (2 х – 3) = log 7 (х – 2).

1) 5; 2) решений нет; 3) 1; 4) 3 (1/3).

7. Найдите сумму корней уравнения: (1/3) +5 (1/3)х – 24 = 0

1) –1; 2) 11; 3) –5; 4) 1 .

8.Укажите промежуток, которому принадлежит корень уравнения: log 5 (3х – 1) = 3

1) (15; 20) ; 2) [ 0; 2]; 3) (20; 40); 4) [ 40; 50]

9. Решите уравнение: 5х+1 + 5х + 5х – 1 = 31.

1) решений нет; 2) 0; 3) 1; 4) 5 .

10. Решите уравнение: : log 5 (2 х – 3) + log 5 6 = log 5 18.

1) 0; 2) 3; 3) 2; 4) 1/3.

11. Найдите сумму корней уравнения: log22 х – 4 log 2 х= 12.

1) 64,25; 2) 8; 3) 4; 4) 60.

12. Укажите промежуток, которому принадлежит корень уравнения: log 5 (3х + 1) = 2

1) (–∞; 0) ; 2) [ 0; 7]; 3) (2; 3); 4) (- 1/3; +∞)

13. Решите уравнение: 3х+2 + 3 х + 1 + 3х = 39

1) –1; 2) 0; 3) 1; 4) 3 .

14. Найдите сумму корней уравнения: 3 – 43х + 3 = 0.

1) 0; 2) 1; 3) 3; 4) 2.

15. Решите уравнение lg(5х+11)–lg(1/2) = lg13

1) –0,9; 2) 2,5; 3) 10/9; 4) 10.

16. Решите уравнение: .

1) –3 ; 2) 3; 3) 5; 4)5/3.























Просмотрено: 0%
Просмотрено: 0%
Скачать материал
Скачать материал "ПЗ по специальности Технология машиностроения,1 курс(2016-2017 уч.г.)"

Получите профессию

Бухгалтер

за 6 месяцев

Пройти курс

Рабочие листы
к вашим урокам

Скачать

Выбранный для просмотра документ ПЗ№36.docx

ПЗ № 36. Решение систем уравнений. Использование свойств и графиков функций для решения уравнений и неравенств.

Задание:

1)Перепишите и заполните пропуски:
Пример 1. Решить систему уравнений:  .

Решение: ,

Выразим у через х из (2) -го уравнения системы и подставим это значение в (1) -ое уравнение системы. Решаем (2) -ое уравнение полученной системы: 2х+2x+2=10, применяем формулу: ax+y=ax∙ay.

2x+2x∙22=10, вынесем общий множитель 2х за скобки: 2х · (1+22)=10 или 2х∙5=10, отсюда 2х=2.

2х=21, отсюда х=... Возвращаемся к системе уравнений. у = х + 1 = 1 + 1 = ...

Ответ: (1; 2).

Пример 2. Решить систему уравнений:  .

Решение: Представляем левую и правую части (1) -го уравнения в виде степеней с основанием 2, а правую часть (2) -го уравнения как нулевую степень числа 5. Если равны две степени с одинаковыми основаниями, то равны и показатели этих степеней — приравниваем показатели степеней с основаниями 2 и показатели степеней с основаниями 5. Получившуюся систему линейных уравнений с двумя переменными решаем методом сложения.

,

Находим х = … и это значение подставляем вместо х во второе уравнение системы, находим у.

2(2 + у) = 7, 2 + у = 3,5 ; у = … Ответ: (2; 1,5).

Пример 3. Решить систему уравнений: .

Решение: ,

Сделаем замену , .

Выразим через . Подставим во 2 уравнение. Решим уравнение с переменной .

. По теореме Виета ,


Возвращаемся к х, у. , х = 2, у = 1. Ответ: (2; 1).

Пример 4. Решить систему уравнений:

Решение: ,

Подставим из 2 уравнения у в 1, решим с переменной х.

х· (4х 15) = 4, 4х2 15х 4 = 0. D = 152 4 = 225 64 = .


По свойству логарифмов х > 0, y > 0, поэтому х = 0,25. Найдем у: у = 4 0,25 15 = 1 + 15 = …

Ответ: (0,25;16).

Пример 5. а)Решить систему уравнений:

Решение: ,

Решаем способом сложения: 2 = 10, = 5, х = …, у = х21 = 25 21 = …

Ответ: (25,4).

б) Решить систему уравнений:

Решение: ,

Из 1 уравнения выразим х и подставим во 2: 2х = 5 у, х = 2,5 0,5у,

, , ,

, D = ()2 4 = 9 352 = …


О.Д.З. : , у2 не уд. О.Д.З., поэтому

Ответ: (3;).

Пример 6. Решить уравнение hello_html_m51b291f3.png

ОДЗ этого уравнения состоит из всех x, одновременно удовлетворяющих условиям hello_html_2eb3b384.png. Это значит, что ОДЗ есть пустое множество. Этим решение уравнения и завершается, т. к. установлено, то ни одно число не может являться решением, т.е. уравнение не имеет корней.

Ответ: решений нет.

Пример 7. Решить неравенство hello_html_m3821e4c8.png

Нахождение ОДЗ неравенства есть трудная задача, поэтому перейдем к равносильной ему системе неравенствhello_html_5d6879e5.pnghello_html_m29a2435e.png.

Третье неравенство имеет решение hello_html_m53fb9541.png. Первое и второе неравенство справедливо лишь для x из промежутка hello_html_m31dbfcc5.png. Поэтому этот промежуток является множеством решений системы.

Ответ: hello_html_m31dbfcc5.png.

Пример 8. Решить неравенство hello_html_29bb9126.png. Существует стандартный прием решения: возведение в квадрат (при условии hello_html_53296a34.pnghello_html_m23cdad48.png0). Мы рассмотрим решение данного неравенства с использованием свойства монотонности. Функция, расположенная в левой части неравенства, монотонно возрастает, в правой части - убывает. Из этого следует, что уравнение hello_html_3124ddfd.png имеет не более одного решения, причем если x0 – решение этого уравнения, то при hello_html_24195d59.png будет hello_html_23029354.png, а решением данного неравенства будет hello_html_4c5db010.png. Значение hello_html_450402f6.png легко подбирается: hello_html_me29bdd5.png.

Ответ: hello_html_m7c58eed1.png.

Пример 9. Решить уравнение hello_html_6f9b3b8f.png

Данное уравнение имеет очевидное решение hello_html_32a3376d.png. Докажем, что других решений нет. Поделим обе части на hello_html_8475fac.png, получим hello_html_m1d3f68cc.png.Левая часть представляет собой монотонно убывающую функцию. Правая часть функция постоянная. Следовательно, каждое свое значение она принимает один раз, то есть данное уравнение имеет единственное решение.Ответ: hello_html_32a3376d.png.

Пример 10.а)

hello_html_2d44b68f.jpg

б)

hello_html_m16d16eed.jpg

2)Решить задание ( по примерам):

  1. Решить систему уравнений:

  2. Решить систему уравнений: 

  3. Решить систему уравнений:

  4. Решить систему уравнений:

  5. а)Решить систему уравнений:

б) Решить систему уравнений:

3)А)Решить задание:

  1. Найти значение выражения ,если и - решение системы уравнений

а) ,б) в) г) ,д)

  1. Найти значение выражения ,если и - решение системы уравнений

а)б) в) г) ,д)

  1. Решить систему уравнений: 


а) б) в)

г) д)

  1. Решите систему уравнений , решение принадлежит отрезку [0;2].


  1. Решить систему уравнений:

 а)б)

В) ТЕСТ. 1-в.

1. Какое неравенство не существует при х = - 6:hello_html_m9cafd1f.gif

1) ;

2) ;

3) ?

2. Укажите промежуток, которому принадлежат корни уравнения :

1) (2;3); 2) ; 3) (-0,5;0); 4) (1;2).

3. Выберите решение неравенства :hello_html_4ec559a2.gifhello_html_m44224fa1.gif

1) ;

2) ;

3) ;

4) .

4. Какой рисунок соответствует уравнению

? (укажите номер рисунка).

5. Укажите приближенное значение корня уравнения :

1) -2; 2) 0,4; 3) -0,3.

2-в.

1. Укажите промежуток, которому принадлежат корни уравнения :hello_html_7a8b474.gif

1) (2;3); 2) ; 3) (0,5;1,5); 4) (0;0,5).

2. Какой рисунок соответствует уравнению ? (укажите номер рисунка).

3. Какое неравенство не существует при х = - 2:

1) ;

2) ;

3) ?hello_html_m7559fa1.gifhello_html_3271fc9f.gif

4. Выберите решение неравенства :

1) ;

2) ;

3) ;

4) .


5. Укажите значение наибольшего корня уравнения :

1) 0,15; 2) 0; 3)1.


Просмотрено: 0%
Просмотрено: 0%
Скачать материал
Скачать материал "ПЗ по специальности Технология машиностроения,1 курс(2016-2017 уч.г.)"

Получите профессию

Секретарь-администратор

за 6 месяцев

Пройти курс

Рабочие листы
к вашим урокам

Скачать

Выбранный для просмотра документ ПЗ№37.docx

ПЗ № 37. Числовая последовательность, способы ее задания, вычисления членов последовательности. Предел последовательности. Бесконечно убывающая геометрическая прогрессия.

Задание:

1)Опорный конспект.

ЧИСЛОВАЯ ПОСЛЕДОВАТЕЛЬНОСТЬ  функция вида y = f(x), x О N, где N– множество натуральных чисел (или функция натурального аргумента), обозначается y = f(n) или y1, y2,…, yn,…. Значения y1, y2, y3,… называют соответственно первым, вторым, третьим, … членами последовательности.

Например, для функции y = n2 можно записать:

y1 = 12 = 1; y2 = 22 = 4; y3 = 32 = 9;…yn = n2;…

Способы задания последовательностей. Последовательности можно задавать различными способами, среди которых особенно важны три: аналитический, описательный и рекуррентный.

1. Последовательность задана аналитически, если задана формула ее n-го члена: yn = f(n).

Пример. yn = 2n – 1  последовательность нечетных чисел: 1, 3, 5, 7, 9, …

2. Описательный способ задания числовой последовательности состоит в том, что объясняется, из каких элементов строится последовательность.

Пример 1. «Все члены последовательности равны 1». Это значит, речь идет о стационарной последовательности 1, 1, 1, …, 1, ….

Пример 2. «Последовательность состоит из всех простых чисел в порядке возрастания». Таким образом, задана последовательность 2, 3, 5, 7, 11, …. При таком способе задания последовательности в данном примере трудно ответить, чему равен, скажем, 1000-й элемент последовательности.

3. Рекуррентный способ задания последовательности состоит в том, что указывается правило, позволяющее вычислить n-й член последовательности, если известны ее предыдущие члены. Название рекуррентный способ происходит от латинского слова recurrere – возвращаться. Чаще всего в таких случаях указывают формулу, позволяющую выразить n-й член последовательности через предыдущие, и задают 1–2 начальных члена последовательности.

Пример 1. y1 = 3; yn = yn–1 + 4, если n = 2, 3, 4,….

Здесь y1 = 3; y2 = 3 + 4 = 7; y3 = 7 + 4 = 11; ….

Можно видеть, что полученную в этом примере последовательность может быть задана и аналитически: yn = 4n – 1.

Пример 2. y1 = 1; y2 = 1; yn = yn–2 + yn–1 , если n = 3, 4,….

Здесь: y1 = 1; y2 = 1; y3 = 1 + 1 = 2; y4 = 1 + 2 = 3; y5 = 2 + 3 = 5; y6 = 3 + 5 = 8;

Арифметическая прогрессия.

Числовую последовательность, каждый член которой, начиная со второго, равен сумме предыдущего члена и одного и того же числа d, называют арифметической прогрессией, а число d – разностью арифметической прогрессии.

Таким образом, арифметическая прогрессия – это числовая последовательность {an}, заданная рекуррентно соотношениями a1 = a, an = an–1 + d (n = 2, 3, 4, …) (a и d – заданные числа).

Пример 1. 1, 3, 5, 7, 9, 11, … – возрастающая арифметическая прогрессия, у которой a1 = 1, d = 2.

Пример 2. 20, 17, 14, 11, 8, 5, 2, –1, –4,… – убывающая арифметическая прогрессия,

у которой a1 = 20, d = –3.

Нетрудно найти явное (формульное) выражение an через n. Величина очередного элемента возрастает на d по сравнению с предыдущим, таким образом, величина n элемента возрастет на величину (n – 1)d по сравнению с первым членом арифметической прогрессии, т.е.

an = a1 + d(n – 1). Это формула n-го члена арифметической прогрессии.

hello_html_m442ca9c.gif. Это формула суммы n членов арифметической прогрессии.

Пример3. При каком значении x числа 3x + 2, 5x – 4 и 11x + 12 образуют конечную арифметическую прогрессию?

Согласно характеристическому свойству, заданные выражения должны удовлетворять соотношению

5x – 4 = ((3x + 2) + (11x + 12))/2.

Решение этого уравнения дает x = –5,5. При этом значении x заданные выражения 3x + 2, 5x – 4 и 11x + 12 принимают, соответственно, значения –14,5, –31,5, –48,5. Это – арифметическая прогрессия, ее разность равна –17.

Геометрическая прогрессия.

Числовую последовательность, все члены которой отличны от нуля и каждый член которой, начиная со второго, получается из предыдущего члена умножением на одно и то же число q, называют геометрической прогрессией, а число q – знаменателем геометрической прогрессии.

Таким образом, геометрическая прогрессия – это числовая последовательность {bn}, заданная рекуррентно соотношениями b1 = b, bn = bn–1 q (n = 2, 3, 4…). (b и q – заданные числа, b ≠ 0, q ≠ 0).

Пример 1. 2, 6, 18, 54, … – возрастающая геометрическая прогрессия b = 2, q = 3.

Пример 2. 2, –2, 2, –2, …  геометрическая прогрессия b = 2, q = –1.

Пример 3. 8, 8, 8, 8, …  геометрическая прогрессия b = 8, q = 1.

Геометрическая прогрессия является возрастающей последовательностью, еслиb1 > 0, q > 1, и убывающей, если b1 > 0, 0 < q < 1.

Одно из очевидных свойств геометрической прогрессии состоит в том, что если последовательность является геометрической прогрессией, то и последовательность квадратов, т.е.

b12, b22, b32, …, bn2,… является геометрической прогрессией, первый член которой равен b12, а знаменатель – q2.

Формула n-го члена геометрической прогрессии имеет вид bn = b1qn–1.

Формула суммы первых n членов прогрессии

 hello_html_470610d0.gif, или hello_html_m6e7205ae.gif, hello_html_763ec99f.gif (в случае hello_html_m61e84691.gif, hello_html_1c16e6b4.gif).

Если геометрическая прогрессия бесконечно убывающая (hello_html_m474ab65f.gif), то ее сумма вычисляется по формуле  hello_html_m3403f07c.gif.

Обращение периодической десятичной дроби в обыкновенную. Предположим, мы хотим обратить периодическую десятичную дробь 0,(3) в обыкновенную. Рассмотрим эту десятичную дробь в следующем виде:

hello_html_m3f36da04.gif

Это бесконечно убывающая геометрическая прогрессия, первый член которой равен 3/10, а разность q = 1/10. В соответствии с выше приведенной формулой эта сумма равна:

hello_html_m22101018.gif

Таким образом, 0,(3) = 1/3.

1)

Бесконечно убывающая геометрическая прогрессия, т.к Sп=<1.

. . . х-1=3. х=4.

2) 1+2х+4х+…+(2х)+…=3,4-1,2х <0,5

Бесконечно убывающая геометрическая прогрессия, т.к <0,5. .

Бесконечно убывающая геометрическая прогрессия (|q|<1). hello_html_m522d392.png.


Пример 1. Найдите сумму всех целых чисел, начиная от 30 и до 80 включительно.

    Решение:

        Сумма всех целых чисел от 30 и до 80 включительно представляет собой сумму членов арифметической прогрессии, где а1 = 30, разность d = 1, а количество членов n = 51. 

        hello_html_m2c236891.png. Ответ: 2805.

Пример 2. Решите уравнение 22∙24∙26∙...∙22n=(0,125)-10 .

    Решение:

        22∙24∙26∙...∙22n=22+4+6+...+2n.  hello_html_m638d9306.png.

        Получили: 22+4+6+...+2n=230 hello_html_3e9ff5b7.png2+4+6+...+2n=30.

        В левой части равенства сумма n членов арифметической прогрессии, где а1 =2, а d = 2. Согласно формуле суммы членов арифметической прогрессии имеем:

        hello_html_m512755f7.png.

        n1 = – 6 не удовлетворяет условию т.к. n отрицательным быть не может.

        n2 =5 удовлетворяет условию. Ответ: n = 5.

Пример 3. В геометрической прогрессии пятый член равен 2, восьмой равен 16. Найти сумму первых десяти членов.

    Решение:Пусть (bn) – геометрическая прогрессия. Известно, что b5 = 2. По формуле n – го члена геометрической прогрессии      b5 = b1 q4 = 2. Аналогично b8 = b1 q7 = 16. Поделив второе равенство на первое, получим q3 = 8, т.е. q = 2. Подставив q = 2 в первое равенство, найдём b1:

        hello_html_6ff45943.png. Согласно формуле hello_html_m65da4e87.pngполучим: hello_html_3f7115ea.png.

Ответ: hello_html_aae12bf.png.

Пример 4. Записать число 0,6222… обыкновенной дробью.

    Решение:

        Запишем число 0,6222…в виде суммы: 0,6222…= 0,6 + 0,02 + 0,002 + 0,0002 + …,

        где 0,02 + 0,002 + 0,0002 + … - сумма членов бесконечно убывающей геометрической прогрессии, которая находится по формуле hello_html_m5ab5ce1b.png, где b1 = 0,02; q = 0,1.

        hello_html_549eecc.png.

Ответ: 28/45 .

2)Решить задание:

  1. Найти произведение третьего и четвёртого членов арифметической прогрессии,

если первый член равен 3, а второй равен – 2.

    2) Между числами – 8,8 и 2 вставьте пять чисел так, чтобы получилась арифметическая прогрессия.

  1. Третий член арифметической прогрессии в три раза меньше шестого, а сумма второго и

пятого равна 16. Определите первый член прогрессии

    4) Найдите знаменатель геометрической прогрессии, если b1=2, b5=162.

    5) В геометрической прогрессии q=0,5, b6=1/32 , найти b1.

    6) Найдите третий член геометрической прогрессии, если её знаменатель равен -2,

а седьмой член 16.

    7) В геометрической прогрессии b1+b2+b3=31, b1+b3=26. Найти b7.

    8) Какое наибольшее число последовательных нечётных чисел, начиная с 1, можно сложить,

чтобы получившаяся сумма осталась меньше 400?

    9) В геометрической прогрессии b2>b1 в два раза, а b=64. Найти b1 .

    10) Найдите сумму первых пяти членов последовательности, общий член которой выражается

формулой hello_html_284c9d.png.

    11) Сумма членов арифметической прогрессии с третьего по одиннадцатый включительно

равна 27. Найти номер члена прогрессии равного 3.

    12) Сумма бесконечной убывающей геометрической прогрессии равна 32, а сумма пяти её

первых членов равна 31. Найдите первый член прогрессии.

    13) Найдите сумму всех чётных натуральных трёхзначных чисел, делящихся на 3.

    14) Вычислить сумму: hello_html_m52b7aa09.png.

    15) Найдите сумму всех положительных членов арифметической прогрессии 10,3; 8,5; ….

    16) В арифметической прогрессии a10=-23 . Найти a3+a17.

    17) В арифметической прогрессии a5+a9=-20. Найти a7.

    18) Произведение девяти первых членов геометрической прогрессии равно hello_html_m6523f58.png.

Какой член геометрической прогрессии можно найти на основании этой информации? Чему он равен?

    19) Решите уравнение hello_html_m14c9512b.png.

    20) Найдите суммуhello_html_m19749d63.png.

    21) Три различных числа a1, a2, a3 в указанном порядке образуют арифметическую прогрессию,

а числа 2a3-a1, a2+a3-a1, a1  в указанном порядке образуют геометрическую прогрессию.

Найти знаменатель геометрической прогрессии.

3)Решить задание:

  1. Пусть – арифметическая прогрессия с разностью d и Sn – сумма n первых членов. Найти:

1. a13, если a5=2; a40=142.

2. a1+a20, если a3+a18=50.

3. n, если a1=3; a2=5; Sn=360.

4. a1 и d, если a17+a20=35; a16a21=150.

5. a1 и d, если Sn=2n2-3n.

6. Сумму всех натуральных трехзначных чисел, не делящихся на 3.

7. Первых 100 натуральных чисел, каждое из которых при делении на 5 дает в остатке 2.

  1. Пусть – геометрическая прогрессия со знаменателем q и Sn – суммой первых n членов. Найти:

8. b6, если b5=36, b7=114.

9. q, если b1=10, b2+b3=60.

10. b13, если b11=25, b15=400.

11. b1 и q, если b1+b2+b3=62, b12+b22+b32=2604.

12. S6, если b1=–2, b6=–486.

13. n, если b1=9, bn=, Sn=25.

14. Какому условию удовлетворяют три числа a1, a2, a3, которые одновременно являются последовательными членами как геометрической, так и арифметической прогрессий?

15. Решить уравнение: .

16. По преданию, индийский шах позволил изобретателю шахматной игры самому назначить себе награду. Изобретатель просил, чтобы ему за первую клетку шахматной доски было дано 1 зерно, за вторую – 2, за третью – 4. В общем случае, за каждую следующую клетку в 2 раза больше, чем за предыдущую. Узнать, сколькими цифрами изображается число зерен, предназначенное изобретателю; найти это число.

17.Три числа образуют геометрическую прогрессию. Если второе число увеличить на 2, то прогрессия станет арифметической, а если после этого последнее число увеличить на 9, то прогрессия станет геометрической. Найти эти числа.

18.Решить уравнение .

19.Найти a1 и d, если a11=6; a16=8,5.

20. Может ли число 75 быть членом геометрической прогрессии , у которой b1=4 и q=?

21.Найти количество всех трехзначных натуральных чисел, делящихся на 7.

22.Доказать, что последовательность с общим членом является арифметической прогрессией.












Просмотрено: 0%
Просмотрено: 0%
Скачать материал
Скачать материал "ПЗ по специальности Технология машиностроения,1 курс(2016-2017 уч.г.)"

Получите профессию

Копирайтер

за 6 месяцев

Пройти курс

Рабочие листы
к вашим урокам

Скачать

Выбранный для просмотра документ ПЗ№38.docx

ПЗ № 38. Производная: механический и геометрический смысл производной. Уравнение касательной в общем виде.

Задание:

1)Опорный конспект.

Механический смысл производной. Рассмотрим простейший случай: движение материальной точки вдоль координатной оси, причём закон движения задан:  координата  x  движущейся точки – известная функция  x ( t ) времени  t. В течение интервала времени от  t0  до  t0 + hello_html_7bc674bb.gif  точка перемещается на расстояние:  x ( t0 + hello_html_7bc674bb.gif ) - x ( t0 ) = hello_html_m1ebb9fce.gif, а её средняя скорость равна:  va = hello_html_m1ebb9fce.gif / hello_html_7bc674bb.gif . При  hello_html_7bc674bb.gif hello_html_2c63fb57.gif 0  значение средней скорости стремится к определённой величине, которая называется мгновенной скоростью  v ( t0 )  материальной точки в момент времени  t0 . Но по определению производной мы имеем:

hello_html_75ac67c4.gif

отсюда,  v t0 ) = x’ t0 ) , т.e. скорость – это производная координаты по времени. В этом и состоит  механический смысл производной.Аналогично, ускорение – это производная скорости по времени:  a = v’ t ).

Пример. Точка движется прямолинейно по закону S (t)= 2t3 – 0,5t2 + 3t (S – путь в метрах, t – время в секундах). Вычислить скорость движения точки в момент времени t=1с.

Решение: vt ) = s ’ t ) = 6t2 t + 3, v(1) = 6 – 1 + 3 = 8.

Геометрический смысл производной заключается в том, что численно производная функции в данной точке равна тангенсу угла, образованного касательной, проведенной через эту точку к данной кривой, и положительным направлением оси Ох:

hello_html_m2921bfa1.jpg

 Уравнение касательной. 

y =  f ( x0 ) +  f ’( x0 ) · ( x – x0  ) .


2)Перепишите и заполните пропуски:

Пример 1. Найти угловой коэффициент касательной к графику функции y = f(x) в точке с

абсциссой х: а) y(x) = x³, x = 1, б) y(x) = ln x, x = 1, в) y(x) = 3x² 4x, x = 2,

г) y(x) = х3 + 7x² 5x+3, x = 3, д) y(x) = ех, x = ln 7, e) y(x) = 7sinx, x = 0,ж) y(x) = е, x = ln 4.

Решение: угловой коэффициент k равен производной от функции в точке, т.е. k = y (x0) ,

найдем производные и вычислим их в точке x0

a)   бв)  

г)

д) е ln 7= …,е) 7cos x, 7 cos 0 = 7 1 = …,

ж) е3 ln 4 = 343 = 364 = …

Ответ: a)3, б)1, в)8,г) 64,д) 7,е)7,ж) 192.

Пример 2. а) Найти угловой коэффициент k, если α = arctg 6, α = - arctg 8.

б) Найти α,если y(x) = х3, x = 2.

Решение: а) k = tgα = tg k = tgα = tg k = tgα = tg

k = tgα = tg

б)

Ответ: а)1, ,6,- 8, б) arctg 4.

Пример 3. Дана функция y = x3. Составить уравнение касательной к графику этой функции в точке x0 = 2.
Решение: Уравнение касательной: y = f  (x0) · (x − x0) + f(x0). Точка x0 = 2 нам дана, а вот значения f (x0) и f (x0) придется вычислять.

Для начала найдем значение функции. Тут все легко: f (x0) = f (2) = 23 = …;
Теперь найдем производную: f  (x) = (x3) = 3x2;
Подставляем в производную x0 = 2: f   (x0) = f  (2) = 3 · 22 = 34 = …;
Итого получаем: y = 12 · (x − 2) + 8 = 12x − 24 + 8 = 12x − 16. 
Это и есть уравнение касательной.

Ответ: y = 12x − 16. 
Пример 4. Составить уравнение касательной к графику функции f (x) = 2sin x + 5 в точке x0 = π/2.

Решение: f (x0) = f (π/2) = 2sin (π/2) + 5 = 2 + 5 = …; f  (x) = (2sin x + 5) = 2cos x;
f  (x0) = f  (π/2) = 2cos (π/2) = 0;

Уравнение касательной: y = 0 · (x − π/2) + 7  y = ...hello_html_44c9db7b.png

Ответ: y = 7.

Пример 5. Составьте уравнение касательной к графику функции  

в точке M(3; – 2).

Решение: Точка M(3; – 2) является точкой касания, так как

1. a = 3 – абсцисса точки касания.2. f(3) = – 2. 3. f '(x) = x2 – 4, f '(3) = 9 4 = …
y = – 2 + 5(x – 3), y = …x – 17 – уравнение касательной.

Ответ: y = 5x – 17.

Пример 6. Напишите уравнения всех касательных к графику функции y = – x2 – 4x + 2, проходящих через точку M(– 3; 6).

Решение: Точка M(– 3; 6) не является точкой касания, так как f(– 3)  6 (рис. 2).hello_html_m126a8a75.png

1. a – абсцисса точки касания.
2. f(a) = – a
2 – 4a + 2.
3. f '(x) = – 2x – 4, f '(a) = – 2a – 4.
4. y = – a2 – 4a + 2 – 2(a + 2)(x – a) – уравнение касательной.

Касательная проходит через точку M(– 3; 6), следовательно, ее координаты удовлетворяют уравнению касательной.

6 = – a2 – 4a + 2 – 2(a + 2)(– 3 – a),
a
2 + 6a + 8 = 0 , D = 62 41 8 = 36 32 = …,

а1= (6 2) : 2 = 8 : 2 = …, а2 = (6 2) : 2 = 4 : 2 = …,

Если a = – 4, то уравнение касательной имеет вид y = 4x + 18.

Если a = – 2, то уравнение касательной имеет вид y = 6.

Ответ: y = 4x + 18 или y = 6.
Пример 7. Напишите уравнения всех касательных к графику функции y = x3 – 3x2 + 3, параллельных прямой y = 9x + 1.

Решение: 1. a – абсцисса точки касания. 2. f(a) = a3 – 3a2 + 3.3. f '(x) = 3x2 – 6x, f '(a) = 3a2 – 6a.

Но, с другой стороны, f '(a) = 9 (условие параллельности). Значит, надо решить уравнение 3a2 – 6a = 9. 3a2 – 6a 9 = 0, hello_html_5a070ee0.png

D = (6)2 43 () = 36 108 = …, а1= (6 12) : 6 = 18 : 6 = …,

а2 = (6 12) : 6 = 6 : 6 = …,

Его корни a = – 1, a = 3 (рис. 3).

4. 1сл.) a = – 1; f(– 1) = – 1– 3 + 3 = …;  f '(– 1) = 3 + 6 = …;

 y = – 1 + 9(x + 1); y = 9x + 8 – уравнение касательной;

2сл.) a = 3; f(3) = 27–27 + 3 = …; f '(3) = 27 – 18 = …;
y = 3 + 9(x – 3); y = 9x – 24 – уравнение касательной.

Ответ: y = 9x + 8 и y = 9x – 24.

Пример 8. Напишите уравнение касательной к графику функции y = 0,5x2 – 3x + 1, проходящей под углом 45° к прямой y = 0 (рис. 4).hello_html_m32119c54.png

Решение: Из условия '(a) = tg 45°, найдем a:  a – 3 = 1 ,a = 3 + 1 = ...

1. a = 4 – абсцисса точки касания.
2. f(4) = 8 – 12 + 1 = ...
3. f '(4) = 4 – 3 = ...
4. y = – 3 + 1(x – 4). y = x – 7 – уравнение касательной.

Ответ: y = x – 7.

Пример 9. На параболе у = х2 взяты две точки с абсциссами 1 и 3. Через эти точки проведена прямая. В какой точке параболы касательная будет параллельна проведенной прямой?

Решение: у = х2 , (1;1), (3;9). Найдем уравнение прямой .

4х – 4 = у – 1. у = 4х – 3.

Прямые параллельны, если их угловые коэффициенты равны.

- угловой коэффициент касательной в точке с абсциссой х0.

 0 = 4. х0 = ... ,

Ответ: в точке (2;4) касательная параллельна заданной прямой.

Пример 10. При каких b и c прямые y = x и y = – 2x являются касательными к графику

функции y = x2 + bx + c?

Решение: Пусть t – абсцисса точки касания прямой y = x с параболой y = x2 + bx + c;

p – абсцисса точки касания прямой y = – 2x с параболой y = x2 + bx + c.

Тогда уравнение касательной y = x примет вид y = (2t + b)x + c – t2, а уравнение

касательной y = – 2x примет вид y = (2p + b)x + c – p2. Составим и решим систему уравнений:

;


2t = 1,5; t = 0,75;

p = – t = …,

c = = = …,

b = 1 – 2t = 1 – 2 0,75 = 1– 1,5 = …

Ответ: b = – 0,5; c = 0,562 5.

3)Решить задание ( по примерам):

  1. Найти угловой коэффициент касательной к графику функции y = f(x) в точке с

абсциссой х: а) y(x) = x4, x = 1, б) y(x) = ln x, x = 2, в) y(x) = 3x² - 4x, x = 4,

г) y(x) = х3 + 7x² - 5x+3, x =5, д) y(x) = ех, x = ln 8, e) y(x) = 9sinx, x = 0,ж) y(x) = е, x = ln 6.

  1. а) Найти угловой коэффициент k, если α = arctg 9, α = - arctg 11.

б) Найти α,если y(x) = х3, x = 4.

  1. Дана функция y = x3. Составить уравнение касательной к графику этой функции в точке x0 = 1.

  2. Составить уравнение касательной к графику функции f (x) = 4sin x + 5 в точке x0 = π/2.

  3. Составьте уравнение касательной к графику функции  в точке M(3; – 1).

  4. Напишите уравнения всех касательных к графику функции y = – x2 – 4x + 2, проходящих через точку M(– 3; 9).

  5. Напишите уравнения всех касательных к графику функции y = x3 – 3x2 + 3, параллельных

прямой y = 24x + 1.

  1. Напишите уравнение касательной к графику функции y = 0,5x2 – 5x + 1, проходящей
    под углом 45° к прямой y = 0 .

  2. На параболе у=х2 взяты две точки с абсциссами 1 и 2. Через эти точки проведена прямая.
    В какой точке параболы касательная будет параллельна проведенной прямой?

  3. При каких b и c прямые y = x и y = – 2x являются касательными к графику
    функции y = x2 + 2bx + c?

  4. Точка движется прямолинейно по закону S (t)= t5t4 + 6 (S – путь в метрах, t – время в секундах). Вычислить скорость движения точки в момент времени t=2с.

  5. Точка движется прямолинейно по закону S (t)= 2t3 – 0,5t2 + 3t (S – путь в метрах, t – время в секундах). Вычислить скорость движения точки в момент времени t= 2с.

  6. Какая из приведенных зависимостей описывает равноускоренное движение:

а) x = 4 + 2t; б) v = 5; в) x = 8 - 2t - 4t2; г) x = 10 + 5t2.

  1. Точка движется вдоль оси х согласно закону х = 8 – 2t – 4t2 . Определите начальную скорость и ускорение . Запишите уравнение для скорости.

4)Решить задание:

  1. Напишите уравнения касательных, проведенных к графику функции y = 2x2 – 4x + 3 в точках пересечения графика с прямой y = x + 3.

  2. При каких значениях a касательная, проведенная к графику функции y = x2 – ax в точке графика с абсциссой x0 = 1, проходит через точку M(2; 3)?

  3. Найдите все общие точки графика функции y = 3x – x3 и касательной, проведенной к этому графику через точку P(0; 16).

  4. На кривой y = x2 – x + 1 найдите точку, в которой касательная к графику параллельна
    прямой y – 3x + 1 = 0.

  5. Найдите угол q между касательными к графику функции y = x3 – 4x2 + 3x + 1, проведенными в точках с абсциссами 0 и 1.

  6. Прямая y = 2x + 7 и парабола y = x2 – 1 пересекаются в точках M и N. Найдите точку K пересечения прямых, касающихся параболы в точках M и N.

  7. При каких значениях b касательная, проведенная к графику функции y = bx3 – 2x2 – 4 в точке с абсциссой x0 = 2, проходит через точку M(1; 8)?

  8. Найти угол между касательными к графику функции , проведенными в точках с абсциссами 1 и 2.

  9. Является ли прямая у = х – 1 касательной к кривой у = х3 – 2х + 1?

  10. Найдите уравнение касательной к графику функции  в точке с абсциссой .

  11. К графику функции у = 3(х + 2) проведены две параллельные касательные, одна из которых проходит через точку графика с абсциссой х0 = – 1. Найдите абсциссу точки, в которой
    другая касательная касается графика данной функции.

  12. Напишите уравнение касательной к графику функции f(x) = x2 – 4x + 5, если эта касательная проходит через точку (0; 4) и абсцисса точки касания положительна.

  13. Напишите уравнение касательной к графику функции f(x) = x2 + 3x + 5, если эта касательная проходит через точку (0; 1) и абсцисса точки касания отрицательна.

  14. Найдите уравнение параболы f(x) = ax2 + bx + 1 касающейся прямой у = 7х + 2
    в точке М (1; 5).

  15. К графику функции провести касательную так, чтобы она была параллельна прямой у = 4х – 5.

  16. Из точки (0; 1) провести касательную к графику функции.

  17. Составить уравнение касательной к графику функции  в точке с абсциссой.

  18. Составить уравнение касательной к графику функции  в точке с абсциссой 

  19. Составить уравнение касательной к графику функции
      в точке с абсциссой  .

  20. Составить уравнение касательной к графику функции > 0, отсекающей от осей координат треугольник, площадь которого равна .

  21. Задача. Пусть X = 2 + 4t2 - sin2πt. Найти: а) мгновенную скорость, б) ускорение, если t = 0,5 c.

  22. Задача. Высота снежка, брошенного вертикально вверх со скоростью U0 с начальной высоты h0, меняется по закону h =h0+U0t-gt2/2, где g =10м/c – ускорение силы тяжести. Покажите, что энергия камня Е= mv2/ 2 + mgh, где m масса снежка, не зависит от времени.

  23. Задача. Количество электричества, протекающее через проводник, задаѐтся формулой

q(t) = t+4/t. В какой момент времени ток в цепи равен нулю?









Просмотрено: 0%
Просмотрено: 0%
Скачать материал
Скачать материал "ПЗ по специальности Технология машиностроения,1 курс(2016-2017 уч.г.)"

Получите профессию

Секретарь-администратор

за 6 месяцев

Пройти курс

Рабочие листы
к вашим урокам

Скачать

Выбранный для просмотра документ ПЗ№39.docx

ПЗ № 39. Правила и формулы дифференцирования, таблица производных элементарных функций.

Задание:

1)Опорный конспект.

Пример 1. Найти производную функции  y = .

Решение: По свойству дифференцирования произведения,

hello_html_m667262fc.png.

Используя формулу для нахождения производной показательной и степенной функций, получим: hello_html_m33ce9c10.png , hello_html_m7a155e45.png

Для нахождения производной использовались правила дифференцирования и таблица производных функций. Ответ: hello_html_m7259a439.png .

Пример 2. Найти производную функции  y = .

Решение: Воспользуемся правилом дифференцирования частного:

hello_html_m6c0be978.png.

Производная суммы/разности равна сумме/разности производных и константу можно выносить за знак производной, поэтому имеем:

hello_html_m6d9e2531.png,

hello_html_m4661b9f4.png,

hello_html_5aeb11a1.png, hello_html_303e1e97.png , hello_html_47c000ce.png .

Ответ: hello_html_6873cbb0.png .

Пример 3. Найти производную функции y =   .

Решение: По правилу дифференцирования частного:

hello_html_m2652334c.png,

Далее воспользуемся формулами из таблицы производных - формулам для производных степенной и тригонометрических функций, а также учитываем тот факт, что производная суммы равна сумме производных:

hello_html_6bfbb3b0.png,

hello_html_4f5a6440.png,

hello_html_345a5152.png, hello_html_3dd92d7d.png .

Ответ: hello_html_5f22ddde.png .

Пример 4. Найти производную функции  hello_html_mb62401c.png .

Решение: По свойству дифференцирования частного получаем:

hello_html_m1bbacdfd.png,

Далее пользуясь формулами для производных логарифмической и степенной функции, получим:

hello_html_m7e5e4019.png, hello_html_7fc6a7d9.png , hello_html_m68b092d7.png .Ответ: hello_html_ecc0376.png .

Пример 5. а) Найти производную функции  .

Решение:

Примените таблицу основных производных и формулы производных линейной комбинации и отношения функций.



Ответ:  .

б) Вычислить производную функции y = cos ln ().

Решение: Примените таблицу основных производных и формулу производной сложной функции.

y / = sin ln (3x2 ) (ln (3x2)) / = sin ln (3x2 ) / =

= sin ln (3x2 ) .

Ответ:  sin ln (3x2 ) .

Задание: Найти производную функции: 

1) y = . 2) y = . 3) y = . 4) .5) а) . б) y = cos ln (2x2).

2)Перепишите и заполните пропуски:

Пример 1. Найдите производные функций: а) y = ex x7 ,б) у=3ех+cos2x, в) у = ехsinx,

г) у= ln2x ,д) , е) , ж)

Решение: а) б) в) = ех cosx; г) ,

д)е)ж)

Ответ: а)б) в) = ех cosx; г) ,д)

е)ж)

Пример 2. Вычислите значение производной функции:

а) у= в точке , б) у=ех sinx + x2 в точке ,

в) у = cos2x + 4x в точке ,г) в точке .

Решение: а)


б)

в)

г) Ответ: а)10,5; б)1;в)4; г)2.

Пример 3. Найдите производные функций: а) б)

в) г) д)
Решение: а) у  (x) = (x 2 + sin x) = (x 2) + (sin x) = …x + cos x;
б) у  (x) = (x 3 · cos x) = (x 3) · cos x + x 3 · (cos x) = …x 2 · cos x + x 3· (− sin x) =

= x 2 · (3cos x  x · sin x),

в) у  (x) = ((x 2 + 7x − 7) · e x ) = (x 2 + 7x − 7) · e x + (x 2 + 7x − 7) · (e x ) = (2x + 7) · e x +

+(x 2 + 7x − 7) · e x = e x · (2x + 7 + x 2 + 7x−7) = (x 2 + …x) · e x = x(x + …) · e x .

г)
д)

По традиции, разложим числитель на множители — это значительно упростит ответ:


Ответ: а) у  (x) = 2x + cos x; б) у  (x) = x 2 · (3cos x  x · sin x), в) у  (x) = x(x + 9) · e x ,

г) д)

Пример 4. Найти производные функций:  f(x) = e 2x + 3g(x) = sin (x 2 + ln x).
Решение: Заметим, что если в функции f(x) вместо выражения 2x + 3 будет просто x, то получится элементарная функция f(x) = e x . Поэтому делаем замену: пусть 2x + 3 = tf(x) = f(t) = e t . Ищем производную сложной функции по формуле:

f  (x) = f  (t) · t  = (e t ) · t  = e t · t . Выполняем обратную замену: t = 2x + 3. Получим:

f  (x) = e t · t  = e 2x + 3 · (2x + 3) = e 2x + 3 · 2 = … · e 2x + 3

Теперь разберемся с функцией g(x). Очевидно, надо заменить x 2 + ln x = t. Имеем:

g  (x) = g  (t) · t  = (sin t) · t  = cos t · t . Обратная замена: t = x 2 + ln x. Тогда:

g  (x) = cos (x 2 + ln x) · (x 2 + ln x) = cos (x 2 + ln x) · (…x + 1/x).

Ответ: f  (x) = 2 · e 2x + 3; g  (x) = (2x + 1/x) · cos (x 2 + ln x).
Пример 5. Найти производную функции :а)б)
Решение: а)

б)
Ответ: а) б)

2)Решить задание ( по примерам):

  1. Найдите производные функций: а) y = 2ex –3x7 ,б) у=5ех+cos3x, в) у = ехcosx,

г) у= – ln4х, д) , е) , ж)

  1. Вычислите значение производной функции:

а) у= в точке , б) у=2ех sinx +3 x2 в точке ,

в) у = cos2x + 8x в точке ,г) в точке .

  1. Найдите производные функций: а) б)

в) г) д)

  1. Найти производные функций:  f(x) = e 4x + 3; g(x) = sin (2x 2 + ln x).

  2. Найти производные функций : а)б)

3)А)Решить задание:

  1. Найдите производную функции y = e -x 2x7 , у= 4х3+ е .

  2. Найдите производную функции у = x2 + sinx в точке х0 =.
    Найдите производную функции у = sinх ex – 9x3 в точке xo=0.

  3. Найдите значение производной функции у = 5cos x – 7x в точке хо = 0 .

  4. Вычислите значение производной функции y = ln(2x+11)+ 5x в точке хо= 5.

  5. Найдите производную функции: а) б)

В) ТЕСТ.

ЧастьА.

А1. Найдите производную функции y = e -x -2x7 . 1) y´= - e-x -14x6; 2) y´= - e-x –; 3) y´= -e-x –2x6; 4) y´= e-x -14x6.

А2. Найдите производную функции у=4х3+ е .

1) у´=12х2 ; 2) у´=12х2 – е ; 3) у´=х4 - е ; 4) у´=12х2 – хе -х-1.

А3. Найдите производную функции у = x2 + sinx в точке х0 =.

1) 2 -1; 2) 2 + 1; 3) 2 -1; 4) 2. А4. Вычислите значение производной функции в точке хо=2. 1) 10; 2) 12; 3) 8; 4) 6.

А5. Найдите производную функции у = sinх ex – 9x3 в точке xo=0. 1) 0; 2) -1; 3) 1; 4) -9.

А6. Найдите значение производной функции у = 5cos x – 7x в точке хо = 0 . 1) -14; 2) -7;3) -9; 4) -2.

А7. Найдите производную функции .

1) 4х – 6+; 2) (2х - 3)2+; 3) 8х – 12 +; 4) 4х – 6 - . А8. Вычислите значение производной функции в точке хо= 4.

1) 21; 2) 24; 3) 0; 4) 3,5.

А9. Вычислите значение производной функции y = ln(2x+11)+ 5x в точке хо= -5. 1) 7; 2) -25; 3) 6; 4) 1. А10. Вычислите значение производной функции в точке хо= .

1) 1; 2) 2; 3) 0; 4) 4.

Часть В.

В1.Найдите производную функции:

1) ; 2) ;3) ; 4) ;

В2. К графику функции проведена касательная через точку с абсциссой . Вычислите тангенс угла наклона этой касательной к оси абсцисс.











Просмотрено: 0%
Просмотрено: 0%
Скачать материал
Скачать материал "ПЗ по специальности Технология машиностроения,1 курс(2016-2017 уч.г.)"

Получите профессию

Секретарь-администратор

за 6 месяцев

Пройти курс

Рабочие листы
к вашим урокам

Скачать

Выбранный для просмотра документ ПЗ№4.docx

ПЗ № 4. Параллельное проектирование и его свойства. Теорема о площади ортогональной проекции многоугольника. Взаимное расположение пространственных фигур.

Задание:

1) Опорный конспект.

ПАРАЛЛЕЛЬНОЕ ПРОЕКТИРОВАНИЕ

    В стереометрии изучаются пространственные фигуры, однако на чертеже они изображаются в виде плоских фигур. Каким же образом следует изображать пространственную фигуру на плоскости? Обычно в геометрии для этого используется параллельное проектирование. 
    Пусть p - некоторая плоскость, 
l - пересекающая ее прямая (рис. 1). Через произвольную точку A, не принадлежащую прямой l, проведем прямую, параллельную прямой l. Точка пересечения этой прямой с плоскостью p называется параллельной проекцией точки A на плоскость p в направлении прямой l. Обозначим ее A'. Если точка A принадлежит прямой l, то параллельной проекцией на плоскость p считается точка пересечения прямой l с плоскостью p.

hello_html_5f1d4cc7.gif

   Таким образом, каждой точке A пространства сопоставляется ее проекция A' на плоскость p. Это соответствие называется параллельным проектированием на плоскость pв направлении прямой l. 
    Пусть 
Ф - некоторая фигура в пространстве. Проекции ее точек на плоскость p образуют фигуру Ф', которая называется параллельной проекцией фигуры Ф на плоскостьp в направлении прямой l. Говорят также, что фигура Ф' получена из фигуры Ф параллельным проектированием. 
    Примеры параллельных проекций дают, например, тени предметов под воздействием пучка параллельных солнечных лучей. 
Метод параллельного проецирования заключается в том, что все проецирующие лучи, проходящие через точки, будут параллельны между собой. Этот метод вытекает из метода центрального проецирования, при этом полюс должен быть удален на бесконечно большое расстояние от плоскости, на которую проецируется предмет. Ортогональный метод проецирования – метод, когда проецирующие лучи параллельны между собой и перпендикулярны к плоскости проекций. Данный метод – частный случай параллельного проецирования. 

Основные свойства параллельного и ортогонального проектирования:

  1. Проекцией точки является точка.

  2. Проекцией прямой является прямая – свойство прямолинейности.

  3. Проекцией точки, лежащей на некоторой прямой, является точка, лежащая на проекции данной прямой – свойство принадлежности.

  4. Проекциями параллельных прямых являются параллельные прямые – свойство сохранения параллельности.

  5. Отношение проекций отрезков, лежащих на параллельных прямых или на одной и той же прямой, равно отношению самих отрезков.

  6. Проекция фигуры не меняется при параллельном переносе плоскости проекций.

  7. Проекция отрезков не может быть больше самого отрезка.

  8. Теорема о прямом угле. Если одна сторона прямого угла параллельны плоскости проекции, а вторая сторона этой плоскости не перпендикулярно, то прямой угол проекцируется на эту плоскость без изкажения.

Простейшим многоугольником является треугольник. Параллельной проекцией треугольника, как следует из свойств параллельного проектирования, является треугольник или отрезок. При этом, если плоскость треугольника параллельна плоскости проектирования, то, как мы выяснили, его проекцией будет треугольник, равный исходному.

Тогда ясно, что треугольник ABC является параллельной проекцией треугольника AB1C на плоскость p в направлении прямой l.

hello_html_5aa809f6.gif

    Рассмотрим теперь параллельную проекцию правильного шестиугольника ABCDEF с центром в точке O (рис. 7). Выберем какой-нибудь треугольник, например, AOB. Его проекцией может быть треугольник A'O'B' на плоскости p (рис. 8), имеющий произвольную форму. Далее отложим O'D'=A'O' и O'E'=B'O'. Теперь из точек A' и D' проведем прямые, параллельные прямой B'O'; из точек B' и E' проведем прямые, параллельные прямой A'O'. Точки пересечения соответствующих прямых обозначим F' и C'. Шестиугольник A'B'C'D'E'F' и будет искомой проекцией правильного шестиугольника ABCDEF. 

Выясним, какая фигура является параллельной проекцией окружности. Пусть F - окружность в пространстве, F'- ее проекция на плоскость p в направлении прямой l. Если прямая l параллельна плоскости окружности или лежит в ней, то проекцией окружности является отрезок, равный диаметру окружности.  Изображение параллелепипеда строится, исходя из того, что все его грани параллелограммы и, следовательно, изображаются параллелограммами (рис. 11).

hello_html_256e21bd.gif

    При изображении куба плоскость изображений обычно выбирается параллельной одной из его граней. В этом случае две грани куба, параллельные плоскости изображений (передняя и задняя), изображаются равными квадратами. Остальные грани куба изображаются параллелограммами (рис. 12). Аналогичным образом изображается прямоугольный параллелепипед (рис. 13). 
    Для того чтобы построить изображение призмы, достаточно построить многоугольник, изображающий ее основание. Затем из вершин многоугольника провести прямые, параллельные некоторой фиксированной прямой, и отложить на них равные отрезки. Соединяя концы этих отрезков, получим многоугольник, являющийся изображением второго основания призмы (рис. 14). 
    Для того чтобы построить изображение пирамиды, достаточно построить многоугольник, изображающий ее основание. Затем выбрать какую-нибудь точку, которая будет изображать вершину пирамиды, и соединить ее с вершинами многоугольника (рис. 15). Полученные отрезки будут изображать боковые ребра пирамиды.

hello_html_m476fd703.gifhello_html_mc0b771.jpg

    Для изображения цилиндра достаточно изобразить его основания в виде двух эллипсов, получающихся друг из друга параллельным переносом, и нарисовать две образующие, соединяющие соответствующие точки этих оснований

(рис. 16). 
    Для изображения конуса достаточно изобразить его основание в виде эллипса, отметить вершину и провести через нее две образующие, являющиеся касательными к этому эллипсу (рис. 17). 

Теорема. Площадь ортогональной проекции многоугольника на плоскость равна площади проектируемого многоугольника, умноженной на косинус угла между плоскостью многоугольника и плоскостью проекций.

hello_html_m3acf510f.jpg

hello_html_7e2a7158.gifhello_html_546111d.gif

hello_html_56b1906f.gif






hello_html_m226890bd.gifhello_html_m65c5f227.gif

hello_html_69020528.gif

hello_html_m5d411a1c.gif





hello_html_5800f7e5.gifhello_html_3986437c.gifhello_html_3986437c.gif




hello_html_2eec2f0e.gifhello_html_7ef82a8b.gif

2)Решить задачи :

1. В правильной четырёхугольной призме через диагональ основания под углом 30 к плоскости основания проведена плоскость, пересекающая боковое ребро.

Найти диагональ основания, если площадь сечения равна 8hello_html_2548fd2c.jpghello_html_246ecaab.jpg3 см2.


  1. В правильной четырёхугольной призме через середины двух смежных сторон основания, равных a, проведена плоскость, пересекающая три боковых ребра и наклонённая к плоскости основания под углом . Найти площадь сечения.


  1. Высота прямой призмы равна 1. В основании призмы лежит ромб со стороной, равной 2, и острым углом 30°. Через сторону основания проведена секущая плоскость с углом наклона к плоскости основания 60°. Найти площадь сечения призмы.


  1. В правильной четырёхугольной призме через середины двух смежных сторон основания, равных a, проведена плоскость, пересекающая три боковых ребра и наклонённая к плоскости основания под углом . Найти площадь сечения.


  1. Высота прямой призмы равна 1. В основании призмы лежит ромб со стороной, равной 2, и острым углом 30°. Через сторону основания проведена секущая плоскость с углом наклона к плоскости основания 60°. Найти площадь сечения призмы.


  1. В кубе ABCDA1B1C1D1 через точки A, D1 и середину ребра BB1 проведено сечение.


Найти угол наклона секущей плоскости к плоскости основания ABCD.


    1. Основанием пирамиды является прямоугольный треугольник, катеты которого равны 3 и 4. Каждая боковая грань пирамиды наклонена к плоскости основания под углом в 60°. Найти площадь полной поверхности пирамиды.


    1. Стороны основания треугольной пирамиды равны 5, 6 и 7. Вершина пирамиды проектируется в центр окружности, вписанной в её основание. Высота пирамиды равна радиусу этой вписанной окружности. Найти площадь поверхности пирамиды.

    2. Основанием пирамиды является ромб с диагоналями 6 и 8. Высота пирамиды равна 1.Найти площадь полной поверхности этой пирамиды, если все двугранные углы при её основании равны.

    1. В правильной усечённой треугольной пирамиде стороны нижнего и верхнего оснований равны соответственно a и b (a b ). Найти площадь полной поверхности усечённой пирамиды, если её боковые грани наклонены к плоскости основания под углом .

    2. В правильной усечённой шестиугольной пирамиде стороны нижнего и верхнего оснований равны соответственно a и b (a b ). Найти площадь полной поверхности усечённой пирамиды, если её боковые грани наклонены к плоскости основания под углом .

  1. Основанием пирамиды служит треугольник, стороны которого равны 12, 10 и 10. Каждая боковая грань наклонена к основанию под углом 45°. Найти площадь полной поверхности пирамиды.


  1. Боковые грани пирамиды наклонены к плоскости основания под углом 60°. В основании пирамиды лежит треугольник со сторонами 8, 9, 11. Найти площадь боковой поверхности пирамиды.



12. Ребро куба ABCDA1B1C1D1 равно a. На рёбрах AA1, BB1, DD1 соответственно взяты точки M, N, P так, что AM MA1, BN 2NB1, D1P 3PD . Найти площадь сечения куба плоскостью MNP.



13. Дан куб ABCDA1B1C1D1 с ребром, равным 1. Точка M середина ребра AD, точка


N центр грани CC1D1D. Найти площадь сечения куба плоскостью, проходящей через точки M, N и параллельной прямой BD.


14. Дана правильная шестиугольная призма ABCDEFA1B1C1D1E1F1 со стороной основания 2 и высотой 10. На боковом ребре CC1 взята точка M. Через точку M проведены две взаимно перпендикулярные плоскости, каждая из которых перпендикулярна плоскости CC1F . Одна из этих плоскостей проходит через точку F1, а другая через точку F. Найти площади сечений призмы проведёнными плоскостями.


  1. Дана правильная треугольная пирамида, площадь каждой боковой грани которой равна 1. Найти площадь сечения пирамиды плоскостью, проходящей через середину высоты пирамиды параллельно её боковой грани.


  1. Дана правильная шестиугольная пирамида, площадь каждой боковой грани которой равна 1. Найти площадь сечения пирамиды плоскостью, проходящей через середину высоты пирамиды параллельно её боковой грани.

3)Решить задачи :

hello_html_709dc4a7.gif



Просмотрено: 0%
Просмотрено: 0%
Скачать материал
Скачать материал "ПЗ по специальности Технология машиностроения,1 курс(2016-2017 уч.г.)"

Получите профессию

HR-менеджер

за 6 месяцев

Пройти курс

Рабочие листы
к вашим урокам

Скачать

Выбранный для просмотра документ ПЗ№40,41.docx

ПЗ № 40. Исследование функции с помощью производной.

Задание:

1)Перепишите и заполните пропуски:

Пример 1. Исследовать и построить график функции:

Решение:

  1. D (f) = R, т.к. f -многочлен.

  2. Выясняем, является ли функция f четной или нечетной. - функция ни четная, ни нечетная.

  3. Функция непериодическая.

  4. Находим точки пересечения графика с осями координат:

а) с осью ОХ: у=0 получаем точки (0;0), (3;0)

б) с осью ОУ: х=0 получаем точки (0;0)

  1. Найдем производную функции:

  2. Найдем критические точки: , т.е. ,х = … или х = ...

Отмечаем эти точки 0 и 2 на числовой прямой, и определяем знак производной в каждом промежутке. −  +  

6(−  1) −  3(−  1)2 = −  6 −  3 = −  9 < 0

0 2 х

Значит, в промежутках и функция убывает и (0;2) – функция возрастает.

х = 0 - точка минимума, т.к. производная меняет знак с минуса на плюс.

Вычислим уmin=

х = 2 – точка максимума, т.к. производная меняет знак с плюса на минус.

Вычислим уmax= .

7.Составляем таблицу для внесения всех данных

x


0

(0;2)

2



− 

0

+

2

− 

f(x)


0


4




min


max


8. Строим график функции.

hello_html_6ac426ef.png


Пример 2. Сколько корней имеет уравнение: x4   4x3   9 = 0?

Решение: р (x) = x4   4x3   9, D(р) = ( hello_html_8781bd9.gif; hello_html_8781bd9.gif).

р ' (x) = 4 x 3 12x 2 = 4 x 2 3) = 0, x1 = 0; 1 петля; x2 = …, р ' (4) = 4 hello_html_m61765ba1.gif16 hello_html_m61765ba1.gif1 > 0






р(x) убывает на интервале (hello_html_8781bd9.gif ; 3]; р (x) возрастает на [3; +hello_html_8781bd9.gif).

x = 3 – min, р min= р (3) = 34   4 hello_html_m61765ba1.gif 33   9 = 81 4hello_html_m61765ba1.gif27 – 9 = 81   117= − < 0, в точке x = 0 график имеет точки перегиба (то есть меняет выпуклость), f(0) = 0   0  9 = ...

Строим эскиз графика

hello_html_331651eb.png

График пересекает ось 0Х в двух точках x1 и x2, следовательно, многочлен, а значит и данное уравнение имеет два корня.

Ответ: два.

Пример 3. Исследовать функцию f(x)= 3x5 3 + 2 и построим ее график.

Решение: 1.D (f ) = R, так как f (x) - многочлен.

2.Функция f не является ни четной, ни нечетной, так как

f (− x) = 3(− x)5 5(− x)3 + 2 = − 3x 5 + 5х3 + 2= −  ( 3x5 − 5х3 − 2) hello_html_382283fb.gif f(x)

3.Найдем координаты точек пересечения графика с осями координат:

а) с осью 0Х, для этого решим уравнение: 3x5 3 + 2 = 0.

Методом подбора можно найти один из корней (x = 1). Другие корни могут быть найдены только приближенно. Поэтому для данной функции остальные точки пересечения графика с осью абсцисс и промежутки знакопостоянства находить не будем.

б) с осью 0У: f(0) = 3hello_html_m61765ba1.gif05  5hello_html_m61765ba1.gif03 + 2 = …

Точка А (0; 2) - точка пересечения графика функции с осью 0У.

Отметили, что промежутки знакопостоянства не будем находить.

4.Найдем промежутки возрастания и убывания функции : а ) f '(x)= 15x4  15х2 = 15х2 hello_html_m61765ba1.gif2   1)

D (f ') = R, поэтому критических точек которых f '(x)не существует, нет.

б) f '(x) = 0, если х2hello_html_m61765ba1.gif2 1)=0 <=> x1 = 0 ; 1 петля ; x2  1= 0, x2 =  1, х2 = …, х3 = …

в) Получим три критические точки, они разбивают координатную прямую на четыре промежутка. Определим знак производной на этих промежутках: f ' (4) = 15 hello_html_m61765ba1.gif16 hello_html_m61765ba1.gif15 > 0






Так как функция непрерывна в точках – 1; 0; 1, то f   возрастает на (– hello_html_8781bd9.gif; – 1] и [1; +hello_html_8781bd9.gif);

f  убывает на [– 1; 0] и [0; 1].

5.Найдем точки экстремума функции и вычислим значения функции в этих точках.

x = − 1 - точка max, f (− 1) = 3hello_html_m61765ba1.gif (− 1) 5  5hello_html_m61765ba1.gif (− 1) 3 + 2 = − 3+ 5 + 2 = 7 – 3 = … ;

x = 1 - точка min, f (1) = 3hello_html_m61765ba1.gif15  5hello_html_m61765ba1.gif13 + 2 = 3− 5 + 2 = 5 – 5 = ...

Полученные результаты занесем в таблицу и построим график .


x


1

(− 1;0)

0

(0;1)

1



+

0

0

0

+

f(x)


4


2


0




max



min



Пример 4. Исследовать и построить график функции:hello_html_m46464d67.png

Решение: проведем исследование функции:

  1. Функция определена и непрерывна на всей числовой

прямой, D (f ) = R .
,

значит, данная функция является четной, ее график симметричен относительно оси ординат.
Очевидно, что функция непериодическая.

  1. Точки пересечения графика с координатными осями, интервалы знакопостоянства функции.
    График функции проходит через начало координат.

 на всей области определения.

  1. Возрастание, убывание, экстремумы функции.



х = 0
 – критическая точка. Определим знаки :
hello_html_5a5cd5d3.jpg

 возрастает на  и убывает на.
В точке х= 0 функция достигает минимума: .

  1. Найдем дополнительные точки и выполним чертёж:


х

0,5

1,5

2

2,5

3

4

5

6


0,08

0,43

0,57

0,68

0,75

0,84

0,89

0,93


hello_html_m1f9f2eb8.jpg


Пример 5. Сколько корней имеет уравнение: ?

Решение: Рассмотрим функцию р(x) =

1) Найдем область определения функции D(р) = (−hello_html_8781bd9.gif; hello_html_8781bd9.gif).

2) Найдем производную р' (x) = x 3 − 3x 2 – x + 3.

3) Найдем критические точки и промежутки возрастания и убывания функции:

р' (x) = 0 <=> x 3 − 3x 2 – х + 3= 0 <=> x 2 hello_html_m61765ba1.gif (х − 3) − (х − 3) = 0 <=> (х − 3) hello_html_m61765ba1.gif ( x 2 − 1) = 0 <=>

х1=3, х2= 1, х3= − 1. Получим три критические точки, они разбивают координатную прямую на четыре промежутка. Определим знак производной на этих промежутках:

р ' (4) = 1 ·15 > 0

hello_html_47ff2293.png




р(x) возрастает на интервалах [1; 1] и [3; +hello_html_8781bd9.gif);

р(x) убывает на (hello_html_8781bd9.gif ; 1] и [1; 3].

4) Найдем точки экстремума и экстремумы функции:

х = − 1 min р min= 1/4 + 1 − 1/2 − 3= 0,25 + 1 – 0,5 – 3 = 1,25 – 3,5 = − … < 0,

x = 1 max р max= 1/4 − 1 − 1/2 + 3 = 0,25 – 1 – 0,5 + 3 = 2 – 0,25=… > 0,

х = 3 min р min = 81/4 − 27 − 9/2 + 9 = 20,25 – 27 – 4,5 + 9 = 29,25 – 31,5 =

= − … < 0.

Строим эскиз графика.

Из рисунка видно, что многочлен имеет 4 корня, следовательно, уравнение имеет 4 решения.

Ответ: уравнение имеет 4 решения.

2)Решить задание ( по примерам):

  1. Исследуйте функцию и постройте ее график.

  2. Сколько корней имеет уравнение: x4 − 4x3 + 9 = 0?

  3. Исследовать функцию f(x)= 3x5 3 + 6 и построим ее график.

  4. Исследовать и построить график функции:.

  5. Сколько корней имеет уравнение: ?

3)Решить задание:

  1. Исследуйте функцию y = 1/3x3 − 3x2 + 8x и постройте ее график.

  2. Сколько корней имеет уравнение: x2 − x3/3− 1= 0?

  3. Исследовать и построить график функции:

а) y = 3x4 − 4x3 − 12x2 + 10; б) y = в) f (х) = x4 − 2х2.

  1. Исследуйте функцию и постройте ее график: f (x) = x4 −2х2 −3.

  2. Найти число корней уравнения: 2x 3 − 3x 2 − 12х − 11= 0.

  3. Исследуйте функцию и постройте ее график: а)б)

  4. Сколько корней имеет уравнение: а)б) в)

  5. Определите промежутки монотонности и экстремумы функции 

  6. Докажите, что функция f(x) = 4x — 3 sin x возрастает на всей числовой прямой.

  7. Исследуйте функцию f(x) = x4 + 4x2  5 и постройте ее график.

  8. Определите промежутки монотонности и экстремумы функции 

  9. Докажите, что функция f(x) = 5 cosx 7x убывает на всей числовой прямой.

  10. Исследуйте функцию f(x) = x4 + 8x2 − 9 и постройте ее график.

  11. Определите промежутки монотонности и экстремумы функции  .

  12. Докажите, что уравнение х5 + 2х3 + 8x + cos 3x = 0 имеет ровно один корень.

  13. Дана функция  
    а) постройте график функции 
    f(х);
    б) сколько корней имеет уравнение 
    f(х) = а?

  14. Дана функция 
    а) постройте график функции 
    f(x); б) сколько корней имеет уравнение f(х) = а?

  15. При каком наибольшем значении параметра а функция  убывает на всей числовой прямой?

  16. Докажите, что уравнение х5 + 4х3 + 7x sin2x = 0 имеет ровно один корень.

  17. При каком наибольшем значении параметра а функция  возрастает на всей числовой прямой?

  18. При каких значениях а функция f(х) = 8ac – a sin 6 7 sin 5х возрастает на всей числовой оси и не имеет стационарных точек?

  19. Проведите исследование и постройте график функции 

  20. Исследуйте функцию f(x) = х3 – 3x2 + 2 и постройте ее график.

  21. Исследуйте функцию f(х) = х3 3х + 2 и постройте ее график.

  22. При каких значениях параметра а функция f(x) = a sin 7x + 8 ax + sin 4 5x убывает на всей числовой оси и не имеет стационарных точек?


ПЗ № 41. Нахождение наибольшего, наименьшего значения и экстремальных значений функции.

Задание:

1)Перепишите и заполните пропуски:

Пример 1. Найти точку максимума функции y= x3 3x2 24x 5.

Решение: Требуется найти критическую точку, в которой знак производной меняется с плюса на минус. Область определения функции: hello_html_4fb18fd6.gif Найдем критические точки функции:



- Критические точки.

Исследуем знак производной на интервалах, разделенных критическими точками:

х

- 4 2

max min

Ответ: x = ...

Пример 2. Найдите точки экстремума функции и определите их характер y= x 4 8x2.

Решение: y = x 4 8x2 , D(y) = R , y = (x 4 8x2) = 4x 3 – 16x, y = 0,

4x 3 – 16x = 0, 4x(x2 4) = 0, 4x(x2) (x2) = 0,

x1= 0 или х2=0 или х2=0

х2 = … х3 =…

х1= 0, х2 = 2, х3 = 2 – это стационарные точки.


2 0 2 х

Функция убывает на ( ;2, на 0; 2. Функция возрастает на 2; 0, на 2; +).

х3 = …, х2 = … – это точки минимума. х1= … – это точка максимума.

Ответ: х3 = 2, х2 = 2– это точки минимума, х1= 0 – это точка максимума.

Пример 3. Найдите точки экстремума функции и определите их характер: y= x2 6x1.

Решение: y = x2 6x1, D(y) =R,

y  = ( x3 x2 6x1)  = x25x6 = (х3)(х2) , y = 0, x 2 5x6 = 0,

x1 = 3, x2 = 2 – это стационарные точки.

х

2 3

Функция возрастает на ( ; 2, на 3; +).Функция убывает на 2; 3.

x2 = … – это точка максимума, х1 = ... – это точка минимума.

Ответ: х2 = 2 – это точка максимума, х1 = 3 – это точка минимума.

Пример 4. Найдите точки экстремума функции и определите их характер.

y= 2x5 5x4 10x3 3.

Решение: y = 2x5 5x4 10x3 3, D(y) = R, y  = (2x5 5x4 10x3 3)  = 25x4 54x3 103x2 = =…x4 …x3 …x2 = 10х2 (х1)(х3), y = 0 , 10x4 20x3 30x2 = 0, 10x2 (x2 + 2x 3) = 0,

x 2 = 0 или х2 2х3=0, х1 = 0, х2 = 1, х3 = 3 – это стационарные точки.


3 0 1 х

Функция возрастает на ( ; 3, на 1; ). Функция убывает на 3; 1.

х3 = – это точка максимума. х2 = … – это точка минимума.

Ответ: х3 = 3 – это точка максимума, х2 = 1 – это точка минимума.

Пример 5. Найти наибольшее и наименьшее значения функции у(x) = 2x3 12x2 18x  3  на отрезке [– 1;2] .

Решение: 1) Вычислим значения функции в критических точках, принадлежащих данному отрезку:

 

Полученное квадратное уравнение имеет два действительных корня:

х1= 1, х2 = 3 – критические точки. Первая критическая точка принадлежит данному

отрезку: х1= 1 . А вот вторая – нет: х2= 1 , поэтому про неё сразу забываем.

Вычислим значение функции в нужной точке:

2)Вычислим значения функции  на концах отрезка:


3) Дело сделано, среди чисел выбираем наибольшее и наименьшее.

Ответ:  

Пример 6. Число 24 представьте в виде суммы двух неотрицательных слагаемых так, чтобы сумма квадратов этих чисел была наименьшей.

Решение: Пусть х – первое слагаемое, тогда (24-х) – второе слагаемое. Сумма квадратов этих чисел По условию задачи Рассмотрим функцию Она на интервале (0;24) непрерывна и дифференцируема. Найдем критические точки.

Это значение единственное, поэтому первое число – 12, второе – 12. Ответ: 24=12+12.
Пример 7. Найдите размеры участка прямоугольной формы, имеющего наибольшую площадь, если его периметр равен 200 м.

Решение: A B

x

C D Так как функция S(x) непрерывная на всей числовой прямой, b

то будем искать ее наибольшее значение на отрезке .

Значит, наибольшей будет площадь участка 2500 м2, а стороны участка равны 50 м и 50 м.

Ответ: 50 м и 50 м.

2)Решить задание ( по примерам):

  1. Найти точку максимума функции y = x3 6x2 15x 3.

  2. Найдите точки экстремума функции а) y = x 4 2x2 , б) y = x2 4x3 ,
    в) y = 2x
    5 10x4 40x3 5 и определите их характер.

  3. Найти наибольшее и наименьшее значения функции f(x) = x3 3x2 – 72x  90 на отрезке [– 4;5] .

  4. Число 12 представьте в виде суммы двух неотрицательных слагаемых так, чтобы сумма квадратов этих чисел была наименьшей.

  5. Найдите размеры участка прямоугольной формы, имеющего наибольшую площадь, если его периметр равен 120 м.

3)Решить задание : 1.Найдите точки экстремума функции

2. Найти наибольшее и наименьшее значения функции f(x) = x4 4x2  8 на отрезке [– 1;2] .

  1. Найти максимальное и минимальное значения функции f(x) = на отрезке [– 8;0] .

  2. Найти наибольшее и наименьшее значения функции f(x) = 2x312x2 – 30x 9 на отрезке [– 4;2] .

  3. Найти наибольшее и наименьшее значение функции  на отрезке [1;4];

  4. Найдите наибольшее значение функции   на отрезке .

  5. а) Найдите наибольшее и наименьшее значения функции f(x) = x33x2  1 на отрезке [– 4;-1/3] .

б )Площадь Х равна 25 см2. Найдите наименьший возможный периметр этого прямоугольника.


Просмотрено: 0%
Просмотрено: 0%
Скачать материал
Скачать материал "ПЗ по специальности Технология машиностроения,1 курс(2016-2017 уч.г.)"

Получите профессию

Менеджер по туризму

за 6 месяцев

Пройти курс

Рабочие листы
к вашим урокам

Скачать

Выбранный для просмотра документ ПЗ№42,43.docx

ПЗ № 42. Интеграл и первообразная. Теорема Ньютона—Лейбница.

Задание:

1)Перепишите и заполните пропуски:

Пример 1. Найти общий вид первообразных данных функций :

1) f(x) = x2  cosx;   2) f(x) = 3; 3) f(x) = 10 sinx;  4) f(x) = 2sin 4x; 5) f(x) = 5x4x2  ; 6) f(x) = (3x – 1)2; 7) f(x) =. .

Решение: 1) F(x) = x3/3 – sinx  C; 2) F(x) =3x  C; 3) F(x) = 10  C; 

4) F(x) = 1/2 cos…x + C;5) F(x) =   x3/3 – 2;  6) F(x) = (3x – 1)3/… + C; 7) F(x) = /… + C.

Ответ: 1) F(x) = x3/3 – sinx  C; 2) F(x) =3x  C; 3) F(x) = 10cosx  C; 

4) F(x) = 1/2 cos 4x + C;5) F(x) = x5  x3/3 – 2;  6) F(x) = (3x – 1)3/9 + C; 7) F(x) =  /3 + C.

Пример 2. Для функции  f(x) = 4x + 1/x2 найти первообразную, график которой проходит через точку M(– 1; 4).

Решение: F(x) = 2x2– 1/x + C. , F(x) = 2()2– 1/() + C = 2 C = 4, 4 = 3 + C, C = …

Ответ: F(x) = 2х2 – 1/х + 1.

Пример 3. Докажите , что функция F(x) является первообразной для функции f(x).

a) f(x) = 2x; F(x) = x2 , б) f(x) = – sin x; F(x) = сos x ,
в) f(x) = 6x2 + 4; F(x) = 2x3 + 4x, г) f(x) = 1/cos2 x; F(x) = tg x .

Решение: a) f(x) = 2x; F(x) = x2 , F (x)= (x2) = … = f(x);

б) f(x) = – sin x; F(x) = сos x , F (x)= (cos x) = – … = f(x);

в) f(x) = 6x2 4; F(x) = 2x3 4x, F (x)= (2x3 4x) = …x2 4 = f(x);

г) f(x) = 1/cos2 x; F(x) = tg x , F (x)= (tg x) = 1/cos2 x= f(x).

Ответ: F(x) является первообразной для f(x). Пример 4. Найдите первообразные функций: a) f(x) = x4 x2 x ; б) g(u) = ;

в) h(x) = (x3 + 1)2 ; г) v(x) = cos (5x ). Решение: Для нахождения первообразных функций воспользуемся таблицей первообразных. а) x5/5 - одна из первообразных функции х4; x3/3 - одна из первообразных функции х2; x2/2 - одна из первообразных функции х; х - одна из первообразных функции 1. По правилу 1 нахождения первообразных F(x) =   - первообразная функции f(х);

б) функцию g(u) запишем в виде g(u) =  u - 1/3 u3/2.

3/2u2/3 - одна из первообразных функции u-1/3; 2/5u5/2 - одна из первообразных функции u3/2; 

G(x) = 3/2u2/3  2/5u5/2  - первообразная функции g(u);

в) h(x) = (x3 + 1)2 = x6 2x3 .

x7/7 - одна из первообразных функции х6; x4/4 - одна из первообразных функции х3; х - одна из первообразных функции 1.

По правилам 1 и 2 нахождения первообразных H(x) = …  1/2x4C - первообразная функции h(х);

г) v(x) = cos (5x ) , sinu - одна из первообразных функции cosu; V(x) = … sin(5x)  C - первообразная функции v(х).

Ответ: a) F(x) =   ; б) G(x) = 3/2u2/3  2/5u5/2  в) H(x) = 1/7x7  1/2x4xC; г) V(x) = 1/5 sin(5x)  C.

Пример 5. Найдите первообразную: a) g(x) = (47x)5 ; б) g(x)= x 2; в) t(x) = (5+ 2x)3.

Решение: a) G (x) = ; б) G(x) = ; в) T(x) = .


Ответ: a) G (x) = ; б) G(x) = ; в) T(x) = .

Пример 6. Вычислить интеграл: а) ,

б) ,

в)

г) д) е)

ж)

2)Решить задание ( по примерам):

  1. Найти общий вид первообразных данных функций :

1) f(x) = 4x3  2cosx;   2) f(x) = 5; 3) f(x) = 5 sinx;  4) f(x) = 3sin 4x;
5)
 f(x) = 6x5x4  ; 6) f(x) = (4x – 1)2; 7) f(x) =. .

  1. Для функции  f(x) = 6x + 1/x2 найти первообразную, график которой проходит через точку M(– 1; 8).

  2. Докажите , что функция F(x) является первообразной для функции f(x).

а)f(x) = 4x; F(x) = 2x2 , б) f(x) = – 2sin x; F(x) = 2сos x ,

в) f(x) = 6x2 7; F(x) = 2x3 7x, г) f(x) = 1/sin2 x; F(x) = ctg x .

  1. Найдите первообразные функций:
    a) f(x) = 10x4 4x2 2x ; б) g(u) = ; в) h(x) = (x3 3)2 ; г) v(x) = cos (7x ).

  2. Найдите первообразную: a) g(x)= (45x)5 ; б) g(x)= 2x- 2; в) t(x) = (54x)5.

3)Решить задание:

  1. Найдите первообразную функции f(x) = 4x3– 3x2 , график которой проходит через

точку M(–1; 2).

  1. Для функции f(х) = еx найти первообразную, график которой проходит через точку М(0; 2).

  2. Найти общий вид первообразной для функции f:

а)f(x) = 1,б) f(x) = х+1,в) f(x) = х9 + 3, г) f(x) =3х32 ,д) f(x) = 5х + ,

е) f(x) = 1+ , ж) f(x) = 4 + 2cosx, з) f(x) =sin2x + x, и) f(x) = sinx + cosx.

  1. При каких значениях k и С функции kcosx + x - 4 является первообразной функции 3sinх + 1? 

  2. При каком значении k и С функция   является первообразной функции  ?

  3. Найдите одну из первообразных функции f:

а) f(x) = 2х5 ,б) f(x) = 3х3+2х1, в) f(x) = 3 cosx 4 sinx ,г) f(x) = (х+1)4,

  1. Докажите, что функция F является первообразной для функции f на множестве R:

а) F(x) = 4хх3 , f(x) = 4 2, б) F(x) = 0,5 sinx, f(x) = cosx.

  1. Найдите первообразные функций: а) f(x) =,б)f(x) = 7 sinx, в) f(x) =г)f(x) =12cosx.

  2. При каком значении к и С функция является первообразной функции 5cosх + 2х?

  3. Вычислить интеграл: а) , б) ,в) , г) , д) , е) , ж)

ПЗ № 43. Применение интеграла к вычислению физических величин и площадей.

Задание:

1)Опорный конспект.

1. ВЫЧИСЛЕНИЕ ПУТИ, ПРОЙДЕННОГО ТОЧКОЙ

Путь, пройденный точкой при неравномерном движении по прямой с переменной скоростью hello_html_m14eaee5.gifза промежуток времени от hello_html_m52f1cc8d.gif до hello_html_m19189cb0.gif вычисляется по формуле hello_html_m12db380c.gif.

Примеры:

1. Скорость движения точки hello_html_dc36de3.gif м/с. Найти путь, пройденный точкой за 4-ю секунду.

Решение: согласно условию, hello_html_65824c05.gif. Следовательно, hello_html_4274c13b.gif

2. Два тела начали двигаться одновременно из одной точки в одном направлении по прямой. Первое тело движется со скоростью hello_html_m7ae6071a.gif м/с, второе — со скоростью v = (4t+5) м/с. На каком расстоянии друг от друга они окажутся через 5 с?

Решение: очевидно, что искомая величина есть разность расстояний, пройденных первым и вторым телом за 5 с:

hello_html_m41c22c75.gif

3. Тело брошено с поверхности земли вертикально вверх со скоростью и = (39,2—9,8^) м/с. Найти наибольшую высоту подъема тела.

Решение: тело достигнет наибольшей высоты подъема в такой момент времени t, когда v = 0, т.е. 39,29,8t = 0, откуда I 4 с. По формуле (1) на ходим

hello_html_129eae47.gif

2. ВЫЧИСЛЕНИЕ РАБОТЫ СИЛЫ

Работа, произведенная переменной силой f(х) при перемещении по оси Ох материальной точки от х = адо х=b, находится по формуле hello_html_95bb59.gifПри решении задач на вычисление работы силы часто используется закон Г у к а: F=kx, (3) где F — сила Н; х—абсолютное удлинение пружины, м, вызванное силой F, а k —коэффициент пропорциональности, Н/м.

Пример:

1. Пружина в спокойном состоянии имеет длину 0,2 м. Сила в 50 Н растягивает пружину на 0,01 м. Какую работу надо совершить, чтобы растянуть ее от 0,22 до 0,32 м?

Решение: используя равенство (3), имеем 50=0,01k, т. е. kК = 5000 Н/м. Находим пределы интегрирования: а = 0,22 — 0,2 = 0,02 (м), b=0,32— 0,2 = 0,12(м). Теперь по формуле (2) получим

hello_html_m396fccd3.gif

3. ВЫЧИСЛЕНИЕ РАБОТЫ, ПРОИЗВОДИМОЙ ПРИ ПОДНЯТИИ ГРУЗА

Задача. Цилиндрическая цистерна с радиусом основания 0,5 м и высотой 2 м заполнена водой. Вычислить работу, которую необходимо произвести, чтобы выкачать воду из цистерны.

Решение: выделим на глубине х горизонтальный слой высотой dх (рис.). Работа А, которую надо произвести, чтобы поднять слой воды весом Р на высоту х, равна Рх.

Изменение глубины х на малую величину dх вызовет изменение объема V на величину dV пr2 dх и изменение веса Р на величину * dР = 9807 r2 dх; при этом совершаемая работа А изменится на величину dА=9807пr2 хdх. Проинтегрировав это равенство при изменении x от 0 до Н, получим

hello_html_78e3df37.gif

4. ВЫЧИСЛЕНИЕ СИЛЫ ДАВЛЕНИЯ ЖИДКОСТИ

Значение силы Р давления жидкости на горизонтальную площадку зависит от глубины погружения х этой площадки, т. е. от расстояния площадки до поверхности жидкости.

Сила давления (Н) на горизонтальную площадку вычисляется по формуле Р =9807 hello_html_m358815e3.gifS x,

где hello_html_m358815e3.gif — плотность жидкости, кг/м3; S — площадь площадки, м2х - глубина погружения

площадки, м.

Если площадка, испытывающая давление жидкости, не горизонтальна, то давление на нее различно на разных глубинах, следовательно, сила давления на площадку есть функция глубины ее погружения Р (х).


Физические приложения интеграла

1. Реши задачи.

Вычислите массу участка стержня от hello_html_m2acd972a.gif, если его линейная плотность задается формулой hello_html_m51532c9b.gif
  1. Вычислите работу за промежуток времени [4;9 ], если мощность вычисляется по формуле hello_html_630bc344.gif.

  1. Вычислите количество электричества, протекшего по проводнику за промежуток времени [ 2;3 ], если сила тока задается формулой hello_html_m4b9cde72.gif

  1. Вычислите работу по переносу единичной массы, совершенную силойhello_html_2919d23.gif [ -1;2].


2)Перепишите и заполните пропуски:


Пример 1. а)Вычислить площадь фигуры, ограниченной линиями у = х2 2, у = 0, х = 2, х = 1.

Решение: Выполним чертеж (обратите внимание, что уравнение у = 0  задает ось  ОХ): Штриховать криволинейную трапецию я не буду, здесь очевидно, о какой площади идет речь. Решение продолжается так:hello_html_m6ea081fc.png

На отрезке[– 2;1]    график функции у = х2 2  расположен над осью ОХ, поэтому:


Ответ: S = 9 eд2.


б)Вычислить площадь фигуры, ограниченной линиями у =  , х = 1  и координатными осями.

Решение: Выполним чертеж: Если криволинейная трапеция расположена под осью OX (или, по крайней мере, не выше данной оси), то её площадь можно найти по формуле: S =  .
В данном случае:
hello_html_m521974c9.png

Ответ: 

Пример 2.а)Найти площадь плоской фигуры, ограниченной линиями  у = 2х , у =  .hello_html_m34e0aa0e.png

Решение: Сначала нужно выполнить чертеж. Найдем точки пересечения параболы у = 2х   и прямой у =   .

Решаем уравнение: =  , 3х = 0, х(3) = 0,

х1 = …, х2 = ...

Значит, нижний предел интегрирования а = 0, верхний предел интегрирования b = 3 .

x = a ,x = b , можно найти по формуле: S = .

Искомая фигура ограничена параболой y = 2х   сверху и прямой у =    снизу.
На отрезке
[0;3]  2х  , по соответствующей формуле

Ответ: S = 4,5 eд2.  . б)Вычислить площадь фигуры, ограниченной линиями , y = x  , y = 0  , x = 3 .

Решение: Сначала выполним чертеж: Площадь фигуры считается с помощью двух определенных интегралов. Действительно:hello_html_m2509795.jpg

1) На отрезке [– 1;1]  над осью OX расположен график прямой y = x   ;

2) На отрезке [1;3]   над осью OX  расположен график гиперболы

Совершенно очевидно, что площади можно (и нужно) приплюсовать, поэтому:

Ответ: .hello_html_dfee4aa.png

Пример 3.a) Вычислить площадь фигуры, ограниченной линиями

 ,2x  .

Решение: Представим уравнения в виде и выполним поточечный чертеж:
Из чертежа видно, что верхний предел у нас «хороший»:
  b = 1.
Найдем точки пересечения прямой
    и параболы
Для этого решаем уравнение:
3x2 = 2x 3x2 2x

D = 4 12 = …, = 4, x1 = , x2 = ... Действительно,a = .

На отрезке по соответствующей формуле: Ответ: .hello_html_3f5273e5.png

б)Найти площадь фигуры, ограниченной линиями y =  , y = 2x  .

Решение: Выполним чертеж:
На отрезке по соответствующей формуле:


Ответ: S = 10 eд2.  .


Пример 4.Найти площадь фигуры, ограниченной линиями x , xy = 3 .hello_html_m2e7c3126.png

Решение: Выполним чертеж . На отрезке  , по соответствующей формуле:
Ответ:  .
hello_html_78b890fb.jpg


Пример 5.a) Найти площадь фигуры, ограниченной параболой у = х2 +10 и касательными к этой параболе, проведёнными из точки (0;1). 

Решение: Неизвестна абсцисса точки касания х = а. Чтобы её найти, составим уравнение касательной:  y = f (x0) .

Имеем f(x) = x2 f (x) = 2x;значит, f(a) = a2 f (a) = 2a; уравнение касательной имеет вид:

y = a2 2 a(x ) = a2 2 ax ;

Уравнение касательной y = (1)

По условию касательная должна проходить через точку (0;1), то есть координаты точки (0;1) должны удовлетворять уравнению (1):

1 = 2a0 ; , a1 = a2 = ...

Подставим найденные значения в уравнение (1):


Если a =  то y = 9 10 Если a = 3 , то y =  .

Получили два уравнения касательных y =  . Параболы y = х2 + 10 они касаются в точках А(3;19) и В(3;19).

Найдём площадь фигуры DACB: SDACB = 2SDCB ,


hello_html_m157678ba.gif






SDACB = 2 9 = ...

Ответ: 18.

б) Вычислить площадь фигуры, ограниченной линиями hello_html_35881468.jpg

у = 4/x, y = х, х = 4.

Решение: SABC = SMBAD SMBCD;

SMBAD = 1/2(MB ) MD = = 1/2 (2 ) 2 = 6;




hello_html_450e27c6.gif


Ответ: 6 – 4ln2.



2)Решить задание ( по примерам):

  1. а)Вычислить площадь фигуры, ограниченной линиями .

б)Вычислить площадь фигуры, ограниченной линиями и координатными осями.

  1. а)Найти площадь плоской фигуры, ограниченной линиями  .

б)Вычислить площадь фигуры, ограниченной линиями.

  1. a) Вычислить площадь фигуры, ограниченной линиями .

б)Найти площадь фигуры, ограниченной линиями y =  , y = 2x  .

  1. a)Найти площадь фигуры, ограниченной линиями  .

б) В каком отношении парабола делит площадь четырёхугольника, вершины которого находятся в точках с координатами (0;0); (2;0); (0;6); (2;6)? 

  1. a)Найти площадь фигуры, ограниченной параболой и касательными к этой параболе, проведёнными из точки (0;1). 

б) Вычислить площадь фигуры, ограниченной линиями .

3)Решить задание:

  1. a)Вычислить площадь фигуры, ограниченной линиями .

б)Вычислить площадь фигуры, ограниченной линиями и координатными осями.

  1. а)Найти площадь плоской фигуры, ограниченной линиями  .

б)Вычислить площадь фигуры, ограниченной линиями.

  1. a)Вычислить площадь фигуры, ограниченной линиями.

б)Найти площадь фигуры, ограниченной линиями .

  1. a)Найти площадь фигуры, ограниченной линиями.

б) В каком отношении парабола делит площадь четырёхугольника, вершины которого находятся в точках с координатами (0;0); (2;0); (0;6); (2;6)? 

  1. a)Найти площадь фигуры, ограниченной параболой и касательными к этой параболе, проведёнными из точки (0;1). 

б) Вычислить площадь фигуры, ограниченной линиями .

  1. Найти площадь фигуры, ограниченной функцией и осями координат.

  2. Найти площадь фигуры, ограниченной функциями и касательной к этой параболе, проведенной в точке (1/2;3/4).

  3. Найти площадь фигуры, ограниченной линиями.

  4. Найти площадь фигуры, ограниченной линиями.

  5. Найти площадь фигуры, ограниченной линиями.

  6. Найти площадь фигуры, ограниченной линиями.

  7. Найти площадь фигуры, ограниченной линиями..

  8. Найти площадь фигуры, ограниченной линиями .

  9. Найти площадь фигуры, ограниченной линиями.

  10. Найти площадь фигуры, ограниченную линиями .

  11. Найти площадь фигуры, ограниченной линиям .

  12. Вычислите площадь фигуры, ограниченной параболой и осью абсцисс.

  13. Вычислите площадь фигуры, ограниченной прямыми , .

  14. Вычислите площадь фигуры, ограниченной прямой и параболой .

  15. Вычислите площадь фигуры, ограниченной линиями и .























Просмотрено: 0%
Просмотрено: 0%
Скачать материал
Скачать материал "ПЗ по специальности Технология машиностроения,1 курс(2016-2017 уч.г.)"

Получите профессию

Методист-разработчик онлайн-курсов

за 6 месяцев

Пройти курс

Рабочие листы
к вашим урокам

Скачать

Выбранный для просмотра документ ПЗ№5.docx

ПЗ № 5. Различные виды многогранников. Их изображения. Сечения, развертки многогранников. Площадь поверхности.

Задание:

1) Опорный конспект.

Многогранником называют геометрическое тело, поверхность которого состоит из конечного числа многоугольников. Каждый из этих многоугольников называется гранью многогранника, стороны и вершины этих многоугольников — соответственно ребрами и вершинами многогранника.

Среди многогранников различают призмы и пирамиды.

Призма это многогранник, поверхность которого состоит из двух равных многоугольников и параллелограммов, имеющих общие стороны с каждым из оснований.

Два равных многоугольника называются основаниями призмы, а параллелограммы — ее боковыми гранями. Боковые грани образуют боковую поверхность призмы. Ребра, не лежащие в основаниях, называются боковыми ребрами призмы.hello_html_6951885f.jpg

Призму называют п-угольной, если ее основаниями

являются п -угольники. На рис. 24.6 изображена четырехугольная призма АВСDА'В'С'D'.

Призму называют прямой, если ее боковыми гранями являются прямоугольники (рис. 24.7).

Призму называют правильной, если она прямая, а ее основания — правильные многоугольники.

Четырехугольную призму называют параллелепипедом, если ее основания — параллелограммы.

Параллелепипед называют прямоугольным, если все его грани — прямоугольники.

Диагональ параллелепипеда — это отрезок, соединяющий его противоположные вершины. У параллелепипеда четыре диагонали.

Доказано, что диагонали параллелепипеда пересекаются в одной точке и делятся этой точкой пополам. Диагонали прямоугольного параллелепипеда равны.

Пирамида — это многогранник, поверхность которого состоит из многоугольника — основания пирамиды, и треугольников, имеющих общую вершину, называемых боковыми гранями пирамиды. Общая вершина этих треугольников называется вершиной пирамиды, ребра, выходящие из вершины, — боковыми ребрами пирамиды.

Перпендикуляр, опущенный из вершины пирамиды на основание, а также длина этого перпендикуляра называется высотой пирамиды.hello_html_534c4034.png


Простейшая пирамида — треугольная или тетраэдр (рис. 24.8). Особенность треугольной пирамиды состоит в том, что любую грань можно рассматривать как основание.

Пирамиду называют правильной, если в основании ее лежит правильный многоугольник, а все боковые ребра равны между собой.

Заметим, что следует различать правильный тетраэдр (т.е. тетраэдр, у которого все ребра равны между собой) и правильную треугольную пирамиду (в ее основании лежит правильный треугольник, а боковые ребра равны между собой, но их длина может отличаться от длины стороны треугольника, который является основанием призмы).

Различают выпуклые и невыпуклые многогранники. Определить выпуклый многогранник можно, если воспользоваться понятием выпуклого геометрического тела: многогранник называют выпуклым. если он является выпуклой фигурой, т.е. вместе с любыми двумя своими точками целиком содержит и соединяющий их отрезок.

Можно определить выпуклый многогранник иначе: многогранник называют выпуклым, если он полностью лежит по одну сторону от каждого из ограничивающих его многоугольников.

Все многогранники, которые до сих пор рассматривались, были выпуклыми (куб, параллелепипед, призма, пирамида и др.). Многогранник, изображенный на рис. 24.9, выпуклым не является.hello_html_6ab29a4f.png

Рассмотрим несколько выпуклых многогранников (таблица 24.1)

hello_html_39176722.jpg

Из этой таблицы следует, что для всех рассмотренных выпуклых многогранников имеет место равенство В - Р + Г= 2. Оказалось, что оно справедливо и для любого выпуклого многогранника. Впервые это свойство было доказано Л. Эйлером и получило название теоремы Эйлера.

Выпуклый многогранник называют правильным, если его гранями являются равные правильные многоугольники и в каждой вершине сходится одинаковое число граней.

Используя свойство выпуклого многогранного угла, можно доказать, что различных видов правильных многогранников существует не более пяти.hello_html_1c57ba51.png

Действительно, если грани многогранника — правильные треугольники, то в одной вершине их может сходиться

3, 4 и 5, так как 60" • 3 < 360°, 60° • 4 < 360°,

60° • 5 < 360°, но 60° • 6 = 360°.

Если в каждой вершине многогранника сходится три правильных треугольника, то получаем правильный тетраэдр, что в переводе с греческого означает «четырехгранник» (рис.).

Если в каждой вершине многогранника сходится четыре правильных треугольника, то получаем октаэдр (рис.). Его поверхность состоит из восьми правильных треугольников.

Если в каждой вершине многогранника сходится пять правильных треугольников, то получаем икосаэдр (рис.). Его поверхность состоит из двадцати правильных треугольников.

Если грани многогранника — квадраты, то в одной вершине их может сходиться только три, так как 90° • 3 < 360°, но 90° • 4 = 360°. Этому условию удовлетворяет только куб. Куб имеет шесть граней и поэтому называется также гексаэдром (рис.).

Если грани многогранника — правильные пятиугольники, то в одной вершине их может сходиться только три, так как 108° • 3 < 360°, пятиугольники и в каждой вершине сходится три грани, называется додекаэдром (рис.). Его поверхность состоит из двенадцати правильных пятиугольников.

Шестиугольными и более грани многогранника не могут быть, так как даже для шестиугольника 120° • 3 = 360°. Чтобы изготовить модель многогранника, нужно сделать его развертку (точнее развертку его поверхности).

Развертка многогранника — это фигура на плоскости, которая получается, если поверхность многогранника разрезать по некоторым ребрам и развернуть ее так, чтобы все многоугольники, входящие в эту поверхность, лежали в одной плоскости.

Сечения:

hello_html_27c25e08.png



Формулы:

hello_html_m4d5320ff.png


hello_html_e83952e.png


2) Задание:

1. Заполнить таблицу (где пустое место):

Теорема Эйлера. Число граней + число вершин - число ребер = 2.








2. Определите количество граней, вершин и рёбер многогранника, изображённого на рисунке. Проверьте выполнимость формулы Эйлера для данного многогранника.

3. Построить развертку многогранника на альбомном листе (или картоне)и склеить.

а) куб и б) тетраэдр

hello_html_45252253.jpg

в) октаэдр

hello_html_m5eba50d7.jpg

г) икосаэдр

hello_html_679d2806.gif


д) додекаэдр

hello_html_m4e1327af.gif


4. Построить развертку многогранника на альбомном листе (или картоне)и склеить.

hello_html_m36eb1203.gif



Просмотрено: 0%
Просмотрено: 0%
Скачать материал
Скачать материал "ПЗ по специальности Технология машиностроения,1 курс(2016-2017 уч.г.)"

Получите профессию

Няня

за 6 месяцев

Пройти курс

Рабочие листы
к вашим урокам

Скачать

Выбранный для просмотра документ ПЗ№6,7.docx

ПЗ № 6. Виды симметрий в пространстве. Симметрия тел вращения и многогранников.

Задание:

1) Опорный конспект.

Симметрия в широком смысле — соответствие, неизменность, проявляемые при каких-либо изменениях, преобразованиях (например: положения, энергии, информации, другого). Так, например, сферическая симметрия тела означает, что вид тела не изменится, если его вращать в пространстве на произвольные углы (сохраняя одну точку на месте). Двусторонняя симметрия означает, что правая и левая сторона относительно какой-либо плоскости выглядят одинаково.

Отсутствие или нарушение симметрии называется асимметрией или аритмией.

Общие симметрийные свойства описываются с помощью теории групп.

Симметрии могут быть точными или приближёнными.

Симметрия в геометрии.

Геометрическая симметрия — это наиболее известный тип симметрии для многих людей. Геометрический объект называется симметричным, если после того как он был преобразован геометрически, он сохраняет некоторые исходные свойства. Например, круг повёрнутый вокруг своего центра будет иметь ту же форму и размер, что и исходный круг. Поэтому круг называется симметричным относительно вращения (имеет осевую симметрию). Виды симметрий, возможных для геометрического объекта, зависят от множества доступных геометрических преобразований и того, какие свойства объекта должны оставаться неизменными после преобразования.

Виды геометрических симметрий:

Зеркальная симметрия

Осевая симметрия

Вращательная симметрия

Центральная симметрия

Скользящая симметрия

Зеркальная симметрия.

Зеркальная симметрия или отражение — движение евклидова пространства, множество неподвижных точек которого является гиперплоскостью (в случае трехмерного пространства — просто плоскостью). Термин зеркальная симметрия употребляется также для описания соответствующего типа симметрии объекта, то есть, когда объект при операции отражения переходит в себя. Это математическое понятие в оптике описывает соотношение объектов и их (мнимых) изображений при отражении в плоском зеркале. Проявляется во многих законах природы (в кристаллографии, химии, физике, биологии и т. д., а также в искусстве и искусствоведении).

Осевая симметрия.

Фигура называется симметричной относительно прямой А, если для каждой точки фигуры симметричная ей точка относительно прямой А также принадлежит этой фигуре.

Центральная симметрия.

Центральной симметрией (иногда центральной инверсией) относительно точки A называют преобразование пространства, переводящее точку X в такую точку X′, что A — середина отрезка XX′. Фигура называется симметричной относительно точки A, если для каждой точки фигуры симметричная ей точка относительно точки A также принадлежит этой фигуре. Точка A называется центром симметрии фигуры. Говорят также, что фигура обладает центральной симметрией. Другие названия этого преобразования — симметрия с центром A. Центральная симметрия в планиметрии является частным случаем поворота, точнее, является поворотом на 180 градусов.

Два треугольника с точечной симметрией отражения в плоскости. Треугольник А’В’С может быть получен из треугольника ABC поворотом на 180 ° вокруг точки O.


hello_html_b0d629b.png



Виды симметрии:


hello_html_ea8bead.jpg


Симметрия тел вращения:

hello_html_464dc77b.jpghello_html_m47b0b663.pnghello_html_mdf3f01e.jpghello_html_7a8edcf0.jpg

2) Примеры:

Пример 1. а)Может ли фигура иметь более одного центра симметрии?

Ответ: Да, например, прямая, плоскость и т.д. имеют бесконечно много центров симметрии.

б) Приведите примеры центрально-симметричных фигур.
Ответ: Центрально-симметричные: куб, прямоугольный параллелепипед, шар и др.;

Пример 2.а) Построить точки А1 и А2 симметричные относительно точки О,

б) Построить треугольники АВС и А1В1С1 симметричные относительно точки О,
Ответ: а) б)

hello_html_27d1ce35.gif

hello_html_2c7a64c8.gif







Пример 3. а) Построить точки А1 и А2 симметричные относительно прямой а.

б) Построить прямоугольники симметричные относительно прямой b.
Ответ: а) б)
hello_html_me2c259c.gif

hello_html_73eae775.gif


Пример 4. Можно ли назвать ножницы симметричной фигурой?hello_html_m43e930ed.jpg

Ответ: Да.




Пример 5. Найди подходящую правую часть робота.

hello_html_m69d1ea91.gif

Ответ:2 и 5; 1 и …; … и 4.

Пример 6. На рисунке укажите буквы латинского алфавита имеющие одну ось симметрии;

hello_html_m79939fd7.png

Ответ: A, …, …, D, …, …, .., U, V, W, Y;

Пример 7. Построить рисунок.

hello_html_17f4ab15.jpg

а

Пример 8. Сколько осей симметрии имеет прямоугольный параллелепипед, гранями которого не являются квадраты?hello_html_76dbeb1f.png

Ответ: 3.hello_html_18121dc3.png

Пример 9. Сколько плоскостей симметрии имеет правильная шестиугольная призма?

Ответ:7.

Пример 10.

Тест по теме «Зеркальная симметрия в призме ».

1)Сколько плоскостей симметрии имеет правильная четырехугольная призма?

а)2 б)4 в)3 г)5 д)12,

2)Сколько плоскостей симметрии имеет прямая призма, в основании которой лежит прямоугольник?

а)2 б)3 в)1 г)4 д)8,

3)Сколько плоскостей симметрии имеет правильная треугольная призма?

а)4 б)3 в)1 г)2 д)5.

3) Задание:

1. Построить таблицу:

Элементы симметрии правильных многогранников.


тетраэдр

октаэдр

икосаэдр

гексаэдр

додекаэдр

Центры симметрии

-

1

1

1

1

Оси симметрии

3

9

15

9

15

Плоскости симметрии

6

9

15

9

15


2.Привести примеры симметрии из природы, в архитектуре, в искусстве.(один рисунок на альбомном листе) .

3. Решить задачи ( по примерам):

  1. а)Может ли фигура иметь ровно два центра симметрии?

б)Приведите примеры не центрально-симметричных фигур.

  1. а) Построить точки В1 и В2 симметричные относительно точки О,

б) Построить прямоугольные треугольники АВС и А1В1С1 симметричные относительно точки О,hello_html_m575e1189.png

  1. а) Построить точки В1 и В2 симметричные относительно прямой а.

б) Построить квадраты симметричные относительно прямой b.

  1. Дорисуй рисунки так, чтобы они стали симметричными.

  2. Можно ли назвать стрекозу насекомым, у которого имеется ось симметрии?hello_html_m3970ee64.jpg

  3. На рисунке укажите буквы латинского алфавита имеющие две оси симметрии;

hello_html_m79939fd7.png

  1. Построить рисунок.

hello_html_3cfea8dd.jpgа аhello_html_m304f2795.jpg

  1. Сколько осей симметрии имеет прямоугольный параллелепипед, две грани которого являются квадратами?

  2. Сколько плоскостей симметрии имеет прямоугольный параллелепипед, гранями которого не являются квадраты?

  3. Определить фигуры:1) обладающие центральной симметрией и указать их центр;
    2) обладающие осевой симметрией и указать ось симметрии;3) имеющие обе симметрии.





4.

hello_html_m4a5c24b8.png

5. Построить.


hello_html_m29026d9f.png

ПЗ № 7. Вычисление площадей и объемов.

Задание:

1) Перепишите и заполните пропуски:hello_html_3a511f3b.png

  • А)Пример 1. Высота правильной треугольной пирамиды 4 см, а ее апофемы 8 см. Вычислите площадь боковой поверхности пирамиды. 
    Решение:  Исходя из того, что MK = 8, MO = 4, синус угла OKM равен  MO/MK = 1/2 , откуда угол равен arcsin 1/2 = 30 °. Откуда  KO / MK = cos 30° , KO / 8 = cos 30° ,

KO = 8 cos 30° .KO = 8/2 = 4 .
Тогда по свойству равностороннего треугольника
  КО = r = a/6.

4 = a /6 , a = 24. 
Теперь, зная размер основания боковой грани и ее апофему, найдем площадь боковой грани как площадь равнобедренного треугольника:
 Sт = 1/224 8 = 12 8 = … см2 .
Откуда площадь боковой поверхности пирамиды
 S = 3 Sт = 3 96 = … см2 . 
Ответ: 288 см2.

Пример 2. Дано: усеченная правильная пирамида, n = 3, h = 4, a1= 16 , a2= 10 . Надо найти площадь полной поверхности усеченной пирамиды .

Решение: r1= a1 / 2  = 16  : 2  = 16 : 2 = …, r2= a2 / 2  = 10  : 2  = 10 : 2 = … ,

l2 = h2 + (r2 r1)2, l2 = 42 + (5 8)2 = 16 + 9 = …, l = … Sn =  /4 (a12 + a22) + 1,5 l(a1 + a2) .

Sn =  /4 ((16 )2 + (10 )2) + 1,5 5(16  + 10 ) =  /4 (768 + 300) + 1,5 5 = =267 + 195  =   .

Ответ: 462 

Пример 3. Дано: усеченная правильная пирамида, n = 4, h = 3, a1= 16, a2= 8 . Надо найти площадь полной поверхности усеченной пирамиды .

Решение: r1= a1 / 2= 16: 2= …, r2= a2 / 2= 8  : 2  = …,

l2 = h2 + (r2 r1)2, l2 = 32 + (4 8)2 = 9 + 16 = …, l = ….

Sn = (a12 + a22) + 2 l(a1 + a2) .Sn = (162 + 82) + 2 5(16 + 8) = 320 + 240 = … .

Ответ: 560

Пример 4. Дано: усеченная правильная пирамида, n = 6, h = 2, a1= 2 , a2= 6 . Надо найти площадь полной поверхности усеченной пирамиды .

Решение: r1= a1 / 2  = 2  : 2  =  , r2= a2 / 2  = 6  : 2  = 3 ,

l2 = h2 + (r2 r1)2, l2 = 22 + ( )2 = 4 + 12 = …, l = ….

Sn =3  /2 (a12 + a22) + 3 l(a1 + a2) .Sn =3  /2 (22 + 62) + 3 4(2 + 6) = …   + .

Ответ: 60   + 96

Пример 5. Дано: усеченная правильная пирамида, n = 4, h = 3, r1=2, r2= 6 . Надо найти площадь полной поверхности усеченной пирамиды .

Решение: l2 = h2 + (r2 r1)2, l2 = 32 + (6 2)2 = 9 + 16 = …, l = ….

Sn = 4 (r12 + r22) + 4 l(r1 + r2) . Sn = 4 (22 + 62) + 2 5(2 + 6) = 160 + 80 = … .

Ответ: 240.

  • В)Пример 1. Площадь грани прямоугольного параллелепипеда равна 12. Ребро,

перпендикулярное этой грани, равно 4. Найдите объем параллелепипеда.
Решение: Каждая грань прямоугольного параллелепипеда –прямоугольник.

Пусть SABCD= a b = 12 , тогда АА1= h = 4, т.к. АА1 АВСD

Используем формулу объема прямоугольного параллелепипеда: V = a b h , V = 12 4 = ...

Ответ: 48 см3.

Пример 2. Объем прямоугольного параллелепипеда равен 12. Одно из его ребер равно 3. Найдите площадь грани параллелепипеда, перпендикулярной этому ребру.

Решение: Пусть АА1 АВСD, V = 12 , АА1= h = 3.

Найдём SABCD. Используем формулу объема прямоугольного параллелепипеда V = a b h, где SABCD= a b, S ABCD 3 = 12,S ABCD = 12 : 3 = ... Ответ: 4 см2.

Пример 3. Два ребра прямоугольного параллелепипеда, выходящие из одной вершины, равны 2, 4. Диагональ параллелепипеда равна 6. Найдите объем параллелепипеда.

Решение: a = 4, b = 2, d = 6. Найдем V.

Формула диагонали прямоугольного параллелепипеда:

d2 = a2 + b2 + h2 , 16 + 4 + h2 = 36, h2 = … , h = ...

Формула объема прямоугольного параллелепипеда: V = abh , V = 4 2 4 = ... Ответ: 32 см3.

Пример 4. Два ребра прямоугольного параллелепипеда, выходящие из одной вершины, равны 2, 3. Объем параллелепипеда равен 36. Найдите его диагональ и высоту.

Решение: a = 3, b = 2. Формула объема прямоугольного параллелепипеда: V = abh , 3 . 2 . h = 36,

6h = 36, h = ..., V = 36. Найдем d. d2 = 9 + 4 + 36, d2 = 49, d = ... Ответ: 7 и 6 см.
Пример 5. Дано: ABCDA1B1C1D1 - прямоугольный параллелепипед, диагональ 
D1= 18 составляет угол в 30° с плоскостью боковой грани, и угол в 45° с боковым ребром (рис. ). Найти: V.
hello_html_m49e41dc8.jpg

 Решение: BC1 - проекция D1на плоскость боковой грани BB1С1С,
поэтому 
D1BC1 = 30°D1BB1= 45°.
Рассмотрим Δ
D1C1BD1C1= 90° (рис.). ∠В = 30°. => D1C1 = 18 : 2 = … см.
Рассмотрим Δ
D1B1- прямоугольный: BB1= 18 cos 45° = 18 : 2 = … см.
Диагональ (d) и измерения (а, b, с) прямоугольного параллелепипеда связаны соотношением:
d2 = a2 + b2 + h2 , 182 = 92 + (9)2 + B1C12 ,(ΔD1B1B: B1B =D1 B1).
B1C12 = 182 92 (9)2 = 324 – 8181 2 = 81, B1C1 = …см. V = 99 9 = … см3.   
Ответ:
V = 729см3.

Пример 6. Стороны основания прямоугольного параллелепипеда 3 и 4. Найти его объём, если высота равна длине диагонали его основания.

Решение: BD - диагональ основания прямоугольного параллелепипеда. BD2 = АВ2 + АD2,
BD2 = 32 + 42 = 9 + 16 = …, BD = …, h = 5. V = 345 = … см3.
Ответ:
60 см3.

Пример 7. Найти объём прямоугольного параллелепипеда, если стороны основания 2 и 3, а диагональ параллелепипеда .

Решение: d2 = a2 + b2 + h2 , ()2 = 22 + 32 + h2 , h 2 = 38 – 49 = 25, h = ... V = 23 5 = … см3.
Ответ: 30 см3.

  • С)Пример 1. Дано: АВСА1В1С1 - прямая призма, АС = ВС, ACB = 90°BN NACNC1 = 45°CC1 = 6 (рис.). Найти: V. Решение: V = Sh , S = BC2 : 2, BC2 = BN2 + CN2 , BN =CN
    (
    ΔABC – прямоугольный,AC =BC), ΔC1CN – прямоугольный,CNC1 = 45°
    CC1 = CN= 6, BC2 =2CN2 = 2 62 = 236 = …, BC = 6 ,
    V = (62 6 : 2 = 36 6 = … см3.    Ответ:216см3.     Пример 2. Дано: ABCDА1В1С1D1 - прямая призма,  ABCD - ромб, BAD = 60° (рис.). ВВ1 = 2, B1DB = 45°. Найти: V. РешениеSp = AB AD sin 60°. ΔABD – равносторонний( AB = AD,BAD = 60° ).
    AB = BD = AD. ΔB1DB –прямоугольный ,
    B1DB = 45°. => ΔB1DB – равнобедренный, ВВ1 = ВD = 2,
    V = AB AD sin 60° BB1= BB13 sin 60° = 23 / 2 = … см3.
    hello_html_m47ca6280.jpghello_html_6c1a9bbb.jpg

Ответ: 4 см3

Пример 3. Дано: ABCDFM...M1 - правильная шестиугольная призма. AD1 = 8 см - наибольшая диагональ.AD1= 30°(рис.).hello_html_72355ff0.jpg

Найти: V. 
Решение: V= S0 · h. h = DD1 в ΔADD1, = 90°. D1 = 30°,

DD1 = AD1 · cos 30°. DD1 = 8 / 2 = … , AD = AD1 : 2 = 8 : 2 = … см,
OD = OC = CD = AD : 2 = 4 : 2 = …
см,
S
0 = 6S ΔOCD = 6 / 4) a2 = 6 / 4) 22 = 6 см. V = 6 = 6 43 = … см3.    

Ответ: 72 см3.   hello_html_m62762a7f.jpg

Пример 4. Дана трапеция, S(BB1C1C) = 8 см2, S(AA1D1D) = 12см2, BH = 5 см (рис.).Найти: Vnp. 
Решение:1)Расстояние между параллельными плоскостями ВВ1С1 и AA1D1 есть длина перпендикуляра ВН, который является высотой трапеции ABCD.

2) Обозначим верхнее основание трапеции - а, нижнее - b, высоту призмы h, тогда S(BB1C1C) = ah, 8 = ah, a = 8 / h, S(AA1D1D) = bh , 12 = bh, b = 12 / h,

3) S0 = (AD + BC)BH : 2 =( a + b ) BH : 2 = (8 / h + 12 / h) 5 : 2 = … / h,

4) V= S0 · h. V= 50 / · h = … см3.  Ответ: 50 см3.

  • Д)Пример 1. В правильной четырехугольной пирамиде высота равна 9 см. Сторона основания 4 см. Найдите объем пирамиды.

Решение: V= 1/3 S0 · h. V= 1/3 a2 · h = 1/3· 42·9 = 1/3 · 16 · 9 = 16 · 3 = … см3. Ответ: 48см3. 

Пример 2. a) Объем правильной четырехугольной пирамиды равен 27 см3, высота 9 см. Найти сторону основания.

Решение: V= 1/3 S0 · h. V= 1/3 a2 · h, a2  = 3V : h = 3 · 27 : 9 = 3 · 3 = ... , a = … см.

Ответ: 3 см.hello_html_m12ec6366.jpg

б) Объем пирамиды равен 56 см3, площадь основания 14 см2. Чему равна высота?

Решение: V= 1/3 S0 · h.  h = 3 V : S0  = 3 · 56 : 14 = 3 · 4 = … см.

Ответ: 12 см.

Пример 3. Дано: ABCD - правильная пирамида.

АВ = a = 3; AD = 2 (рис.).Найти: aSocн.; б) АО; в) DO; г) V.

 Решение:

а) S0 = 0,25 · a2  = 0,25 · 32 = 2,25 (используем формулу для вычисления площади правильного треугольника). 

б) AO = R = 2/3h = 1/3 a  (формула радиуса описанной окружности через сторону правильного треугольника). AO = 1/3 · 3 = .

в) DO2 = AD2AO2, (по теореме Пифагора).

DO2 = (2)2 – ()2 = 4 · 3 – 3 = … , DO = h = 3.hello_html_5fef969e.jpg

г) V= 1/3 S0 · h. V= 1/3 · 2,25 · 3 = … см3.

Ответ: aSocн. = 2,25 см2; б) АО = см; в) DO = 3см; г) V = 2,25 см3 .

Пример 4. Дано: ABCDF - правильная пирамида. 

FCO = 45°FO = 2 (рис.). Найти: a) Socн.; б) V. 

Решение:

1) Рассмотрим ΔFOC= 90°= 45°, значит, = 45°. Следовательно, ΔFOC - равнобедренный, ОС ≈ FO = h= 2.

2) АС = 2OС = 4. AC = AD (по свойству диагонали квадрата, d2 = 2а2).

Тогда  AD = AC / = 4 / = 2 .

3) ABCD - квадрат (пирамида правильная). S0 = AD2 = (2)2 = 2 · 4 = ...

4) V= 1/3 S0 · h. V= 1/3 · 8 · 2 = 16/3 5,3. Ответ: a) 8; 6) 5,3.hello_html_6d91a8cf.jpg

Пример 5. Дано: ABCA1B1C1 – усеченная пирамида. ΔАВС – прямоугольный,
AB = 18 дм, BC = 24 дм, AA1 = BB1 = СС1 = 12,5 дм, k = 0,5. Найти V.

Решение: S1 = SABC = 1/2 · AB · BC = 1/2 · 18 · 24 = 9 · 24 = … ,
S
2 = S(A1B1C1) = 1/2· A1B1 · B1C1 = 1/2 (k · AB) · (k · BC) =
= 1/2· 0,5 · 18 · 0,5 · 24 = 6 · 9 = … ,
S = S
1 + S2 + = = 216 + 54 + = 216 + 54 + 54 = … ,
V = 1/3 · h · S = 1/3 378 h = 126 h, R
1 = abc/4S1 ,

c = = = … , R1 = = = …, hello_html_m78f1984a.jpg

R2 = R1 : 2 = 7,5; h2 = 12,52 – (15 – 7,5)2 = 12,52 – 7,52 = (12,5 – 7,5) · (12,5 + 7,5) =

= 5 · 20 = … , h = … ,

V = 126 h = 126 · 10 = … (дм3).
Ответ: 1260 (дм3).

Пример 6. усеченная пирамида а) n = 3, а1 = 2, а2 = 5, h = 12, V =?
Решение: A = 22 + 52 + 2 · 5 = 39, V = · h · A = · 12 · 39 = … . Ответ: 39 .

б) n = 4, a1 = 3, a2 = 8, h = 6, V = ?
Решение: A = 32 + 82 + 3 · 8 = 97, V = 1/3 · 6 · 97 = 2 · 97 = ...

Ответ: 194.



2)Решить задачи ( по примерам):

А)

  1. Высота правильной треугольной пирамиды 8 см, а ее апофемы 16 см. Вычислите площадь боковой поверхности пирамиды. 

  2. Дано: усеченная правильная пирамида, n = 3, h = 8, a1 = 14 , a2 = 2 . Надо найти площадь полной поверхности усеченной пирамиды .

  3. Дано: усеченная правильная пирамида, n = 4, h = 8, a1 = 16, a2 = 4 . Надо найти площадь полной поверхности усеченной пирамиды .

  4. Дано: усеченная правильная пирамида, n = 6, h = 2, a1 = 4 , a2 = 8 . Надо найти площадь полной поверхности усеченной пирамиды .

  5. Дано: усеченная правильная пирамида, n = 4, h = 3, r1 = 5, r2 = 9 . Надо найти площадь полной поверхности усеченной пирамиды .

В)

  1. Площадь грани прямоугольного параллелепипеда равна 15. Ребро, перпендикулярное этой грани, равно 6. Найдите объем параллелепипеда.

  2. Объем прямоугольного параллелепипеда равен 24. Одно из его ребер равно 3. Найдите площадь грани параллелепипеда, перпендикулярной этому ребру.

  3. Два ребра прямоугольного параллелепипеда, выходящие из одной вершины, равны 3, 4. Диагональ параллелепипеда равна 13. Найдите объем параллелепипеда.

  4. Два ребра прямоугольного параллелепипеда, выходящие из одной вершины, равны 3, 6. Объем параллелепипеда равен 108. Найдите его диагональ и высоту.

  5. Дано: ABCDA1B1C1D1 - прямоугольный параллелепипед, диагональ  D1= 12 составляет угол в 30° с плоскостью боковой грани, и угол в 45° с боковым ребром. Найти: V.

  6. Стороны основания прямоугольного параллелепипеда 6 и 8. Найти его объём, если высота равна длине диагонали его основания.

  7. Найти объём прямоугольного параллелепипеда, если стороны основания 4 и 6, а диагональ параллелепипеда .

С)

  1. Дано: АВСА1В1С1 - прямая призма, АС = ВС, ACB =90°BN NACNC1 = 45°CC= 8 (рис.). Найти: V.

  2. Дано: ABCDА1В1С1D1 - прямая призма,  ABCD - ромб, BAD = 60° (рис.). ВВ1 = 4, B1DB = 45°. Найти: V.

  3. Дано: ABCDFM...M1 - правильная шестиугольная призма. AD1 = 16 см - наибольшая диагональ.AD1= 30° (рис.). Найти: V. 

  4. Дана трапеция, S(BB1C1C) = 10 см2, S(AA1D1D) = 14см2, BH = 10 см (рис.). Найти: Vnp. 

Д)

  1. В правильной четырехугольной пирамиде высота равна 6 см. Сторона основания 5 см. Найдите объем пирамиды.

  2. a)Объем правильной четырехугольной пирамиды равен 48 см3, высота 4 см. Найти сторону основания. б) Объем пирамиды равен 28 см3, площадь основания 4 см2. Чему равна высота?

  3. Дано: ABCD - правильная пирамида. АВ = a = 6; AD = 4 . Найти: aSocн.; б) АО; в) DO; г) V.

  4. Дано: ABCDF - правильная пирамида.  FCO = 45°FO = 4 . Найти: a) Socн.; б) V. 

  5. Дано: ABCA1B1C1усеченная пирамида. ΔАВС прямоугольный, AB = 12 дм,BC = 16 дм, AA1 = BB1 = СС1 = 13 дм, k = 0,5. Найти V.

  6. а) n = 3, а1 = 2, а2 = 5, h = 24, V =?, б) n = 4, a1 = 3, a2 = 8, h = 3, V = ?,

3)Решить задачи :

  1. Дана правильная четырехугольная пирамида со стороной основания a=12 см и
    высотой
    h=8 см. Найдите площадь полной поверхности пирамиды.
    hello_html_m3eb51a34.png

  2. Найдите объем прямоугольного параллелепипеда с ребрами 3 см, 5 см и 8 см.

а) 120 см3; б) 60 см3; в) 32 см3; г) другой ответ.

  1. Длина прямоугольной комнаты в 2 раза больше ширины и на 2 м больше высоты. Найдите объем комнаты, если ее длина равна 6 м. а) 432 м3; б) 144 м3; в) 72 м3; г) другой ответ.

  2. Три ребра прямоугольного параллелепипеда, выходящие из одной вершины, равны 1; 0,5 и 16. Найдите ребро равновеликого ему куба.












Просмотрено: 0%
Просмотрено: 0%
Скачать материал
Скачать материал "ПЗ по специальности Технология машиностроения,1 курс(2016-2017 уч.г.)"

Получите профессию

Технолог-калькулятор общественного питания

за 6 месяцев

Пройти курс

Рабочие листы
к вашим урокам

Скачать

Выбранный для просмотра документ ПЗ№8,9.docx

ПЗ № 8. Векторы. Действия с векторами. Декартова система координат в пространстве. Уравнение окружности, сферы, плоскости.

Задание:

1) Опорный конспект.

Физические величины, имеющие не только абсолютное значение, но и направление, называются векторными.

Скорость, сила, ускорение — векторы. Для них важно «сколько» и важно «куда». Например, ускорение свободного падения hello_html_m28faee77.png направлено к поверхности Земли, а величина его равна 9,8 м/с2. Импульс, напряженность электрического поля, индукция магнитного поля — тоже векторные величины.

Вектор — это направленный отрезок. Длиной вектора называется длина этого отрезка. Обозначается: hello_html_m2a88039c.png или hello_html_20991715.png

Равными называются векторы, имеющие одинаковые длины и одинаковое направление. Это значит, что вектор можно перенести параллельно себе в любую точку плоскости.
Единичным
 называется вектор, длина которого равна 1. Нулевым — вектор, длина которого равна нулю, то есть его начало совпадает с концом.

Сложение векторов

Для сложения векторов есть два способа.

1. Правило параллелограмма. Чтобы сложить векторы hello_html_6ccfdd8b.png и hello_html_8a9ad90.png, помещаем начала обоих в одну точку. Достраиваем до параллелограмма и из той же точки проводим диагональ параллелограмма. Это и будет сумма векторов hello_html_6ccfdd8b.png и hello_html_8a9ad90.png.

hello_html_m7e7837b3.png

Помните басню про лебедя, рака и щуку? Они очень старались, но так и не сдвинули воз с места. Ведь векторная сумма сил, приложенных ими к возу, была равна нулю.

2. Второй способ сложения векторов — правило треугольника. Возьмем те же векторы hello_html_6ccfdd8b.png и hello_html_8a9ad90.png. К концу первого вектора пристроим начало второго. Теперь соединим начало первого и конец второго. Это и есть сумма векторов hello_html_6ccfdd8b.png и hello_html_8a9ad90.png.

hello_html_m16fda0fa.png

По тому же правилу можно сложить и несколько векторов. Пристраиваем их один за другим, а затем соединяем начало первого с концом последнего.

hello_html_m180d670c.png

Представьте, что вы идете из пункта А в пункт В, из В в С, из С в D, затем в Е и в F. Конечный результат этих действий — перемещение из А в F.

Умножение вектора на число

При умножении вектора hello_html_6ccfdd8b.png на число k получается вектор, длина которого в k раз отличается от длины hello_html_6ccfdd8b.png. Он сонаправлен с вектором hello_html_6ccfdd8b.png, если k больше нуля, и направлен противоположно hello_html_6ccfdd8b.png, если k меньше нуля.

hello_html_73835e2b.pnghello_html_7e06bfe9.jpg

Трехмерные чертежи выполнять тяжко, поэтому ограничусь одним вектором, который для простоты отложу от начала координат:

Перед вами ортонормированный базис hello_html_5b0c516f.gif трехмерного пространства и прямоугольная система координат, единичные векторы hello_html_37b6494a.gif данного базиса попарно ортогональны: hello_html_6b98b57d.gif и hello_html_50e1b5f7.gif. Ось hello_html_m4710f128.gif наклонена под углом 45 градусов только для того, чтобы складывалось визуальное впечатление пространства. Любой вектор hello_html_m14956daf.gif трехмерного пространства можно единственным способом разложить по ортонормированному базису hello_html_5b0c516f.gif: 
hello_html_m6fd59a9f.gif, где hello_html_m4e4977fa.gif – координаты вектора hello_html_754949a.gif (числа) в данном базисе.

Пример с картинки: hello_html_5f103d13.gif. Давайте посмотрим, как здесь работают правила действий с векторами. Во-первых, умножение вектора на число: hello_html_m196faf04.gif (красная стрелка), hello_html_6278e414.gif (зеленая стрелка) и hello_html_b8930de.gif (малиновая стрелка). Во-вторых, перед вами пример сложения нескольких, в данном случае трёх, векторов: hello_html_6c8ea6a4.gif.  Вектор суммы hello_html_m49782ef1.gif начинается в исходной точке отправления (начало вектора hello_html_m196faf04.gif) и утыкается в итоговую точку прибытия (конец вектора hello_html_b8930de.gif).

Все векторы трехмерного пространства, естественно, тоже свободны, попробуйте мысленно отложить вектор hello_html_m49782ef1.gif от любой другой точки, и вы поймёте, что его разложение hello_html_6c8ea6a4.gif  «останется при нём».

Аналогично плоскому случаю, помимо записи hello_html_5f103d13.gif широко используются версии  со скобками: hello_html_m1450ba4d.gif либо hello_html_6f699bd2.gif.

Если в разложении отсутствует один (или два) координатных вектора, то вместо них ставятся нули. Примеры:
вектор
 hello_html_m7a0cbf9e.gif (дотошно hello_html_171ec4e7.gif) – запишем hello_html_5bd901b6.gif;
вектор
 hello_html_m4cf51bb.gif (дотошно hello_html_3bf2c75b.gif) – запишем hello_html_m26e182f0.gif;
вектор
 hello_html_mbf68e03.gif (дотошно hello_html_2380cf95.gif) – запишем hello_html_m7c8d9d32.gif.

Базисные векторы записываются следующим образом:
hello_html_7deb8e75.gif

Если даны две точки пространства hello_html_578b0f29.gif и hello_html_4270881d.gif, то вектор hello_html_63db5e27.gif имеет следующие координаты:
hello_html_m145ad02f.gif

То есть, из координат конца вектора нужно вычесть соответствующие координаты начала вектора.

Если даны две точки пространства hello_html_578b0f29.gif и hello_html_4270881d.gif, то длину отрезка hello_html_28104f0d.gif можно вычислить по формуле hello_html_292c92b6.gif

Действия над векторами

Пусть в трехмерном пространстве заданы векторы hello_html_4cd93f33.pngсвоими координатами. Имеют место следующие операции над ними: линейные (сложение, вычитание, умножение на число и проектирование вектора на ось или другой вектор); не линейные – различные произведения векторов (скалярное, векторное, смешанное).

1). Сложение двух векторов производится поэлементно, то есть если hello_html_ddb270d.png, то в координатной форме записывается:

hello_html_m39f935fc.pnghello_html_7df27acf.png(2.18)

Данная формула имеет место для произвольного конечного числа слагаемых.

Геометрически два вектора складываются по двум правилам:

а) правило треугольника – результирующий вектор суммы двух соединяет начало первого из них с концом второго; для суммы hello_html_m68d67358.png векторов – результирующий вектор суммы соединяет начало первого из них с концом последнего вектора–слагаемого при условии, что начало последующего слагаемого совпадает с концом предыдущего;

б) правило параллелограмма (для двух векторов) – параллелограмм строится на векторах–слагаемых как на сторонах, приведенных к одному началу; диагональ параллелограмма исходящая из их общего начала, является суммой векторов.

2). Вычитание двух векторов производится поэлементно, аналогично сложению, то есть если hello_html_46ec41.png, то в координатной форме записывается

hello_html_7ec0d2fc.pnghello_html_73424de9.png(2.19)

Геометрически два вектора складываются по уже упомянутому правилу параллелограмма с учетом того, что разностью векторов является диагональ, соединяющая концы векторов, причем результирующий вектор направлен из конца вычитаемого в конец уменьшаемого вектора.

3). Умножение вектора на число hello_html_745ce834.png покоординатно: hello_html_m1bf67fb0.png.

При hello_html_m66aa3952.png – вектор сонаправлен первоначальному;

hello_html_7012cc6f.png – вектор противоположно направлен первоначальному;

hello_html_2473ec23.png – длина вектора увеличивается в hello_html_745ce834.png раз;

hello_html_6d30a424.png – длина вектора уменьшается в hello_html_745ce834.png раз.

4). Пусть в пространстве задана направленная прямая (ось l)вектор hello_html_18de2c7d.png=hello_html_m694d1127.png задан координатами конца и начала. Обозначим проекции точек hello_html_100c518e.png и hello_html_6344936f.png на ось hello_html_278ee096.png соответственно через hello_html_m2223bfd4.png и hello_html_5303a217.png.

Проекцией вектора hello_html_18de2c7d.png на ось hello_html_278ee096.png называется длина вектора hello_html_m7dc13367.png, взятая со знаком «+», если вектор hello_html_m7dc13367.png и ось lсонаправлены, и со знаком «-», если hello_html_m7dc13367.png и l противоположно направлены.hello_html_m11b24da8.png

Если в качестве оси l взять некоторый другой вектор hello_html_6b05d8d6.png, то получим проекцию вектора hello_html_18de2c7d.png на вектор hello_html_6b05d8d6.png.

Уравнение окружности, сферы, плоскости.


hello_html_7a030643.jpg



hello_html_59f1dd46.gifhello_html_1781d02f.jpg


2) Решить задачи:

1. Укажите какое правило использовали и постройте чертеж.

Сложение векторов

Правило параллелограмма: диагональ параллелограмма - сумма двух векторов с общим началом.

Правило треугольника: от конца первого вектора отложить второй вектор, тогда их суммой будет вектор, начало которого совпадает с началом первого вектора, а конец с концом второго вектора.

hello_html_m4ccfc737.png

Вычитание векторов

Вычитание векторов - это сумма положительного и отрицательного вектора.

hello_html_62112a3f.png


2.Перепишите и заполните пропуски:
Пример 1. Сфера задана уравнением x 2 + (y + 3)2 + (z – 2)2 = 25.

Найдите координаты центра и радиуса сферы.

Решение: О - центр сферы, О(0,3,2), R = = ...
Ответ:
О(0,3,2), R = 5.

Пример 2. Напишите уравнение сферы радиуса = 7 с центром в точке А(2; 0; 1). Решение: (x …)2 + y 2 + (z + …)2 = 72. (x2)2 + y 2 + (z + 1)2 = …
Ответ: (x2)2 + y 2 + (z + 1)2 = 49.

Пример 3. Лежит ли А(2; 1; 4) на сфере, заданной уравнением  (x + 2)2 + (y 1) 2 + (z 3)2 = 1. Решение: Подставим координаты точки А в уравнение сферы (2 + 2)2 + (1 1) 2 + (4 3)2 = 1, 1 = 1(верно), точка А лежит на сфере.
Ответ:
точка А лежит на сфере.

Пример 4. Найти координаты центра и радиус сферы x2 + y2 + z2 + 4y - 2z = 4. Решение: x2 + y2 + z2 + 4y 2z = 4 выделим квадрат двучлена:
х
2 + у2 + 4у + 4 4 + z2  4z + 1 1 = 4, х2 + (у + 2)2 + (z 1)2 = 9, центр окружности С(…; …; …), радиус R = ...
Ответ:
С(0; 2; 1), R = 3.

Пример 5. Дано: уравнение сферы, х2 + у2z2 + 2у 4= 4.

Найти: а) О(х0; у0z0), R; б) m, при котором А(0; m; 2) и В(1; 1; m2) принадлежат сфере.

Решение: а) x 2 + y 2 +2у + z 2 – 4z = 4, x 2 + y 2 +2у +11 + z 2 – 4z + 4 4 = 4,
x 2 + (y + 1)2 + (z – 2)2 = 9. О(...,…,…), R = = ...
б) А(0; 
m; 2) и В(1; 1; m2)


 , , ,

, m = 2. При m = … точки A и В принадлежат сфере. Ответ: а) О(0; 1; 2), R = 3; б) при m = 2.

Пример 6. Найдите длину вектора КА АС.

hello_html_m436d4048.gif

Ответ: 20 см.

3)Решить задачи ( по примерам):

  1. Сфера задана уравнением (x – 1)2 + y 2 + (z – 2)2 = 9.

Найдите координаты центра и радиуса сферы.

  1. Напишите уравнение сферы радиуса = 4 с центром в точке А(2; 1; 0).

  2. Лежит ли А(5; 1; 4) на сфере, заданной уравнением  (x 3)2 + (y+ 1) 2 + (z 4)2 = 4.

  3. Найти координаты центра и радиус сферы x2 – 6x + y2 + z2 = 0.

  4. Дано: уравнение сферы, х2 + у2z2 + 4у 2= 4.

Найти: а) О(х0; у0z0), R; б) m, при котором А(0; m; 1) и В(1; 0; m2) принадлежат сфере.

  1. Найдите длину вектора КА АС, диагонали ромба 6 и 8 см.

3)Решить задачи :

  1. Точки А(3; – 5; 6) и В(5; 7; – 1) являются концами одного из диаметров сферы. Составьте уравнение этой сферы.

  2. Дана сфера x2 + y2 + z2 = 450  . Найти координаты точек пересечения сферы с прямой, проходящей через начало координат и точку А(4; 5; 3).

  3. Даны точки А(– 1; 3; 2), В(0; 3; 1), С(2; – 2; 0), D(– 4; 2; 2), Е(5; 7; 8). Какие из этих точек принадлежат сфере с центром О(– 2; 1; 0) и радиусом 3?

  4. Составьте уравнение сферы с центром О (2; 3; 4) и радиусом R=5.

  5. Точки А(7; – 2; 4) и В(9; – 8; 6) лежат на поверхности сферы и на прямой, проходящей через её центр. Составьте уравнение сферы.

  6. Сфера задана уравнением x 2 + y 2 + z 2 + 2y – 4z = 4. a)Найдите координаты центра и радиуса сферы. б) Найдите значение m, при котором точки А(0; m; 2) и В(1; 1; m – 2) принадлежат данной сфере.

  7. Диаметр сферы – отрезок АВ с концами А(2; – 1; 4) и В(2; 7; 10). a) Составьте уравнение сферы. б) Найдите кратчайшее расстояние от точки данной сферы до плоскости Оxy.

  8. Сфера задана уравнением (x – 1)2 + y 2 + (z – 2)2 = 9. а)Найдите координаты центра и радиуса сферы. б)Определите, принадлежат ли данной сфере точки А(1; 3; 1) и В(2; 2; 1).

  9. Составить уравнение сферы в каждом из следующих случаев:

а) сфера имеет центр С(0; 0; 0) и радиус r = 9;

б) сфера имеет центр С(5; 3; 7) и радиус r = 2;

в) сфера проходит через начало координат и имеет центр С(4; 4;2);

г) сфера проходит через точку А(2; 1; 3) и имеет центр С(3; 2; 1);

д) точки А(2; 3; 5) и В(4; 1; 3) являются концами одного из диаметров сферы;

  1. Сфера задана уравнением x2 + у2 + z2 + 2у 4z = 4.

а) Найдите координаты центра и радиус сферы.

б) Найдите значение m, при котором точки А(0; m; 2) н В (1; 1; m2) принадлежат данной сфере.

  1. Найдите координаты центра и радиус сферы, заданной уравнением

(x – 2)2 + (y + 3) 2 + z2 = 25. 

  1. Напишите уравнение сферы радиуса R = 7 с центром в точке А(2; 0; 1).

  2. Лежит ли А(2; 0; 3) на сфере, заданной уравнением (x + 2)2 + (y 1) 2 + (z 3)2 = 1. 

  3. Могут ли все вершины прямоугольного треугольника с катетами 4 см и 2см лежать на сфере радиуса см?

  4. Найти координаты центра и радиус сферы x2 + 6х + y2 + z 2 = 0. 

  5. Найдите координаты центра и радиус сферы, заданной уравнением

(x + 3)2 + y 2 + (z 1)2 = 16. 

  1. Напишите уравнение сферы радиуса R = 4 с центром в точке А(2; 1; 0).

  2. Лежит ли точка А(5; 1; 4) на сфере, заданной уравнением

(x –3)2 + (y + 1) 2 + (z 4)2 = 4. 

  1. Могут ли все вершины прямоугольного треугольника с катетами 4 см и 2см лежать на сфере радиуса см?

  2. Найти координаты центра и радиус сферы x2 + y2 + 6у + z2 = 0. 

  3. Составить уравнение сферы с центром в точке А (– 3; 4; – 9) и проходящую через

точку N (– 2; 6; 1).

  1. Составить уравнение сферы которая касается каждой из координатных плоскостей и проходит через точку M (2;1;3).

  2. Составьте уравнение сферы с центром в точке О(– 1;0;2), если известно, что этой сфере принадлежит точка А(3;1;1).

  3. Даны точки А(2; – 5;8) В(8; – 2;5) С(5; – 8:2)и Д(– 2; – 8; – 5).Составьте уравнение сферы, если известно, что эти точки лежат на её поверхности.

  4. Точка А лежит на сфере с центром О(3; 0; 0).

  1. Напишите уравнение сферы.

  2. Принадлежат ли этой сфере точки с координатами и (4; – 1; 0)?

  1. Составьте уравнение сферы, радиус которой равен 2, если известно, что центр сферы лежит в плоскости ОХZ, а сама сфера проходит через начало координат и точку А(1; 1; 0).

  2. Составьте уравнение сферы с радиусом, равным 3, если известно, что центр сферы лежит на оси OZ и сфера проходит через точку К(– 2; – 2; 1).

  3. Найти уравнение сферы, проходящей через точки  (0;0;0), (4;0;0),(0;6;0) и (0;0;8).

  4. Найдите координаты центра и радиус сферы, заданной уравнением:

1)x² + y² + z² = 49,

2)(х 3)² + (у + 1)² + (z + 3)² = 1,

3)х² + (y 4)² + z² = 3,

4)(x 1)² + y² + (z + 2)² = 25.

  1. Найти координаты центра и радиус сферы, заданной уравнением
    x2 + y2 + z2 – x + 2y + 1 = 0.


ПЗ № 9. Расстояние между точками. Действия с векторами, заданными координатами.

Задание:

1)Перепишите и заполните пропуски:
Пример 1. Дано:



Решение:

  1. Находим координаты вектора

;

  1. Затем находим координаты вектора


  1. Теперь находим аналогично координаты вектора


  1. Теперь находим сумму данных векторов, складывая соответствующие координаты:


Ответ:
Пример 2. Дано: , . Найдите  

Решение: Первый случай

  1. Находим координаты вектора

;

  1. Затем находим разность векторов

;

  1. Теперь находим длину вектора :

Второй случай

  1. Находим координаты вектора

;

  1. Находим координаты вектора

;

  1. Затем находим сумму векторов

;

  1. Теперь находим длину вектора : =

Ответ:

Пример 3. Даны векторы   и . Найти

Решение: Для действий с векторами справедлив обычный алгебраический приоритет: сначала умножаем, потом складываем:

=  3 - 2= - =
= = .

= + 4 {7; -9 ;1 } = + = =

=
Ответ:  ,

Пример 4. Найдите сумму векторов: и .

Решение: , .

Ответ:

Пример 5. Даны векторы , Найдите координаты векторов

Решение: , , с ,

, .

Ответ: .

Пример 6. Дано: ΔАВС, А(2; 0; 1), В(1; 2; 3), С(8; 4; 9). ВМ - медиана.

Найти: координаты вектора .

Решение: По определению медианы, М - середина отрезка АС. Следовательно, координаты М найдем по формулам координат середины отрезка  M ((82)/2, (4 + 0)/2,(9 + 1)/2), M(…,…,…).{3 + 1,22,53}, {…,… ,…}. Ответ: {4; 4; 2}.

Пример 7. Дано: А(1; 5; 3), В(7; 1; 3), С(3; 2; 6). Доказать: ΔABC - прямоугольный.

Решение: По формуле расстояния между двумя точками найдем длины отрезков АВ, АС, ВС.
AB2 = (7 + 1)2 + (5 + 1)2 + (3 – 3)2, AB2 = 64 + 36 = … , BC2 = (7– 3)2 + (– 2 + 1)2 + (6 – 3)2,
BC2 = 16 + 1 + 9 = … , AC2 = (3 + 1)2 + (5 + 2)2 + (6 – 3)2, AC2 = 16 + 49 + 9 = ...

Проверим равенство АВ2 = ВС2 + АС2, 100 = 26 + 74 верно.

По теореме обратной теореме Пифагора делаем вывод, что ΔABC - прямоугольный
с гипотенузой АВ.

Пример 8. Дано: ΔАВС; М, N, К - середины сторон соответственно АВ, ВС, АС. М(3; 2; 5), 
N(3,5; 1; 6), К(1,5; 1; 2). Найти: координаты А, В, С.

Решение: Пусть A (х1; у1z1), В(х2; у2z2), С(х3; у3z3). По формулам координат середины отрезка составим системы для абсцисс, ординат и аппликат. Пользуясь методом сложения, решим эту систему:

Ответ: А(2; 0; 1), В(8;4; 9), С(1; 2; 3).

Пример 9. Дано: А(-2; 1; 2), B(-6; 3; -2), С  оси OZ; АС = ВС. Найти: координаты точки С.

Решение: По условию С  оси OZ, значит она имеет координаты С(0; 0; z) и АС = ВС. Составим уравнение, пользуясь формулой расстояния между двумя точками: 4 + 1 + (z 2)2 = 36 + 9 + (z + 2)2, 5 + z2 – 4z + 4 = 45 + z2 + 4z + 4, 8z = 40, z = …

Ответ: (0; 0;5).

Пример 10. Дано: А(2; 1; 2), B(6; 3; 2), С (0; 0; 5); АС = ВС. Найти: SABC).

Решение: По формуле координат середины отрезка АВ найдем координаты точки М — середины:
M ((62)/2, (1 + 3)/2,(22)/2), M(4,2,0). AB2 = (6 + 2)2 + ( 31)2 + (2 + 2)2 = 16 + 4 + 16 = …, AB = ... СМ-высота равнобедренного ΔABC.
CM2 = (40)2 + (20)2 + (0 (5))2 = 16 + 4 + 25 = … , CM = 3 ,
SABC) = AB · CM : 2 = 6 · 3 : 2 = … .

Ответ: 9.

2)Решить задачи ( по примерам):

  1. Дано: А(2;1;6), В (2;0; 1), С(1; 5; 0)


  1. Дано: , , ; 2).

  2. Даны векторы   и ,  . Найти

  3. Найдите сумму векторов: и .

  4. Даны векторы , , . Найдите координаты векторов

и

  1. Дано: ΔАВС; А(1; 2; 3), B(1; 0; 4), С(3; 2; 1). AM - медиана. Найти: координаты вектора

  2. Дано: А(1; 5; 3), В(1; 3; 9), С(3; 2; 6).Доказать: ΔAВС - прямоугольный.

  3. Дано: ΔАВС, М, N, К - середины сторон соответственно ABBС, AС. М(3; 2; 4), 
    N(6; 4; 10), К(7; 2; 12).Найти: координаты вершин А, В, С.

  4. Дано: A(4; 5; 4), B(2; 3; 4); С  оси  OXAC = ВС. Найти: координаты точки С.

  5. Дано: А(4; 5; 4), B(2; 3; 4), С(1; 0; 0), АС = ВС. Найти: S(ΔABC).

3)Решить задачи :

А)

  1. Найдите координаты вектора , если

  2. Даны векторы {1;3; 3} и . Найдите координаты и длину вектора.

  3. Даны векторы {3;1; 2} и . Найдите координаты вектора ,

  4. Найдите длину вектора , , если {2;1; 5} и .

  5. Из точки А построен вектор . Найдите координаты точки В , если:

А(3;1; 2), .

  1. Даны точки А(4;6; –2) и В (–10;6; 0) . Найти длину отрезка АВ.

  2. Даны точки: А(10;14; 4), В (10;8; 12) , С (18;8; 18) 

Выяснить, равнобедренный ли треугольник, построенный на этих точках.

  1. а) Даны два вектора:  и .Найти .

б) Даны четыре вектора: .

Найти координаты векторов  

  1. В кубе АВСDА1В1С1D1, сторона которого равна 3, на диагоналях граней АD1 и D1В1 взяты точки Е и К так, что D1Е : АD1 = 2 : 3, D1K : D1B1 = 1 : 3. Найдите длину отрезка DK.

  2. Дано:

  3. Найдите длину вектора КА АС, диагонали ромба 6 и 8 см.

  4. Даны точки А(2;3; –1) и В (–5;3; 0) . Найти длину отрезка АВ.

  5. Даны точки: А(5;7; 2), В (5;4; 6) , С (9;4; 9) Выяснить, равнобедренный ли треугольник.

  6. Даны два вектора:  и .Найти .

В)

  1. Дано: A (10, 4, 3), B (6, 2, 1). Найти координаты точки M – середины отрезка AB.

  2. Дано: A (5, 4, 7), B (10, 10, 0). Найти координаты вектора .

  3. Дано: {0, 5, 0}, {2, 2, 1}. Найти длину векторов.

  4. Даны точки А (1,5; 1; –2), B (2; 2; –3); и C (2; 0; –1). Найдите: периметр треугольника ABC.

  5. Дано: М(–4; 7; 0) N(0; –1; 2).Найти: расстояние от начала координат до середины

отрезка MN.

  1. В кубе АВСDА1В1С1D1, сторона которого равна 3, на диагоналях граней АD1 и D1В1
    взяты точки Е и К так, что
    D1Е : АD1 = 1 : 3, D1K : D1B1 = 2 : 3. Найдите длину отрезка DK.

  2. Даны четыре вектора: .

Найти координаты векторов  

  1. Дано:

  2. Даны векторы и   Найдите координаты вектора .

  3. Даны векторы,. Найдите координаты вектора 

  4. На каком расстоянии от плоскости (хОу) находится точка А(2; 3; 5).

  5. На каком расстоянии от начала координат находится точка А(3; 4; 0).

  6. Найти длину вектора  если А(5; 3; 2), В(3; 1; 4).

  7. На каком расстоянии от плоскости (yOz) находится точка В(3; 2; 4).

  8. Даны векторы и  . Найдите  

  9. Изобразить систему координат Оху: и построить точку А(1; 2; 4).
    Найти расстояние от этой точки до координатных плоскостей.

  10. Вершины ΔАВС имеют координаты А(2; 0; 1), В(1; 2; 3), С(8; 4; 9).
    Найдите координаты вектора
      если ВМ - медиана ΔABC.

  11. Даны точки А(1; 5; 3) В(7; 1; 3) С(3; 2; 6). Доказать, что ΔАВС - прямоугольный.

  12. Даны точки А(2; 1; 2), В(6; 3; 2) на оси аппликат.
    Найти точку С, равноудаленную от точек А и В.

  13. Дано: А(2; 5; 8), В(6; 1;0).На оси ординат найти точку С, равноудаленную от точек А и В.
    Найти: площадь Δ
    ABC.

  14. Даны точки А(2; 1; 2), В(6; 3;2) на оси аппликат. Найти точку С, равноудаленную от
    точек А и В. Найти площадь ΔАВС.

  15. Середины сторон ΔАВС имеют координаты: М(3; 2; 4). N(6; 4; 10), К(7; 2; 12).
    Найдите координаты вершин ΔАВС.

  16. Даны точки А(4; 5; 4), В(2; 3; 4) на оси абсцисс. Найти точку С, равноудаленную от точек А и В. Найти площадь ΔABC

  17. Даны точки А(3; 1; 2) и В(1; 1; 2). Найдите: а) координаты середины отрезка АВ;

б) координаты и длину вектора  в) координаты точки С, если .

  1. Даны точки А(0; 4; 0), В(2; 0; 0), С(4; 0; 4) и D(2; 4; 4). Докажите, что ABCD - ромб.

  2. Даны точки А(0; 1; 2), В(√2 ; 1; 2), С(; 2; 1) и D(0; 2; 1). Докажите, что ABCD- квадрат.

  3. Даны точки А(2; 1; 8), В(1;5; 0), С(8; 1; 4).

Докажите, что ΔАВС - равнобедренный и найдите длину средней линии треугольника, соединяющей середины боковых сторон.

  1. Даны координаты трех вершин параллелограмма ABCD:
    А(
    6; 4; 0), В(6; 6; 2), С(10; 0; 4). Найдите координаты точки D и
    угол между векторами
      и .
    hello_html_793497f7.jpg

  2. Дано: О(0; 0; 0), А(4; 0; 0), В(0; 6; 0), С(0; 0; 2).
    Δ
    AОВ - вписанный в окружностьW(D; r).
    Найти: а) координаты центра окружности
     D;
    б)
     r- радиус окружности.

  3. Дано: ΔАВС - прямоугольный; АС, ВС - катеты;AC = b = 9 ;BC = a = 12;
    CD = m = 4; CD  (ABC); М - середина гипотенузы АВ. Найти: DM.









Просмотрено: 0%
Просмотрено: 0%
Скачать материал
Скачать материал "ПЗ по специальности Технология машиностроения,1 курс(2016-2017 уч.г.)"

Получите профессию

Копирайтер

за 6 месяцев

Пройти курс

Рабочие листы
к вашим урокам

Скачать

Выбранный для просмотра документ ПР - ТМ-1.docx

ПЗ № 1. Признаки взаимного расположения прямых. Угол между прямыми. Взаимное расположение прямых и плоскостей.hello_html_m4f96c18b.jpg

Задание:

1)А)Перепишите и заполните пропуски:

  1. Найти точки пересечения прямых и плоскостей.

а) АВ ∩ АD = А , б) АА1 ∩ А1В1 = …, в) СС1∩ВС=…,

г) (АА1 D1D) ∩ (АА1 В1В) = АА1 ,

д) ( АВСD ) ∩ ( СDD1С1 ) = …,

е) ( АВСD ) ∩ ( АВD1С1 ) = …,

  1. Определить взаимное расположение прямых: вставить знак

( ||, ∩, · )

а) АВ и А1В1, б) А1D и АВ, в) DС1 и АВ ,

г) ВС и В1С1 , д) В1В и ВС, е) АD и ВС .

  1. а) Точка М не лежит в плоскости ромба АВСD. Докажите, что прямая АВ параллельна плоскости DМС. hello_html_m63271b33.jpg

Доказательство: АВ || СD , СD DМС,=> АВ|| DМС

б) АВСD- трапеция, МК- средняя линия трапеции. Докажите, что прямая МК параллельна плоскости α, в которой лежит основание трапеции АD и не лежит ВС.

Доказательство:

МК || АВ, АВ α, => МК ... α hello_html_4d3b9131.jpg

в) В параллелограмме АОВК сторона ОВ параллельна прямой m , а АО и m –скрещивающиеся прямые. Найти угол между скрещивающимися прямыми , если один из углов параллелограмма равен а) 22°, б) 34°,в) 110°,г) 140°.

Решение: а) φ = 22 °,б) φ = …, в) φ = 180° – 110° =70 °,

г ) φ = 180 °– 140°= …

  1. Точка С лежит на отрезке АВ. Через точку А проведена плоскость α, а через точки В и С- параллельные прямые , пересекающие эту плоскость соответственно в точках В1 и С1. Найти длину отрезка СС1, если а) ВВ1 = 10 см ; б) АС : СВ = 3 : 2 и ВВ1 = 10 см. hello_html_bd13706.jpg

Решение:

а) СС1 = ВВ1 : 2 = 10 : 2 = …

б) СС1 = ВВ1 · 3: (3 + 2) = 10 · 3: (3 + 2) =…

5) Дано: ABCD - параллелограмм; а || ВС; а  (ABCD) (рис. 1).hello_html_m456621d.jpg

Доказать: а и CD - скрещивающиеся.

Найти: угол между а и CD, если BCD = 50°.

Решение:

I. 1) Так как а || ВС, то проведем через них плоскость α.

2) D  α, так как иначе DC  α, то есть α совпала бы с плоскостью ABCD и а  (ABCD), что противоречит условию.

3) Тогда DC  α в точке С  а;

4) Вывод: по теореме а и CD - скрещивающиеся.

II. Проведем через точку С прямую, параллельную прямой а. Это будет прямая СВ. Значит, угол между а к СВ равен углу между прямыми СВ и CD, то есть BCD = ...°. (Ответ: 50°.)

В) Пример 1. Дано: а || α, b α, а || b, с - секущая, 2 1= 30°,

Найти: 1, 2.hello_html_69440865.png

Решение: Углы 1 и 2 внутренние односторонние,

их сумма равна 180°, т. е.  l + 2 = 180°. (1)

Обозначим градусную меру угла 1 через х.

По условию 2 х = 30°, или 2 = 30° + x.

Подставим в равенство (1) значения углов 1 и 2, получим 
х + 30° + х = 180°, 2х = 150°,

Решая это уравнение, получим х = …°, т. е. 

1 = 75°, a 2 = 180° 75° = …°.

Ответ: 1 = 75°, a 2 = 105°.

Пример 2. Две параллельные прямые, одна из которых лежит в плоскости α, пересечены третьей. Известно, что сумма двух внутренних накрест лежащих углов равна 150°. Чему равны эти углы и остальные шесть?hello_html_2ed53250.png

Дано: а || α, bα, а || b, с - секущая, l + 2 = 150°,

Найти: 1, 2, 3, 4, 5, 6, 7, 8.

Решение: Углы 1 и 2 внутренние накрест лежащие, следовательно, они равны.

Сумма этих углов по условию задачи равна 150°, тогда 1 = 2 = 150° : 2 = ...

Найдем остальные углы . 1 = 3 = 75° и 2 = 7 = 75° (вертикальные). Углы 4 и 5, 6 и 8 равны как вертикальные, a 5 = 6 как внутренние накрест лежащие. Все перечисленные углы 4, 5, 6 и 8 равны между собой , так как 4 + 3 = 180°,

то 4 = 180° 3 = 180 ° – 75 ° = ...

Получили четыре угла по 75°, четыре угла по 105°.

Ответ: 1 = 2 = 3 = 7 = 75°, 4 = 5 = 6 = 8 = 105°.hello_html_m40fa69e3.jpg

Пример 3. Дано: а || α, bα, а || b, с - секущая, 1 = 150°,

Найти: 2, 3, 4, 5, 6, 7, 8.

Решение: 3 = 1 = 150°(верт.), 3 = 5 = 150°(н.леж.),

5 = 7 = 150°(верт.), 1 + 2 = 180°(смежные),

2 = 180° 1 = 180° 150° = …°,

2 = 4 = 30°(верт.), 4 = 6 = 30°(н.леж.), 6 = 8 = 30°(верт.).

Ответ: 3 = 5 = 7 = 150°, 2 = 4 = 6 = 8 = 30°.

Пример 4. Дано: а || α, bα, а || b, с - секущая, 4 = 70°,

Найти: 1, 2, 3, , 5, 6, 7, 8.

Решение: 2 = 4 = …°(верт.), 4 = 6 = 70°(н.леж.),

6 = 8 = …°(верт.), 4 + 3 = 180°(смежные),

3 = 180° 4 = 180° 70° = …°,

3 = 1 = 110°(верт.), 3 = 5 = …°(н.леж.), 5 = 7 = …°(верт.).

Ответ: 1 = 3 = 5 = 7 = 110°, 2 = 6 = 8 = 70°.

Пример 5.Плоскости α и β пересекаются по прямой АВ. Прямая а || α, а || β. Докажите, что а || АВ.

Док-во: Через точку А проведем АМ || α. Так как а || α, а || β, то АМ α, АМ β. Таким образом, αβ =АМ, т.е. она совпадает с АВ. Следовательно, АВ || а.

Пример 6. Дано: АВСD- прямоугольник, М ( АВСD), (СВМ) = α. Докажите, что АD || α.

Док-во: АD || ВС, ВС α. Следовательно, АD || α.

Пример 7. Дано: AC || BD, AC ∩ α = A; BD ∩ α = B. AC = 8 cm, BD = 6 см, AB = 4 см (рис.). Доказать: CD ∩ α = E.Найти: BE. 

Решение: 1) Проведем плоскость (ACDB), если CD || АВ, то ACDB - параллелограмм, то есть АС = BD, но это противоречит условию, значит, CDAB = Е.hello_html_4fbb0191.jpg

Рассмотрим ΔАСЕ и ΔBDE. CAE = DBE, АСЕ = BDE - как соответственные при параллельных прямых, значит,

ΔEDB ~ ΔЕСА (по 3 углам) следовательно,

= , то есть ,  

BE = 12 (см).

Ответ: BE = 12 см.

Пример 8. Через основание AD трапеции ABCD проведена плоскость α. ВС α. Докажите, что прямая, проходящая через середины сторон АВ и CD, параллельна плоскости α. рис.

hello_html_m20ed3981.jpg

Дано: ABCD - трапеция; AD α, СВ α; АК = КВ, CN = ND (рис.).

Доказать: KN || α.

Доказательство:1. KN - средняя линия трапеции, значит KN || AD.

2. KN || AD , AD α, KN || α (по теореме о параллельности прямой и плоскости).

С)1)Построить таблицу:

hello_html_m7f53789a.png

2)Приведите примеры взаимного расположения прямых в пространстве из окружающего мира

Параллельные

Пересекающиеся

Скрещивающиеся

Лампы дневного света

циркуль

Башенный кран

Батареи отопления

Перекрёсток дорог

Вертолёт, самолёт

Ножки стола

стрелки часов

антенна

.

.

.

.

.

.


3) Тест «Прямые в пространстве. Параллельность прямых, прямой и плоскости»

1.Прямая а, параллельная прямой b, пересекает плоскость α. Прямая с параллельна прямой b, тогда:

а) прямые а и с пересекаются; б) прямая с лежит в плоскости α;

в) прямые а ис скрещиваются; г) прямые а и с параллельны.

2. Каким может быть взаимное расположение прямых а и b, если через прямую а можно провести плоскость, параллельную прямой b?

а) скрещиваются или пересекаются;

б) скрещиваются или параллельны;                      

в) только скрещиваются;

г) только параллельны.

3. Прямые а  и  в лежат в параллельных плоскостях, следовательно эти прямые                             а)скрещиваются или пересекаются; б) скрещиваются или параллельны;                    

в) только скрещиваются; г) только параллельны.

4. Каким может быть взаимное расположение двух прямых, если обе они параллельны одной плоскости?

а) только параллельны; б) все случаи взаимного расположения;

в) только скрещиваются; г) только пересекаются.

5. Прямая а параллельна плоскости α. Какое из следующих утверждений верно?

а) Прямая а параллельна любой прямой, лежащей в плоскости α;

б) прямая а не пересекает ни одну прямую, лежащую в плоскости α; 

в) прямая а скрещивается со всеми прямыми плоскости α;

г) прямая а имеет общую точку с плоскостью  .

4) Задание - отгадайте зашифрованное слово - две прямые, находящиеся в разных плоскостях.

Вопросы:

1. Раздел геометрии, изучающий свойства фигур в пространстве (12 букв).

2.Утверждение, не требующее доказательства.

3. Простейшая фигура планиметрии и стереометрии (6 букв).

4. Раздел геометрии, изучающий свойства фигур на плоскости (11 букв).

5. Защитное приспособление воина в виде круга, овала, прямоугольника.

6. Теорема, задающая свойства предметов.

7. Направленный отрезок (6 букв).

8. Планиметрия - плоскость, стереометрия -…

9. Женская одежда в форме трапеции (4 буквы).

10. Точка, принадлежащая обеим прямым.

11. Какую форму имеют гробницы фараонов в Египте? (8 букв)

12. Какую форму имеет кирпич? (14 букв)

13. Одна из основных фигур стереометрии.

14. Она может быть прямой, кривой, ломаной.


hello_html_m30062b56.pnghello_html_m4f96c18b.jpg

2) Решите задачи(по примерам):

А)

  1. АВСDА1В1С1D1- параллелепипед.

Найти точки пересечения прямых и плоскостей.

а) DС ∩ D1D = ? , б) С1В1 ∩ А1В1 = ?,

в) (СС1 В1В) ∩ (А1В1С1D1) = ? , г) ( А А1 D1D ) ∩ ( DD1С1С ) = ?

  1. Определить взаимное расположение прямых:hello_html_33135d75.jpg

а) АС и А1С1, б) С1D и DВ, в) DВ1 и АС ,

г) ВА и В1А1 , д) В1D и А1В1, е) В1D и АС .

  1. а) Докажите, что прямая АВ параллельна плоскости α, если АВСD параллелограмм, (А1В1 С D )= α и

А1В1 С D-трапеция.

б) Докажите, что прямая МК параллельна плоскости α, в которой лежит основание АВ треугольника АВС ,
а МК- средняя линия треугольника АВС .
hello_html_4d3b9131.jpg

в) В параллелограмме АОВК сторона ОВ параллельна прямой m , а АО и m –скрещивающиеся прямые. Найти угол между скрещивающимися прямыми , если один из углов параллелограмма равен 117°.

  1. Точка С лежит на отрезке АВ. Через точку А проведена плоскость α, а через точки В и С- параллельные прямые , пересекающие эту плоскость соответственно в точках В1 и С1. hello_html_bd13706.jpg

Найти длину отрезка СС1 ,если

а) ВВ1 = 8 см; б) АС : СВ = 3 : 2 и ВВ1 = 30 см.

В)

  1. Дано: а || α, bα, а || b, с - секущая, 2 1= 40°,Найти: 1, 2.

  2. Дано: а || α, bα, а || b, с - секущая, l + 2 = 150°,Найти: 1, 2, 3, 4, 5, 6, 7, 8.

  3. Дано: а || α, bα, а || b, с - секущая, 1 = 145°,Найти: 2, 3, 4, 5, 6, 7, 8.

  4. Дано: а || α, bα, а || b, с - секущая, 4 = 50°,Найти: 1, 2, 3, , 5, 6, 7, 8.

  5. Плоскости α и β пересекаются по прямой МС. Прямая а || α, а || β. Докажите, что а || МС.

  6. Дано: АВСD- прямоугольник, К ( АВСD), (СВК) = α. Докажите, что АD || α.

  7. Дано: AC || BD, AC ∩ α = A; BD ∩ α = B. AC = 10 см, BD = 8 см, AB = 2 см.

Доказать: CD ∩ α = E.Найти: BEhello_html_m41773e40.jpg

  1. Дано: ABCD - трапеция; AD α, АЕ = ЕВ, CF = FD (рис. ). Доказать: EF || α.

3) Решите задачи:

А)

  1. Дано: ΔABC, AC α, AD = DB, BE = EC. Доказать: DE || α. (рис.) рис.  hello_html_m7f9b8754.jpg

  2. Дан ΔВСЕ. Плоскость, параллельная прямой СЕ, пересекает BE в точке Е1, а ВС - в точке С1. Найдите ВС1, если С1Е1 : СЕ = 3 : 5, ВС = 30 см.

  3. Дано: А, В, С, D; В (ACD). Е, F, М, К- середины сторон АВ, ВС, CD, AD; AC = 8 см, BD = 14 см. Доказать: EFMK - параллелограмм. Найти: P(EFMK).  

  4. Дано: ΔАВК, М (АВК); E.D- точки пересечения медиан ΔМВК и ΔАВМ; АК = 24 см.

Доказать: ADEK - трапеция. Найти: DE

  1. Дано: AC || BD, AC ∩ α = A; BD ∩ α = B. AC = 12 см, BD = 10 см, AB = 3 см.

Доказать: CD ∩ α = E.Найти: BEhello_html_m4c348d2f.png

В)

1)Дана пирамида ABCS

Укажите:

1.Прямые, которые лежат в плоскости BSC

2. Прямые, пересекающие плоскость АВС

А С

2) Дан куб ABCDA1B1C1D1

  1. Рёбра, которые лежат на прямых, параллельных hello_html_m7b6c3827.png

ребру АА1

  1. Рёбра, которые лежат на прямых, пересекающих

ребро АА1

  1. Прямые, которые скрещиваются с прямой АА1

3) Дана пирамида ABCD Укажите: hello_html_m21257e9f.png

1.плоскости, в которых лежат прямые РЕ, МК, DB, АВ, ЕС;

2.точки пересечения прямой DK с плоскостью ABC, прямой СЕ с плоскостью ADB;

3. точки, лежащие в плоскостях ADB и DBC;

4.прямые, по которым пересекаются плоскости ABC и DCB, ABD и CDA, PDC и ABC.


С)

1.Дан куб ABCDA1B1C1D1. Точка М– середина ребра B1C1,

 N– серединаC1D1,K– середина DC,О– точка пересечения диагоналей основания ABCD.

Укажите взаимное расположение между следующими прямыми:

а) АA1 и 1; б)A1C1 и B1D1; в)A1C1 и C1D1; г) A1М и CC1; д)A1D и DC1; е)A1C1 и BD;

ё) A1C и АС; ж)A1B и D1С; з)A1C и ВB1; и) A1D и АВ; й)A1М и ВС; к)A1М и ВК;

л) C1К и B1N; м)C1О и AB1; н)A1О и B1D.

2. Дан тетраэдр ABCD. Точка К– середина ребра AD,L– середина DB,М– середина АС,

N– середина ВС. Определите взаимное расположение прямых и плоскостей.

а) DB и AMN; б)MN и ABC; в) КС и DMN;

г) MN и ABD; д)KL и DMN; е)LN и KML;

ё) CL и ADN; ж)LN и DMK.

3.Точка М не лежит в плоскости ромба ABCD. На отрезке ВМ выбрана точка F тaк,

что MF: FB = 1 : 3.

а)    Постройте точку К - точку пересечения прямой МС с плоскостью AFD.

б)    Найдите FK, если AD = 16 см.

4. 1)Пользуясь данным рисунком, назовите: а) четыре точки, лежащие в плоскости SAB; б) плоскость, в которой лежит прямая MN; в) прямую, по которой пересекаются плоскостиSAC и SBC.hello_html_m79d99f9a.jpg

2) Точка С - общая точка плоскости α и β. Прямая проходит через точку С. Верно ли, что плоскости аир пересекаются по прямой с? Ответ объясните.

3) Через прямую а и точку А можно провести две различные плоскости. Каково взаимное расположение прямой а и точки А? Ответ объясните.

4) Пользуясь данным рисунком, назовите: а) четыре точки, лежащие в плоскости ABC; б) плоскость, в которой лежит прямая KN; в) прямую, по которой пересекаются плоскости SAC и САВ.

5) Плоскости α и β имеют три общие точки. Верно ли, что эти плоскости совпадают? Ответ объясните.

6) Через А, В и С можно провести две различные плоскости. Каково взаимное расположение точек А, В и С? Ответ объясните. 

5.Дан прямоугольник АВСД, О - точка пересечения его диагоналей. Известно, что точки А, В, О лежат в плоскости α. Докажите, что точки С и Д также лежат в плоскости α. Вычислите площадь прямоугольника, если АС = 8 (см), AOB = 60°.

6.  Прямые а и b пересекаются в точке О, А  α, В  b, Р  АВ. Докажите, что прямые а и b и точка Р лежат в одной плоскости.

D)

  1. Дано: а || α, bα, а || b, с - секущая, 2 1= 50°,Найти: 1, 2.

  2. Две параллельные прямые, одна из которых лежит в плоскости α, пересечены третьей. Известно, что сумма двух внутренних накрест лежащих углов равна 110°. Чему равны эти углы и остальные шесть?

  3. Дано: а || α, bα, а || b, с - секущая, 1 = 135°,Найти: 2, 3, 4, 5, 6, 7, 8.

  4. Дано: а || α, bα, а || b, с - секущая, 4 = 20°,Найти: 1, 2, 3, , 5, 6, 7, 8.

  5. Плоскости α и β пересекаются по прямой МК. Прямая а || α, а || β. Докажите, что а || МК.

  6. Дано: ОРТЕ- прямоугольник, К ( ОРТЕ), (РТК) = α. Докажите, что ОЕ || α.

  7. Дано: АВСD- трапеция, М ( АВСD), (СВМ) = α. Докажите, что АD || α.

  8. Дано: АВСD- трапеция, АВ || α, С α, MN – средняя линия трапеции. Докажите, что MN || α.

  9. Дано: АВСD- параллелограмм, К ( АВСD), (АВК) = α. Докажите, что СD || α.hello_html_m477473c3.png

  10. Дано: А α, В α, С α, М – середина АС, К – середина ВС. Докажите, что MК || α.

  11. hello_html_me6b84c4.png


  1. Плоскость α проходит через середины боковых сторон АВ и CD трапеции ABCD- точки M и N. а) Докажите, что AD׀׀α.
    б) Найдите ВС, если
    AD=10 см, MN=8см.

  2. Плоскость α проходит через основание АD трапеции ABCD. Точки M и Nсередины боковых сторон трапеции. а) Докажите, что MN׀׀α.

б) Найдите AD, если BC = 4 см, MN = 6см.hello_html_m3e0c4470.jpg

  1. По готовому рисунку: а) докажите, что: KMEF;
    б) найдите
    KM , если EF=8 см.

  2. Дан ΔВСЕ. Плоскость, параллельная прямой СЕ, пересекает BE
    в точке Е1,а ВС - в точке С1. Найдите ВС1, если С1Е1 : СЕ = 3 : 8, ВС = 28 см.

  3. Через основание AD трапеции ABCD проведена плоскость α. ВС α. Докажите, что прямая, проходящая через середины сторон АВ и CD,
    параллельна плоскости α.
    hello_html_m1a1d6694.jpg

  4. Дано: А, В, С, D; В (ACD). Е, F, М, К- середины сторон АВ, ВС, CD, AD;
    AC = 6 см, BD = 8 см. Доказать: EFMK - параллелограмм. Найти: P(EFMK).  

  5. Дано: ΔАВК, М (АВК); E.D- точки пересечения медиан ΔМВК и ΔАВМ;
    АК = 14 см. Доказать: ADEK - трапеция. Найти: DE

 ПЗ № 2. Перпендикуляр и наклонная к плоскости. Угол между прямой и плоскостью. Теоремы о взаимном расположении прямой и плоскости. Теорема о трех перпендикулярах.

Задание:

1) A)а) Записать по рисунку:

  • какой отрезок является перпендикуляр, наклонная, проекция наклонной,

  • угол между наклонной и плоскостью α.

АС - …, АВ - …, СВ – …, АВ2 = ВС2 + АС2.

- угол между наклонной и плоскостью α.

б)Перепишите и заполните пропуски:

Пример 1. Из точки, не принадлежащей данной плоскости , проведены к ней
две наклонные, равные 10см и 18см. Сумма длин их проекций на

плоскость равна 16см. Найти проекцию каждой наклонной.(рис.1)

Дано: ОС - перпендикуляр, АС и ВС - наклонные, АО и ОВ – их проекции, рис.1

АС = 10 см, СВ = 18 см, АО + ОВ = 16 см,

Найти: АО, ОВ

Решение: АС = 10, СВ = 18, АО + ОВ = 16, АО = х, ОВ = 16 х,

АС2 АО2 = ВС2 – ОВ2 , 102 х2 = 182 – (16 х)2, 100 х2 = 324 – 256 + 32 х х2 ,

32 х = 32, х = … , АО = 1, ОВ = 16 – 1 = .... Ответ: 1 и 15 см.

Пример 2. Из точки к плоскости проведены две наклонные. Одна из них длиной 12см наклонена к плоскости под углом 60°, проекция другой на эту плоскость равна 6 см.

Найти длину этой наклонной.

Дано: ОС - перпендикуляр, АС и ВС - наклонные, АО и ОВ – их проекции,

СА = 12 см , САО = 60°, ОВ = 6 см ,

Найти: СВ

Решение: Δ АОС- прямоугольный, АСО = 90 ° 60 ° = 30°, АО = СА : 2 = 12: 2 = … ,

СО2 = СА2 –АО2 = 122 – 62 = 144 – 36 = … ,

СВ2 = СО2 + ОВ2 = 108 + (6 )2 = 108 + 36 6 = 108 + 216 = … , СВ = … см. Ответ: 18 см.

Пример 3. Из точки С к данной плоскости проведены перпендикуляр СО = 6см и две наклонные. Каждая из наклонных образует с плоскостью угол 60°. Угол между наклонными 120°. Найти расстояние между основаниями наклонных.

Дано: ОС - перпендикуляр, АС и ВС - наклонные, АО и ОВ – их проекции,

СО = 6см, САО = СВО = 60°, АСВ = 120°,

Найти: АВ
Решение: sin САО = СО : АС, АС = ВС = СО : sin САО = 6: sin60 ° = 6 : = 12 : = 4 ,

Δ АВС – равнобедренный, АВ2 = АС2 + ВС2 – 2АС ВС cos АСВ =

= (4)2 + (4)2 – 24 cos 120° = 16 3 + 16 3 - 216 3( – ) = 48 + 48 + 48 = … ,

АВ = … см. Ответ: АВ = 12 см.

Пример 4. Из точки С к данной плоскости проведены перпендикуляр СО и две наклонные СВ и АС. ОВ= 4,САО = 30°, СВО = 60°, а угол между наклонными 90°. Найти расстояние между основаниями наклонных.

Дано: ОС - перпендикуляр, АС и ВС - наклонные, АО и ОВ – их проекции,

ОВ= 4,САО = 30°, СВО = 60°, АСВ = 90°,

Найти: АВ

Решение: ΔСОВ – прямоугольный, СВО = 60°, ОСВ = 90 ° - 60 ° = 30 °,

ВС= 2 ОВ = 24 = … , СО2 = ВС2 – ОВ2 = 82 – 42 = 64 – 16 = … , СО = = 4,

АС = 2 СО = 24 = … , ΔАСВ - прямоугольный, АВ2 = АС2 + ВС2 = (8)2 + 82 =

= 64 3 + 64 = … , АВ = … см. Ответ: АВ = 16 см.
Пример 5. Диагонали квадрата АВСD пересекаются в точке О.


Из точки О проведён к плоскости квадрата перпендикуляр ОМ. Найти расстояние от точки М до

стороны ВС, если AD = 6см, ОМ = 4см. (рис.2)

Дано: АВСD - квадрат, ОМ - перпендикуляр,
О - точка пересечения диагоналей квадрата,

МК - расстояние от точки М до стороны ВС, AD = 6см, ОМ = 4см.

Найти: МК

Решение: ОК = АВ : 2 = AD : 2 = 6 : 2 = … , ΔМОК - прямоугольный, Рис.2

МК2 = ОМ2 + ОК2 = 42 + 32 = 16 + 9 = … , МК = ... Ответ: МК = 5 см.
Пример 6. Прямые АВ, АС и AD попарно перпендикулярны . Найдите отрезок CD, если: АВ = 3 см, ВС = 7 см, AD = 1,5 см;

Дано: АВ, АС и AD попарно перпендикулярны, АВ = 3 см, ВС = 7 см, AD = 1,5 см; Найти: CD

Решение: Δ САВ – прямоугольный, АС2 = СВ2 – АВ2, АС2 = 72 – 32 = 49 – 9 = … ,

Δ САD – прямоугольный, СD2 = АС2 + АD2, СD2 = 40 + 1,52 = 40 + 2,25 = … ,

СD = … см. Ответ: СD = 6,5 см.hello_html_2b8f3f4.png

Пример 7.

hello_html_2b8f3f4.png

Решение:

hello_html_2b8f3f4.png

Пример 8. Дан параллелепипед АВСDА1В1С1D1 , у которого основание квадрат. hello_html_75ed7ad2.jpg

Докажите, что а) СD В1С1 , б) С1D1 АD .

Доказательство: а) СD || A1B1, A1B1 В1С1 СD В1С1 ( по лемме),

б) С1D1 || ВС , ВС АD С1D1 АD ( по лемме) .

B) Тест на опознание.

  1. Перпендикуляр от точки до прямой – это …

  2. Перпендикуляр к плоскости – это …

  3. Если прямая перпендикулярна к прямой плоскости, то …

  4. Чтобы прямая была перпендикулярна к плоскости, необходимо, чтобы она была …

  5. Наклонная к плоскости – это прямая …

  6. Если наклонные, проведенные к плоскости из одной точки, равны, то равны и …

  7. Если проекции наклонных, проведенных к плоскости из одной точки, равны, то равны и …

  8. Если прямая плоскости перпендикулярна к наклонной., то …

  9. Если прямая перпендикулярна к проекции наклонной, то …

  10. Если наклонная перпендикулярна к прямой плоскости, то …

  11. Если проекция наклонной перпендикулярна к прямой плоскости, то …

Какие из формулировок относятся к теореме о трех перпендикулярах?

  1. Если прямая, пересекающая плоскость, перпендикулярна к плоскости, то она перпендикулярна ко всем ее прямым.

  2. Если прямая плоскости перпендикулярна к наклонной, то она перпендикулярна и ее проекции.

  3. Если прямая, пересекающая плоскость, перпендикулярна двум пересекающимся прямым плоскости, то она перпендикулярна и самой плоскости.

  4. Прямая плоскости, перпендикулярная проекции наклонной, перпендикулярна и самой наклонной.

C) Тест на различение.

  1. Какие из прямых MA, MD или MC перпендикулярны AD?

  2. Какие из прямых AM, NM или DM перпендикулярны AD?

  3. Какие из прямых MA или MC перпендикулярны AC?

  4. Какие из прямых MA, MN или MC перпендикулярны AC?

1. 2.















3. 4.

Тест на различение.


Какие из прямых перпендикулярны к прямой AD?

  1. Какие из прямых: AB1; DB1; CB1 перпендикулярны к AD или DC?

  2. Какие из прямых: AB1; DB1; CB1 перпендикулярны к AD или DC? :

  3. Какие из прямых: AB1 или DC1 перпендикулярны к AD

  4. Как построить перпендикуляр из точки пространства к прямой плоскости?










2)Решить задачи ( по примерам):

  1. Из точки, не принадлежащей данной плоскости, проведены к ней две наклонные,
    равные 20 см и 36 см. Сумма длин их проекций на плоскость равна 32 см.
    Найти проекцию каждой наклонной.

  2. Из точки к плоскости проведены две наклонные. Одна из них длиной 24 см наклонена к плоскости под углом 60°, проекция другой на эту плоскость равна 12 см. Найти длину этой наклонной.

  3. Из точки С к данной плоскости проведены перпендикуляр СО = 12 см и две наклонные. Каждая из наклонных образует с плоскостью угол 60°. Угол между наклонными 120°.
    Найти расстояние между основаниями наклонных.

  4. Из точки С к данной плоскости проведены перпендикуляр СО и две наклонные СВ и АС.
    ОВ= 8,
    САО = 30°, СВО = 60°, а угол между наклонными 90°. Найти расстояние между основаниями наклонных.

  5. Диагонали квадрата АВСD пересекаются в точке О. Из точки О проведён к плоскости квадрата перпендикуляр ОМ. Найти расстояние от точки М до стороны ВС, если AD = 12 см, ОМ = 8 см.

  6. Прямые АВ, АС и AD попарно перпендикулярны.
    Найдите отрезок CD, если: АВ = 6 см, ВС = 14 см, AD = 3 см;

  7. Дан параллелепипед АВСDА1В1С1D1 , у которого основание квадрат.
    Докажите, что
    а) С1D1 ВС, б) СD А1D1 .

3)Решить задачи :

  1. Дано: АС - перпендикуляр, АВ - наклонная,
    а)
    АВ = 10 см, ВС = 6 см, АС = ?, б) АС = 12 см, ВС = 5 см, АВ = ? (Указание:АВ2 = ВС2 + АС2 )

  2. Дано: Δ АВС – равнобедренный, АК(АВС), АК = 12 см, АВ = АС = 5 см, ВС = 6 см,
    КМ
    ВС. Найти: КМ, АМ.

(Указание: АВ = АС => КВ = КС => Δ СКВ – равнобедренный, КМ ВС => ВМ- медиана,

ВМ = МС = ВС : 2, КС2 = АК2 + АС2 , КМ2 = КС2 - МС2 , АМ2 = АС2 - МС2 )

  1. Дано: АО - перпендикуляр, АВ и АС - наклонные, АВ = АС, ОАВ = ВАС = 60°,
    АО = 2,5 см.
    Найти: ВС. (Указание: Δ ВАС – равносторонний, ВС = АВ = АС = 2АО)

  2. Телефонная проволока длиной 15 м протянута от телефонного столба, где она прикреплена на высоте 8 м от поверхности земли, к дому, где ее прикрепили на высоте 20 м. Найдите расстояние между домом и столбом, предполагая, что проволока не провисает.

Дано: AB = 15 м, АС = 8 м, BD = 20 м, Найти: CD.

(Указание: Δ BKА – прямоугольный, АK2 = AB2 - BK 2)

  1. Дан куб АВСDА1В1С1D1 . Найдите следующие двугранные углы: а) АВ В1С , б) АDD1В,
    в) А
    1ВВ1К, где К- середина А1D1.

  2. Из вершины равностороннего треугольника АВС проведен перпендикуляр АК к плоскости треугольника. Найдите длину АК, если ВС = 3 см, КС = 3 см.

  3. Найдите тангенс угла между диагональю куба и плоскостью одной из его граней.

  4. Дано: ABCD квадрат; AM - прямая; АМ  (ABCD); АС  BD = О. Доказать: a) BD  (АМО); б) МО  BD.

  5. Прямые АВ и CD перпендикулярны некоторой плоскости и пересекают ее в точках В и D соответственно. Найдите AС, если АВ = 9, CD = 15, BD = 8.

  6. Отрезок МН пересекает некоторую плоскость в точке К. Через концы отрезка проведены прямые HP и ME, перпендикулярные плоскости и пересекающие ее в точках Р и Е. Найдите РЕ, если HP = 4 см, НК = 5 см, ME = 12 см.

  7. Треугольник ABC правильный, точка О - его центр. Прямая ОМ перпендикулярна плоскости AВС. Докажите, что МА = MB = МС. Найдите МА,если АВ = 6 см, МО = 2 см.hello_html_m754188c5.jpg

  8. ABCD - квадрат . АЕ - перпендикулярно плоскости квадрата, К  BE. Найти: (ВС; АК).

  9. ABCD прямоугольник. Отрезок АЕ перпендикулярен к плоскости ABC. ЕВ = 15, ЕС = 24, ED = 20. Докажите, что треугольник EDC прямоугольный, и найдите АЕ.

  10. Точка А принадлежит окружности, АК - перпендикуляр к ее плоскости, АК = 1 см, АВ - диаметр, ВС — хорда окружности, составляющая с АВ угол 45°. Радиус окружности равен 2 см. Докажите, что треугольник КСВ прямоугольный, и найдите КС.

  11. Дано: α; АВ - отрезок;AB α = O,AD α; BC α; AD α = D,BC α = C, AD = 6 см, BC = 2 см, OC = 1,5 см. Найти: АВ.

  12. Прямые АВ, АС и AD попарно перпендикулярны . Найдите отрезок CD, если: АВ = 6 см, ВС = 14 см, AD = 3 см;

  13. Через точки А и В проведены прямые, перпендикулярные плоскости α, пересекающие ее в точках С и D соответственно. Найдите расстояние между точками А и В, если АС = 8 см, BD = 20 см, CD = 5см и отрезок АВ не пересекает плоскость α.

  14. Телефонная проволока длиной 26 м протянута от телефонного столба, где она прикреплена на высоте 6 м от поверхности земли, к дому, где ее прикрепили на высоте 30 м. Найдите расстояние между домом и столбом, предполагая, что проволока не провисает.

  15. К плоскости треугольника из центра, вписанной в него окружности радиуса 1 м восставлен перпендикуляр длиной 2,4 м. Найдите расстояние от конца этого перпендикуляра до сторон треугольника.

  16. Дано: ΔАВС; АВ = АС = ВС; CD  (ABC); AM = MB, DM = 17, CD = 8. Найти: SΔADB.


ПЗ № 3. Признаки и свойства параллельных и перпендикулярных плоскостей. Расстояние от точки до плоскости, от прямой до плоскости, расстояние между плоскостями, между скрещивающимися прямыми, между произвольными фигурами в пространстве.

Задание:

1)Перепишите и заполните пропуски:

А)Пример 1. Два отрезка длин а и b упираются концами в две параллельные плоскости. Проекция первого отрезка (длины а) на плоскость равна с. Найдите проекцию второго отрезка, если а = 17 , b = 10, с = 15 см.

Дано: α || β, а = 17 , b = 10, с = 15 см. Найти: х

Решение:

а2 – с2 = b2 – х2, х2 = b2а2 + с2 , х2 = 102 – 172 + 152 =

= 100 – 289 + 225 = …, х = … см.
Ответ: х = 6 см.

Пример 2.

Две параллельные плоскости расстояние между

которыми 2 дм, пересечены прямой, составляющей с каждой из

плоскости угол в 300. Найти длину отрезка этой прямой, заключенной

между плоскостями.

Дано: α || β, АВα = А, АВβ = В, АВС = 30°, АС = 2 дм.

Найти: АВ

Решение: Δ АСВ – прямоугольный, АВС = 30°, АС = 2 дм.

АВ = 2 АС = 2 2 = … дм.
Ответ: АB = 4 дм.

Пример 3. Расстояние между параллельными плоскостями равно 8 см. Отрезок прямой длина которого 17 см расположен между ними так, что его конец принадлежит плоскости. Найти проекцию этого отрезка на другую плоскость.

Дано: α || β, АВα = А, АВβ = В, АВ = 17 см, АС = 8 см.

Найти: ВС

Решение: Δ АСВ – прямоугольный, ВС2 = АВ2 – АС2 = 172 – 82 = 289 – 64 = …, ВС = … см.

Ответ: BС = 15 см.

Пример 4. На параллельных плоскостях α и β, выбрано по паре точек А12 и В12 соответственно так, что прямые А1В1 и А2В2 пересекаются в точке S Вычислите SА1 и SВ2, если А1В1= 6см;
2 = 2,5см; SВ2 : SА2 = 3 : 1 . S

Дано: α || β, А1 А2В1 В2 = S, А1, А2 α, В12 β,

А1В1= 6см; SА2 = 2,5см; SВ2 : SА2 = 3 : 1

Найти: 1, SВ2

Решение: Δ SА1 А2 ~ Δ SВ1В2 , (α || β), SВ2 : SА2 = 3 : 1, SА2 = 2,5см,

2 = 3 2,5 = … см. 1 : SА1 = 3 : 1, А1В1= 6см, SА1 = х ,

( х + 6 ) : х = 3 : 1, 3х = х + 6 , 2х = 6, х = …, SА1 = … см.

Ответ:1 = 3 см, SВ2 = 7,5 см .

Пример 5.

Дано: α || β, а α, bβ, а || b, с - секущая, 1 = 150°,

Найти: 2, 3, 4, 5, 6, 7, 8.

Решение: 3 = 1 = 150°(верт.), 3 = 5 = 150°(н.леж.),hello_html_m40fa69e3.jpg

5 = 7 = 150°(верт.), 1 + 2 = 180°(смежные),

2 = 180° – 1 = 180° – 150° = …°,

2 = 4 = 30°(верт.), 4 = 6 = …°(н.леж.), 6 = 8 = …°(верт.).

Ответ: 3 = 5 = 7 = 150°, 2 = 4 = 6 = 8 = 30°.

В)hello_html_41ca87ca.jpg

Пример 1. Из точек А и В, лежащих в двух перпендикулярных плоскостях, опущены перпендикуляры АС и ВD на прямую пересечения плоскостей. Найдите длину отрезка АВ если:

а) АС = 6 м, ВD = 7 м, СD = 6 м, б) АD = ВС = 5 м, СD = 1 м.

Решение: а) Пусть плоскости α и β перпендикулярны. СD – прямая пересечения плоскостей , тогда АС СВ и ВD АD. Тогда в Δ АСВ: АВ2 = АС2 + ВС2, но из Δ СDВ следует ,что: ВС2 = СD2 + ВD2 , так что АВ2 = АС2 + СD2 + ВD2.

АВ2 = 62 + 72 + 62 = 36 + 49 + 36 = …, АВ = …

б) АВ2 = АС2 + ВС2, но из Δ СDА следует ,что: АС2 = АD2 СD2 ,

так что АВ2 = АD2 СD2 + ВС2. АВ2 = 52 12 + 52 = 251 + 25 = …, АВ = …

Ответ: а) 11 м, б) 7 м.hello_html_552bcaf5.jpg

Пример 2. Точка А находится на расстоянии а = 24 см и b = 10 см от двух перпендикулярных плоскостей α и β. Найдите расстояние от этой точки до прямой пересечения плоскостей.

Решение: Пусть α β и α β = с. Проведем перпендикуляры АВ, АD, АС. Тогда четырехугольник АВСD – прямоугольник. АС2 = а2 + b2 ,

АС –искомое расстояние. ВС - проекция АС на плоскость α, поэтому по теореме о 3 – х перпендикулярах ВС с, ВС β. Так как АD β, то по теореме АD||ВС, а, значит, АD и ВС лежат в одной плоскости.

Итак , АС2 = 242 + 102 = 576 + 100 = … , АС = …hello_html_mf971f4e.jpg

Ответ: АС = 26 см.

Пример 3. Плоскости α и β перпендикулярны. В плоскости α взята точка А, расстояние от которой до прямой с ( линия пересечения плоскостей ) равно 0,5 м. В плоскости β проведена прямая b, параллельная прямой с и отстоящая от нее на 1,2 м. Найдите расстояние от точки А до прямой b.

Решение: Пусть α β , b || с, ВС = 1, АВ = 0,5м , где АВ с и ВС b.

Тогда по теореме о 3 – х перпендикулярах АС b. Так что

АС – искомое расстояние и АС2 = АВ2 + ВС2 = 1,22 + 0,52 = 1,44 + 0 ,25 = …, АС = …

Ответ: АС = 1,3 м.

Пример 4. Перпендикулярные плоскости α и β пересекаются по прямой с. Плоскости α проведена прямая а|| с, в плоскости β – прямая b || с. Найдите расстояние между прямыми а и b , если расстояние между прямыми а и с равно 1,5 м, а между прямыми b и с – 0,8 м. hello_html_m209580fa.jpg

Решение: Возьмем в плоскости α точку А на прямой а. По теореме о 3 – х параллельных прямых получаем, что а || b (так как а || с, b ||с). Проведем

АС с и СВ b. Тогда по теореме о 3 – х перпендикулярах АВ b.

Так что АВ – искомое расстояние и АВ СВ, так как α β
( по условию), из прямоугольного треугольника АВС по теореме Пифагора имеем: АВ
2 = СВ2 + АС2 = 1,52 + 0,82 = 2,25 + 0,64 = … , АВ = …

Ответ: АВ = 1,7 м.

С) Решение теста. Методические рекомендации к выполнению теста:

  1. Прочитать вопрос, ответить на его и записать букву , под которой записан правильный ответ.

  2. Решив задачу, нужно выбрать правильный ответ и записать номер, под которым он записан.

Задание: 1 часть.

1. Расстоянием от точки до плоскости называется

а) длина перпендикуляра, опущенного из точки на эту плоскость.

б) длина перпендикуляра, проведенного из плоскости к этой точке.

в)длина перпендикуляра, проведенного из любой точки одной

плоскости ко второй плоскости, на которой лежит эта точка.

г) расстояние от этой точки до любой из точек лежащих на плоскости.

2. Расстоянием от прямой до параллельной ей плоскости называется

а) длина перпендикуляра, опущенного из произвольной точки на эту плоскость.

б) длина перпендикуляра, опущенного из произвольной точки прямой на эту плоскость.

в) расстояние от точки лежащей на прямой, до любой из точек лежащих на плоскости.

г) длина перпендикуляра, опущенного из произвольной точки плоскости на эту прямую.

3. Найдите расстояние между скрещивающимися прямыми, содержащими

диагональ куба и ребро куба, если ребро куба равно см.

а) см ,б) 1 см, в) 0,5 см, г) 2 см.

4.Если угол между двумя прямыми равен 90°, то эти прямые:

а) пересекаются, б) параллельны, в) скрещиваются, г) перпендикулярны, д) совпадают.


5.Расстояния от точки М до сторон прямоугольного треугольника АВС

(угол С равен 90°) равны. Какое из следующих утверждений верно?

а) плоскости МАВ и АВС перпендикулярны,

б) плоскости МВС и АВС перпендикулярны,

в) плоскости МАС и АВС перпендикулярны,

г) плоскости МАС и МВС перпендикулярны, д) условия в пунктах а - г неверны.

2 часть .

  1. Расстояние от некоторой точки до плоскости квадрата равно 4 см, а до каждой из его вершин – 6 см. Найдите диагональ квадрата. А) 2 см; Б) 5 см; В) 5 см; Г) другой ответ.

2. Найдите расстояние от середины отрезка АВ, пересекающего плоскость α,до плоскости α, если расстояния от точек А и В до плоскости равны соответственно 7 см и 9 см.

А) 8 см; Б) 1 см; В) 4 см; Г) другой ответ.

3.Из вершины равностороннего треугольника АВС проведен перпендикуляр АК к плоскости треугольника. Точка D – середина стороны ВС. Найдите длину АК, если ВС = см, КD = 8 см.

А) 14 см; В) 12 см; В) 7 см; Г) другой ответ.

4.Расстояние от некоторой точки до плоскости прямоугольника равно см, а до всех его вершин – 3 см. Найдите диагональ прямоугольника. А) 4 см; Б) 2 см; В) 5 см; Г) другой ответ.

5. Найдите расстояние от середины отрезка АВ, пересекающего плоскость α ,до плоскости α, если расстояния от точек А и В до плоскости равны соответственно 4 см и 10 см.

А) 7 см; Б) 3 см; В) 2 см; Г) другой ответ.

6. Расстояния от вершин А, В, С параллелограмма ABCD, не пересекающего плоскость α, до плоскости α равны соответственно 19 см, 6 см и 16 см. Найдите расстояние от вершины D до плоскости α .

А) 23 см; Б) 11 см; В) 29 см; Г) другой ответ.

D) Построить таблицу:

Расстояние от точки до плоскости — это длина перпендикуляра, опущенного из этой точки на плоскость.

Проводим KM α

 (M α). KM = ρ (K; α).

hello_html_m432887ec.gif

SO α.

Проводим KM || SO. 

Тогда KM αи KM = ρ (K; α).

hello_html_m9bde0f8.gif

Проводим через точку K плоскость β α (β пересекает α по AB).

Проводим

KM AB. 

Тогда KM α и KM = ρ (K; α).

hello_html_m7f69f547.gif

Расстояние между прямой и параллельной ей плоскостью

Расстоянием от прямой до параллельной ей плоскости называется расстояние от произвольной точки этой прямой до плоскости.

a || α, A a, ρ (a; α) = ρ (A; α).
Выбираем на прямой a произвольную точку A и находим расстояние от этой точки до плоскости α.

hello_html_41c2f592.gif

Расстояние между параллельными плоскостями

Расстоянием между двумя параллельными плоскостями называется расстояние от произвольной точки одной плоскости до второй плоскости.

β || α, B β, ρ (β; α) = ρ (B; α).
Выбираем в плоскости β произвольную точку B и находим расстояние от этой точки до плоскости α.

hello_html_78b104b6.gif

Расстояние между скрещивающимися прямыми

Общим перпендикуляром к двум скрещивающимся прямым называется отрезок с концами на этих прямых, перпендикулярный каждой из них.

Расстоянием между скрещивающимися прямыми называется длина их общего перпендикуляра. Она равна расстоянию между параллельными плоскостями, которые проходят через эти прямые.

hello_html_m19408d45.gif

AB a, AB b; ρ (a; b) = AB.
Прямые a и b — скрещивающиеся.


2)Решить задачи ( по примерам):

  1. Два отрезка длин а и b упираются концами в две параллельные плоскости. Проекция первого отрезка (длины а) на плоскость равна с. Найдите проекцию второго отрезка, если а = 13 , b = 15, с = 5 см.

  2. Две параллельные плоскости расстояние между которыми 6 дм, пересечены прямой, составляющей с каждой из плоскости угол в 300. Найти длину отрезка этой прямой, заключенной между плоскостями.

  3. Расстояние между параллельными плоскостями равно 10 см. Отрезок прямой длина которого 26 см расположен между ними так, что его конец принадлежит плоскости. Найти проекцию этого отрезка на другую плоскость.

  4. На параллельных плоскостях α и β, выбрано по паре точек А12 и В12 соответственно так, что прямые А1В1 и А2В2 пересекаются в точке S Вычислите SА1 и SВ2, если А1В1= 12см;
    2 = 4,5см; SВ2 : SА2 = 3 : 1.

  5. Дано: α || β, а α, bβ, а || b, с - секущая, 1 = 140°. Найти: 2, 3, 4, 5, 6, 7, 8.

  6. Из точек А и В, лежащих в двух перпендикулярных плоскостях, опущены перпендикуляры АС и ВD на прямую пересечения плоскостей. Найдите длину отрезка АВ если:

а) АС = 3 м, ВD = 4 м, СD = 12 м, б) АD = 4 м, ВС = 7 м, СD = 1 м.

  1. Точка А находится на расстоянии а = 17 см и b = 8 см от двух перпендикулярных плоскостей α и β. Найдите расстояние от этой точки до прямой пересечения плоскостей.

  2. Плоскости α и β перпендикулярны. В плоскости α взята точка А, расстояние от которой до прямой с ( линия пересечения плоскостей ) равно 0,9 м. В плоскости β проведена прямая b, параллельная прямой с и отстоящая от нее на 1,2 м. Найдите расстояние от точки А до прямой b.

  3. Перпендикулярные плоскости α и β пересекаются по прямой с. Плоскости α проведена прямая а|| с, в плоскости β – прямая b || с. Найдите расстояние между прямыми а и b , если расстояние между прямыми а и с равно 4,5 м, а между прямыми b и с – 2,4 м.

3)Решить задачи :

  1. Через точку O, которая находится между параллельными плоскостями α и β, проведены прямые c и d, пересекающие плоскости так, что точки A и B находятся в плоскости α, а точки C и D - в плоскости β ,AB=15 см, DO=29 см и AC=3AO.Вычислить: BD;CD.  

  2. Дан треугольник АВС. Плоскость, параллельная прямой АВ, пересекает сторону АС этого треугольника в точке А1, а сторону ВС в точке В1. Найдите длину отрезка А 1В 1,если АВ = 8 см, АА1 : А1С = 5 : 3.

  3. Сторона правильного треугольника ABC равна 4. Треугольник DBC — равнобедренный
    (DB = DC). Их плоскости взаимно перпендикулярны. Плоскость ADC составляет с плоскостью АВС угол 60°. Найдите площадь треугольника DBC.

  4. В треугольнике ABC синус угла B равен , . отрезок перпендикулярный плоскости данного треугольника. Найдите расстояние от точки A до плоскости (SBC).

  5. В треугольнике ABC сторона . отрезок перпендикулярный плоскости данного треугольника. Найдите расстояние от точки A до плоскости (QBC), если площадь треугольника BQC равна 6,5.

  6. Площадь треугольника ABC равна 3, . отрезок перпендикулярный плоскости данного треугольника. Найдите расстояние от точки A до плоскости (PBC).

  7. В единичном кубе найдите .

ПЗ № 4. Параллельное проектирование и его свойства. Теорема о площади ортогональной проекции многоугольника. Взаимное расположение пространственных фигур.

Задание:

1) Опорный конспект.

ПАРАЛЛЕЛЬНОЕ ПРОЕКТИРОВАНИЕ

    В стереометрии изучаются пространственные фигуры, однако на чертеже они изображаются в виде плоских фигур. Каким же образом следует изображать пространственную фигуру на плоскости? Обычно в геометрии для этого используется параллельное проектирование. 
    Пусть p - некоторая плоскость, 
l - пересекающая ее прямая (рис. 1). Через произвольную точку A, не принадлежащую прямой l, проведем прямую, параллельную прямой l. Точка пересечения этой прямой с плоскостью p называется параллельной проекцией точки A на плоскость p в направлении прямой l. Обозначим ее A'. Если точка A принадлежит прямой l, то параллельной проекцией на плоскость p считается точка пересечения прямой l с плоскостью p.

hello_html_5f1d4cc7.gif

   Таким образом, каждой точке A пространства сопоставляется ее проекция A' на плоскость p. Это соответствие называется параллельным проектированием на плоскость pв направлении прямой l. 
    Пусть 
Ф - некоторая фигура в пространстве. Проекции ее точек на плоскость p образуют фигуру Ф', которая называется параллельной проекцией фигуры Ф на плоскостьp в направлении прямой l. Говорят также, что фигура Ф' получена из фигуры Ф параллельным проектированием. 
    Примеры параллельных проекций дают, например, тени предметов под воздействием пучка параллельных солнечных лучей. 
Метод параллельного проецирования заключается в том, что все проецирующие лучи, проходящие через точки, будут параллельны между собой. Этот метод вытекает из метода центрального проецирования, при этом полюс должен быть удален на бесконечно большое расстояние от плоскости, на которую проецируется предмет. Ортогональный метод проецирования – метод, когда проецирующие лучи параллельны между собой и перпендикулярны к плоскости проекций. Данный метод – частный случай параллельного проецирования. 

Основные свойства параллельного и ортогонального проектирования:

  1. Проекцией точки является точка.

  2. Проекцией прямой является прямая – свойство прямолинейности.

  3. Проекцией точки, лежащей на некоторой прямой, является точка, лежащая на проекции данной прямой – свойство принадлежности.

  4. Проекциями параллельных прямых являются параллельные прямые – свойство сохранения параллельности.

  5. Отношение проекций отрезков, лежащих на параллельных прямых или на одной и той же прямой, равно отношению самих отрезков.

  6. Проекция фигуры не меняется при параллельном переносе плоскости проекций.

  7. Проекция отрезков не может быть больше самого отрезка.

  8. Теорема о прямом угле. Если одна сторона прямого угла параллельны плоскости проекции, а вторая сторона этой плоскости не перпендикулярно, то прямой угол проекцируется на эту плоскость без изкажения.

Простейшим многоугольником является треугольник. Параллельной проекцией треугольника, как следует из свойств параллельного проектирования, является треугольник или отрезок. При этом, если плоскость треугольника параллельна плоскости проектирования, то, как мы выяснили, его проекцией будет треугольник, равный исходному.

Тогда ясно, что треугольник ABC является параллельной проекцией треугольника AB1C на плоскость p в направлении прямой l.

hello_html_5aa809f6.gif

    Рассмотрим теперь параллельную проекцию правильного шестиугольника ABCDEF с центром в точке O (рис. 7). Выберем какой-нибудь треугольник, например, AOB. Его проекцией может быть треугольник A'O'B' на плоскости p (рис. 8), имеющий произвольную форму. Далее отложим O'D'=A'O' и O'E'=B'O'. Теперь из точек A' и D' проведем прямые, параллельные прямой B'O'; из точек B' и E' проведем прямые, параллельные прямой A'O'. Точки пересечения соответствующих прямых обозначим F' и C'. Шестиугольник A'B'C'D'E'F' и будет искомой проекцией правильного шестиугольника ABCDEF. 

Выясним, какая фигура является параллельной проекцией окружности. Пусть F - окружность в пространстве, F'- ее проекция на плоскость p в направлении прямой l. Если прямая l параллельна плоскости окружности или лежит в ней, то проекцией окружности является отрезок, равный диаметру окружности.  Изображение параллелепипеда строится, исходя из того, что все его грани параллелограммы и, следовательно, изображаются параллелограммами (рис. 11).

hello_html_256e21bd.gif

    При изображении куба плоскость изображений обычно выбирается параллельной одной из его граней. В этом случае две грани куба, параллельные плоскости изображений (передняя и задняя), изображаются равными квадратами. Остальные грани куба изображаются параллелограммами (рис. 12). Аналогичным образом изображается прямоугольный параллелепипед (рис. 13). 
    Для того чтобы построить изображение призмы, достаточно построить многоугольник, изображающий ее основание. Затем из вершин многоугольника провести прямые, параллельные некоторой фиксированной прямой, и отложить на них равные отрезки. Соединяя концы этих отрезков, получим многоугольник, являющийся изображением второго основания призмы (рис. 14). 
    Для того чтобы построить изображение пирамиды, достаточно построить многоугольник, изображающий ее основание. Затем выбрать какую-нибудь точку, которая будет изображать вершину пирамиды, и соединить ее с вершинами многоугольника (рис. 15). Полученные отрезки будут изображать боковые ребра пирамиды.

hello_html_m476fd703.gifhello_html_mc0b771.jpg

    Для изображения цилиндра достаточно изобразить его основания в виде двух эллипсов, получающихся друг из друга параллельным переносом, и нарисовать две образующие, соединяющие соответствующие точки этих оснований

(рис. 16). 
    Для изображения конуса достаточно изобразить его основание в виде эллипса, отметить вершину и провести через нее две образующие, являющиеся касательными к этому эллипсу (рис. 17). 

Теорема. Площадь ортогональной проекции многоугольника на плоскость равна площади проектируемого многоугольника, умноженной на косинус угла между плоскостью многоугольника и плоскостью проекций.

hello_html_m3acf510f.jpg

hello_html_7e2a7158.gifhello_html_546111d.gif

hello_html_56b1906f.gif






hello_html_m226890bd.gifhello_html_m65c5f227.gif

hello_html_69020528.gif

hello_html_m5d411a1c.gif





hello_html_5800f7e5.gifhello_html_3986437c.gifhello_html_3986437c.gif




hello_html_2eec2f0e.gifhello_html_7ef82a8b.gif

2)Решить задачи :

1. В правильной четырёхугольной призме через диагональ основания под углом 30 к плоскости основания проведена плоскость, пересекающая боковое ребро.

Найти диагональ основания, если площадь сечения равна 8hello_html_2548fd2c.jpghello_html_246ecaab.jpg3 см2.


  1. В правильной четырёхугольной призме через середины двух смежных сторон основания, равных a, проведена плоскость, пересекающая три боковых ребра и наклонённая к плоскости основания под углом . Найти площадь сечения.


  1. Высота прямой призмы равна 1. В основании призмы лежит ромб со стороной, равной 2, и острым углом 30°. Через сторону основания проведена секущая плоскость с углом наклона к плоскости основания 60°. Найти площадь сечения призмы.


  1. В правильной четырёхугольной призме через середины двух смежных сторон основания, равных a, проведена плоскость, пересекающая три боковых ребра и наклонённая к плоскости основания под углом . Найти площадь сечения.


  1. Высота прямой призмы равна 1. В основании призмы лежит ромб со стороной, равной 2, и острым углом 30°. Через сторону основания проведена секущая плоскость с углом наклона к плоскости основания 60°. Найти площадь сечения призмы.


  1. В кубе ABCDA1B1C1D1 через точки A, D1 и середину ребра BB1 проведено сечение.


Найти угол наклона секущей плоскости к плоскости основания ABCD.


    1. Основанием пирамиды является прямоугольный треугольник, катеты которого равны 3 и 4. Каждая боковая грань пирамиды наклонена к плоскости основания под углом в 60°. Найти площадь полной поверхности пирамиды.


    1. Стороны основания треугольной пирамиды равны 5, 6 и 7. Вершина пирамиды проектируется в центр окружности, вписанной в её основание. Высота пирамиды равна радиусу этой вписанной окружности. Найти площадь поверхности пирамиды.

    2. Основанием пирамиды является ромб с диагоналями 6 и 8. Высота пирамиды равна 1.Найти площадь полной поверхности этой пирамиды, если все двугранные углы при её основании равны.

    1. В правильной усечённой треугольной пирамиде стороны нижнего и верхнего оснований равны соответственно a и b (a b ). Найти площадь полной поверхности усечённой пирамиды, если её боковые грани наклонены к плоскости основания под углом .

    2. В правильной усечённой шестиугольной пирамиде стороны нижнего и верхнего оснований равны соответственно a и b (a b ). Найти площадь полной поверхности усечённой пирамиды, если её боковые грани наклонены к плоскости основания под углом .

  1. Основанием пирамиды служит треугольник, стороны которого равны 12, 10 и 10. Каждая боковая грань наклонена к основанию под углом 45°. Найти площадь полной поверхности пирамиды.


  1. Боковые грани пирамиды наклонены к плоскости основания под углом 60°. В основании пирамиды лежит треугольник со сторонами 8, 9, 11. Найти площадь боковой поверхности пирамиды.



12. Ребро куба ABCDA1B1C1D1 равно a. На рёбрах AA1, BB1, DD1 соответственно взяты точки M, N, P так, что AM MA1, BN 2NB1, D1P 3PD . Найти площадь сечения куба плоскостью MNP.


13. Дан куб ABCDA1B1C1D1 с ребром, равным 1. Точка M середина ребра AD, точка


N центр грани CC1D1D. Найти площадь сечения куба плоскостью, проходящей через точки M, N и параллельной прямой BD.


14. Дана правильная шестиугольная призма ABCDEFA1B1C1D1E1F1 со стороной основания 2 и высотой 10. На боковом ребре CC1 взята точка M. Через точку M проведены две взаимно перпендикулярные плоскости, каждая из которых перпендикулярна плоскости CC1F . Одна из этих плоскостей проходит через точку F1, а другая через точку F. Найти площади сечений призмы проведёнными плоскостями.


  1. Дана правильная треугольная пирамида, площадь каждой боковой грани которой равна 1. Найти площадь сечения пирамиды плоскостью, проходящей через середину высоты пирамиды параллельно её боковой грани.


  1. Дана правильная шестиугольная пирамида, площадь каждой боковой грани которой равна 1. Найти площадь сечения пирамиды плоскостью, проходящей через середину высоты пирамиды параллельно её боковой грани.

3)Решить задачи :

hello_html_709dc4a7.gif


ПЗ № 5. Различные виды многогранников. Их изображения. Сечения, развертки многогранников. Площадь поверхности.

Задание:

1) Опорный конспект.

Многогранником называют геометрическое тело, поверхность которого состоит из конечного числа многоугольников. Каждый из этих многоугольников называется гранью многогранника, стороны и вершины этих многоугольников — соответственно ребрами и вершинами многогранника.

Среди многогранников различают призмы и пирамиды.

Призма это многогранник, поверхность которого состоит из двух равных многоугольников и параллелограммов, имеющих общие стороны с каждым из оснований.

Два равных многоугольника называются основаниями призмы, а параллелограммы — ее боковыми гранями. Боковые грани образуют боковую поверхность призмы. Ребра, не лежащие в основаниях, называются боковыми ребрами призмы.hello_html_6951885f.jpg

Призму называют п-угольной, если ее основаниями

являются п -угольники. На рис. 24.6 изображена четырехугольная призма АВСDА'В'С'D'.

Призму называют прямой, если ее боковыми гранями являются прямоугольники (рис. 24.7).

Призму называют правильной, если она прямая, а ее основания — правильные многоугольники.

Четырехугольную призму называют параллелепипедом, если ее основания — параллелограммы.

Параллелепипед называют прямоугольным, если все его грани — прямоугольники.

Диагональ параллелепипеда — это отрезок, соединяющий его противоположные вершины. У параллелепипеда четыре диагонали.

Доказано, что диагонали параллелепипеда пересекаются в одной точке и делятся этой точкой пополам. Диагонали прямоугольного параллелепипеда равны.

Пирамида — это многогранник, поверхность которого состоит из многоугольника — основания пирамиды, и треугольников, имеющих общую вершину, называемых боковыми гранями пирамиды. Общая вершина этих треугольников называется вершиной пирамиды, ребра, выходящие из вершины, — боковыми ребрами пирамиды.hello_html_534c4034.png

Перпендикуляр, опущенный из вершины пирамиды на основание, а также длина этого перпендикуляра называется высотой пирамиды.


Простейшая пирамида — треугольная или тетраэдр (рис. 24.8). Особенность треугольной пирамиды состоит в том, что любую грань можно рассматривать как основание.

Пирамиду называют правильной, если в основании ее лежит правильный многоугольник, а все боковые ребра равны между собой.

Заметим, что следует различать правильный тетраэдр (т.е. тетраэдр, у которого все ребра равны между собой) и правильную треугольную пирамиду (в ее основании лежит правильный треугольник, а боковые ребра равны между собой, но их длина может отличаться от длины стороны треугольника, который является основанием призмы).

Различают выпуклые и невыпуклые многогранники. Определить выпуклый многогранник можно, если воспользоваться понятием выпуклого геометрического тела: многогранник называют выпуклым. если он является выпуклой фигурой, т.е. вместе с любыми двумя своими точками целиком содержит и соединяющий их отрезок.

Можно определить выпуклый многогранник иначе: многогранник называют выпуклым, если он полностью лежит по одну сторону от каждого из ограничивающих его многоугольников.hello_html_6ab29a4f.png

Все многогранники, которые до сих пор рассматривались, были выпуклыми (куб, параллелепипед, призма, пирамида и др.). Многогранник, изображенный на рис. 24.9, выпуклым не является.

Рассмотрим несколько выпуклых многогранников (таблица 24.1)

hello_html_39176722.jpg

Из этой таблицы следует, что для всех рассмотренных выпуклых многогранников имеет место равенство В - Р + Г= 2. Оказалось, что оно справедливо и для любого выпуклого многогранника. Впервые это свойство было доказано Л. Эйлером и получило название теоремы Эйлера.

Выпуклый многогранник называют правильным, если его гранями являются равные правильные многоугольники и в каждой вершине сходится одинаковое число граней.

Используя свойство выпуклого многогранного угла, можно доказать, что различных видов правильных многогранников существует не более пяти.

Действительно, если грани многогранника — правильные треугольники, то в одной вершине их может сходиться hello_html_1c57ba51.png

3, 4 и 5, так как 60" • 3 < 360°, 60° • 4 < 360°,

60° • 5 < 360°, но 60° • 6 = 360°.

Если в каждой вершине многогранника сходится три правильных треугольника, то получаем правильный тетраэдр, что в переводе с греческого означает «четырехгранник» (рис.).

Если в каждой вершине многогранника сходится четыре правильных треугольника, то получаем октаэдр (рис.). Его поверхность состоит из восьми правильных треугольников.

Если в каждой вершине многогранника сходится пять правильных треугольников, то получаем икосаэдр (рис.). Его поверхность состоит из двадцати правильных треугольников.

Если грани многогранника — квадраты, то в одной вершине их может сходиться только три, так как 90° • 3 < 360°, но 90° • 4 = 360°. Этому условию удовлетворяет только куб. Куб имеет шесть граней и поэтому называется также гексаэдром (рис.).

Если грани многогранника — правильные пятиугольники, то в одной вершине их может сходиться только три, так как 108° • 3 < 360°, пятиугольники и в каждой вершине сходится три грани, называется додекаэдром (рис.). Его поверхность состоит из двенадцати правильных пятиугольников.

Шестиугольными и более грани многогранника не могут быть, так как даже для шестиугольника 120° • 3 = 360°. Чтобы изготовить модель многогранника, нужно сделать его развертку (точнее развертку его поверхности).

Развертка многогранника — это фигура на плоскости, которая получается, если поверхность многогранника разрезать по некоторым ребрам и развернуть ее так, чтобы все многоугольники, входящие в эту поверхность, лежали в одной плоскости.

Сечения:

hello_html_27c25e08.png

Формулы:

hello_html_m4d5320ff.png


hello_html_e83952e.png


2) Задание:

1. Заполнить таблицу (где пустое место):

Теорема Эйлера. Число граней + число вершин - число ребер = 2.

hello_html_7e4f3f17.jpg

Многогранник

тетраэдр

октаэдр

икосаэдр

додекаэдр

куб

Число граней

4


20

12


Число вершин

4

6


20

8

Число ребер


12

30


12

2. Определите количество граней, вершин и рёбер многогранника, изображённого на рисунке. Проверьте выполнимость формулы Эйлера для данного многогранника.

3. Построить развертку многогранника на альбомном листе (или картоне)и склеить.

а) куб и б) тетраэдр

hello_html_45252253.jpg

в) октаэдр

hello_html_m5eba50d7.jpg

г) икосаэдр

hello_html_679d2806.gif


д) додекаэдр

hello_html_m4e1327af.gif


4. Построить развертку многогранника на альбомном листе (или картоне)и склеить.

hello_html_m36eb1203.gif

ПЗ № 6. Виды симметрий в пространстве. Симметрия тел вращения и многогранников.

Задание:

1) Опорный конспект.

Симметрия в широком смысле — соответствие, неизменность, проявляемые при каких-либо изменениях, преобразованиях (например: положения, энергии, информации, другого). Так, например, сферическая симметрия тела означает, что вид тела не изменится, если его вращать в пространстве на произвольные углы (сохраняя одну точку на месте). Двусторонняя симметрия означает, что правая и левая сторона относительно какой-либо плоскости выглядят одинаково.

Отсутствие или нарушение симметрии называется асимметрией или аритмией.

Общие симметрийные свойства описываются с помощью теории групп.

Симметрии могут быть точными или приближёнными.

Симметрия в геометрии.

Геометрическая симметрия — это наиболее известный тип симметрии для многих людей. Геометрический объект называется симметричным, если после того как он был преобразован геометрически, он сохраняет некоторые исходные свойства. Например, круг повёрнутый вокруг своего центра будет иметь ту же форму и размер, что и исходный круг. Поэтому круг называется симметричным относительно вращения (имеет осевую симметрию). Виды симметрий, возможных для геометрического объекта, зависят от множества доступных геометрических преобразований и того, какие свойства объекта должны оставаться неизменными после преобразования.

Виды геометрических симметрий:

Зеркальная симметрия

Осевая симметрия

Вращательная симметрия

Центральная симметрия

Скользящая симметрия

Зеркальная симметрия.

Зеркальная симметрия или отражение — движение евклидова пространства, множество неподвижных точек которого является гиперплоскостью (в случае трехмерного пространства — просто плоскостью). Термин зеркальная симметрия употребляется также для описания соответствующего типа симметрии объекта, то есть, когда объект при операции отражения переходит в себя. Это математическое понятие в оптике описывает соотношение объектов и их (мнимых) изображений при отражении в плоском зеркале. Проявляется во многих законах природы (в кристаллографии, химии, физике, биологии и т. д., а также в искусстве и искусствоведении).

Осевая симметрия.

Фигура называется симметричной относительно прямой А, если для каждой точки фигуры симметричная ей точка относительно прямой А также принадлежит этой фигуре.

Центральная симметрия.

Центральной симметрией (иногда центральной инверсией) относительно точки A называют преобразование пространства, переводящее точку X в такую точку X′, что A — середина отрезка XX′. Фигура называется симметричной относительно точки A, если для каждой точки фигуры симметричная ей точка относительно точки A также принадлежит этой фигуре. Точка A называется центром симметрии фигуры. Говорят также, что фигура обладает центральной симметрией. Другие названия этого преобразования — симметрия с центром A. Центральная симметрия в планиметрии является частным случаем поворота, точнее, является поворотом на 180 градусов.

Два треугольника с точечной симметрией отражения в плоскости. Треугольник А’В’С может быть получен из треугольника ABC поворотом на 180 ° вокруг точки O.


hello_html_b0d629b.png



Виды симметрии:


hello_html_ea8bead.jpg


Симметрия тел вращения:

hello_html_464dc77b.jpghello_html_m47b0b663.pnghello_html_mdf3f01e.jpghello_html_7a8edcf0.jpg

2) Примеры:

Пример 1. а)Может ли фигура иметь более одного центра симметрии?

Ответ: Да, например, прямая, плоскость и т.д. имеют бесконечно много центров симметрии.

б) Приведите примеры центрально-симметричных фигур.
Ответ: Центрально-симметричные: куб, прямоугольный параллелепипед, шар и др.;

Пример 2.а) Построить точки А1 и А2 симметричные относительно точки О,

б) Построить треугольники АВС и А1В1С1 симметричные относительно точки О,
Ответ: а) б)

hello_html_27d1ce35.gif

hello_html_2c7a64c8.gif







Пример 3. а) Построить точки А1 и А2 симметричные относительно прямой а.

б) Построить прямоугольники симметричные относительно прямой b.
Ответ: а) б)
hello_html_me2c259c.gif

hello_html_73eae775.gif


Пример 4. Можно ли назвать ножницы симметричной фигурой?hello_html_m43e930ed.jpg

Ответ: Да.




Пример 5. Найди подходящую правую часть робота.

hello_html_m69d1ea91.gif

Ответ:2 и 5; 1 и …; … и 4.

Пример 6. На рисунке укажите буквы латинского алфавита имеющие одну ось симметрии;

hello_html_m79939fd7.png

Ответ: A, …, …, D, …, …, .., U, V, W, Y;

Пример 7. Построить рисунок.

hello_html_17f4ab15.jpg

а

Пример 8. Сколько осей симметрии имеет прямоугольный параллелепипед, гранями которого не являются квадраты?hello_html_76dbeb1f.png

Ответ: 3.hello_html_18121dc3.png

Пример 9. Сколько плоскостей симметрии имеет правильная шестиугольная призма?

Ответ:7.

Пример 10.

Тест по теме «Зеркальная симметрия в призме ».

1)Сколько плоскостей симметрии имеет правильная четырехугольная призма?

а)2 б)4 в)3 г)5 д)12,

2)Сколько плоскостей симметрии имеет прямая призма, в основании которой лежит прямоугольник?

а)2 б)3 в)1 г)4 д)8,

3)Сколько плоскостей симметрии имеет правильная треугольная призма?

а)4 б)3 в)1 г)2 д)5.

3) Задание:

1. Построить таблицу:

Элементы симметрии правильных многогранников.


тетраэдр

октаэдр

икосаэдр

гексаэдр

додекаэдр

Центры симметрии

-

1

1

1

1

Оси симметрии

3

9

15

9

15

Плоскости симметрии

6

9

15

9

15


2.Привести примеры симметрии из природы, в архитектуре, в искусстве.(один рисунок на альбомном листе) .

3. Решить задачи ( по примерам):

  1. а)Может ли фигура иметь ровно два центра симметрии?

б)Приведите примеры не центрально-симметричных фигур.

  1. а) Построить точки В1 и В2 симметричные относительно точки О,

б) Построить прямоугольные треугольники АВС и А1В1С1 симметричные относительно точки О,hello_html_m575e1189.png

  1. а) Построить точки В1 и В2 симметричные относительно прямой а.

б) Построить квадраты симметричные относительно прямой b.

  1. Дорисуй рисунки так, чтобы они стали симметричными.

  2. Можно ли назвать стрекозу насекомым, у которого имеется ось симметрии?hello_html_m3970ee64.jpg

  3. На рисунке укажите буквы латинского алфавита имеющие две оси симметрии;

hello_html_m79939fd7.png

  1. Построить рисунок.

hello_html_3cfea8dd.jpgа аhello_html_m304f2795.jpg

  1. Сколько осей симметрии имеет прямоугольный параллелепипед, две грани которого являются квадратами?

  2. Сколько плоскостей симметрии имеет прямоугольный параллелепипед, гранями которого не являются квадраты?

  3. Определить фигуры:1) обладающие центральной симметрией и указать их центр;
    2) обладающие осевой симметрией и указать ось симметрии;3) имеющие обе симметрии.





4.

hello_html_m4a5c24b8.png

5. Построить.


hello_html_m29026d9f.png

ПЗ № 7. Вычисление площадей и объемов.

Задание:

1) Перепишите и заполните пропуски:hello_html_3a511f3b.png

  • А)Пример 1. Высота правильной треугольной пирамиды 4 см, а ее апофемы 8 см. Вычислите площадь боковой поверхности пирамиды. 
    Решение:  Исходя из того, что MK = 8, MO = 4, синус угла OKM равен  MO/MK = 1/2 , откуда угол равен arcsin 1/2 = 30 °. Откуда  KO / MK = cos 30° , KO / 8 = cos 30° ,

KO = 8 cos 30° .KO = 8/2 = 4 .
Тогда по свойству равностороннего треугольника
  КО = r = a/6.

4 = a /6 , a = 24. 
Теперь, зная размер основания боковой грани и ее апофему, найдем площадь боковой грани как площадь равнобедренного треугольника:
 Sт = 1/224 8 = 12 8 = … см2 .
Откуда площадь боковой поверхности пирамиды
 S = 3 Sт = 3 96 = … см2 . 
Ответ: 288 см2.

Пример 2. Дано: усеченная правильная пирамида, n = 3, h = 4, a1= 16 , a2= 10 . Надо найти площадь полной поверхности усеченной пирамиды .

Решение: r1= a1 / 2  = 16  : 2  = 16 : 2 = …, r2= a2 / 2  = 10  : 2  = 10 : 2 = … ,

l2 = h2 + (r2 r1)2, l2 = 42 + (5 8)2 = 16 + 9 = …, l = … Sn =  /4 (a12 + a22) + 1,5 l(a1 + a2) .

Sn =  /4 ((16 )2 + (10 )2) + 1,5 5(16  + 10 ) =  /4 (768 + 300) + 1,5 5 = =267 + 195  =   .

Ответ: 462 

Пример 3. Дано: усеченная правильная пирамида, n = 4, h = 3, a1= 16, a2= 8 . Надо найти площадь полной поверхности усеченной пирамиды .

Решение: r1= a1 / 2= 16: 2= …, r2= a2 / 2= 8  : 2  = …,

l2 = h2 + (r2 r1)2, l2 = 32 + (4 8)2 = 9 + 16 = …, l = ….

Sn = (a12 + a22) + 2 l(a1 + a2) .Sn = (162 + 82) + 2 5(16 + 8) = 320 + 240 = … .

Ответ: 560

Пример 4. Дано: усеченная правильная пирамида, n = 6, h = 2, a1= 2 , a2= 6 . Надо найти площадь полной поверхности усеченной пирамиды .

Решение: r1= a1 / 2  = 2  : 2  =  , r2= a2 / 2  = 6  : 2  = 3 ,

l2 = h2 + (r2 r1)2, l2 = 22 + ( )2 = 4 + 12 = …, l = ….

Sn =3  /2 (a12 + a22) + 3 l(a1 + a2) .Sn =3  /2 (22 + 62) + 3 4(2 + 6) = …   + .

Ответ: 60   + 96

Пример 5. Дано: усеченная правильная пирамида, n = 4, h = 3, r1=2, r2= 6 . Надо найти площадь полной поверхности усеченной пирамиды .

Решение: l2 = h2 + (r2 r1)2, l2 = 32 + (6 2)2 = 9 + 16 = …, l = ….

Sn = 4 (r12 + r22) + 4 l(r1 + r2) . Sn = 4 (22 + 62) + 2 5(2 + 6) = 160 + 80 = … .

Ответ: 240.

  • В)Пример 1. Площадь грани прямоугольного параллелепипеда равна 12. Ребро,

перпендикулярное этой грани, равно 4. Найдите объем параллелепипеда.
Решение: Каждая грань прямоугольного параллелепипеда –прямоугольник.

Пусть SABCD= a b = 12 , тогда АА1= h = 4, т.к. АА1 АВСD

Используем формулу объема прямоугольного параллелепипеда: V = a b h , V = 12 4 = ...

Ответ: 48 см3.

Пример 2. Объем прямоугольного параллелепипеда равен 12. Одно из его ребер равно 3. Найдите площадь грани параллелепипеда, перпендикулярной этому ребру.

Решение: Пусть АА1 АВСD, V = 12 , АА1= h = 3.

Найдём SABCD. Используем формулу объема прямоугольного параллелепипеда V = a b h, где SABCD= a b, S ABCD 3 = 12,S ABCD = 12 : 3 = ... Ответ: 4 см2.

Пример 3. Два ребра прямоугольного параллелепипеда, выходящие из одной вершины, равны 2, 4. Диагональ параллелепипеда равна 6. Найдите объем параллелепипеда.

Решение: a = 4, b = 2, d = 6. Найдем V.

Формула диагонали прямоугольного параллелепипеда:

d2 = a2 + b2 + h2 , 16 + 4 + h2 = 36, h2 = … , h = ...

Формула объема прямоугольного параллелепипеда: V = abh , V = 4 2 4 = ... Ответ: 32 см3.

Пример 4. Два ребра прямоугольного параллелепипеда, выходящие из одной вершины, равны 2, 3. Объем параллелепипеда равен 36. Найдите его диагональ и высоту.

Решение: a = 3, b = 2. Формула объема прямоугольного параллелепипеда: V = abh , 3 . 2 . h = 36,

6h = 36, h = ..., V = 36. Найдем d. d2 = 9 + 4 + 36, d2 = 49, d = ... Ответ: 7 и 6 см.
Пример 5. Дано: ABCDA1B1C1D1 - прямоугольный параллелепипед, диагональ 
D1= 18 составляет угол в 30° с плоскостью боковой грани, и угол в 45° с боковым ребром (рис. ). Найти: V.
hello_html_m49e41dc8.jpg

 Решение: BC1 - проекция D1на плоскость боковой грани BB1С1С,
поэтому 
D1BC1 = 30°D1BB1= 45°.
Рассмотрим Δ
D1C1BD1C1= 90° (рис.). ∠В = 30°. => D1C1 = 18 : 2 = … см.
Рассмотрим Δ
D1B1- прямоугольный: BB1= 18 cos 45° = 18 : 2 = … см.
Диагональ (d) и измерения (а, b, с) прямоугольного параллелепипеда связаны соотношением:
d2 = a2 + b2 + h2 , 182 = 92 + (9)2 + B1C12 ,(ΔD1B1B: B1B =D1 B1).
B1C12 = 182 92 (9)2 = 324 – 8181 2 = 81, B1C1 = …см. V = 99 9 = … см3.   
Ответ:
V = 729см3.

Пример 6. Стороны основания прямоугольного параллелепипеда 3 и 4. Найти его объём, если высота равна длине диагонали его основания.

Решение: BD - диагональ основания прямоугольного параллелепипеда. BD2 = АВ2 + АD2,
BD2 = 32 + 42 = 9 + 16 = …, BD = …, h = 5. V = 345 = … см3.
Ответ:
60 см3.

Пример 7. Найти объём прямоугольного параллелепипеда, если стороны основания 2 и 3, а диагональ параллелепипеда .

Решение: d2 = a2 + b2 + h2 , ()2 = 22 + 32 + h2 , h 2 = 38 – 49 = 25, h = ... V = 23 5 = … см3.
Ответ: 30 см3.

  • С)Пример 1. Дано: АВСА1В1С1 - прямая призма, АС = ВС, ACB = 90°BN NACNC1 = 45°CC1 = 6 (рис.). Найти: V. Решение: V = Sh , S = BC2 : 2, BC2 = BN2 + CN2 , BN =CN
    (
    ΔABC – прямоугольный,AC =BC), ΔC1CN – прямоугольный,CNC1 = 45°
    CC1 = CN= 6, BC2 =2CN2 = 2 62 = 236 = …, BC = 6 ,
    V = (62 6 : 2 = 36 6 = … см3.    Ответ:216см3.     Пример 2. Дано: ABCDА1В1С1D1 - прямая призма,  ABCD - ромб, BAD = 60° (рис.). ВВ1 = 2, B1DB = 45°. Найти: V. РешениеSp = AB AD sin 60°. ΔABD – равносторонний( AB = AD,BAD = 60° ).
    AB = BD = AD. ΔB1DB –прямоугольный ,
    B1DB = 45°. => ΔB1DB – равнобедренный, ВВ1 = ВD = 2,
    V = AB AD sin 60° BB1= BB13 sin 60° = 23 / 2 = … см3.
    hello_html_m47ca6280.jpghello_html_6c1a9bbb.jpg

Ответ: 4 см3

Пример 3. Дано: ABCDFM...M1 - правильная шестиугольная призма. AD1 = 8 см - наибольшая диагональ.AD1= 30°(рис.).hello_html_72355ff0.jpg

Найти: V. 
Решение: V= S0 · h. h = DD1 в ΔADD1, = 90°. D1 = 30°,

DD1 = AD1 · cos 30°. DD1 = 8 / 2 = … , AD = AD1 : 2 = 8 : 2 = … см,
OD = OC = CD = AD : 2 = 4 : 2 = …
см,
S
0 = 6S ΔOCD = 6 / 4) a2 = 6 / 4) 22 = 6 см. V = 6 = 6 43 = … см3.    

Ответ: 72 см3.   hello_html_m62762a7f.jpg

Пример 4. Дана трапеция, S(BB1C1C) = 8 см2, S(AA1D1D) = 12см2, BH = 5 см (рис.).Найти: Vnp. 
Решение:1)Расстояние между параллельными плоскостями ВВ1С1 и AA1D1 есть длина перпендикуляра ВН, который является высотой трапеции ABCD.

2) Обозначим верхнее основание трапеции - а, нижнее - b, высоту призмы h, тогда S(BB1C1C) = ah, 8 = ah, a = 8 / h, S(AA1D1D) = bh , 12 = bh, b = 12 / h,

3) S0 = (AD + BC)BH : 2 =( a + b ) BH : 2 = (8 / h + 12 / h) 5 : 2 = … / h,

4) V= S0 · h. V= 50 / · h = … см3.  Ответ: 50 см3.

  • Д)Пример 1. В правильной четырехугольной пирамиде высота равна 9 см. Сторона основания 4 см. Найдите объем пирамиды.

Решение: V= 1/3 S0 · h. V= 1/3 a2 · h = 1/3· 42·9 = 1/3 · 16 · 9 = 16 · 3 = … см3. Ответ: 48см3. 

Пример 2. a) Объем правильной четырехугольной пирамиды равен 27 см3, высота 9 см. Найти сторону основания.

Решение: V= 1/3 S0 · h. V= 1/3 a2 · h, a2  = 3V : h = 3 · 27 : 9 = 3 · 3 = ... , a = … см.

Ответ: 3 см.hello_html_m12ec6366.jpg

б) Объем пирамиды равен 56 см3, площадь основания 14 см2. Чему равна высота?

Решение: V= 1/3 S0 · h.  h = 3 V : S0  = 3 · 56 : 14 = 3 · 4 = … см.

Ответ: 12 см.

Пример 3. Дано: ABCD - правильная пирамида.

АВ = a = 3; AD = 2 (рис.).Найти: aSocн.; б) АО; в) DO; г) V.

 Решение:

а) S0 = 0,25 · a2  = 0,25 · 32 = 2,25 (используем формулу для вычисления площади правильного треугольника). 

б) AO = R = 2/3h = 1/3 a  (формула радиуса описанной окружности через сторону правильного треугольника). AO = 1/3 · 3 = .

в) DO2 = AD2AO2, (по теореме Пифагора).

DO2 = (2)2 – ()2 = 4 · 3 – 3 = … , DO = h = 3.hello_html_5fef969e.jpg

г) V= 1/3 S0 · h. V= 1/3 · 2,25 · 3 = … см3.

Ответ: aSocн. = 2,25 см2; б) АО = см; в) DO = 3см; г) V = 2,25 см3 .

Пример 4. Дано: ABCDF - правильная пирамида. 

FCO = 45°FO = 2 (рис.). Найти: a) Socн.; б) V. 

Решение:

1) Рассмотрим ΔFOC= 90°= 45°, значит, = 45°. Следовательно, ΔFOC - равнобедренный, ОС ≈ FO = h= 2.

2) АС = 2OС = 4. AC = AD (по свойству диагонали квадрата, d2 = 2а2).

Тогда  AD = AC / = 4 / = 2 .

3) ABCD - квадрат (пирамида правильная). S0 = AD2 = (2)2 = 2 · 4 = ...

4) V= 1/3 S0 · h. V= 1/3 · 8 · 2 = 16/3 5,3. Ответ: a) 8; 6) 5,3.hello_html_6d91a8cf.jpg

Пример 5. Дано: ABCA1B1C1 – усеченная пирамида. ΔАВС – прямоугольный,
AB = 18 дм, BC = 24 дм, AA1 = BB1 = СС1 = 12,5 дм, k = 0,5. Найти V.

Решение: S1 = SABC = 1/2 · AB · BC = 1/2 · 18 · 24 = 9 · 24 = … ,
S
2 = S(A1B1C1) = 1/2· A1B1 · B1C1 = 1/2 (k · AB) · (k · BC) =
= 1/2· 0,5 · 18 · 0,5 · 24 = 6 · 9 = … ,
S = S
1 + S2 + = = 216 + 54 + = 216 + 54 + 54 = … ,
V = 1/3 · h · S = 1/3 378 h = 126 h, R
1 = abc/4S1 ,

c = = = … , R1 = = = …, hello_html_m78f1984a.jpg

R2 = R1 : 2 = 7,5; h2 = 12,52 – (15 – 7,5)2 = 12,52 – 7,52 = (12,5 – 7,5) · (12,5 + 7,5) =

= 5 · 20 = … , h = … ,

V = 126 h = 126 · 10 = … (дм3).
Ответ: 1260 (дм3).

Пример 6. усеченная пирамида а) n = 3, а1 = 2, а2 = 5, h = 12, V =?
Решение: A = 22 + 52 + 2 · 5 = 39, V = · h · A = · 12 · 39 = … . Ответ: 39 .

б) n = 4, a1 = 3, a2 = 8, h = 6, V = ?
Решение: A = 32 + 82 + 3 · 8 = 97, V = 1/3 · 6 · 97 = 2 · 97 = ...

Ответ: 194.



2)Решить задачи ( по примерам):

А)

  1. Высота правильной треугольной пирамиды 8 см, а ее апофемы 16 см. Вычислите площадь боковой поверхности пирамиды. 

  2. Дано: усеченная правильная пирамида, n = 3, h = 8, a1 = 14 , a2 = 2 . Надо найти площадь полной поверхности усеченной пирамиды .

  3. Дано: усеченная правильная пирамида, n = 4, h = 8, a1 = 16, a2 = 4 . Надо найти площадь полной поверхности усеченной пирамиды .

  4. Дано: усеченная правильная пирамида, n = 6, h = 2, a1 = 4 , a2 = 8 . Надо найти площадь полной поверхности усеченной пирамиды .

  5. Дано: усеченная правильная пирамида, n = 4, h = 3, r1 = 5, r2 = 9 . Надо найти площадь полной поверхности усеченной пирамиды .

В)

  1. Площадь грани прямоугольного параллелепипеда равна 15. Ребро, перпендикулярное этой грани, равно 6. Найдите объем параллелепипеда.

  2. Объем прямоугольного параллелепипеда равен 24. Одно из его ребер равно 3. Найдите площадь грани параллелепипеда, перпендикулярной этому ребру.

  3. Два ребра прямоугольного параллелепипеда, выходящие из одной вершины, равны 3, 4. Диагональ параллелепипеда равна 13. Найдите объем параллелепипеда.

  4. Два ребра прямоугольного параллелепипеда, выходящие из одной вершины, равны 3, 6. Объем параллелепипеда равен 108. Найдите его диагональ и высоту.

  5. Дано: ABCDA1B1C1D1 - прямоугольный параллелепипед, диагональ  D1= 12 составляет угол в 30° с плоскостью боковой грани, и угол в 45° с боковым ребром. Найти: V.

  6. Стороны основания прямоугольного параллелепипеда 6 и 8. Найти его объём, если высота равна длине диагонали его основания.

  7. Найти объём прямоугольного параллелепипеда, если стороны основания 4 и 6, а диагональ параллелепипеда .

С)

  1. Дано: АВСА1В1С1 - прямая призма, АС = ВС, ACB =90°BN NACNC1 = 45°CC= 8 (рис.). Найти: V.

  2. Дано: ABCDА1В1С1D1 - прямая призма,  ABCD - ромб, BAD = 60° (рис.). ВВ1 = 4, B1DB = 45°. Найти: V.

  3. Дано: ABCDFM...M1 - правильная шестиугольная призма. AD1 = 16 см - наибольшая диагональ.AD1= 30° (рис.). Найти: V. 

  4. Дана трапеция, S(BB1C1C) = 10 см2, S(AA1D1D) = 14см2, BH = 10 см (рис.). Найти: Vnp. 

Д)

  1. В правильной четырехугольной пирамиде высота равна 6 см. Сторона основания 5 см. Найдите объем пирамиды.

  2. a)Объем правильной четырехугольной пирамиды равен 48 см3, высота 4 см. Найти сторону основания. б) Объем пирамиды равен 28 см3, площадь основания 4 см2. Чему равна высота?

  3. Дано: ABCD - правильная пирамида. АВ = a = 6; AD = 4 . Найти: aSocн.; б) АО; в) DO; г) V.

  4. Дано: ABCDF - правильная пирамида.  FCO = 45°FO = 4 . Найти: a) Socн.; б) V. 

  5. Дано: ABCA1B1C1усеченная пирамида. ΔАВС прямоугольный, AB = 12 дм,BC = 16 дм, AA1 = BB1 = СС1 = 13 дм, k = 0,5. Найти V.

  6. а) n = 3, а1 = 2, а2 = 5, h = 24, V =?, б) n = 4, a1 = 3, a2 = 8, h = 3, V = ?,

3)Решить задачи :

  1. Дана правильная четырехугольная пирамида со стороной основания a=12 см и
    высотой
    h=8 см. Найдите площадь полной поверхности пирамиды.
    hello_html_m3eb51a34.png

  2. Найдите объем прямоугольного параллелепипеда с ребрами 3 см, 5 см и 8 см.

а) 120 см3; б) 60 см3; в) 32 см3; г) другой ответ.

  1. Длина прямоугольной комнаты в 2 раза больше ширины и на 2 м больше высоты. Найдите объем комнаты, если ее длина равна 6 м. а) 432 м3; б) 144 м3; в) 72 м3; г) другой ответ.

  2. Три ребра прямоугольного параллелепипеда, выходящие из одной вершины, равны 1; 0,5 и 16. Найдите ребро равновеликого ему куба.

ПЗ № 8. Векторы. Действия с векторами. Декартова система координат в пространстве. Уравнение окружности, сферы, плоскости.

Задание:

1) Опорный конспект.

Физические величины, имеющие не только абсолютное значение, но и направление, называются векторными.

Скорость, сила, ускорение — векторы. Для них важно «сколько» и важно «куда». Например, ускорение свободного падения hello_html_m28faee77.png направлено к поверхности Земли, а величина его равна 9,8 м/с2. Импульс, напряженность электрического поля, индукция магнитного поля — тоже векторные величины.

Вектор — это направленный отрезок. Длиной вектора называется длина этого отрезка. Обозначается: hello_html_m2a88039c.png или hello_html_20991715.png

Равными называются векторы, имеющие одинаковые длины и одинаковое направление. Это значит, что вектор можно перенести параллельно себе в любую точку плоскости.
Единичным
 называется вектор, длина которого равна 1. Нулевым — вектор, длина которого равна нулю, то есть его начало совпадает с концом.

Сложение векторов

Для сложения векторов есть два способа.

1. Правило параллелограмма. Чтобы сложить векторы hello_html_6ccfdd8b.png и hello_html_8a9ad90.png, помещаем начала обоих в одну точку. Достраиваем до параллелограмма и из той же точки проводим диагональ параллелограмма. Это и будет сумма векторов hello_html_6ccfdd8b.png и hello_html_8a9ad90.png.

hello_html_m7e7837b3.png

Помните басню про лебедя, рака и щуку? Они очень старались, но так и не сдвинули воз с места. Ведь векторная сумма сил, приложенных ими к возу, была равна нулю.

2. Второй способ сложения векторов — правило треугольника. Возьмем те же векторы hello_html_6ccfdd8b.png и hello_html_8a9ad90.png. К концу первого вектора пристроим начало второго. Теперь соединим начало первого и конец второго. Это и есть сумма векторов hello_html_6ccfdd8b.png и hello_html_8a9ad90.png.

hello_html_m16fda0fa.png

По тому же правилу можно сложить и несколько векторов. Пристраиваем их один за другим, а затем соединяем начало первого с концом последнего.

hello_html_m180d670c.png

Представьте, что вы идете из пункта А в пункт В, из В в С, из С в D, затем в Е и в F. Конечный результат этих действий — перемещение из А в F.

Умножение вектора на число

При умножении вектора hello_html_6ccfdd8b.png на число k получается вектор, длина которого в k раз отличается от длины hello_html_6ccfdd8b.png. Он сонаправлен с вектором hello_html_6ccfdd8b.png, если k больше нуля, и направлен противоположно hello_html_6ccfdd8b.png, если k меньше нуля.

hello_html_73835e2b.pnghello_html_7e06bfe9.jpg

Трехмерные чертежи выполнять тяжко, поэтому ограничусь одним вектором, который для простоты отложу от начала координат:

Перед вами ортонормированный базис hello_html_5b0c516f.gif трехмерного пространства и прямоугольная система координат, единичные векторы hello_html_37b6494a.gif данного базиса попарно ортогональны: hello_html_6b98b57d.gif и hello_html_50e1b5f7.gif. Ось hello_html_m4710f128.gif наклонена под углом 45 градусов только для того, чтобы складывалось визуальное впечатление пространства. Любой вектор hello_html_m14956daf.gif трехмерного пространства можно единственным способом разложить по ортонормированному базису hello_html_5b0c516f.gif: 
hello_html_m6fd59a9f.gif, где hello_html_m4e4977fa.gif – координаты вектора hello_html_754949a.gif (числа) в данном базисе.

Пример с картинки: hello_html_5f103d13.gif. Давайте посмотрим, как здесь работают правила действий с векторами. Во-первых, умножение вектора на число: hello_html_m196faf04.gif (красная стрелка), hello_html_6278e414.gif (зеленая стрелка) и hello_html_b8930de.gif (малиновая стрелка). Во-вторых, перед вами пример сложения нескольких, в данном случае трёх, векторов: hello_html_6c8ea6a4.gif.  Вектор суммы hello_html_m49782ef1.gif начинается в исходной точке отправления (начало вектора hello_html_m196faf04.gif) и утыкается в итоговую точку прибытия (конец вектора hello_html_b8930de.gif).

Все векторы трехмерного пространства, естественно, тоже свободны, попробуйте мысленно отложить вектор hello_html_m49782ef1.gif от любой другой точки, и вы поймёте, что его разложение hello_html_6c8ea6a4.gif  «останется при нём».

Аналогично плоскому случаю, помимо записи hello_html_5f103d13.gif широко используются версии  со скобками: hello_html_m1450ba4d.gif либо hello_html_6f699bd2.gif.

Если в разложении отсутствует один (или два) координатных вектора, то вместо них ставятся нули. Примеры:
вектор
 hello_html_m7a0cbf9e.gif (дотошно hello_html_171ec4e7.gif) – запишем hello_html_5bd901b6.gif;
вектор
 hello_html_m4cf51bb.gif (дотошно hello_html_3bf2c75b.gif) – запишем hello_html_m26e182f0.gif;
вектор
 hello_html_mbf68e03.gif (дотошно hello_html_2380cf95.gif) – запишем hello_html_m7c8d9d32.gif.

Базисные векторы записываются следующим образом:
hello_html_7deb8e75.gif

Если даны две точки пространства hello_html_578b0f29.gif и hello_html_4270881d.gif, то вектор hello_html_63db5e27.gif имеет следующие координаты:
hello_html_m145ad02f.gif

То есть, из координат конца вектора нужно вычесть соответствующие координаты начала вектора.

Если даны две точки пространства hello_html_578b0f29.gif и hello_html_4270881d.gif, то длину отрезка hello_html_28104f0d.gif можно вычислить по формуле hello_html_292c92b6.gif

Действия над векторами

Пусть в трехмерном пространстве заданы векторы hello_html_4cd93f33.pngсвоими координатами. Имеют место следующие операции над ними: линейные (сложение, вычитание, умножение на число и проектирование вектора на ось или другой вектор); не линейные – различные произведения векторов (скалярное, векторное, смешанное).

1). Сложение двух векторов производится поэлементно, то есть если hello_html_ddb270d.png, то в координатной форме записывается:

hello_html_m39f935fc.pnghello_html_7df27acf.png(2.18)

Данная формула имеет место для произвольного конечного числа слагаемых.

Геометрически два вектора складываются по двум правилам:

а) правило треугольника – результирующий вектор суммы двух соединяет начало первого из них с концом второго; для суммы hello_html_m68d67358.png векторов – результирующий вектор суммы соединяет начало первого из них с концом последнего вектора–слагаемого при условии, что начало последующего слагаемого совпадает с концом предыдущего;

б) правило параллелограмма (для двух векторов) – параллелограмм строится на векторах–слагаемых как на сторонах, приведенных к одному началу; диагональ параллелограмма исходящая из их общего начала, является суммой векторов.

2). Вычитание двух векторов производится поэлементно, аналогично сложению, то есть если hello_html_46ec41.png, то в координатной форме записывается

hello_html_7ec0d2fc.pnghello_html_73424de9.png(2.19)

Геометрически два вектора складываются по уже упомянутому правилу параллелограмма с учетом того, что разностью векторов является диагональ, соединяющая концы векторов, причем результирующий вектор направлен из конца вычитаемого в конец уменьшаемого вектора.

3). Умножение вектора на число hello_html_745ce834.png покоординатно: hello_html_m1bf67fb0.png.

При hello_html_m66aa3952.png – вектор сонаправлен первоначальному;

hello_html_7012cc6f.png – вектор противоположно направлен первоначальному;

hello_html_2473ec23.png – длина вектора увеличивается в hello_html_745ce834.png раз;

hello_html_6d30a424.png – длина вектора уменьшается в hello_html_745ce834.png раз.

4). Пусть в пространстве задана направленная прямая (ось l)вектор hello_html_18de2c7d.png=hello_html_m694d1127.png задан координатами конца и начала. Обозначим проекции точек hello_html_100c518e.png и hello_html_6344936f.png на ось hello_html_278ee096.png соответственно через hello_html_m2223bfd4.png и hello_html_5303a217.png.

Проекцией вектора hello_html_18de2c7d.png на ось hello_html_278ee096.png называется длина вектора hello_html_m7dc13367.png, взятая со знаком «+», если вектор hello_html_m7dc13367.png и ось lсонаправлены, и со знаком «-», если hello_html_m7dc13367.png и l противоположно направлены.hello_html_m11b24da8.png

Если в качестве оси l взять некоторый другой вектор hello_html_6b05d8d6.png, то получим проекцию вектора hello_html_18de2c7d.png на вектор hello_html_6b05d8d6.png.

Уравнение окружности, сферы, плоскости.


hello_html_7a030643.jpg



hello_html_59f1dd46.gifhello_html_1781d02f.jpg


2) Решить задачи:

1. Укажите какое правило использовали и постройте чертеж.

Сложение векторов

Правило параллелограмма: диагональ параллелограмма - сумма двух векторов с общим началом.

Правило треугольника: от конца первого вектора отложить второй вектор, тогда их суммой будет вектор, начало которого совпадает с началом первого вектора, а конец с концом второго вектора.

hello_html_m4ccfc737.png

Вычитание векторов

Вычитание векторов - это сумма положительного и отрицательного вектора.

hello_html_62112a3f.png


2.Перепишите и заполните пропуски:
Пример 1. Сфера задана уравнением x 2 + (y + 3)2 + (z – 2)2 = 25.

Найдите координаты центра и радиуса сферы.

Решение: О - центр сферы, О(0,3,2), R = = ...
Ответ:
О(0,3,2), R = 5.

Пример 2. Напишите уравнение сферы радиуса = 7 с центром в точке А(2; 0; 1). Решение: (x …)2 + y 2 + (z + …)2 = 72. (x2)2 + y 2 + (z + 1)2 = …
Ответ: (x2)2 + y 2 + (z + 1)2 = 49.

Пример 3. Лежит ли А(2; 1; 4) на сфере, заданной уравнением  (x + 2)2 + (y 1) 2 + (z 3)2 = 1. Решение: Подставим координаты точки А в уравнение сферы (2 + 2)2 + (1 1) 2 + (4 3)2 = 1, 1 = 1(верно), точка А лежит на сфере.
Ответ:
точка А лежит на сфере.

Пример 4. Найти координаты центра и радиус сферы x2 + y2 + z2 + 4y - 2z = 4. Решение: x2 + y2 + z2 + 4y 2z = 4 выделим квадрат двучлена:
х
2 + у2 + 4у + 4 4 + z2  4z + 1 1 = 4, х2 + (у + 2)2 + (z 1)2 = 9, центр окружности С(…; …; …), радиус R = ...
Ответ:
С(0; 2; 1), R = 3.

Пример 5. Дано: уравнение сферы, х2 + у2z2 + 2у 4= 4.

Найти: а) О(х0; у0z0), R; б) m, при котором А(0; m; 2) и В(1; 1; m2) принадлежат сфере.

Решение: а) x 2 + y 2 +2у + z 2 – 4z = 4, x 2 + y 2 +2у +11 + z 2 – 4z + 4 4 = 4,
x 2 + (y + 1)2 + (z – 2)2 = 9. О(...,…,…), R = = ...
б) А(0; 
m; 2) и В(1; 1; m2)


 , , ,

, m = 2. При m = … точки A и В принадлежат сфере. Ответ: а) О(0; 1; 2), R = 3; б) при m = 2.

Пример 6. Найдите длину вектора КА АС.

hello_html_m436d4048.gif

Ответ: 20 см.

3)Решить задачи ( по примерам):

  1. Сфера задана уравнением (x – 1)2 + y 2 + (z – 2)2 = 9.

Найдите координаты центра и радиуса сферы.

  1. Напишите уравнение сферы радиуса = 4 с центром в точке А(2; 1; 0).

  2. Лежит ли А(5; 1; 4) на сфере, заданной уравнением  (x 3)2 + (y+ 1) 2 + (z 4)2 = 4.

  3. Найти координаты центра и радиус сферы x2 – 6x + y2 + z2 = 0.

  4. Дано: уравнение сферы, х2 + у2z2 + 4у 2= 4.

Найти: а) О(х0; у0z0), R; б) m, при котором А(0; m; 1) и В(1; 0; m2) принадлежат сфере.

  1. Найдите длину вектора КА АС, диагонали ромба 6 и 8 см.

3)Решить задачи :

  1. Точки А(3; – 5; 6) и В(5; 7; – 1) являются концами одного из диаметров сферы. Составьте уравнение этой сферы.

  2. Дана сфера x2 + y2 + z2 = 450  . Найти координаты точек пересечения сферы с прямой, проходящей через начало координат и точку А(4; 5; 3).

  3. Даны точки А(– 1; 3; 2), В(0; 3; 1), С(2; – 2; 0), D(– 4; 2; 2), Е(5; 7; 8). Какие из этих точек принадлежат сфере с центром О(– 2; 1; 0) и радиусом 3?

  4. Составьте уравнение сферы с центром О (2; 3; 4) и радиусом R=5.

  5. Точки А(7; – 2; 4) и В(9; – 8; 6) лежат на поверхности сферы и на прямой, проходящей через её центр. Составьте уравнение сферы.

  6. Сфера задана уравнением x 2 + y 2 + z 2 + 2y – 4z = 4. a)Найдите координаты центра и радиуса сферы. б) Найдите значение m, при котором точки А(0; m; 2) и В(1; 1; m – 2) принадлежат данной сфере.

  7. Диаметр сферы – отрезок АВ с концами А(2; – 1; 4) и В(2; 7; 10). a) Составьте уравнение сферы. б) Найдите кратчайшее расстояние от точки данной сферы до плоскости Оxy.

  8. Сфера задана уравнением (x – 1)2 + y 2 + (z – 2)2 = 9. а)Найдите координаты центра и радиуса сферы. б)Определите, принадлежат ли данной сфере точки А(1; 3; 1) и В(2; 2; 1).

  9. Составить уравнение сферы в каждом из следующих случаев:

а) сфера имеет центр С(0; 0; 0) и радиус r = 9;

б) сфера имеет центр С(5; 3; 7) и радиус r = 2;

в) сфера проходит через начало координат и имеет центр С(4; 4;2);

г) сфера проходит через точку А(2; 1; 3) и имеет центр С(3; 2; 1);

д) точки А(2; 3; 5) и В(4; 1; 3) являются концами одного из диаметров сферы;

  1. Сфера задана уравнением x2 + у2 + z2 + 2у 4z = 4.

а) Найдите координаты центра и радиус сферы.

б) Найдите значение m, при котором точки А(0; m; 2) н В (1; 1; m2) принадлежат данной сфере.

  1. Найдите координаты центра и радиус сферы, заданной уравнением

(x – 2)2 + (y + 3) 2 + z2 = 25. 

  1. Напишите уравнение сферы радиуса R = 7 с центром в точке А(2; 0; 1).

  2. Лежит ли А(2; 0; 3) на сфере, заданной уравнением (x + 2)2 + (y 1) 2 + (z 3)2 = 1. 

  3. Могут ли все вершины прямоугольного треугольника с катетами 4 см и 2см лежать на сфере радиуса см?

  4. Найти координаты центра и радиус сферы x2 + 6х + y2 + z 2 = 0. 

  5. Найдите координаты центра и радиус сферы, заданной уравнением

(x + 3)2 + y 2 + (z 1)2 = 16. 

  1. Напишите уравнение сферы радиуса R = 4 с центром в точке А(2; 1; 0).

  2. Лежит ли точка А(5; 1; 4) на сфере, заданной уравнением

(x –3)2 + (y + 1) 2 + (z 4)2 = 4. 

  1. Могут ли все вершины прямоугольного треугольника с катетами 4 см и 2см лежать на сфере радиуса см?

  2. Найти координаты центра и радиус сферы x2 + y2 + 6у + z2 = 0. 

  3. Составить уравнение сферы с центром в точке А (– 3; 4; – 9) и проходящую через

точку N (– 2; 6; 1).

  1. Составить уравнение сферы которая касается каждой из координатных плоскостей и проходит через точку M (2;1;3).

  2. Составьте уравнение сферы с центром в точке О(– 1;0;2), если известно, что этой сфере принадлежит точка А(3;1;1).

  3. Даны точки А(2; – 5;8) В(8; – 2;5) С(5; – 8:2)и Д(– 2; – 8; – 5).Составьте уравнение сферы, если известно, что эти точки лежат на её поверхности.

  4. Точка А лежит на сфере с центром О(3; 0; 0).

  1. Напишите уравнение сферы.

  2. Принадлежат ли этой сфере точки с координатами и (4; – 1; 0)?

  1. Составьте уравнение сферы, радиус которой равен 2, если известно, что центр сферы лежит в плоскости ОХZ, а сама сфера проходит через начало координат и точку А(1; 1; 0).

  2. Составьте уравнение сферы с радиусом, равным 3, если известно, что центр сферы лежит на оси OZ и сфера проходит через точку К(– 2; – 2; 1).

  3. Найти уравнение сферы, проходящей через точки  (0;0;0), (4;0;0),(0;6;0) и (0;0;8).

  4. Найдите координаты центра и радиус сферы, заданной уравнением:

1)x² + y² + z² = 49,

2)(х 3)² + (у + 1)² + (z + 3)² = 1,

3)х² + (y 4)² + z² = 3,

4)(x 1)² + y² + (z + 2)² = 25.

  1. Найти координаты центра и радиус сферы, заданной уравнением
    x2 + y2 + z2 – x + 2y + 1 = 0.


ПЗ № 9. Расстояние между точками. Действия с векторами, заданными координатами.

Задание:

1)Перепишите и заполните пропуски:
Пример 1. Дано:



Решение:

  1. Находим координаты вектора

;

  1. Затем находим координаты вектора


  1. Теперь находим аналогично координаты вектора


  1. Теперь находим сумму данных векторов, складывая соответствующие координаты:


Ответ:
Пример 2. Дано: , . Найдите  

Решение: Первый случай

  1. Находим координаты вектора

;

  1. Затем находим разность векторов

;

  1. Теперь находим длину вектора :

Второй случай

  1. Находим координаты вектора

;

  1. Находим координаты вектора

;

  1. Затем находим сумму векторов

;

  1. Теперь находим длину вектора : =

Ответ:

Пример 3. Даны векторы   и . Найти

Решение: Для действий с векторами справедлив обычный алгебраический приоритет: сначала умножаем, потом складываем:

=  3 - 2= - =
= = .

= + 4 {7; -9 ;1 } = + = =

=
Ответ:  ,

Пример 4. Найдите сумму векторов: и .

Решение: , .

Ответ:

Пример 5. Даны векторы , Найдите координаты векторов

Решение: , , с ,

, .

Ответ: .

Пример 6. Дано: ΔАВС, А(2; 0; 1), В(1; 2; 3), С(8; 4; 9). ВМ - медиана.

Найти: координаты вектора .

Решение: По определению медианы, М - середина отрезка АС. Следовательно, координаты М найдем по формулам координат середины отрезка  M ((82)/2, (4 + 0)/2,(9 + 1)/2), M(…,…,…).{3 + 1,22,53}, {…,… ,…}. Ответ: {4; 4; 2}.

Пример 7. Дано: А(1; 5; 3), В(7; 1; 3), С(3; 2; 6). Доказать: ΔABC - прямоугольный.

Решение: По формуле расстояния между двумя точками найдем длины отрезков АВ, АС, ВС.
AB2 = (7 + 1)2 + (5 + 1)2 + (3 – 3)2, AB2 = 64 + 36 = … , BC2 = (7– 3)2 + (– 2 + 1)2 + (6 – 3)2,
BC2 = 16 + 1 + 9 = … , AC2 = (3 + 1)2 + (5 + 2)2 + (6 – 3)2, AC2 = 16 + 49 + 9 = ...

Проверим равенство АВ2 = ВС2 + АС2, 100 = 26 + 74 верно.

По теореме обратной теореме Пифагора делаем вывод, что ΔABC - прямоугольный
с гипотенузой АВ.

Пример 8. Дано: ΔАВС; М, N, К - середины сторон соответственно АВ, ВС, АС. М(3; 2; 5), 
N(3,5; 1; 6), К(1,5; 1; 2). Найти: координаты А, В, С.

Решение: Пусть A (х1; у1z1), В(х2; у2z2), С(х3; у3z3). По формулам координат середины отрезка составим системы для абсцисс, ординат и аппликат. Пользуясь методом сложения, решим эту систему:

Ответ: А(2; 0; 1), В(8;4; 9), С(1; 2; 3).

Пример 9. Дано: А(-2; 1; 2), B(-6; 3; -2), С  оси OZ; АС = ВС. Найти: координаты точки С.

Решение: По условию С  оси OZ, значит она имеет координаты С(0; 0; z) и АС = ВС. Составим уравнение, пользуясь формулой расстояния между двумя точками: 4 + 1 + (z 2)2 = 36 + 9 + (z + 2)2, 5 + z2 – 4z + 4 = 45 + z2 + 4z + 4, 8z = 40, z = …

Ответ: (0; 0;5).

Пример 10. Дано: А(2; 1; 2), B(6; 3; 2), С (0; 0; 5); АС = ВС. Найти: SABC).

Решение: По формуле координат середины отрезка АВ найдем координаты точки М — середины:
M ((62)/2, (1 + 3)/2,(22)/2), M(4,2,0). AB2 = (6 + 2)2 + ( 31)2 + (2 + 2)2 = 16 + 4 + 16 = …, AB = ... СМ-высота равнобедренного ΔABC.
CM2 = (40)2 + (20)2 + (0 (5))2 = 16 + 4 + 25 = … , CM = 3 ,
SABC) = AB · CM : 2 = 6 · 3 : 2 = … .

Ответ: 9.

2)Решить задачи ( по примерам):

  1. Дано: А(2;1;6), В (2;0; 1), С(1; 5; 0)


  1. Дано: , , ; 2).

  2. Даны векторы   и ,  . Найти

  3. Найдите сумму векторов: и .

  4. Даны векторы , , . Найдите координаты векторов

и

  1. Дано: ΔАВС; А(1; 2; 3), B(1; 0; 4), С(3; 2; 1). AM - медиана. Найти: координаты вектора

  2. Дано: А(1; 5; 3), В(1; 3; 9), С(3; 2; 6).Доказать: ΔAВС - прямоугольный.

  3. Дано: ΔАВС, М, N, К - середины сторон соответственно ABBС, AС. М(3; 2; 4), 
    N(6; 4; 10), К(7; 2; 12).Найти: координаты вершин А, В, С.

  4. Дано: A(4; 5; 4), B(2; 3; 4); С  оси  OXAC = ВС. Найти: координаты точки С.

  5. Дано: А(4; 5; 4), B(2; 3; 4), С(1; 0; 0), АС = ВС. Найти: S(ΔABC).

3)Решить задачи :

А)

  1. Найдите координаты вектора , если

  2. Даны векторы {1;3; 3} и . Найдите координаты и длину вектора.

  3. Даны векторы {3;1; 2} и . Найдите координаты вектора ,

  4. Найдите длину вектора , , если {2;1; 5} и .

  5. Из точки А построен вектор . Найдите координаты точки В , если:

А(3;1; 2), .

  1. Даны точки А(4;6; –2) и В (–10;6; 0) . Найти длину отрезка АВ.

  2. Даны точки: А(10;14; 4), В (10;8; 12) , С (18;8; 18) 

Выяснить, равнобедренный ли треугольник, построенный на этих точках.

  1. а) Даны два вектора:  и .Найти .

б) Даны четыре вектора: .

Найти координаты векторов  

  1. В кубе АВСDА1В1С1D1, сторона которого равна 3, на диагоналях граней АD1 и D1В1 взяты точки Е и К так, что D1Е : АD1 = 2 : 3, D1K : D1B1 = 1 : 3. Найдите длину отрезка DK.

  2. Дано:

  3. Найдите длину вектора КА АС, диагонали ромба 6 и 8 см.

  4. Даны точки А(2;3; –1) и В (–5;3; 0) . Найти длину отрезка АВ.

  5. Даны точки: А(5;7; 2), В (5;4; 6) , С (9;4; 9) Выяснить, равнобедренный ли треугольник.

  6. Даны два вектора:  и .Найти .

В)

  1. Дано: A (10, 4, 3), B (6, 2, 1). Найти координаты точки M – середины отрезка AB.

  2. Дано: A (5, 4, 7), B (10, 10, 0). Найти координаты вектора .

  3. Дано: {0, 5, 0}, {2, 2, 1}. Найти длину векторов.

  4. Даны точки А (1,5; 1; –2), B (2; 2; –3); и C (2; 0; –1). Найдите: периметр треугольника ABC.

  5. Дано: М(–4; 7; 0) N(0; –1; 2).Найти: расстояние от начала координат до середины

отрезка MN.

  1. В кубе АВСDА1В1С1D1, сторона которого равна 3, на диагоналях граней АD1 и D1В1
    взяты точки Е и К так, что
    D1Е : АD1 = 1 : 3, D1K : D1B1 = 2 : 3. Найдите длину отрезка DK.

  2. Даны четыре вектора: .

Найти координаты векторов  

  1. Дано:

  2. Даны векторы и   Найдите координаты вектора .

  3. Даны векторы,. Найдите координаты вектора 

  4. На каком расстоянии от плоскости (хОу) находится точка А(2; 3; 5).

  5. На каком расстоянии от начала координат находится точка А(3; 4; 0).

  6. Найти длину вектора  если А(5; 3; 2), В(3; 1; 4).

  7. На каком расстоянии от плоскости (yOz) находится точка В(3; 2; 4).

  8. Даны векторы и  . Найдите  

  9. Изобразить систему координат Оху: и построить точку А(1; 2; 4).
    Найти расстояние от этой точки до координатных плоскостей.

  10. Вершины ΔАВС имеют координаты А(2; 0; 1), В(1; 2; 3), С(8; 4; 9).
    Найдите координаты вектора
      если ВМ - медиана ΔABC.

  11. Даны точки А(1; 5; 3) В(7; 1; 3) С(3; 2; 6). Доказать, что ΔАВС - прямоугольный.

  12. Даны точки А(2; 1; 2), В(6; 3; 2) на оси аппликат.
    Найти точку С, равноудаленную от точек А и В.

  13. Дано: А(2; 5; 8), В(6; 1;0).На оси ординат найти точку С, равноудаленную от точек А и В.
    Найти: площадь Δ
    ABC.

  14. Даны точки А(2; 1; 2), В(6; 3;2) на оси аппликат. Найти точку С, равноудаленную от
    точек А и В. Найти площадь ΔАВС.

  15. Середины сторон ΔАВС имеют координаты: М(3; 2; 4). N(6; 4; 10), К(7; 2; 12).
    Найдите координаты вершин ΔАВС.

  16. Даны точки А(4; 5; 4), В(2; 3; 4) на оси абсцисс. Найти точку С, равноудаленную от точек А и В. Найти площадь ΔABC

  17. Даны точки А(3; 1; 2) и В(1; 1; 2). Найдите: а) координаты середины отрезка АВ;

б) координаты и длину вектора  в) координаты точки С, если .

  1. Даны точки А(0; 4; 0), В(2; 0; 0), С(4; 0; 4) и D(2; 4; 4). Докажите, что ABCD - ромб.

  2. Даны точки А(0; 1; 2), В(√2 ; 1; 2), С(; 2; 1) и D(0; 2; 1). Докажите, что ABCD- квадрат.

  3. Даны точки А(2; 1; 8), В(1;5; 0), С(8; 1; 4).

Докажите, что ΔАВС - равнобедренный и найдите длину средней линии треугольника, соединяющей середины боковых сторон.

  1. Даны координаты трех вершин параллелограмма ABCD:
    А(
    6; 4; 0), В(6; 6; 2), С(10; 0; 4). Найдите координаты точки D и
    угол между векторами
      и .
    hello_html_793497f7.jpg

  2. Дано: О(0; 0; 0), А(4; 0; 0), В(0; 6; 0), С(0; 0; 2).
    Δ
    AОВ - вписанный в окружностьW(D; r).
    Найти: а) координаты центра окружности
     D;
    б)
     r- радиус окружности.

  3. Дано: ΔАВС - прямоугольный; АС, ВС - катеты;AC = b = 9 ;BC = a = 12;
    CD = m = 4; CD  (ABC); М - середина гипотенузы АВ. Найти: DM.




ПЗ № 10. Скалярное произведение векторов. Векторное уравнение прямой и плоскости. Использование векторов при доказательстве теорем стереометрии.

Задание:

1) Опорный конспект.

hello_html_m4c520382.jpg

hello_html_3a89eddb.png

hello_html_m5533c7c2.jpg


hello_html_ma8301e3.jpg

2) Перепишите и заполните пропуски:
1. м.диктант.

2. Дано

hello_html_m28d7893e.gif{-hello_html_m10ab8df5.gif; -hello_html_m10ab8df5.gif; -2}, hello_html_m355018cf.gif{hello_html_m10ab8df5.gif/2; hello_html_m10ab8df5.gif/2; 1}

Найдите угол между векторами hello_html_m28d7893e.gif и hello_html_m355018cf.gif.

Решение:

1) hello_html_m5f0379c8.gif. hello_html_49d81ced.gif.

2) hello_html_m506bc32c.gif– 2 ∙ 1 = –1 – 1 – 2 = –4 .

3) cos α =hello_html_22bfaa40.gif= –1 hello_html_70dfa0d8.gif α = ...°.

Ответ: 180°.

3.Дано: ,,. Выяснить: а) и, б) , .

Решение:

а)

, значит угол ….

б) = ++ = ,

в) == ++ = , угол ...

Ответ: а) тупой, б) острый, в) прямой.

4. а)Дано: ^=^=60º,

Вычислить: (+

Решение:

(+ = +

= )=1 =2=…

2)= )=2=4=…

3) (+ =+ = 1+2=…

Ответ: 3.

б)Дано: 3-5+ , -5

Найти: (- 2-2)

Решение:

1) , , 2 , , ,2

2)- 2 , -2

3)(- 2-2)=3+(-7)=…

Ответ: 28

5. ABCD - тетраэдр, =90º, AB=BD=2, BC=1.

а) Вычислите косинус угла между прямой, проходящей через середины ребер AD и BC,и прямой AC.

Решение:1). =


2) А (2, 0, 0) , В (0, 0, 0) , С (0, 1, 0) , D (0, 0, 2)

Т.к. М – середина [AD], то , т.е.

М (1, 0, 1), аналогично N (0, , 0)

,



= =

Ответ:

б) Вычислить синус угла между прямой MN и плоскостью грани ABD.

Решение:

- нормаль.

, .

=

Ответ:

6.Составить параметрические уравнения прямой, проходящей через точку M0 (1; 2; 0) и параллельной вектору s = (2; 0; −3).

Решение:





x = m t + x0

Запишем параметрические уравнения прямой: y = n t + y0





z = p t + z0


В нашем случае направляющий вектор прямой равен s = (2;0; −3), т.е. m = 2 , n = 0 и p = −3 .

Прямая проходит через точку M0 (1; 2; 0), поэтому

x0=1 , y0 = 2 z0 = 0 .

Подставим значения m = 2 ,

n = 0 , p = −3 , x0 =1 , y0 = 2

z0 = 0 в

параметрические уравнения прямой


x = m t + x0

x = 2 t +1

x = 2 t +1


y = n t

+ y0

y = 0 t + 2 ,




z = p t


z = − 3 t +0




Заметим, что прямая лежит в плоскости y = 2 , перпендикулярной оси

ординат. Поэтому прямая также перпендикулярна этой оси.

7. Найти уравнение плоскости, проходящей через точки A(1; 0; 2), B (2 ; 1; 1) и

параллельной оси OY .

Решение. В уравнении плоскости отсутствует переменная y , так как плоскость параллельна оси OY . Следовательно, уравнение плоскости, проходящей через

точку A(1; 0; 2), имеет вид

a ( x x0 ) + b ( y y0 ) + c ( z z 0 )= 0 a (x x0 )+ 0 ( y y0 ) + c ( z z 0 )= 0

a ( x 1) + c ( z 2 )= 0

Подставим в это уравнение координаты второй точки B (2 ; 1; 1). Получаем:

a ( x 1) + c ( z 2 )= 0 a ( 2 1) + c (1 2 )= 0 , a c = 0

Заметим, что один из неизвестных коэффициентов можно принять равным любому ненулевому числу (что повлияет только на изменение длины нормали плоскости). Например, полагаем a =1 , тогда из равенства ac = 0 находим c =1 . Подставим эти значения в уравнение искомой плоскости

a ( x 1) + c ( z 2 )= 0 x 1 + z 2 = 0 , x + z = 3

Ответ. x + z = 3 .

8. а)Если одна из двух параллельных прямых перпендикулярна к третьей, то и другая прямая перпендикулярна к этой прямой

a b

a c


bc

…………, bc.


б) Если одна из двух параллельных прямых перпендикулярна к плоскости, то и другая прямая перпендикулярна к этой плоскости

a b

a

b

Пусть х- произвольная прямая плоскости , тогда


a а…х

)…..)=

b…х b.

в)

hello_html_1d9831e5.png

9.


hello_html_35a224f7.gif

10.


hello_html_13920dcf.png




3)Решить задачи :


1.Вычислите скалярное произведение векторов и , если

2. Дан куб АBCDA1B1C1D1. Найдите угол между прямыми AD1 и ВМ, где М – середина ребра DD1.

3. При движении прямая а отображается на прямую а1, а плоскость α – на плоскость α1. Докажите, что если а║α, то а1║ α1.

4.а) Найти косинус угла между прямыми MN и DC;

б) Найти синус угла между прямой MN и плоскостью BDC.

  1. Дан квадрат ABCD. Найдите угол между векторами и .

  2. Найдите скалярный квадрат вектора = 7.

  3. Найдите скалярное произведение если = 60˚.

  4. Вычислите скалярное произведение векторов если

  5. ABCDA1B1C1D1 - куб, ребро которого равно 1. Найдите скалярное произведение

векторов и .

  1. Вычислите угол между прямыми АВ и CD, если А(; 1; 0), С( 0; 2; 0 ),

В(0; 0; 2), D().

11. Найдите координаты вектора hello_html_1dc502d9.gif, если А (5; –1; 3), В (2; –2; 4).

12. Даны векторы hello_html_54ee4636.gif(3; 1; –2) и hello_html_m9b9f52c.gif(1; 4; –3). Найдите hello_html_5b70e5.gif.

13. Изобразите систему координат Oxyz и постройте точку А (1; –2; –4). Найдите расстояния от этой точки до координатных плоскостей.

  1. Дан квадрат ABCD. Найдите угол между векторами и .

  2. Найдите скалярный квадрат вектора = 5+2.

  3. Найдите скалярное произведение если = 150˚.

  4. Вычислите скалярное произведение векторов если - 3 + и = 4 - .

  5. ABCDA1B1C1D1 - куб, ребро которого равно 2. Найдите скалярное произведение

векторов и .

  1. Вычислите угол между прямыми АВ и CD, если А(1; 1 ; 5 ), С(8 ; 5 ; 5 ),

В(4; 7; 5), D(5).

20. Доказать, что прямая, проведенная через середины оснований трапеции, проходит через точку пресечения продолжений боковых сторон.

21. Точка С середина отрезка АВ, а О – произвольная точка плоскости. Доказать, что

22.

hello_html_68aa67d4.pnghello_html_68aa67d4.png




ПЗ № 11. История развития комбинаторики, теории вероятностей и статистики и их роль в различных сферах человеческой жизнедеятельности. Правила комбинаторики. Решение комбинаторных задач.

1)Опорный конспект.

Человеку часто приходится иметь дело с задачами, в которых нужно подсчитать число всех возможных способов расположения некоторых предметов или число всех возможных способов осуществления некоторого действия.

В повседневной жизни нередко перед нами возникают проблемы, которые имеют не одно, а несколько различных вариантов решения. Чтобы сделать правильный выбор, очень важно не упустить ни один из них. Для этого надо осуществить перебор всех возможных вариантов или хотя бы подсчитать их число. Такого рода задачи называют комбинаторными.

Комбинаторика – ветвь математики, изучающая комбинации и перестановки предметов, казалось, долгое время лежала вне основного русла развития математики и ее приложений. На протяжении двух с половиной столетий основную роль в изучении природы играл математический анализ. Процессы, имевшие атомистическую природу, заменялись непрерывными, чтобы можно было применить к ним развитый аппарат математики. Положение коренным образом изменилось после создания быстродействующих вычислительных машин, компьютеров. С их помощью стало возможным делать переборы, ранее требовавшие сотен и тысяч лет. В эпоху расцвета дискретной математики изменилась и роль древнейшей области дискретной математики – комбинаторики. Из области, интересовавшей большей частью составителей занимательных задач и находившей основные применения в кодировании и расшифровке древних письменностей, она превратилась в область, находящуюся на магистральном пути развития науки. Стали выходить журналы по комбинаторике, печататься книги, посвященные этой науке.

В нынешнее время комбинаторика имеет огромное значение в различных областях науки и сферы. С комбинаторными величинами приходится иметь дело представителям многих специальностей: ученому – химику, биологу, конструктору, диспетчеру и т.п.

Ещё первобытный вождь понимал, что у десятка охотников вероятность поразить копьём зубра гораздо больше, чем у одного. Поэтому и охотились тогда коллективно.

Позднее, с опытом, человек всё чаще стал взвешивать случайные события, классифицировать их исходы как невозможные, возможные и достоверные. Он заметил, что случайностями не так уж редко управляют объективные закономерности.

Наиболее интересные для начинающих задачи комбинаторики и теории вероятностей возникли в области азартных игр. Этому, по-видимому, способствовало наличие монеты или игральной кости.

Одним из первых занялся подсчетом числа различных комбинаций при игре в кости итальянский математик Тарталья. Он составил таблицу, показывавшую, сколькими способами могут выпасть р костей. Однако при этом не учитывалось, что одна и та же сумма очков может быть получена разными способами.

Со временем появились различные игры (нарды, карты, шашки, шахматы и т.д.). В каждой из этих игр приходилось рассматривать различные сочетания фигур, и выигрывал тот, кто их лучше изучил, знал выигрышные комбинации и умел избегать проигрышных. Не только азартные игры давали пищу для комбинаторных размышлений математиков. Еще с давних пор дипломаты, стремясь к тайне переписки, изобретали сложные шифры, а секретные службы других государств пытались эти шифры разгадать. Стали применять шифры, основанные на комбинаторных принципах, например, на различных перестановках букв с использованием ключевых слов и т. д.

Как раздел математики комбинаторика возникла в XVI веке, так как для решения вероятностных задач необходимо было подсчитать число различных комбинаций элементов. Первые научные исследования по комбинаторике принадлежат итальянским ученым Дж. Кардано, Н. Тарталье (ок. 1499-1557 гг.), Г. Галилею (1564-1642 гг.) Дальнейшее развитие комбинаторики связано с трудами Б. Паскаля (1623 – 1662 гг.) и П. Ферма (1601 – 1665 гг.) по теории азартных игр. Позднее крупный вклад в развитие комбинаторных методов был сделан Г. Лейбницем (1646 – 1716 гг.), Я. Бернулли (1654 – 1705 гг.) и Л. Эйлером (1707 – 1783 гг.). В их работах были даны определения основных понятий комбинаторики, развиты первые комбинаторные методы и указаны их применения, а также прослежена связь комбинаторики с исчислением вероятностей. Именно комбинаторика послужила фундаментальной основой началам теории вероятностей. Немецкий учёный Г.Лейбниц в своей работе «Об искусстве комбинаторики», опубликованной в 1666 г., впервые выделил комбинаторику как самостоятельный раздел математики. Он также впервые ввел термин «комбинаторика».


Комбинаторика в различных областях жизнедеятельности человека.

Комбинаторика в литературе

В басне Ивана Андреевича Крылова «Квартет»: «проказница Мартышка, Осёл, Козёл да косолапый Мишка» устроили любопытный эксперимент, они исследовали влияние взаимного расположения музыкантов на качество исполнения.

Математика на шахматной доске и в играх.

Профессиональный интерес математиков к шахматам проявился довольно давно и был связан с двумя направлениями: математической логикой и комбинаторикой. Первое — рассмотрение игры с точки зрения построения ее формальной модели, удобной для логического анализа на основе действующих соревновательных правил. Второе — исследование конкретных позиций или их классов в игре для достижения определенных результатов, например матовой позиции за определенное число ходов. Последнее направление породило множество изящных логико-вычислительных проблем. Некоторые из них и по сей день предлагаются на различных математических и программистских олимпиадах, а также для развлечения на досуге.

Необыкновенно популярна головоломка - кубик Рубика, изобретенный в 1975 г. преподавателем архитектуры из Будапешта Эрне Рубиком для развития пространственного воображения у студентов. Задача поиска оптимального (по числу ходов) алгоритма сбора кубика Рубика является самой сложной и не решенной пока математической задачей.

Пароли и коды в нашей жизни.

Вся наша жизнь состоит из множества разнообразных программ. Чтобы запустить ту или иную программу нужно ввести соответствующий верный пароль.

В качестве кода в зависимости от рода программы могут выступать всевозможные цифры, слова или комбинации слов, поведение или действие, и так далее...

Когда мы узнаем что-то новое, развиваемся, к нам приходит жизненный опыт, он то, как раз и есть ничто иное, как набор всевозможных паролей, комбинаций. Ведь опытный человек всегда найдет лучшее решение в конкретной ситуации, потому – что он располагает большей комбинацией паролей.

Мебельная комбинаторика.

Мебельная комбинаторика позволяет рассматривать различные варианты комплектации предметов мебели и выбирать из них наилучшее, комфортнее и практичнее.

hello_html_m22e78f5d.jpg

Задачи на перебор возможных вариантов

Задачи на перебор возможных вариантов решаются следующим образом: выдвигается некая гипотеза, которая подтверждается или опровергается в ходе дальнейших рассуждений. Гипотезы выдвигаются до тех пор, пока не найдется единственно правильный вариант. Решение задач сопровождается записью рассуждений и там, где это необходимо для наглядности, графической иллюстрацией. В рассуждениях выдвижение гипотезы начинается со слова «Пусть». Если при рассмотрении какой-то гипотезы получен ответ на вопрос задачи, надо обязательно проверить и другие варианты для подтверждения, того, что найденное решение является единственным истинным.

1. Сколько двузначных чисел можно составить из цифр 1, 4 и 7? 

1

4

7

1

4

7

Перебором:

Ответ: 9 чисел.

2. Сколько чётных двузначных чисел можно составить из цифр 0, 1, 2, 4, 5, 9? 

0

2

4

1

10

12

14

2

20

22

24

4

40

42

44

5

50

52

54

9

90

92

94

5 * 3 = … чётных двузначных чисел

3. На завтрак Вова может выбрать плюшку, бутерброд, пряник или кекс, а запить их он может кофе, соком или кефиром. Из скольких вариантов завтрака Вова может выбрать?

Плюшка

Бутерброд

Пряник

Кекс

Кофе

Плюшка
Кофе

Бутерброд
Кофе

Пряник
Кофе

Кекс
Кофе

Сок

Плюшка
Сок

Бутерброд
Сок

Пряник
Сок

Кекс
Сок

Кефир

Плюшка
Кефир

Бутерброд
Кефир

Пряник
Кефир

Кекс
Кефир

  1. * 4 = 12 – вариантов завтрака/

4. В футбольном турнире участвуют несколько команд. Оказалось, что все они для трусов и футболок использовали белый, красный, синий и зеленый цвета, причем были представлены все возможные варианты. Сколько команд участвовали в турнире?
Решение. Трусы могут быть белого, красного, синего или зеленого цвета, т.е. существует 4 варианта. Каждый из этих вариантов имеет 4 варианта цвета майки.
4 х 4 = 16. Ответ: 16 команд.

2)Решить задание:

  1. Запишите все двузначные числа, в записи которых используются только цифры 3, 5, 7, 9. Сколько двузначных чисел можно записать, если использовать при записи числа каждую цифру только один раз?

  2. В четверг в первом классе должно быть три урока: русский язык, математика и физкультура. Сколько различных вариантов расписания можно составить на этот день?
    Указание: Перебирая варианты введите обозначения:
    Р – русский язык, М – математика, Ф – физкультура.

  3. Саша выбрал в библиотеке 5 книг, но одновременно можно взять только две книги. Сколько вариантов выбора двух книг есть у Саши?

  4. Школьники из Волгограда собрались на каникулы поехать в Москву, посетив по дороге Нижний Новгород. Сколькими различными способами могут ребята осуществить свое путешествие, если из Волгограда в Нижний Новгород можно отправиться на теплоходе ли поезде, а из Нижнего Новгорода в Москву – на самолете, теплоходе, поезде или автобусе?

  5. Девять школьников, сдавая экзамены по математике и английскому языку, получили отметки «4» и «5». Можно ли утверждать, что по крайней мере двое из них получили по каждому предмету одинаковые отметки?

  6. Сколько существует двузначных чисел, у которых первая цифра больше второй?

  7. Несколько стран в качестве символа своего государства решили использовать флаг в виде четырех вертикальных полос, одинаковых по ширине, но разных по цвету: белый, синий, красный, зеленый. У каждой страны свой, отличный от других, флаг.
    а) Сколько всего стран могут использовать такую символику?
    б) Сколько всего стран могут использовать такую символику с верхней белой полосой?
    в) Сколько всего стран могут использовать такую символику с нижней зеленой полосой?
    г) Сколько всего стран могут использовать такую символику с синей и красной полосами, расположенными рядом?

  8. За свои рисунки ученик получил две положительные оценки. Какими они могут быть? Сколько вариантов?

  9. Сколько флагов можно составить из трех разных цветов, если имеются полосы синего, белого, красного цветов?

  10. В понедельник в пятом классе 5 уроков. Сколькими способами можно составить расписание на понедельник?

  11. Из десяти учащихся надо выбрать старосту, физорга и культорга. Сколькими способами это можно сделать?

  12. Сколькими способами можно переставить 5 различных геометрических фигур?

  13. Пять человек обменялись друг с другом фотографиями. Сколько всего фотографий было?

  14. На плоскости отмечены 6 точек. Каждые две точки соединили отрезком. Сколько получилось отрезков?

  15. Сколькими способами можно расставить на полке 4 различные книги?

  16. Алфавит племени тумба-юмба состоит из букв А, У, С. Словом является любая последовательность из 4 букв. Сколько слов в языке этого племени?

  17. «Проказница Мартышка, Осел, Козел и косолапый Мишка затеяли сыграть квартет». Сколькими способами они могут распределить четыре имеющихся у них инструмента?

  18. а)Сколько двузначных чисел можно составить из цифр 1,4,5,7, используя в записи числа каждую из них не более одного раза?

б) На завтрак Вова может выбрать бутерброд, пряник или кекс, а запить их он может кофе, соком или кефиром. Из скольких вариантов завтрака Вова может выбирать?

  1. Сколькими способами можно представлять друг с другом цифры 1, 2, 3, 4?

  2. За столом пять мест. Сколькими способами можно расставить пятерых гостей?

  3. Путешественник из пункта А в пункт С может попасть, доехав до промежуточного пункта В по одной из трёх существующих автомагистралей, а из В в С доехать либо поездом, либо на такси. Сколько существует различных маршрутов между пунктами А и С?

  4. Стас решил пойти на новогодний карнавал в костюме мушкетёра. В ателье проката ему предложили на выбор различные по фасону и цвету предметы: 3 пары брюк, 4 камзола, 3 шляпы, 2 пары сапог. Сколько различных карнавальных костюмов можно составить из этих предметов?

  5. Олеся, Оксана и Юля купили билеты на концерт симфонического оркестра на 1, 2 и 3-е места первого ряда. Сколько существует способов размещения девочек на эти места?

  6. Сергей, Игорь и Миша могут занять 1-е, 2-е и 3-е призовые места в соревнованиях по шахматам. Перечислить всевозможные последовательности из имён мальчиков, где порядковый номер в последовательности соответствует занятому мальчиком месту в соревнованиях. Подсчитать их количество.

  7. В школьной столовой имеются помидоры, огурцы и лук. Сколько различных салатов можно приготовить, если в каждый из них должны входить в равных долях 2 различных вида овощей? Записать все сочетания овощей в составленных салатах.

  8. Девятиклассники Миша, Дима, Антон и Саша побежали на перемене к теннисному столу, за которым уже шла игра. Сколькими способами подбежавшие к столу четверо девятиклассников могут занять очередь для игры в настольный теннис?

  9. Сколько различных смешанных пар для игры в теннис можно образовать из восьми юношей и шести девушек?

  10. Для подарков первоклассникам закупили книги пяти разных авторов и игрушки шести разных видов. Сколько различных подарков можно составить, если в каждый должна входить одна книга и одна игрушка?

  11. В финальном забеге на 100 м участвуют Иванов, Громов и Орлов. Назовите возможные варианты распределения призовых мест.

  12. В кружок бального танца записались Петя, Коля, Витя, Олег, Таня, Оля, Наташа, Света. Какие танцевальные пары девочки и мальчика могут образоваться?

  13. Запишите все возможные варианты расписания пяти уроков на день из предметов: математика, русский язык, история, английский язык, физкультура, причем математика должна быть вторым уроком.

  14. Маша, Оля, Вера, Ира, Андрей, Миша и Игорь готовились стать ведущими на Новогоднем празднике. Назовите возможные варианты, если ведущими могут быть только одна девочка и один мальчик.

  15. 6 учеников сдают зачет по математике. Сколькими способами их можно расположить в списке?

  16. Сколько четных двузначных чисел можно составить из цифр 0, 2, 3, 4, 6, 7?

ПЗ № 12. Размещения, сочетания и перестановки. Бином Ньютона и треугольник Паскаля. Прикладные задачи.

1)Опорный конспект.

hello_html_m3c3aae1b.jpg

Понятие бинома Ньютона
Биномом Ньютона называют разложение вида:
hello_html_3109c0eb.gif
Но, строго говоря, всю формулу нельзя назвать биномом, так как «бином» переводится как «двучлен». Кроме того, формула разложения была известна еще до Ньютона, Исаак Ньютон распространил это разложение на случай n<0 и n – дробного.
Цель изучения бинома Ньютона – упрощение вычислительных действий.
Компоненты формулы «бином Ньютона»:правая часть формулы – разложение бинома;
hello_html_m7f7221a0.gif


hello_html_m3030b558.gif – биномиальные коэффициенты, их можно получить с помощью треугольника Паскаля (пользуясь операцией сложения).


Практическая значимость треугольника Паскаля заключается в том, что с его помощью можно запросто восстанавливать по памяти не только известные формулы квадратов суммы и разности, но и формулы куба суммы (разности), четвертой степени и выше.
Например, четвертая строчка треугольника как раз наглядно демонстрирует биномиальные коэффициенты для бинома четвертой степени:
hello_html_4d3e168.gif
Альтернатива треугольнику Паскаля:

  1. перемножить почленно четыре скобки:
    hello_html_c44bb7f.gif;

  1. вспомнить разложение бинома Ньютона четвертой степени:
    hello_html_m32884135.gif
    общий член разложения бинома n-й степени: 
    hello_html_m6cb3ccf5.gif,
    где Т – член разложения; 
    hello_html_m5cd3cd0b.gif – порядковый номер члена разложения.
    Свойства бинома и биномиальных коэффициентов

  1. hello_html_m40d1b2c3.gif

  2. Число всех членов разложения на единицу больше показателя степени бинома, то есть равно hello_html_m38da65e0.gif

  3. Сумма показателей степеней a и b каждого члена разложения равна показателю степени бинома, то есть n. Доказательство: Рассмотрим hello_html_m5cd3cd0b.gif-й член разложения: hello_html_583ec35b.gif
    Сумма показателей степеней 
    a и bhello_html_me119c4e.gifЧ.т.д

  1. Биномиальные коэффициенты членов разложения, равноотстоящих от концов разложения, равны между собой: hello_html_m6c57fe34.gif (правило симметрии)

  2. Сумма биномиальных коэффициентов всех членов разложения равна hello_html_29daa9a6.gif
    hello_html_3bb7339e.gif
     
    hello_html_5a27b673.gifhello_html_m61d3eb50.gif

  1. Сумма биномиальных коэффициентов, стоящих на нечетных местах, равна сумме биномиальных коэффициентов, стоящих на четных местах и равнаhello_html_m4cf34de5.gif


hello_html_m63a4870a.gif

  1. Правило Паскаля: hello_html_2b56e16.gif

  1. Любой биномиальный коэффициент, начиная со второго, равен произведению предшествующего биномиального коэффициента и дроби hello_html_2d89af74.gif,hello_html_m714b47b0.gif
    Типовые задачи по теме «Бином Ньютона»
    К типовым (стандартным) заданиям по данной теме можно отнести задачи на вычисление, среди которых:

    1. Найти член (номер члена) разложения бинома

    2. Вывести бином по известным членам разложения (по известной сумме)


Вычислить сумму биномиальных коэффициентов разложения бинома
и другие.
Продемонстрируем на примерах (их решение несложное, поэтому большинство предлагаем решить самостоятельно).
Пример .В биномиальном разложении hello_html_7d2a897c.gif найти член разложения, не содержащий х
Решение:
hello_html_m34d61411.gif
Так как в разложении мы ищем член не содержащий 
х, то hello_html_1568eb8e.gif
Тогда 
hello_html_m660919.gif
Ответ: 
hello_html_44d6f621.gif
Задачи, сводящиеся к использованию формулы бинома Ньютона 
(нестандартные задачи по теме «Бином Ньютона»)
К нестандартным заданиям по данной теме можно отнести такие, в которых нет явного намека на необходимость использования бинома. Однако в итоге, решение сводится к нему и выглядит очень интересным.
Пример1. Доказать, что для любых hello_html_44cfd3a4.gif и для любых hello_html_66e6eb8a.gif верно неравенство Бернулли:

hello_html_m19bbb27c.gif . Доказательство: Пусть hello_html_m761619fc.gif
Так как 
hello_html_44cfd3a4.gif, то hello_html_35376388.gif
Переформулируем требование: Доказать, что 
hello_html_m1fdc89ff.gif, где hello_html_m6810b779.gif

hello_html_6388f0a4.gif
Так как 
hello_html_m4f82866c.gif, значит в разложении как минимум три члена разложения, тогда:
hello_html_5b8534f5.gif
Это означает, что 
hello_html_m19bbb27c.gifЧ.т.д.
Пример 2 . Доказать, что при любом натуральном n число hello_html_1f8d1336.gif делится на 9

Доказательство:hello_html_1dac8434.gif
hello_html_m2a8a54c5.gifЧ.т.д.
Пример 3.Решить уравнение hello_html_m10b576fe.gif

Решение: Осуществим замену: hello_html_22b9cd2c.gif
Тогда уравнение перепишем: 
hello_html_6f72da60.gif
Применим формулу бинома к левой части уравнения: 
hello_html_7c84c5a2.gif
В итоге 
hello_html_m45e4118a.gif
Ответ: 
hello_html_7054713a.gif

2)Перепишите и заполните пропуски:

Пример 1. а) Сколько трехзначных чисел можно записать, используя цифры 1,3,6,7,9, если каждая их них может быть использована в записи только один раз?

Решение: по формуле получаем: способов.

Ответ:60.

б) Из 20 учащихся надо выбрать старосту, его заместителя и редактора газеты. Сколькими способами это можно сделать?

Решение: по формуле получаем: способов.

Ответ:6840.

в)Сколькими способами четверо юношей могут пригласить четырех из шести девушек на танец?

Решение: Два юноши не могут одновременно пригласить одну и ту же девушку. И варианты,

при которых одни и те же девушки танцуют с разными юношами, считаются разными, поэтому:

Ответ: 360.

Пример 2. а)Сколькими способами можно представлять друг с другом цифры 1, 2, 3, 4?

Решение: Р4 = 4!= = …

Ответ: 24.

б)За столом пять мест. Сколькими способами можно расставить пятерых гостей?

Решение: Р5 = 5! =

Ответ:120.

в)Сколькими способами можно выложить в ряд красный, черный, синий и зеленый шарики?

Решение: На первое место можно поставить любой из четырех шариков (4 способа), на

второе – любой из трех оставшихся (3 способа), на третье место – любой из

оставшихся двух (2 способа), на четвертое место – оставшийся последний шар.

Всего 4 · 3 · 2 · 1 = 24 способа. Р4 = 4! = 1 · 2 · 3 · 4 = 24. Ответ: 24 способа.

г)Сколькими способами можно переставить буквы слова «ананас»?

Решение: всего букв 6. Из них одинаковы n1«а»=3, n2«н»=2, n3«с»=1. Следовательно, число различных перестановок равно

Ответ:60.

Пример 3. а) Из 15 членов туристической группы надо выбрать трех дежурных. Сколькими способами можно сделать этот выбор?

Решение: каждый выбор отличается от другого хотя бы одним дежурным. Значит, здесь речь идет о сочетаниях из 15 элементов по 3. Следовательно, по формуле получаем

Ответ:455.

б) На полке стоит 12 книг: англо-русский словарь и 11 художественных произведений на английском языке. Сколькими способами читатель может выбрать 3 книги, если :

1) словарь нужен ему обязательно; 2) словарь ему не нужен?

Решение:

1) 2)

Ответ: 1) 55,2) 165.

в) Учащимся дали список из 10 книг, которые рекомендуется прочитать во время каникул. Сколькими способами ученик может выбрать из них 6 книг?


Решение: Выбор 6 из 10 без учёта порядка: способов.

Ответ: 210 способов.

г) Сколько трехкнопочных комбинаций существует на кодовом замке (все три кнопки нажимаются одновременно), если на нем всего 10 цифр.

Решение: Так как кнопки нажимаются одновременно, то выбор этих трех кнопок – сочетание. Отсюда возможно вариантов.

Ответ:120.

Пример 4. а) Имеются 10 различных книг, три из которых – справочники. Сколькими способами

можно расставить эти книги на полке так, чтобы все справочники стояли рядом?

Решение: Т.к. в справочники должны стоять рядом, то будем рассматривать их как одну книгу. Тогда на полке надо расставить 10 – 3 + 1= … книг. Это можно сделать P8 способами. Для каждой из полученных комбинаций можно сделать P3 перестановок справочников.

Поэтому число способов расположения книг на полке равно произведению:

P8 · P3 = 8! · 3! = 40320 · 6 = ...

Ответ: 241920.

б) Сколько всего существует результатов опыта, заключающегося в подбрасывании двух одинаковых игральных костей?

Решение: Формула числа сочетаний из m элементов по n элементов с повторениями имеет вид:

,
Ответ: 21.

Пример 5. Сколько различных перестановок можно образовать из букв слова «задача»?

Решение: Образовать какую – либо перестановку из букв слова «задача» - это значит на шесть занумерованных мест, каким – нибудь образом поставить одну букву «з», одну букву «д», одну букву «ч» и три буквы «а». Если буквы «з», «д» и «ч» как – то поставлены, то остальные места заполняются буквами «а». Но сколькими способами можно поставить три различные буквы на шесть мест? Очевидно, что число способов равно числу всех трёхэлементных упорядоченных подмножеств шестиэлементного множества, т.е. равно

А63 = 6 ∙5 ∙4 = …

Ответ: 120 различных перестановок.

Пример 6. а) Сколькими способами можно разместить на полке 5 книг?

Решение: Задача сводится к подсчёту числа перестановок из пяти элементов:

Р5 = 5! = 1 ∙2 ∙3 ∙4 ∙5 = … способов.

Ответ: 120 способов

б) Сколькими способами могут быть расставлены 8 участниц финального забега на 8-ми беговых дорожках?

Решение: Р8 = 8!= 1 ∙2 ∙3 ∙4 ∙5 ∙ 6 ∙7 ∙8 = 40320 способов.

Ответ: 40320 способов забега.

Пример 7. В чемпионате страны по футболу (высшая лига) участвуют 18 команд, причём каждые две команды встречаются между собой 2 раза. Сколько матчей играется в течение сезона?

Решение: В первом круге состоится столько матчей, сколько существует двухэлементных подмножеств у множества, содержащего 18 элементов, т.е. их число равно С182. По формуле (4) получаем

18∙17

С218 = 2 =153.

Во втором круге играется столько же матчей, поэтому в течение сезона состоится … встреч.

Ответ: 306 встреч.

Пример 8. Олеся, Оксана и Юля купили билеты на концерт симфонического оркестра на 1, 2 и 3-е места первого ряда. Сколько существует способов размещения девочек на эти места?

Решение: Количество различных способов равно числу перестановок из 3 элементов:

Р3 = 3! = 1×2×3 = …способов.

Ответ: 6 способов.

Пример 9. Сколькими способами можно составить флаг, состоящий из трех горизонтальных полос различных цветов, если имеется материал пяти цветов?

Решение. Искомое число трехполосных флагов:

hello_html_469d74c0.gif

Пример 10. Сколькими способами можно в игре “Спортлото” выбрать 5 номеров из 36?

Искомое число способов
hello_html_4df0ad63.gif


3)Решить задание ( по примерам):

  1. а) Сколько трехзначных чисел можно записать, используя цифры 1,2,4,6,7,9, если каждая их них может быть использована в записи только один раз?

б) Из 15 учащихся надо выбрать старосту, его заместителя и редактора газеты. Сколькими способами это можно сделать?

в)Сколькими способами четверо юношей могут пригласить четырех из пяти девушек на танец?

  1. а)Сколькими способами можно представлять друг с другом цифры 1, 2, 3, 4,5?

б)За столом семь мест. Сколькими способами можно расставить семерых гостей?

в)Сколькими способами можно выложить в ряд красный, черный, синий, белый и зеленый шарики?

г)Сколькими способами можно переставить буквы слова «Миссисипи»?

  1. а) Из 25 членов туристической группы надо выбрать трех дежурных. Сколькими способами можно сделать этот выбор?

б) На полке стоит 15 книг: англо-русский словарь и 14 художественных произведений на английском языке. Сколькими способами читатель может выбрать 3 книги, если :
1) словарь нужен ему обязательно; 2) словарь ему не нужен?

в) Учащимся дали список из 10 книг, которые рекомендуется прочитать во время каникул. Сколькими способами ученик может выбрать из них 7 книг?

г) Сколько четырехкнопочных комбинаций существует на кодовом замке (все три кнопки нажимаются одновременно), если на нем всего 10 цифр.

  1. а)Сколько двузначных чисел можно составить из цифр 1,4,5,7, используя в записи числа каждую из них не более одного раза?

б) На завтрак Вова может выбрать бутерброд, пряник или кекс, а запить их он может кофе, соком или кефиром. Из скольких вариантов завтрака Вова может выбирать?

  1. а)Имеются 10 различных книг, 6 из которых – справочники. Сколькими способами

можно расставить эти книги на полке так, чтобы все справочники стояли рядом?
б)
Сколько всего существует результатов опыта, заключающегося в подбрасывании трех одинаковых игральных костей?

4)Решить задание:
Подсчет числа размещений, перестановок, сочетаний.

  1. «Вороне где-то Бог послал кусочек сыра», колбасы, хлеба и шоколада. «На ель Ворона взгромоздясь, позавтракать совсем уж было собралась, да призадумалась»: если есть кусочки по очереди, то из скольких вариантов придется выбирать?

  2. Сколькими способами можно из 25 учащихся выбрать 5 для участия в школьном марафоне?

  3. Сколькими способами могут быть распределены золотая и серебряная медали по итогам первенства по футболу, если число команд 12?

  4. В классе 7 человек успешно занимаются математикой. Сколькими способами можно выбрать из них двоих для участия в математической олимпиаде?

  5. Из 12 солдат нужно в разведку послать 5. Сколькими способами это можно сделать?

  6. Сколько пятизначных чисел можно составить, используя только цифры 3 и 5?

  7. «Проказница Мартышка, Осел, Козел и косолапый Мишка затеяли сыграть квартет». Сколькими способами они могут распределить четыре имеющихся у них инструмента?

  8. «Проказница Мартышка, Осел, Козел и косолапый Мишка затеяли сыграть квартет». На складе 12 музыкальных инструментов. Мишке поручили принести со склада 8 любых инструментов. Сколько вариантов выбора есть у мишки?

  9. Гера, Афина и Афродита попросили Париса не только назвать самую красивую из них, но и указать, кто «на втором и третьем местах». Сколько есть вариантов ответа?

  10. В магазине «Филателия» продается 8 различных наборов марок, посвященных «Дню Победы». Сколькими способами можно сформировать из них 3 набора?

  11. В классе 27 учеников, из которых нужно выбрать троих: первый ученик должен решить задачу, второй – сходить за мелом, третий – пойти дежурить в столовую. Сколькими способами это можно сделать?

  12. Сколькими способами можно из 6 человек составить комиссию, состоящую из двух человек?

  13. В соревновании участвуют 10 человек. Сколькими способами могут распределиться между

  14. Сколькими способами можно переставить 5 различных геометрических фигур?

  15. Сколькими способами можно выбрать гласную и согласную буквы из слова «здание»?

  16. За свои рисунки ученик получил две положительные оценки. Какими они могут быть? Сколько вариантов?

  17. В соревновании участвуют 10 человек. Сколькими способами могут распределиться между ними места?

  18. У одного человека 7 книг по математике, а у второго – 9. Сколькими способами они могут обменять друг у друга две книги на две книги.

  19. Имеется пять различных стульев и семь рулонов обивочной ткани различных цветов. Сколькими способами можно осуществить обивку стульев.

  20. На ферме есть 20 овец и 24 свиньи. Сколькими способами можно выбрать одну овцу и одну свинью? Если такой выбор уже сделан, сколькими способами можно сделать его еще раз?

  21. Сколько существует четных пятизначных чисел, начинающихся нечетной цифрой?

  22. В 9 классе учатся 7 учащихся, в 10 - 9 учащихся, а в 11 - 8 учащихся. Для работы на пришкольном участке надо выделить двух учащихся из 9 класса, трех – из 10, и одного – из 11 . Сколько существует способов выбора учащихся для работы на пришкольном участке?

  23. Сколько наборов из семи пирожных можно составить, если в продаже имеется четыре сорта пирожных?

  24. На памятные сувениры в «Поле Чудес» спонсоры предлагают кофеварки, утюги, телефонные аппараты, духи. Сколькими способами 9 участников игры могут получить эти сувениры? Сколькими способами могут быть выбраны 9 предметов для участников игры?

  25. Сколькими способами можно выложить в ряд красный, черный, синий и зеленый шарики?

  26. Сколько экзаменационных комиссий, состоящих из 7 членов, можно образовать из 14 преподавателей?

  27. Сколькими способами могут быть расставлены 6 участниц финального забега на 6-ми беговых дорожках?

  28. Сколькими способами можно разместить на полке 4 книг?

  29. Сколько различных двузначных чисел можно составить из цифр 1, 2, 3, 4, 5, 6, 7, 8 при условии, что в записи числа каждая цифра используется только один раз?

  30. Из слова РОТ перестановкой букв можно получить еще такие слова: ТОР, ОРТ, ОТР, ТРО, РТО. Их называют анаграммами. Сколько анаграмм можно составить из слова ЛОГАРИФМ?

ПЗ № 13. Классическое определение вероятности, свойства вероятностей, теорема о сумме вероятностей.

1)Опорный конспект.

При классическом способе подсчета вероятность события А вычисляется по формуле: Р(А)=m/n, где:

все элементарные исходы равновозможны, т.е. ни один из них не является более возможным, чем другой;m – число элементарных исходов испытания, благоприятствующих появлению события А;

n – общее число всех возможных элементарных исходов испытания.

Отметим основные свойства вероятности.
1. Вероятность любого события заключена между нулем и единицей:hello_html_m54c11b95.png

2. Вероятность достоверного события I, т. е. такого события, которое при испытании обязательно произойдет, равна единице: hello_html_m74a48814.png

3. Вероятность невозможного события О, т. е. события, которое в результате испытания не может произойти, равна нулю: hello_html_m15aeba51.png

4. Сумма вероятностей двух противоположных событий А и А, т. е. таких событий, что появление одного из них исключает появление другого, равна единице: hello_html_m2bca0899.png

Вероятностью события А называют отношение числа m исходов испытаний, благоприятствующих наступлению события А, к общему числу n всех равновозможных несовместных исходов: Р(А)=m/n.

Условной вероятностью события А (или вероятностью события А при условии, что наступило событие В), называется число РВ(А) = Р(АВ)/Р(В), где А и В – два случайных события одного и того же испытания.

Суммой конечного числа событий называется событие, состоящее в наступлении хотя бы одного из них. Сумма двух событий обозначается А+В.

Правила сложения вероятностей:

правило сложения вероятностей совместных событий А и В:
Р(А+В) = Р(А)+Р(В)-Р(АВ), где Р(А) – вероятность события А, Р(В) – вероятность события В, Р(А+В) – вероятность появления хотя бы одного из двух событий, Р(АВ)- вероятность совместного появления двух событий.

правило сложения вероятностей несовместных событий А и В:
Р(А+В) = Р(А)+Р(В), где Р(А) – вероятность события А, Р(В) – вероятность события В.

2) Перепишите и заполните пропуски:
Пример 1. a)В партии из 100 деталей имеется 5 бракованных. Определить вероятность того, что, взятая наугад, деталь окажется стандартной.

Решение: А: взятая наугад деталь оказалась стандартной.

Число исходов, благоприятствующих наступлению события А, равно 95.Поэтому вероятность события равна P(A) = m/ n = 95/100 = … .hello_html_m27618eb7.gif Ответ: 0,95.

б) Из пяти букв разрезной азбуки составлено слово «книга». Ребенок, не умеющий читать, рассыпал эти буквы, а затем собрал их в произвольном порядке. Найти вероятность того, что у него снова получится слово «книга».

Решение: А: из рассыпанных букв сложится слово «книга»

Число всех возможных исходов равно n = Pn = 5! = 120.

Число исходов, благоприятствующих событию А равно m =1.

Вероятность события А равна P(A) = m/ n = 1/120 = … .hello_html_m27618eb7.gif 

Ответ: 0,0083.

Пример 2.a) В коробке лежат 8 зеленых, 7 синих и 15 красных карандашей. Вычислить вероятность того, что взятый наугад карандаш будет, синим или зеленым.

Решение: А: взяли синий карандаш, В: взяли зеленый карандаш, С: взяли синий или зеленый карандаш. Событие С равно сумме событий А и В: С = А + В

Вероятность события А равна P(A) = m/ n = 7/30. 

Вероятность события В равна P(B) = m/ n = 8/30. 

Вероятность события С равна P(C) = P(A) = 7/30 8/30 = 15/30 = ... Ответ: 0,5. б) В урне лежат шары, двузначные номера которых составлены из цифр 1,2,3,4,5. Какова вероятность вынуть шар с номером 15? Решение: А: вынут шар с номером 15.

Число всех возможных исходов равно n =

Число исходов, благоприятствующих событию А, m = 1.

Вероятность события А равна P(A) = m/ n = 1/20 = … .

Ответ: 0,05.

Пример 3.a) Набирая номер телефона, абонент забыл последние две цифры и, помня лишь, что эти цифры различны, набрал их наугад. Найти вероятность того, что набраны нужные цифры.

Решение: А: абонент наугад набрал нужные цифры.

Число всех возможных исходов равно n =

Число исходов, благоприятствующих событию А, m = 1

Вероятность события А равна P(A) = m/ n = 1/90 = .... 

Ответ: 0,011.

б) Устройство содержит два независимо работающих элемента. Вероятности отказа элементов равны соответственно 0,05 и 0,08. Найти вероятности отказа устройства, если для этого достаточно, чтобы отказал хотя бы один элемент. 

Решение: Пусть событие А — «устройство не работает», В1 — «отказал первый элемент», 

В2 — « отказал второй элемент». Событие А соответствует тому, что может отказать один из «цементов либо оба элемента. События  В1 и В2  независимы в совокупности, поэтому:

q1 = 10,05 = 0,95,   q2 = 10,08 = 0,92. P(A) = 1 q1q2= 10,950,92 = 10,874 = ...

Ответ:  0,126.

Пример 4. a)Вероятность хотя бы одного попадания в цель при четырех выстрелах равна 0,9984. Найти вероятность попадания в цель при одном выстреле.

Решение: Пусть p - вероятность попадания в цель при одном выстреле. Введем событие 

X = {при четырех выстрелах есть хотя бы одно попадание} и противоположное ему событие  

= {при четырех выстрелах нет ни одного попадания}.

Вероятность события  равна P(  ) = (1p)4, тогда вероятность события Х равна 

P(X) =1P(  ) = 1 (1p)4. По условию эта вероятность равна 0,9984, откуда получаем уравнение относительно p: 1 (1p)4 = 0,9984, (1p)4 = 0,0016, (1p) = 0,2, p = ...

Таким образом, вероятность попадания в цель при одном выстреле равна 0,8.

Ответ: 0,8.

б)На полке в случайном порядке расставлено 40 книг, среди которых находится трехтомник Пушкина. Найти вероятность того, что эти тома стоят в порядке возрастания номера слева направо, но не обязательно рядом.

Решение: Используем классическое определение вероятности: P = m/n, где n- число всех равновозможных элементарных исходов, m - число элементарных исходов, благоприятствующих осуществлению события  A = (Тома стоят в порядке возрастания номера слева направо, но не обязательно рядом). n= 40⋅39⋅38 =59280, так как первый том можно поставить на любое из 40 мест, второй - на любое из 39 мест и третий - на любое из оставшихся 38 мест.

А число m= 40! / (37! 3!) = (40⋅39⋅38) : (1⋅2⋅3) = ...

Тогда искомая вероятность P(A)= m/n = 9880/59280 = 1/6. Ответ: 1/6.

Пример 5. а)В коробке имеется 250 лампочек, из них 100 по 90Вт, 50 - по 60Вт, 50 - по 25Вт и 50 – по 15Вт. Определить вероятность того, что мощность любой наугад взятой лампочки не превысит 60Вт.

Решение: 1. Рассматриваем следующие события: А = {мощность лампочки равна 90Вт}, вероятность Р(А) = 100/250 = 0,4; В = {мощность лампочки равна 60Вт}; С = {мощность лампочки равна 25Вт}; D = {мощность лампочки равна 15Вт}.

2. События А, В, С, D образуют полную систему, так как все они несовместны и одно из них обязательно наступит в данном опыте (выборе лампочки). Вероятность наступления одного из них есть достоверное событие, тогда Р (А)Р (В)Р (С)Р (D) = 1.

3. События {мощность лампочки не более 60Вт} (т.е. меньше или равна 60Вт), и {мощность лампочки более 60Вт} (в данном случае – 90Вт) являются противоположными. По свойству противоположных чисел Р (В)Р (С)Р (D) = 1Р (А).

4. Учитывая, что Р (В)Р (С)Р (D) = Р (ВСD), получим

Р (В СD) = 1Р (А) = 10,4 = ... Ответ: 0,6. 

б) Вероятность поражения цели первым стрелком при одном выстреле равна 0,7, а вторым стрелком – 0,9. Найти вероятность того, что 

1) цель будет поражена только одним стрелком; 2) цель будет поражена хотя бы одним стрелком.

Решение: 1. Рассматриваем следующие события:
А
1 = {первый стрелок поражает цель}, Р (А1) = 0,7 из условия задачи;
1 = {первый стрелок промахнулся}, при этом Р (А1)Р (1) = 1, поскольку А1 и А̄1 – противоположные события. Отсюда Р (1) = 10,7 = …;
А
2 = {второй стрелок поражает цель}, Р (А2) = 0,9 из условия задачи;
2 = {второй стрелок промахнулся}, при этом Р (2) = 10,9 = …

2. Событие А={цель поражена только одним стрелком} означает, что наступило одно из двух несовместных событий: либо А12, либо 1А2.
По правилу сложения вероятностей Р (А) = Р (А12) + Р (1А2).По правилу умножения вероятностей независимых событий:
Р (А12) = Р (А1)Р (2) = 0,70,1= 0,07; Р (1А2) = Р (1)Р (А2) = 0,30,9 = ...
Тогда Р (А)= Р (А12) Р (1А2) = 0,070,27 = ...

3. Событие B ={цель поражена хотя бы одним стрелком} означает, что либо цель поразил первый стрелок, либо цель поразил второй стрелок, либо цель поразили оба стрелка.

Событие = {цель не поражена ни одним стрелком} является противоположным событию В, а значит Р(В) = 1Р ().
Событие B̄ означает одновременное появление независимых событий 1 и 2, следовательно Р () = Р (12) = Р (1)Р (2) = 0,30,1 = 0,03. Тогда Р (В) = 1Р () = 10,03 = ...

Ответ: 1) 0,34; 2) 0,97.

3.Решить задачи ( по примерам):

  1. a)В партии из 100 деталей имеется 3 бракованных. Определить вероятность того, что, взятая наугад, деталь окажется стандартной.

б) Из 4 букв разрезной азбуки составлено слово «мама». Ребенок, не умеющий читать, рассыпал эти буквы, а затем собрал их в произвольном порядке. Найти вероятность того, что у него снова получится слово «мама».

  1. a)В коробке лежат 5 зеленых, 3 синих и 12 красных карандашей. Вычислить вероятность того, что взятый наугад карандаш будет, синим или зеленым.

б) В урне лежат шары, двузначные номера которых составлены из цифр 1,2,3,4. Какова вероятность вынуть шар с номером 123?

  1. a)Набирая номер телефона, абонент забыл последние 3 цифры и, помня лишь, что эти цифры различны, набрал их наугад. Найти вероятность того, что набраны нужные цифры.

б) Устройство содержит два независимо работающих элемента. Вероятности отказа элементов равны соответственно 0,04 и 0,09. Найти вероятности отказа устройства, если для этого достаточно, чтобы отказал хотя бы один элемент. 

  1. a)Вероятность хотя бы одного попадания в цель при четырех выстрелах равна 0,9919. Найти вероятность попадания в цель при одном выстреле.

б)На полке в случайном порядке расставлено 21 книга, среди которых находится трехтомник Пушкина. Найти вероятность того, что эти тома стоят в порядке возрастания номера слева направо, но не обязательно рядом.

  1. a)В коробке имеется 200 лампочек, из них 60 по 90Вт, 60 - по 60Вт, 40 - по 25Вт и 40 – по 15Вт. Определить вероятность того, что мощность любой наугад взятой лампочки не превысит 60Вт.

б) Вероятность поражения цели первым стрелком при одном выстреле равна 0,4, а вторым стрелком – 0,7. Найти вероятность того, что 

1) цель будет поражена только одним стрелком;2) цель будет поражена хотя бы одним стрелком.

  1. Экспедиция издательства отправила газеты в три почтовых отделения. Вероятность своевременной доставки газет в первое отделение равна 0,95, во второе – 0,9, в третье – 0,8. Найти вероятность того, что, хотя бы одно отделение получит газеты с опозданием.

  2. Автомобилист проезжает два поста дорожно-патрульной службы. Вероятность того, что его остановят на первом посту, равна 0,4, на втором – 0,1. Найти вероятность того, что автомобилиста остановят хотя бы на одном посту.

  3. Производится наблюдение за группой, состоящей из четырех однородных объектов. Каждый из них за время наблюдения может быть обнаружен или не обнаружен. Рассматриваются события:

D — обнаружено ровно два объекта; Е — обнаружено ровно три объекта;

F — обнаружены все четыре объекта. Указать, в чем состоит событие: D + Е + F

  1. В урне a – белых, b – черных, c – красных шаров. Вероятность какого события определяется формулой /

  2. Найти вероятность того, что при извлечении наугад одного шара из корзины, в которой находятся 2 белых, 3 зеленых и 5 красных шаров, извлеченный шар окажется зеленым.

4.Решить задачи :

  1. В урне находится 10 шаров, из них 6 белых и 4 черных шара. Вынули из урны 2 шара. Какова вероятность того, что оба шара - белые?

  2. В секретном замке на общей оси 4 диска, каждый из которых разделен на 5 секторов, на которых написаны различные цифры. Замок открывается, если диски установлены так, что цифры на них составляют определенное четырехзначное число. Найти вероятность того, что при произвольной установке дисков замок будет открыт.

  3. Батарея, состоящая из 10 орудий, ведет огонь по 15 кораблям неприятеля. Найти вероятность того, что все орудия стреляют: а) по одной цели; б) по разным целям/

  4. В партии из 10 деталей 7 стандартных. Найти вероятность того, что среди шести взятых наудачу деталей 4 стандартных.

  5. Набирая номер телефона, абонент забыл последние 3 цифры и, помня лишь, что эти цифры различны, набрал их на удачу. Найти вероятность того, что набраны нужные цифры.

  6. В ящике 50 годных и 16 дефектных деталей. Сборщик наудачу достает 8 деталей. Найти вероятность того, что среди них: а) нет дефектных; б) 3 дефектных.

  7. В группе из 30 студентов на контрольной работе 6 студентов получили «5», 10 студентов – «4», 9 студентов – «3», остальные – «2». Найти вероятность того, что 3 студента, вызванные к доске, получили по контрольной работе «2».

  8. В почтовом отделении имеются открытки 6 видов. Какова вероятность того, что среди 4 проданных открыток все открытки различны?

  9. Выбирают наугад число от 1 до 100. Определить вероятность того, что в этом числе не окажется цифры 3.

  10. По условию лотереи «Спортлото 5 из 36» участник, угадавший 4 цифры из 5, получает второй приз. Найдите вероятность такого выигрыша.

  11. Набирая номер телефона, абонент забыл одну цифру и набрал ее наудачу. Найти вероятность того, что набрана нужная цифра.

  12. В кармане 3 пятикопеечные монеты и 7 десятикопеечных монет. Наугад берется одна за другой две монеты. Вторая оказалась десятикопеечной. Определить вероятность того, что и первая десятикопеечная.

  13. Набирая номер телефона, абонент забыл последние две цифры и, помня лишь, что эти цифры различны, набрал их наудачу. Найти вероятность того, что набраны нужные цифры.

  14. Собрание, состоящее из 30 человек, среди которых 8 женщин, выбирает делегацию из 3 человек. Найти вероятность того, что в делегацию войдет одна женщина.

  15. Брошены две игральные кости. Найти вероятность того, что: а) сумма выпавших очков не превосходит семи; б) на обеих костях выпадет одинаковое число очков; в) произведение выпавших очков делится на 4; г) хотя бы на одной кости выпадет 6.

  16. Из 20 сбербанков 10 расположены за чертой города. Для обследования случайным образом отобрано 5 сбербанков. Какова вероятность того, что среди отобранных окажется 3 сбербанка в черте города?

  17. 8 шахматистов, среди которых 3 гроссмейстера, путем жеребьевки делятся на две подгруппы по 4 человека. Какова вероятность того, что два гроссмейстера попадут в одну подгруппу?

  18. В группе 10 юношей и 10 девушек. Для дежурства на вечере путем жеребьевки выделяют 5 человек. Какова вероятность того, что в число дежурных войдут: а) 5 юношей; б) 2 юноши и 3 девушки.

  19. Для постановки танца хореограф выбирает 8 человек. Определить вероятность того, что из выбранных можно составить 4 пары, если в танцевальной студии занимается 12 девочек и 8 мальчиков.

  20. Лотерея выпущена на общую сумму 1000000 рулей. Цена одного билета 50 рублей. Ценные выигрыши падают на каждый десятый билет. Определить вероятность выигрыша при покупке: а) одного билета; б) двух билетов.



ПЗ № 14. Вычисление вероятностей. Прикладные задачи. Представление числовых данных. Прикладные задачи.

1)Опорный конспект.

Определение . Пусть событие А может произойти только совместно с одним из событий Н1, Н2,…, Нп, образующих полную группу несовместных событий. Тогда события Н1, Н2,…, Нп называются гипотезами

Теорема . Вероятность события А, наступающего совместно с гипотезами Н1, Н2,…, Нп, равна: где p(Hi) – вероятность i- й гипотезы, а p(A/Hi) – вероятность события А при условии реализации этой гипотезы. Формула (3.1) носит название формулы полной вероятности.

Пример. Имеются три одинаковые урны с шарами. В первой из них 3 белых и 4 черных шара, во второй – 2 белых и 5 черных, в третьей – 10 черных шаров. Из случайно выбран-ной урны наудачу вынут шар. Найти вероятность того, что он белый.

Решение. Будем считать гипотезами Н1, Н2 и Н3 выбор урны с соответствующим номером. Так как по условию задачи все гипотезы равновозможны, то Найдем условную вероятность А при реализации каждой гипотезы: Тогда

Формула Байеса (теорема гипотез).

Пусть известен результат опыта, а именно то, что произошло событие А. Этот факт может изменить априорные (то есть известные до опыта) вероятности гипотез. Например, в предыдущем примере извлечение из урны белого шара говорит о том, что этой урной не могла быть третья, в которой нет белых шаров, то есть р (Н3/А) = 0. Для переоценки вероятностей гипотез при известном результате опыта используется формула Байеса:

Пример. После двух выстрелов двух стрелков, вероятности попаданий которых равны 0,6 и 0,7, в мишени оказалась одна пробоина. Найти вероятность того, что попал первый стрелок.

Решение. Пусть событие А – одно попадание при двух выстрелах, а гипотезы: Н1 – первый попал, а второй промахнулся, Н2 – первый промахнулся, а второй попал, Н3 – оба попали, Н4 – оба промахнулись. Вероятности гипотез: р(Н1) = 0,6·0,3 = 0,18, р(Н2) = 0,4·0,7 = 0,28, р(Н3) = 0,6·0,7 = 0,42, р(Н4) = 0,4·0,3 = 0,12. Тогда р(А/Н1) = р(А/Н2) = 1, р(А/Н3) = р(А/Н4) = 0. Следовательно, полная вероятность р(А) = 0,18·1 + + 0,28·1 + 0,42·0 + 0,12·0 = 0,46. Применяя формулу Байеса, получим Закон распределения дискретной случайной величины.
Как известно, случайной величиной называется переменная величина, которая может принимать те или иные значения в зависимости от случая. Случайные величины обозначают заглавными буквами латинского алфавита (X, Y, Z), а их значения – соответствующими строчными буквами (x, y, z). Случайные величины делятся на прерывные (дискретные) и непрерывные.
Дискретной случайной величиной называется случайная величина, принимающая лишь конечное или бесконечное (счетное) множество значений с определенными ненулевыми вероятностями.
Функция, связывающая значения случайной величины с соответствующими им вероятностями, называется
 законом распределения дискретной случайной величины. Закон распределения может быть задан одним из следующих способов.

  1. Закон распределения может быть задан таблицей:

Значения xi

x1

x2

x3

...

xn

Вероятности pi

p1

p2

p3

...

pn


События X = xi (i = 1, 2, 3,…,n) являются несовместными и единственно возможными, т.е. они образуют полную систему событий. Поэтому сумма их вероятностей равна единице: р123+…+рn = ∑pi =1

2. Закон распределения может быть задан аналитически (формулой) P(X = xi) = ϕ(xi). Например:а) с помощью биномиального распределения: Pn(X=k) = Сnk pk qn-k, 0<р<1, k = 0, 1, 2, …, n;

б) с помощью распределения Пуассона: hello_html_mc1ab25c.jpg

где λ>0, k = 0, 1, 2, … .

в) с помощью функции распределения F(x), определяющей для каждого значения x вероятность того, что случайная величина X примет значение, меньшее x, т.е. F(x) = P(X < x).

- свойства функции F(x)

3. Закон распределения может быть задан графически – многоугольником (полигоном) распределения

Основные числовые характеристики дискретной случайной величины:

Mатематическое ожидание (среднее значение) дискретной случайной величины M(X)=Σ xipi.
Для биномиального распределения M(X)=np, для распределения Пуассона M(X)=λ

Дисперсия дискретной случайной величины D(X)= M[X–M(X)]2 или D(X) = M(X2)−[M(X)]2. Разность X–M(X) называют отклонением случайной величины от ее математического ожидания.
Для биномиального распределения D(X)=npq, для распределения Пуассона D(X)=λ

Среднее квадратическое отклонение (стандартное отклонение) σ(X)=√D(X).

Задача 1.

Найти математическое ожидание числа очков, выпадающих при бросании игральной кости.

Решение. Случайная величина X числа очков принимает значения 1, 2, 3, 4, 5, 6. Вероятность того, что выпадет одно из данных значений равна 1/6. Закон распределения представим в виде таблицы:


Значения xi

1

2

3

4

5

6

Вероятности pi

1/6

1/6

1/6

1/6

1/6

1/6

Найдем математическое ожидание величины Х:

М(Х) = 1*1/6 + 2*1/6 + 3*1/6 + 4*1/6 + 5*1/6 + 6*1/6 = (1+2+3+4+5+6)/6 = 21/6 = 3,5

Задача 2.

Выпущено 1000 лотерейных билетов: на 5 из них выпадает выигрыш в сумме 500 рублей, на 10 – выигрыш в 100 рублей, на 20 – выигрыш в 50 рублей, на 50 – выигрыш в 10 рублей. Определить закон распределения вероятностей случайной величины X – выигрыша на один билет.

Решение. По условию задачи возможны следующие значения случайной величины X: 0, 10, 50, 100 и 500.

Число билетов без выигрыша равно 1000 – (5+10+20+50) = 915, тогда P(X=0) = 915/1000 = 0,915.

Аналогично находим все другие вероятности: P(X=0) = 50/1000=0,05, P(X=50) = 20/1000=0,02, P(X=100) = 10/1000=0,01, P(X=500) = 5/1000=0,005. Полученный закон представим в виде таблицы:

Значения xi

0

10

50

100

500

Вероятности pi

0,915

0,05

0,02

0,01

0,005


2) Перепишите и заполните пропуски:

Пример 1. В пирамиде 5 винтовок, три из которых снабжены оптическим прицелом. Вероятность того, что стрелок поразит мишень при выстреле из винтовки с оптическим прицелом, равна 0,95; для винтовки без оптического прицела эта вероятность равна 0,7. Найти вероятность того, что мишень будет поражена, если стрелок производит один выстрел из наудачу взятой винтовки.

Решение: по классическому определению:    – вероятности  выбора винтовки с оптическим и без оптического прицела соответственно.

По условию,  – вероятности попадания в мишень из соответствующих типов винтовок.

По формуле полной вероятности:
– вероятность того, что стрелок поразит мишень из наугад выбранной винтовки.

Ответ: 0,85.

Пример 2. Число грузовых машин, проезжающих мимо бензоколонки, относится к числу проезжающих там же легковых машин как 3:2. Вероятность того, что будет заправляться грузовая машина, равна 0,1; для легковой машины эта вероятность равна 0,2. На заправку подъехала машина. Найдите вероятность того, что подъехавшая машина грузовая.
Решение. Пусть событие А состоит в том, что проезжающая машина остановилась на заправку, а гипотезы Н1 и H2 соответственно означают, что проезжающая машина грузовая или легковая. Нам нужно найти вероятность Р(Н1/А). Из условия следует, что Р(Н1) = 0,6, Р(Н2) = 0,4, Р(А/Н1) = 0,1 и Р(А/Н2) = 0,2. Сначала по формуле полной вероятности находим Р(А) = 0,6 * 0,1 + 0,4 * 0,2 = ... Далее применяем формулу Байеса:
Р(Н1/А)= (Р(А/Н1)*Р(Н1))/Р(А)= (0,1*0,6)/0,14 = 6/14= … ≈ 0,4286.

Пример 3. На склад поступило 2 партии изделий: первая – 4000 штук, вторая – 6000 штук. Средний процент нестандартных изделий в первой партии составляет 20%, а во второй – 10%. Наудачу взятое со склада изделие оказалось стандартным. Найти вероятность того, что оно: а) из первой партии, б) из второй партии.

Первая часть решения состоит в использовании формулы полной вероятности. Иными словами, вычисления проводятся в предположении, что испытание ещё не произведено и событие «изделие оказалось стандартным» пока не наступило.

Рассмотрим две гипотезы:
 – наудачу взятое изделие будет из 1-й партии;
 – наудачу взятое изделие будет из 2-й партии.

Всего: 4000 + 6000 = … изделий на складе. По классическому определению:
. Контроль:
 

Рассмотрим зависимое событие: hello_html_m518765a0.gif – наудачу взятое со склада изделие будет стандартным.

В первой партии 100% – 20% = 80% стандартных изделий, поэтому:  – вероятность того, что наудачу взятое на складе изделие будет стандартным при условии, что оно принадлежит 1-й партии.

Аналогично, во второй партии 100% – 10% = 90% стандартных изделий и- вероятность того, что наудачу взятое на складе изделие будет стандартным при условии, что оно принадлежит 2-й партии.

По формуле полной вероятности:
 – вероятность того, что наудачу взятое на складе изделие будет стандартным.

Часть вторая. Пусть наудачу взятое со склада изделие оказалось стандартным. Эта фраза прямо прописана в условии, и она констатирует тот факт, что событие hello_html_m518765a0.gif произошло.

По формулам Байеса:

а)   – вероятность того, что выбранное стандартное изделие принадлежит 1-й партии;

б)  – вероятность того, что выбранное стандартное изделие принадлежит 2-й партии.

После переоценки гипотезы, , разумеется, по-прежнему образуют полную группу:
 
(проверка)

Ответ:a) б)   


Пример 4. Выпущено 1000 лотерейных билетов: на 5 из них выпадает выигрыш в сумме 500 рублей, на 10 – выигрыш в 100 рублей, на 20 – выигрыш в 50 рублей, на 50 – выигрыш в 10 рублей. Определить закон распределения вероятностей случайной величины X – выигрыша на один билет.

Решение: По условию задачи возможны следующие значения случайной величины X:

0, 10, 50, 100 и 500.

Число билетов без выигрыша равно 1000 – (5+10+20+50) = …, тогда P(X=0) = 915/1000 = ...

Аналогично находим все другие вероятности: P(X=0) = 50/1000=…, P(X=50) = 20/1000=…,

P(X=100) = 10/1000=…, P(X=500) = 5/1000=... Полученный закон представим в виде таблицы:


Вероятности pi

0,915

0,05

0,02

0,01

0,005


Пример 5. a)Закон распределения случайной величины X имеет вид:

X

1

0

1

2

P

0,1

0,2

0,3

0,4

Вычислить Dx   и Ϭx .

Решение: Найдем вначале математическое ожидание случайной величины X:

Mx = .

Вычислим дисперсию Dx :Dx = .

Тогда среднее квадратическое отклонение: Ϭx = .

Ответ: Dx = 1, Ϭx = 1.

б) Закон распределения случайной величины X имеет вид:

X

0

1

2

P

0,1

0,2

x

Найти x. Составить функцию распределения. Вычислить: P{X > 0,7} , Mx , Dx и Ϭx .

Решение. Согласно условию нормировки имеем уравнение: 0,1   Отсюда x = 0,7 . Далее, воспользовавшись рядом распределения, найдем:

P{X > 0,7} = P {X = 1}P{X = 2} = 0,2 0,7 = …; Mx =

Dx = ; Ϭx = .

Ответ: x = 0,7 ; P{X > 0,7} = 0, 9; Mx Dx ; Ϭx

Пример 6. a)Известно, что случайная величина X, принимающая два значения  x1 = 2 и x2 = 3 , имеет математическое ожидание, равное 2,2. Построить ряд распределения случайной величины X, найти дисперсию, среднее квадратическое отклонение и составить функцию распределения.

Решение. Пусть P{X = 2} = p . Тогда, согласно условию нормировки,P{X = 3} = 1  . Используя определение математического ожидания, получим Mx = 2p . Имеем уравнение 3 , откуда находим p = 0,8 . Ряд распределения имеет вид:

X

2

3

P

0,8

0,2

Теперь вычислим дисперсию и среднее квадратическое отклонение:

Dx = ; Ϭx =  .

Согласно определению функция распределения имеет вид

Fx(x) =

Ответ: Dx ; Ϭx =   Fx(x) =

б) Возможные значения случайной величины X таковы: x1 = 2 , x2 = 3, x3 = 3 . Известно, что Mx = 2,3 ,α2 = 5,9 . Найти вероятности, соответствующие возможным значениям X, и записать ряд распределения.

Решение. Ряд распределения, с учетом возможных значений случайной величины X, будет выглядеть следующим образом:

X

1

2

3

P

p1

p2

p3

Найдем вероятности p1 , p2 и p3, соответствующие возможным значениям X.

По условию Mx = 2,3 , поэтому имеем первое уравнение, связывающее p1p2 и p3 :

 . Аналогично из условия α2 = 5,9   получим второе уравнение:

 . Третье уравнение возникает из условия нормировки:

p1 p2 p3 = 1. Итак, имеем систему:


Ответ: ряд распределения имеет вид

X

1

2

3

P

0,2

0,3

0,5

3)Решить задачи :

  1. На склад поступают телефоны трех заводов, причем доля телефонов первого завода составляет 25%, второго - 60%, третьего - 15%. Известно также, что средний процент бракованных телефонов для первой фабрики составляет 2%, второй - 4%, третьей - 1%. Найти вероятность того, что наудачу выбранный телефон бракованный.

  2. В офисе: 4 ноутбука изготовлены компанией A, 6- компанией B, 8 -компанией  C и 2 - компанией D . Гарантии, что ноутбуки этих компаний будут работать в течение гарантийного срока без ремонта составляют 70%, 80%, 85%, и 55% для каждой из них. Найти вероятность того, что выбранный ноутбук будет работать без ремонта в течение гарантийного срока.

  3. Экспортно-импортная фирма собирается заключить контракт на поставку оборудования в одну из развивающихся стран. Если основной конкурент фирмы не станет одновременно претендовать на заключение контракта, то вероятность получения контракта оценивается в 0,45; в противном случае — в 0,25. По оценкам экспертов компании вероятность того, что конкурент выдвинет свои предложения по заключению контракта, равна 0,40. Чему равна вероятность заключения контракта для этой фирмы?

  4. Каждому из 3 первоклассников - Пете, Коле и Мише - предложили одинаковое количество загадок. Петя отгадывает в среднем 3 загадки из 4. Коля 5 из 6. Миша 9 из 10. Наугад выбранный школьник не отгадал загадку. Какова вероятность того, что это был Коля?

  5. На каждые 100 электрических ламп завода «А» в среднем приходится 83 стандартных, завода «В» - 63 стандартных. В магазин поступает 70% лампочек с завода «А» и 30% - с завода «В». Купленная лампочка оказалась стандартной. Найти вероятность того, что лампочка изготовлена на заводе «А».

  6. В студенческой группе 70% - юноши. 20% юношей и 60% девушек имеют сотовый телефон. После занятий в аудитории был найден кем-то забытый телефон. Найти вероятность того, что он принадлежал юноше

  7. Сотрудники отдела маркетинга полагают, что в ближайшее время ожидается рост спроса на продукцию фирмы. Вероятность этого они оценивают в 80%. Консультационная фирма, занимающаяся прогнозом рыночной ситуации, подтвердила предположение о росте спроса. Положительные прогнозы консультационной фирмы сбываются с вероятностью 95%, а отрицательные – с вероятностью 99%. Какова вероятность того, что положительный прогноз сбудется? 

  8. Имеются три одинаковые урны. В первой урне находятся 4 белых и 7 черных шаров, во второй – только белые и в третьей – только черные шары. Наудачу выбирается одна урна и из неё наугад извлекается шар. Какова вероятность того, что этот шар чёрный?

  9. Двигатель работает в трёх режимах: нормальном, форсированном и на холостом ходу. В режиме холостого хода вероятность его выхода из строя равна 0,05, при нормальном режиме работы – 0,1, а при форсированном – 0,7. 70% времени двигатель работает в нормальном режиме, а 20% – в форсированном. Какова вероятность выхода из строя двигателя во время работы?

  10. В магазине три холодильника в которых заканчивается мороженое. В первом 4 белых  и 6 шоколадных, во втором - 2 белых и 8 шоколадных, в третьем - 3 белых и 7 шоколадных. Наугад выбирают холодильник и вынимают из него мороженое. Определить: a) вероятность того, что оно белое, б) вероятность того, что мороженное извлекли из второго холодильника.

  11. Число грузовых машин, проезжающих мимо бензоколонки, относится к числу проезжающих там же легковых машин как 3:2. Вероятность того, что будет заправляться грузовая машина, равна 0,2; для легковой машины эта вероятность равна 0,3. На заправку подъехала машина. Найдите вероятность того, что подъехавшая машина грузовая.

  12. На склад поступают телефоны трех заводов, причем доля телефонов первого завода составляет 25%, второго - 60%, третьего - 15%. Известно также, что средний процент бракованных телефонов для первой фабрики составляет 2%, второй - 4%, третьей - 1%. Найти вероятность того, что:  а) наугад взятый телефон окажется с браком; б) телефон изготовлен на первом заводе, если он бракованный;  в) на каком заводе скорее был изготовлен телефон, если он сделан качественно ?

  13. Выпущено 200 лотерейных билетов: на 5 из них выпадает выигрыш в сумме 500 рублей, на 10 – выигрыш в 100 рублей, на 20 – выигрыш в 50 рублей, на 40 – выигрыш в 10 рублей. Определить закон распределения вероятностей случайной величины X – выигрыша на один билет.

  14. a)Закон распределения случайной величины X имеет вид:

X

1

0

1

2

P

0,1

0,15

0,3

0,45

Вычислить Dx и Ϭx . б) Закон распределения случайной величины X имеет вид:

X

0

1

2

P

0,2

0,3

x

Найти x. Составить функцию распределения. Вычислить: P{X > 0,7} , Mx , Dx и Ϭx .

  1. a)Известно, что случайная величина X, принимающая два значения  x1 = 2 и x2 = 3 , имеет математическое ожидание, равное 2,4. Построить ряд распределения случайной величины X, найти дисперсию, среднее квадратическое отклонение и составить функцию распределения.

б) Возможные значения случайной величины X таковы: x1 = 2 , x2 = 3, x3 = 3 . Известно, что Mx = 2,5 ,α2 = 6,7 . Найти вероятности, соответствующие возможным значениям X, и записать ряд распределения.

  1. Вероятность того, что телевизор имеет скрытые дефекты, равна 0,2. На склад поступило 20 телевизоров. Какое событие вероятнее: что в этой партии имеется два телевизора со скрытыми дефектами или три?

  2. В работе телефонной станции происходят в среднем 3 сбоя в час. Определить вероятность хотя бы одного сбоя за 1 час.

  3. В честь национального праздника состоялся массовый забег на дистанцию 10 км. В забеге приняли участие 250 человек. Обычно в забегах такого типа из каждых десяти участников 8 доходят до финиша. Какова вероятность того, что до финиша дойдут 200 человек?

  4. Радиотелеграфная станция передает цифровой текст. В силу наличия помех каждая цифра независимо от других может быть неправильно принята с вероятностью 0,01. Найдите вероятность того, что в принятом тексте, содержащем 1100 цифр, будет менее 20 ошибок.

  5. Предприятие имеет проблемы с поставками сырья. Вероятность того, что в каждом отдельном месяце предприятие будет полностью обеспечено сырьем, равна 0,9. Какова вероятность того, что за полугодовой период предприятие будет полностью обеспечено сырьем:

а) ровно в трех месяцах? б) не менее чем в двух месяцах?

  1. В работе телефонной станции происходят в среднем 3 сбоя в час. Определить вероятность пяти сбоев за 2 часа.

  2. Радиотелеграфная станция передает цифровой текст. В силу наличия помех каждая цифра независимо от других может быть неправильно принята с вероятностью 0,01. Найдите вероятность того, что в принятом тексте, содержащем 1100 цифр, будет 15 ошибок.

  3. В честь национального праздника состоялся массовый забег на дистанцию 10 км. В забеге приняли участие 250 человек. Обычно в забегах такого типа из каждых десяти участников 8 доходят до финиша. Какова вероятность того, что до финиша дойдут от 180 до 220 человек?

  4. Всхожесть семян данного растения составляет 90%. Найти вероятность того, что из 800 посеянных семян взойдёт не менее 700.

ПЗ № 15. Арифметические действия над числами, нахождение приближенных значений величин и погрешностей вычислений (абсолютной и относительной), сравнение числовых выражений.

1)Опорный конспект.

В процессе вычислений весьма часто приходится иметь дело с приближенными числами. Пусть А точное значение некоторой величины, называемое в дальнейшем точным числом А. Под приближенным значением величины А, или приближенным числам, называется число а, заменяющее точное значение величины А. Если а < А, то а называется приближенным значением числа А по недостатку. Если а > А, — то по избытку. Например, 3,14 является приближенным значением числа π по недостатку, а 3,15 — по избытку. Для характеристики степени точности данного приближения пользуются понятием погрешности или ошибки.

Погрешностью Δа приближенного числа а называется разность вида

Δа = А — а, (1.1)

где А — соответствующее точное число.

Определение. Абсолютной погрешностью А приближенного числа а называется абсолютная величина погрешности этого числа

Δ = |А — а|. (1.2)

В силу того, что точное число А, как правило, неизвестно, то пользуются понятием предельной абсолютной погрешности.

Определение. Предельной абсолютной погрешностью Δa приближенного числа а называется число, не меньшее абсолютной погрешности этого числа, т. е.

Δa ≥ Δ. (1.3)

Из (1.3) имеем

Δa |А — а|,

следовательно,

а - Δa £ А £ а + Δa, (1.4)

т. е. а - Δa является приближением числа А по недостатку, а а + Δa — приближением числа А по избытку. Формулу (1.4) кратко записывают в виде А = а ± Δa.

На практике под точностью измерений обычно понимают предельную абсолютную погрешность. Например, если расстояние между двумя пунктами, равное S = 900 м, получено с точностью до 0,5 м, то точное значение величины S заключено в границах 899,5 м £ S £ 900,5 м.

Введение абсолютной или предельной абсолютной погрешностей совершенно недостаточно для характеристики степени точности приближенных чисел. Существенным показателем точности приближенных чисел является их относительная погрешность.

Определение. Относительной погрешностью δ приближенного числа а называется отношение абсолютной погрешности Δ этого числа к модулю соответствующего точного числа А (А ¹ 0)

. (1.5)

Определение. Предельной относительной погрешностью приближенного числа а называется число δа не меньшее относительной погрешности этого числа, т. е.

δаδ. (1.6)

Из (1.6) имеем Δ £ |А|δа

Следовательно, можно считать, что предельная абсолютная погрешность числа а равна

Δа £ |А|δа. (1.7)

Если принять А » а, то формула (1.7) примет вид

Δа £ |а|δа. (1.8)

Следовательно, точное число А лежит в следующих границах:

а(1 - δа) £ А £ а(1 + δа).

Формула (1.8) позволяет определять предельную абсолютную погрешность по заданной предельной относительной погрешности и наоборот.

Пример 1. Округляя точные числа hello_html_m518765a0.gif до трех значащих цифр, определить абсолютную hello_html_m120caa96.gif и относительнуюhello_html_7030a9e7.gif погрешности полученных приближенных чисел.

Дано: hello_html_m600c8ad6.gif hello_html_7d22ad7.gif . Найти: hello_html_m171d5a2f.gif

Решение:

hello_html_5972f99.gif- приближенное значение числа A

Абсолютная погрешность: hello_html_5dc3a3d6.gif

Относительная погрешность: hello_html_79e9a63b.gif

Ответ: hello_html_c7ee83f.gif; hello_html_m69af9963.gif

Пример 2. Определить абсолютную погрешность приближенных чисел hello_html_31311e32.gif по их относительной погрешности hello_html_7030a9e7.gif.

Дано: hello_html_5716c6a.gif hello_html_m652ddba.gif. Найти: hello_html_m120caa96.gif

Решение:

Абсолютная погрешность: hello_html_m116fefef.gif

Ответ: hello_html_7262f7c7.gif

Пример 3 . При измерении длины с точностью до 5 м получено hello_html_m6a0990d6.gif км, а при определении другой длины с точностью до 0.5 см, получено hello_html_m4a535580.gif метров. Какое измерение по своему качеству лучше?

Дано: hello_html_m1623fc6.gifКм, hello_html_m56eb8e11.gifМ, hello_html_mb27ab20.gifМ, hello_html_m175c9b8b.gifСм

Сравнить: hello_html_50409580.gif и hello_html_17d9b7cc.gif

Решение: Итак, по 1-му измерению, результат hello_html_m1623fc6.gifКм = hello_html_m2ee8bf0.gifМ с точностью до hello_html_mb27ab20.gifМ (hello_html_m2cafe90a.gif - абсолютная погрешность величины hello_html_m6a0990d6.gif).

Тогда относительная погрешность: hello_html_293aab59.gif%

По 2-му измерению, результат hello_html_m56eb8e11.gifКм с точностью до hello_html_m175c9b8b.gifСм =hello_html_3b05be7.gifМ (hello_html_7cc15c1e.gif - абсолютная погрешность величины hello_html_m4a535580.gif).

Тогда относительная погрешность: hello_html_m587113e.gif%

Так как hello_html_5928e3a2.gif, то измерение hello_html_m4a535580.gif можно считать по качеству лучше, чем hello_html_m6a0990d6.gif.

Ответ: измерение hello_html_m4a535580.gif по качеству лучше, чем hello_html_m6a0990d6.gif.

2)Решить задание:

А)Вычислить:

  1. а) (– 2,35 – 4,65) · 5 : (16,9 – 2,9),
    б) (7,63 + (– 5,13)) · 0,4 : (3,17 + 6,83),

  2. а) 30,3 · (124,9 – (48,96 : 6,8 + 36,04) : 9,2),

б) 73, 2 · 48, 3 – 37,4 · (166,02 + 219,38) : 1,64,

  1. а) 3,44 : 0,4 + 24,56 , б) 684 · 245 – 675 · 246,

  2. а) (93 · 7 + 141) : 72 , б) 7091 + 9663 – (243916 + 75446) : 527 : 3,

в) (15,964 · 5,2 – 12) · 0,1 , г) (96,6 + 98,6) : 6,4 · 1,2 – 0,2,

  1. а) ((27,12+ 43,08) · 0,007 – 0,0314) · 100,
    б) 1,53 · 54 – 0,42 · (512 – 491,2) + 1,116,

в) (867000 : 2125 – 396,4) · 2,15,

  1. а) 51,6 + (70,2 – 4,4 · (73,73 : 7,3)) · 1,6,
    б) 18,305 : 0,7 – 0,0368 : 0,4 + 0,492 : 1,2,

в) (0,6739 + 1,4261) · 557, 55 : (16,7 · 2,9 – 42,13),

г) 702,3 – (59 – 389,64 : 6,8) · (59,3 – 5,64 : 9,4),

  1. а) 316219 – (27090 : 43 +16422 : 119), б) 565,3 – 465,3 : ((1,25 + 5,8) · (55,8 – 49,2)),

в) 74 : 100 – 0,4 : 10 + 17,8 : 1000, г) 0,35 · 10 + 0,0237 · 100 – 0,00087 · 1000,

  1. а) 0,7 : 0,1 + 0,0474 : 0,01 – 0,00174 : 0,001, б) 12,3 + 7,7 · 187,2 : 4,5 : 6,4 – 3,4,

в) 10,1 + 9,9 · 107,1 : 3,5 · 6,8 – 4,85, г) 37 · 0,01 – 0,2 · 0,1 + 8,9 · 0,001.

  1. Найди значения выражений:
    а) (18370+23679):7, 156-96:(12:4):2, б) (800035
    784942)∙6,

в) 98560:7 ,83216:4, 8656:4 ,91620:4, г) 73170:9 ,3726:9 ,91728:9, 705355:5.

  1. Найди значения выражений:
    а) (10283+16789):9, 5∙(125+75):20+80, б) (200496
    134597)∙2,

в) 54663:7, 80395:5, 6543:9, 860073:3, г) 1836:4,7542:9, 3906:6, 9150:3,

д)795 ·504 248.952:492,

  1. Реши примеры на деление:

114595 : 215 =

200064 : 384 =

404758 : 922 =

5370 : 358 =

396204 : 548 =

263082 : 978 =

181116 : 387 =

118956 : 276 =

115419 : 487 =

140070 : 435 =

223925 : 689 =

420210 : 435 =


  1. а)1098 + (1453 – 564) · 176  + 195 539– 352 004,

б)30257 · 8 + 7 280400 · 5 5 897 · 6 3504: 8.

В)

  1. Найдите значение выражения:

а) 48 – 29 + 37 – 19;  б) 156 + 228 – 193 – 66; в) 39 • 45 : 65 • 2;  г) 1024 : 128 • 15 : 10; 
д) 245 : 7 – 224 : 16 + 35 • 11;  е) 322 : 23 • 70 – 161 • 9 : 69; 

  1. а) 315 : (162 + 12 • 24 - 11 • 39) + 558 : 31;  б) (24 • 7 - 377 : 29) • (2378 : 58 – 38); 
    в) (120 + 16 • 7) • 240 : (300 – 5 • 44);  г) (372 + 118 • 6) : (38 • 35 – 34 • 37) - 12; 
    д) 3124 : (3 • 504 – 4 • 307) + 10 403 : 101;  е) 15 + (12 322 : (24 + 37) – 12 • 15) : (35 • 2 – 59).

  2. Измените порядок действий на основании свойств сложения, вычитания и умножения для удобства вычислений:

а) 348 + 54 + 46;                      г) 54 • 2 • 50; 
б) 543 + 89 – 43;                       д) 34 • 8 + 66 • 8; 
в) 427 – 33 – 67;                        е) 135 • 12 – 35 • 12.

  1. Выполните действия по схеме .

hello_html_m7f78617e.jpg

  1. Найдите частное:

а) 1 989 680 : 187;                            в) 9 018 009 : 1001; 
б) 572 163 : 709;                               г) 533 368 000 : 83 600.

С) ТЕСТ.


Найти абсолютную погрешность приближения числа 3,9 числом 4

1) 0,9; 2) – 0,9; 3) 0,1; 4) – 0,1

  1. Какое из чисел является более точным приближением числа 3,464 ?

1) 3

2) 3,4

3) 3,6

4) 4

  1. Записать оценку величины n в виде двойного неравенства, если n = 0,385 0,001

1) 0,384 0,386

2) 0,3860,384

3) 0,384 0,386

4) 0,384

  1. Округлить число 734,256 до десятых

1) 734,2 2) 734,3 3) 730 4) 734,26

  1. Найти относительную погрешность приближения числа числом 0,14

  1. Записать число 0,00018 в стандартном виде

  1. Найти значение выражения ( 2,5 . 103 ) : ( 5 . 10 – 2 )


D)

1.Округляя точные числа hello_html_m518765a0.gif до трех значащих цифр, определить абсолютную hello_html_m120caa96.gif и относительнуюhello_html_7030a9e7.gif погрешности полученных приближенных чисел.

Дано: А= 0,1766 , hello_html_7d22ad7.gif . Найти: hello_html_m171d5a2f.gif.

2. Определить абсолютную погрешность приближенных чисел hello_html_31311e32.gif по их относительной погрешности hello_html_7030a9e7.gif.

Дано: а =\ 4,782, hello_html_m652ddba.gif. Найти: hello_html_m120caa96.gif

3.Вычислить:

а)

б)


в)

hello_html_1598fcc6.gif

г)

hello_html_346bb90f.gif

д)

hello_html_2f57140e.gif




ПЗ № 16. Вычисление и сравнение корней. Выполнение расчетов с радикалами.

Задание:

1) Перепишите и заполните пропуски:
А)Пример 1. Вычислите: . Решение: .

Ответ: 1.

Пример 2. Вычислите: .

Решение: .

Ответ: 0,2.

Пример 3. Упростите выражение: .

Решение: .

Ответ: 3.

Пример 4. Вычислите: а) б)

в) г)д)

е) ж)

з)

Ответ: а) 6, б) hello_html_15da1ba3.gif8, в) 3,5; г) 40, д) 2, е) 10, ж) 1,25; з) 250.

Пример 5. Выполнить действия: Решение: Ответ:

Пример 6. Решите уравнения:

Решение: Ответ: а) х = 4, б) х = 2.

Пример 7. а) Вынести множитель из-под знака корня: ,

внесите множитель под знак корня: .

Ответ: а) б) .

Пример 8. Сравните числа .

.

Пример 9.: Вычислите:

Чтобы выполнить вычисление, нужно преобразовать числитель, для этого во второй скобке представим составные числа в виде простых:

Получаем:

После преобразований получаем дробь:

Несложно заметить в полученном выражении формулу разности квадратов, свернем ее:


Пример 10. а)Вычислите: .

Сначала вычислим внутренний корень:


После преобразования получили выражение:


б) упростить выражение: .

Выделяем полный квадрат:

Получаем:

Комментарий: число  отрицательное, имеем право раскрыть модуль.

В)

  1. Упростите иррациональные выражения:

1. 2. 3.

4.,5.,6. ,

7. ,8.

  1. Вычислите:

а)

б)

  1. Найдите значение выражения:

а)

б)

в)

  1. Вычислите:

а) , б) ,

в) , г)

д)

е)

ж)

  1. Найти значение выражения:

  2. Упростите выражение:


  1. Разложите на множители:

а) , б) ,

в) ,

  1. Сократите дробь:

а)

б)

в)

2)Решить задание ( по примерам):

  1. Вычислите: .

  2. Вычислите: .

  3. Упростите выражение: .

  4. Вычислите:

а) б) в) г)д)

е) ж) з)

  1. Выполнить действия:

  2. Решите уравнения:

  3. а) Вынести множитель из-под знака корня: ,

внесите множитель под знак корня: .

3)Решить задание :

  1. Вычислите: а) ,б) ,в) ,г) ,д) ,

е) ,ж) , з) .

  1. Решите уравнения: а) , б) , в) , г)

  2. Упростите выражение: .

  3. Вычислите: а) б) в)

  4. а) вынести множитель из-под знака корня:

б) внесите множитель под знак корня:

  1. Вычислите а) ,б) ,в) г) ,д) .

  2. Упростите выражение


  1. Найдите значения выражения при у = 16.

  2. Найдите значение выражения:

  3. Сократите дробь: .

  4. Найдите значения выражения при а = 4, b = 5.

  5. Найдите значение выражения при р = 49.

  6. Упростите выражение .

  7. Упростите выражение .

  8. Вычислите

  9. Упростите выражение .

  10. Найдите значение выражения:

  11. Найдите значение выражения: .

  12. Упростите выражение .

  13. Упростите выражение



ПЗ № 17. Нахождение значений степеней с рациональными показателями. Сравнение степеней.

Задание:

1)Перепишите:

Определение. Степенью числа с рациональным показателем , где m-целое число, а n-натуральное (), называется число . Итак, по определению .

Свойства степени с рациональным показателем,

где r,s-рациональные числа, ,.





Замечание. При рациональная степень числа а не определяется.

Пример 1.

Пример 2. Сравните числа .

.

Пример 3.

Пример 4.

Пример 5.

Пример 6. Упростите выражение:

Решение:

=======

==27

2)Решить задание:

  1. Найдите значение числового выражения .

  2. Сравните числа .

  3. Вычислите: а) ; б) ; в) ; г) .



  1. Упростите выражение:

а) ; б); в) ; г) ; д) .

  1. Представьте выражение в виде степени и найдите его значение при у = 8.

  2. Сократите дробь: а) ; б).

  3. Вычислите:

  1. Вычислите:

  1. Данные     выражения    представить  в  виде    степеней   с одинаковыми показателями и сравнить их по величине:

1) 42 и 28;  2)  273  и  96 ; 3)  1252 и 253; 4) 4300 и 3400;   5) — 1/8   и   (—1/32)3;  6) ( — 6/7 )4  и (36/49)6;

  1. Найдите значение выражения .

3)Решить задание (тест):

А)Часть 1.

1. Представьте выражение в виде степени с основанием a. Ответ: ________________

2. Какое из данных выражений не равно ?

А. Б. В. Г.

3. Найдите значение выражения при m =

А. -16 Б. В. Г. 16

4. Решите уравнение: . Ответ: ________________

5. Разложите на множители:

А. ()() Б. ()() В. ()() Г. ()()

6. Найдите значение выражения А. 60 Б. 30 В. 12 Г.

Часть 2.

7. Упростите выражение: .


  1. Вычислите.

а) [а) 15 б) 30 в) 60 г) 10] б) [а) 1,5 б) в) 2,5 г) 0,4]

в) ( [а) - б) в) г) ] г) 10∙10∙10 [а)10 б)10 в) 10 г)10]

д) (5∙ [а)20 б) в)9 г)-20] е) 4: 2[а)2 б) 2 в)4 г) 4]

ж) 8 - 8 [а) 0 б) 1 в) 4 г) 1] з) [а)6 б) 246 в) 6 г)108]

и) [а) 24 б) 12 в) г)1] к) [а) б) 5 в) 13 г) 4]


В)

1 вариант.

А1. Вычислите .

1) 2; 2) 3; 3) 9; 4) .

А2. Вычислите .

1) 2; 2) 4; 3) 2; 4) 4.

А3. Упростите выражение

1) ; 2) ; 3) а; 4) .

А4. Вычислите

1) 0,25; 2) 0,5; 3) 0,15; 4) 5.

А5. Найдите значения выражения при у = 18.

1) 9(4+3); 2) ; 3) 4+3; 4) 9.

А.6. Упростите выражение

1) 2) 3) 4)

А7. Найдите значение выражения: .


1) 12; 2) 6; 3) 3; 4) –3.


А8. Найдите значение выражения: .

1) ; 2) 1,2; 3) ; 4) .

А9. Найдите значение выражения:

1) 4; 2) 9; 3) 5; 4) 5.


А10. Сократите дробь:

1) а; 2) ; 3) ; 4) а+1.


2 вариант.

А1. Вычислите .

1) 5; 2) 4; 3) 25; 4) .

А2. Вычислите .

1) 2; 2) 4; 3)2; 4) 4.


А3. Упростите выражение

1) ; 2) ; 3) а; 4) .

А4. Вычислите

1) 0,09; 2) 0,03; 3) 0,3; 4) 3.

А5. Найдите значения выражения при а = 4, b = 5.

1) ; 2) 2; 3) 0; 4) .

А.6. Упростите выражение .

1) 2) 3) 4)

А7. Найдите значение выражения: .


1) 45; 2) 5; 3) 3; 4) –45.

А8. Найдите значение выражения: .

1) 5,5; 2) 2; 3) ; 4) .

А9. Найдите значение выражения:

1) 4; 2) 25; 3) 9; 4) 16.

А10. Найдите значение выражения при р = 49.

1) 49; 2) ; 3) ; 4) 7.




ПЗ № 18. Преобразования выражений, содержащих степени. Решение прикладных задач.

Задание:

1) Перепишите:

hello_html_m7ca5a23c.png

hello_html_mea6a4e4.png

hello_html_1f9e2b77.png

2) Решение теста по теме «Степень с рациональным, действительным показателем, ее свойства».

( заполните пропуски и запишите номер правильного ответа)

А1. Вычислите .

1) 2; 2) 4; 3) 9; 4) .

А2. Вычислите .

1) 2; 2) 4; 3) 5; 4) 4.

А3. Упростите выражение

1) ; 2) ; 3) а2; 4) .


А4. Вычислите 1) 0,25; 2) 0,3; 3) 0,15; 4) 5.

А5. Найдите значения выражения при у = 16.

1) 9(4+3); 2) ; 3) 4+3; 4) .

А.6. Упростите выражение

1) 2) 3) 4)

А7. Найдите значение выражения: .


1) 12; 2) 6; 3) 8; 4) –3.


А8. Найдите значение выражения: .

1) ; 2)1,4; 3) ; 4)

А9. Найдите значение выражения:

1) 4; 2) 9; 3) 45; 4) 5.

А10. Сократите дробь:

1) 0,5а; 2) ; 3) ; 4) а+1.

А11. Вычислите . 1) 9; 2) 4; 3) 25; 4) .

А12. Вычислите .

1) 2; 2) 4; 3)6; 4) 4.

А13. Упростите выражение

1) ; 2) ; 3) а; 4) .

А14. Вычислите

1) 0,09; 2) 0,03; 3) 0,8; 4) 3.

А15. Найдите значения выражения

при а = 4, b = 5.

1) ; 2) 2; 3) 0; 4) .

3)Решить задание :

  1. Упростите иррациональные выражения:

1. ,2. , 3. , 4. ,5. ,

6. ,7. ,8.

  1. Упростите выражения: а) б) в)


  1. Разложите на множители:

а) ,б) ,в) ,г) ,д)

е) ,ж) ,з) ,и) ,к)

  1. Вычислите:

1.

2.

3.

4.

5.

6.

7.

  1. Найдите значение выражения:

а) б) в) г)

д) е) ж)

  1. Сократите дробь:

а) б)

в) г)


д)


  1. Вычислите:

а) , б) , в) ,г)

д) е)
ж) з) и)

  1. Упростите выражение:

а) 60,7/ 60,3; б) к -5,3∙4к 0,1; в) а7/6-5/6;

г) (а3/2: а-7/2)∙а3;д) 4с2/7+ 3(с1/7 )2 ; е) (4∙ 4-2а) -2.

  1. Вычислите:

а) б) в)

г) д) е)

ж) з)

  1. Найти значение выражения

hello_html_6f3d862e.png

  1. Найти значение выражения

hello_html_ma7e682b.png

  1. Упростите выражение.

hello_html_1432db50.png




ПЗ № 19. Нахождение значений логарифма по произвольному основанию. Переход от одного основания к другому.

Задание:

1)Перепишите:

1.Вычислим пример по формуле

2. Вычислим пример по формуле

3. Вычислим пример по формуле


4.Вычислим пример по формуле


5.Найдите значение выражения:

6. Найдите значение выражения:

7. Найдите значение выражения:
8. Найдите значение выражения:

9. Найдите значение выражения:


10. Найдите значение выражения:


11. Найдите значение выражения:

12. Найдите значение выражения:

13. Найдите значение выражения:

14. Найдите значение выражения:

15. Найдите значение выражения: .

16. Найдите  , если


2) Перепишите и заполните пропуски:

1) 2) 3)

4) 5) 6) 7)

8) 9) 10)

11) 12)

13)

14)

15) 16)

17) 18)

19)

20)

21)

22)

23)

24)

25) 26)

27)

28)


30)

hello_html_m6da07957.png


3)Решить задание :Найдите значение выражения:





ПЗ № 20. Вычисление и сравнение логарифмов. Логарифмирование и потенцирование выражений. Приближенные вычисления и решения прикладных задач.

Задание:

1) Перепишите и заполните пропуски:
Пример 1. log3 9 = 2, так как 32 = 9, log5 25 = 2, так как 52 = 25, log3 81 = 4, так как 34 = 81,

Ответ: 2,2,4.

Пример 2. Вычислите : а) log2 16, б) log3 3, в) , г) , д) log2 2 log3 81, е) log12 2 + log12 72, ж) log5 75 – log5 3.
Решение:
а) log2 16 = 4, б) log3 3 = …, в) = 16, г) = = …,

д) log2 2 log3 81= 1· 4 = …, е) log12 2 + log12 72 = log12 (2 ·72) = log12 144 = …,

ж) log5 75 – log5 3= log5 (75:3) = log5 25 = …

Ответ: а) 4, б) 1, в) 16, г) 8, д) 4, е) 2, ж) 2.

Пример 3. Найдите х, если logx 36 = 2 и log2 x = – 2.

Решение: logx 36 = 2, х2 = 36, х = log2 x = - 2, х = 2 -2 = 1 / 4 = …

Ответ: 0,25
Пример 4. Вычислите: а) , б) , в) .

Решение: а) = log2 16=…, б) = 5 · = 5 · 3 = … ,

в) = = 17 = 1296 – 17 = …

Ответ: – 4, 15, 1279.

Пример 5. Упростите выражение :

а) ;

б)

;

в) ;

Ответ: 24, 7, 2.

2) Опорный конспект.

Вспомним основные приемы сравнения положительных чисел а и b.


1) Составить разность a-b и сравнить ее с нулем.


2) Составить частное a:b и сравнить его с единицей.

Свойства логарифмической функции или свойства логарифмов чисел применяются при сравнении логарифмов чисел:

Монотонность логарифмической фунции; Свойства: 0 1 2

loga X1 < loga X2, если a >1

loga X1 > loga X2, если 0 < а < 1

Пример 1. Сравнить и ,

a = 15 >1, > .

Пример 2. Сравнить и ,


знак меняется , <


Пример 3. Сравнить и .

Решение:

значит, значит,

Известными способами сравнить можно не все числа. Рассмотрим следующий пример. Основания разные, логарифмируемые числа разные. Также, как и при сравнении иррациональных чисел, применяется метод «оценки» или сравнение с каким-нибудь «хорошим» числом.

Пример 4. Сравнить и .

, а, значит, .

Пример 5. Сравнить и .


Основное логарифмическое тождество

,

Логарифм произведения — это сумма логарифмов

,

Логарифм частного — это разность логарифмов

,

Свойства степени логарифмируемого числа и основания логарифма

Показатель степени логарифмируемого числа  

Показатель степени основания логарифма,

в частности если m = n, мы получаем формулу: ,

например:

Переход к новому основанию, частности, если c = b, то , и тогда: .


Логарифмирование – это нахождение логарифмов заданных чисел или выражений.

                                                              b
Пример: Найдем логарифм x = a2 · — .
                                                             c

Решение.

Последовательно воспользуемся сразу всеми тремя основными свойствами логарифмов, которые изложены выше (логарифм произведения, логарифм частного и логарифм степени):
                  
    b
lg x = lg (a2 · —) = lg a2 + lg b lg c = 2lg a + lg b lg c.
                      c

Потенцирование – это нахождение чисел или выражений по данному логарифму числа (выражения).

Потенцировать – значит освобождаться от значков логарифмов в процессе решения логарифмического выражения.

Пример: Ответ:

Десятичный логарифм — логарифм с основанием 10, который обозначается как lg.

lg100=2log10100=2, так как 102=100 .

Натуральный логарифм — логарифм с основанием e, обозначается ln.


hello_html_5eccaba.gif

hello_html_m34eaff33.gif


Задача 2hello_html_m74ce2fe.gif


hello_html_615d882b.gif

Задача 3

По данным газеты «Зори» от 12 апреля 2011 года из доклада П. Е. Шишкина население в городе Старый Оскол за один год увеличилось с 256100 человек до 257135 человек. Через сколько лет население этого города увеличится в 1,5 раза?
Решение. Для решения этой задачи применим формулу сложных процентов: A=a(1+p/100)x. Примем население города, которое было, за а=256100тогда А=257135-это население, которое стало, х -неизвестно. р=((257135256100)/257135)100≈0,4%

Сделав подстановку в формулуполучим 256100∙1,5=256100(1+0,4/100) x

Чтобы решить это показательное уравнение прологарифмируем его.

xlg 1,004=lg1,5, откуда x =lg 1,5 /lg1,004

Найдя по таблице lg1,5 и lg1,004 , получим x=0,18/0,002≈90.

Ответ : примерно через 90 лет.

Задача 4

Высота над уровнем моря вычисляется по формуле h=(8000/0,4343)lg(p/p),

где p0 =760 мм рт.ст., р - давление на высоте h м.
Давление в городе Старый Оскол на 15 апреля 2011 года равно 738 мм рт. ст. Вычислим, на какой высоте находится наш город.

Решение. Найдем высоту, на которой находится наш город:
h=(8000/0,4343)lg(760/738) ≈235 м

Ответ: 235м.

Задача 5

В начальный момент времени было 8 бактерий. Через 2 часа после помещения бактерий в питательную среду, их число возросло до 100. Через сколько времени с момента размещения в питательную среду следует ожидать появления 500 бактерий?

Решение.

Для решения данной задачи, необходимо вспомнить понятия скорости и ускорения.

Было -8

Стало- 100

hello_html_7da3953.gif

1 изменение:


=> hello_html_m73c4f0ec.gif конечное значение скорости распространения бактерий при первом изменении - hello_html_32205d0c.gif

Было -8

Стало- 500

hello_html_m5e8cc156.gif

2 изменение:


=> hello_html_m73c4f0ec.gifконечное значение скорости распространения бактерий при втором изменении - hello_html_83a73c1.gif.

Составим формулу для ускорения, учитывая, что начальная скорость hello_html_40273d1b.gif (т.е. было -8, стало -8):

hello_html_m2617e0a1.gif hello_html_m71fb25b1.gif

hello_html_bdff533.gif hello_html_m1c3702f.gif

Т.к. ускорение постоянно => hello_html_2e81f73.gif =>

hello_html_m7ee7891c.gif

Перейдем к натуральному основанию логарифмов, для того, чтобы можно было воспользоваться табличными значениями:

hello_html_25013b08.gif

hello_html_m264b8706.gif

Ответ: приблизительно 3часа 15 минут.

3)А)Решить задание ( по примерам):

  1. Вычислите а) log3 27, б) log4 1,в) log1/2 4,

  2. Вычислите а) log2 32, б) log3 9, в) , г) ,

д) log3 3· log2 8, е) lg 5 + lg 2, ж) log3 15 – log3 5.

  1. Найдите х, если log2 4 = x и log6 x = 2.

  2. Вычислить а) б)

в) .

  1. а)

б)

в)

г)

д)

е)

Б) 1)log9 81 ; 2) 3) log3 1; 4) log5 5 ; 5)

6) 7) log2 log3 9; 8) lg100; 9) 92 log9 5 ; 10) log64 8

Ответы: вставьте номер задания

задания


2

7

10



5



1

ответ

2

4

1

0,5

1

0

-2

25

-1

2


В) 1.Сравнить и ,

2. Сравнить и

3. Сравнить и .

4. Прологарифмируйте по основанию 10 (a > 0,b > 0), x = a2b3.

5. Прологарифмируйте по основанию 3 (a > 0,b > 0), x = 7a3b.

6.Найдите x, если

7. Найдите x, если .

С) Задания для практической работы:

hello_html_3b2eb585.gif

1-2.




hello_html_m41b85ba0.gif


3-4.

hello_html_3139472f.gif







5.Для обогрева помещения, температура в котором равна Тп = 200С, через радиатор отопления, пропускают горячую воду температурой  Тв = 1000С. Расход проходящей через трубу воды m = 0,2 кг/с. Проходя по трубе расстояние x (м), вода охлаждается до температуры Т0С, при чём

hello_html_m119072aa.gif, где с = 4200Дж/кг*С — теплоемкость воды

hello_html_m1c48a4a9.gif = 42 Вт/м * 0С— коэффициент теплообмена , = 1,4 — постоянная.

До какой температуры (в градусах Цельсия) охладится вода, если длина трубы 28 м?

4)Решить задание :

1. Вычислите (по свойству степени):

1) , 2) , 3) , 4) , 5) ,

6) , 7) , 8) ,9) , 10) , 11) ,

12) , 13) , 14) , 15) , 16) .

2. Вычислите (по основному лог. тождеству):

1) , 2) , 3) , 4) , 5) , 6) ; 7) ,

8) , 9) , 10) , 11) , 12) , 13) ,

14) ,15) , 16) , 17) , 18) , 19) , 20) .

3. Вычислите: а),б) , в) ,

г), д).

4. Упростите выражение :

а) ;б) ,в);

5.Вычислить логарифмы: log381,ln e, lg1000, log7343,ln7,29, lg0,001.

6.Вычислить логарифмы: log432 + log42, log552, log2(8 128), log654 + log64, log3108 – log34.

7.Вычислить логарифмы:

8.

9. Вычислите: 1) ; 2) ; 3) ; 4) ; 5) ;

6) , 7) , 8) .

10. Выразите данный логарифм через натуральный и вычислите на микрокалькуляторе с точностью до 0,01 : 1) ; 2) .

ПЗ № 21. Радианный метод измерения углов вращения и связь с градусной мерой.


Задание:

1)Перепишите и заполните пропуски:
Пример 1. Найти радианную меру угла, выраженного в градусах: а) α = 40°, б) α = 120°, в) α = 150°.

Решение:

Ответ:

Пример 2. Найти градусную меру угла, выраженного в радианах:

Решение:

Ответ:

Пример 3. Вычислите:

Решение:



Ответ:

Пример 4. Вычислите:

Решение:



Ответ:

Пример 5.

Решение:



Ответ:

2)Решить задание ( по примерам):

  1. Найти радианную меру угла, выраженного в градусах: а) α = 75°, б) α = 32°, в) α = 140°.

  2. Найти градусную меру угла, выраженного в радианах:

  3. Вычислите:

  4. Вычислите:

3)Решить задание :

  1. Вычислите:

  2. Радианная мера двух углов треугольника равна   и  . Найдите градусную меру каждого из углов треугольника. 

  3. Выразите в градусной мере величину угла: .

  4. Выразите величину угла в радианах: .

  5. Найдите знак произведения, используя правило знаков по четвертям:

  6. Вычислите значение выражения:

  7. Вычислите:

  8. Вычислите:

  9. Найдите знак произведения:

  10. Вычислить значения и ,если α =120°.

  11. Вычислите значение тригонометрических функций: sin π/3;cos 7π/6;tg π;sin π/4;tg 2π/3;ctg π/2.

  12. Найдите радианную меру углов треугольника, если их величины относятся как 2:3:4.

  13. Может ли косинус быть равным:  

  14. Может ли синус быть равным:

  15. Вычислите:

  16. Вычислите :

  17. Известно, что Вычислите значение выражения:

  18. Известно, что . Вычислите значение выражения:

  19. Вычислите : .

  20. Известно, что . Вычислите: .

  21. Известно, что Вычислите:

  22. Найдите значение выражения: 5sin²3х – 6,если cos²3х = 0,6.

  23. Найдите значение выражения: 5sin²4х – 6,если cos²4х = 0,8.

  24. Известно, что . Вычислите: .

  25. Известно, что Вычислите:

ПЗ № 22. Основные тригонометрические тождества, формулы сложения, удвоения, преобразование суммы тригонометрических функций в произведение, преобразование произведения тригонометрических функций в сумму.

Задание:

1) Опорный конспект.

Основные тригонометрические тождества:

Формулы сложения:



Формулы двойного аргумента:

Формулы преобразования сумм или разности в произведение.



Формулы преобразования произведения тригонометрических функций в сумму:

2)Перепишите и заполните пропуски:
Пример 1.Вычислить : а) cos 18° cos 12° sin 18° sin 12°; б) cos 107° cos 17°sin 107° sin 17°;

в) sin 17° cos 13° sin 13° cos 17°; г) sin 43° cos 13° sin 13° cos 43°;

д) , е) .

Решение: а) cos 18° cos 12° sin 18° sin 12° = cos(18°12°) = cos 30° = …,

б) cos 107° cos 17° sin 107° sin 17° = cos(107°17°) = cos 90° = …,

в) sin 17° cos 13° sin 13° cos 17° = sin(17°13°) = sin 30° = …,

г) sin 43° cos 13° sin 13° cos 43° = sin(43°13°) = sin 30° = …,

д) = tg (9°51°) = tg 60° = …, е) = tg (65°20°) = tg 45° = … .

Ответ: а); б) 0; в) 0,5; г) 0,5; д) ; е) 1.

Пример 2.Вычислить : а) cos π /7 cos /21 sin π/ 7sin /21;

б) sin π /3 cos π /12  cos π /3sin π /12; в) .

Решение: а) cos π /7 cos /21 sin π /7sin /21 = cos /7 4π /21) = cos (3π /21 4π /21) =

= cos /21 = cos π /3 = …,

б) sin π /3 cosπ /12 cos π /3 sin π /12 = sin /3 π /12) = sin (4π /12π /12) = sin /12 =

= sin π /4 = …,

в) = tg (π /7 4π /21) = tg π /3 = …

Ответ: а) 0,5; б) /2; в).

Пример 3. Упростить: а) cos α cos 3α sinα sin3α; б) sin 2α cos α cos 2α sin α;

в) sin α cos 3α cos α sin 3α; г) .

Решение: а) cos α cos 3α sinα sin3α = cos (α 3α) = cosα;

б) sin 2α cos α cos 2α sin α = sin (2α α) = sin α;

в) sin α cos 3α cos α sin 3α = sin (αα) = sinα; г) = tg (x 3x) = tgx.

Ответ: а) cos 4α; б) sin α; в) sin 4α; г) tg 4x.

Пример 4. Упростить : а) cos α cos β sin α sin β, если α = 42 °, β = 18 °;

б) cos(x y) cos(x + y) + sin(x y) sin(x + y).

Решение: а) cos α cos β sin α sin β = cos (α β) = cos (42 ° 18 °) = cos 60 ° = …,

б) cos(x y) cos(x + y) + sin(x y) sin(x + y) = cos ((x  y) – (x + y)) = cos (–2y) = cos 2y.

Ответ: а) 0,5; б) cos 2y .

Пример 5. Упростить выражение:

Решение: Ответ: 1.

Пример 6. Вычислите: cos630°– sin1470°– сtg1125°.

Решение: cos630°– sin1470°– сtg1125° = cos(360° + 270°)– sin(4360° + 30°)сtg(3360 ° + 45°) =

= cos270°– sin30°– сtg45° = 0 – 0,5 – 1= … Ответ: – 1,5.

Пример7. Вычислить

Решение: Ответ:

Пример 8. Вычислить

Решение: Ответ:

Пример 9. Вычислить sin2α, если sinαcosα =

Решение: Возведем обе части равенства в квадрат: (sinαcosα)2 = ,

sin2α – 2sinαcosα + cos2α = , 2sinαcosα = – 1, 2sinαcosα = , sin2α = …

Ответ: .

Пример 10. Вычислить sin2α, если sinα = 0,6,

Решение: sin2α = 2sinα cosα . Т.к. ,то cosα < 0,

cos α =

sin2α = 2() () = ...

Ответ: 0,96.

Пример 11. Вычислить sinα/2, cosα/2, tgα/2, ctgα/2, если cosα = 0,8,

Решение: cos2 α/2 = (1 + cosα) : 2 = 1,8 : 2 = 0,9, cosα/2 = .

sin2 α/2 = (1 cosα) : 2 = 0,2 : 2 = 0,1, sinα/2 = .

tgα/2 = sinα/2 : cosα/2 = 0,33 : 0,95 = 33/95, ctgα/2 = cosα/2 : sinα/2 = 0,95 : 0,33 = 95/33.

Ответ: 0,33; 0,95;33/95; 95/33.

Пример 12. Пусть Найдем sin2, cos2, tg2.

Решение:


Ответ:

Пример 13. Докажите справедливость равенства cos 5π/ 12 + cos7π/ 12 =0 .

Решение:  cos 5π/ 12 + cos 7π/ 12 = 2cos π/ 2 cos( − π /12 ) = 20cos( − π/ 12 ) = … , 

Пример 14. Вычислить hello_html_39d0c6cc.gif

Решение: sin 70° + sin 10° = 2sin 40° cos30° = sin 40°.

cos70° − cos10° =  − 2 sin 40° sin 30° =  −  sin 40°.

hello_html_39d0c6cc.gif= sin 40° : (−  sin 40° ) =  −  .

Ответ: −  .

Пример 15. Вычислить cos11 π/12 – cos 5 π/12 .

Решение: cos11 π/12 – cos 5 π/12 = − 2sin(11 π/12 + 5 π/12) : 2 sin(11 π/12−5 π/12) : 2 = = − 2 sin 8π/12 sin 3π/12 = − 2 sin 2π/3 sin π/4 = − 2 = .

Ответ:.

Пример 16. Вычислить sin7 π/12 – sin π/12 .

Решение: sin7 π/12 – sin π/12 = 2 sin(7 π/12 − π/12) : 2 cos(7 π/12+ π/12) : 2 =

= 2 sin π/4 sin π/3 = 2 1/2 =

Ответ: .

Пример 17. Вычислим значение выражения .


Пример 18. Преобразуйте в сумму произведение Ответ.

2)Решить задание ( по примерам):

  1. Вычислить : а) cos 38° cos 22° sin 38° sin 22°; б) cos 55° cos 10°sin 55° sin 10°;

в) sin 47° cos 13° sin 13° cos 47°; г) sin 103° cos 13° sin 13° cos 103°;

д) , е) .

  1. Вычислить : а) cos  π /5 cos π /20 sin π/ 5sin π /20;

б) sin π /4 cos π /12  cos π /4sin π /12; в) .

  1. Упростить: а) cos 2α cos 6α – sin 2α sin 6α; б) sin 3α cos α cos 3α sin α;

в) sin 2α cos 3α cos 2α sin 3α; г) .

  1. Упростить : а) cos α cos β sin α sin β, если α = 42 °, β = 48 °;

б) cos(2x y) cos(2x + 3y) + sin(2x y) sin(2x + 3y).

  1. Упростить выражение:

  2. Вычислите: cos450°– sin750°сtg765°.

  3. Вычислить

  4. Вычислить

  5. Вычислить sin2α, если sinαcosα = 1/4.

  6. Вычислить cos 2α, если sinα = 0,8,

  7. Вычислить sinα/2, cosα/2, tgα/2, ctgα/2, если sinα = 0,8,

  8. Пусть Найдем sin2, cos2, tg2.

  9. Преобразовать в произведение .

  10. Преобразовать в произведение

  11. Преобразуйте в сумму произведение

  12. Вычислим значение выражения

3)Решить задание :

  1. Упростите выражение: sin(3π/2 – αcos(π/2 + α) + sin(2 π – α) + cos(3π/2 + α) + cosα ·sinα.

  2. Найдите cosß, если tgß = 7/24 и ß (π; 3π/2).

  3. Найдите значение выражения: 2sin²2х – 9cos²2х, если cos2х = – 0,9.

  4. Вычислите:3ctg60º· (sin310ºcos70º sin70ºcos310º).

  5. Найдите значение выражения:5 cos(3π/2 + α) , если α = 7π/6.

  6. Найдите значение выражения: 4 + 5tg²х • cos²х, если sinх = 0,4.

  7. Найдите значение выражения:7 cos(π + α) – sin(3π/2 + α), если cosα = 0,6.

  8. Упростить выражение 4⋅(tg(π t) + ctg(π t) + ctg(3π/2 t)) ctg(π t).

  9. Упростите выражение: .

  10. Вычислите:

  1. Упростите выражение: .

  2. Вычислите

  1. Найди значение выражения sin1050°+cos4620°+tg1035°.

  2. Вычислите:

  3. Упростите выражение 

  4. Вычислите:

  5. Упростите выражение:

  6. Вычислите: а) sin810°cos900o + tg585octg l845o + cos l35osin405°;
    б)
    cosl05°sinl95° + sin(-135°);

  7. Найдите значение выражения sin (х + у), если sin х= 9/41; cos у =40/41; х - угол II четверти.

  8. Найдите , если  и.

  9. Найдите значение выражения, если .

  10. Вычислить


  1. Дано: cos х =-12/13; 180 º < х < 270 º. Найти: cos х/2,tg x/2.

  2. Упростите выражение 

  3. Упростите выражение 

  4. Найти значение выражения: 2sin150 cos150.

  5. Найдите sin 2α, cos 2α,tg2α,  если и  .

  6. Найдите 24cos2α, если sinα = - 0,2 .

ПЗ № 23. Простейшие тригонометрические уравнения и неравенства.

Задание:

1)Перепишите и заполните пропуски:
Пример 1.Решите уравнение sin4xcos2x = 0.

Решение: sin4x – cos2x = 0 , 2sin2x cos2x – cos2 x = 0 , сos2x(2sin2x – 1)=0 ,

сos2x=0 или sin2x=1/2 .

2x = π/2 + π k, k , 2x = (1) n π/6 + π n, n Z .

х1= π/…+ π k/2 , k Z, x2 = (1)n π/…+ π n/2 , n Z .

Ответ: х1= π/4+ π k/2 , k Z; x2 = (1)n π/12+ π n/2 , n Z .

Пример 2. Решите уравнение (2 sin x – 1) (tg x) = 0.

Решение: ( 2 sin x – 1) hello_html_m61765ba1.gif (tg x) = 0,

2 sin x – 1= 0 или tg x = 0,

sin x = 1/2 tg x = ,

х1= (–1) n π/… + π n, n Z , х2 = π/… + π k, k .

Ответ: х1= (–1) n π/6 + π n, n Z , х2 = π/3 + π k, k .

Пример 3. Решите уравнение ( ctg x – 1) hello_html_m61765ba1.gif (2sin + 1) = 0.

Решение: ( ctg x – 1) hello_html_m61765ba1.gif (2sin + 1) = 0,

ctg x – 1 = 0 или 2sin + 1 = 0,

ctg x = 1 sin = – 1/2, х/2 = (–1) n +1 π/6 + π n, n Z,

х1 = π/… + π k, k , х2 = (–1) n +1 π/… + 2π n, n Z.

Ответ: х1 = π/4 + π k, k , х2 = (–1) n +1 π/3 + 2π n, n Z.

Пример 4. Решите уравнение . Решение: , cos (4x 3x) =, cos x =, x = 2πn, nZ. Ответ: x = 2πn, nZ.

Пример 5. Решите уравнение 2cos( х + π/3) = .

Решение: 2cos( х + π/3) = , cos( х + π/3) = –, х + π/3 = ± 5π/6+2πn, nZ, x = – π/3 ± 5π/6 + 2πn, nZ. x1 = – π/3 + 5π/6+2πn, nZ, x1 = π/… +2πn, nZ,

x2 = π/3 – 5π/6+2πn, nZ, x2 = 7π/… +2πn, nZ.

Ответ: x1 = π/2 +2πn, nZ, x2 = –7π/6 +2πn, nZ.

Пример 6. Решите уравнение sin( 2х + π/2) = 0.

Решение: sin( 2х + π/2) = 0, 2х + π/2 = πn, nZ,2х = – + πn, nZ,х = – , nZ.

Ответ: х = = – , nZ

Пример 7. Решите уравнение a) arccos б)arcsin (5x+2) = 0.

Решение: a) arccos cos(arccos ,

б) sin(arcsin (5x+2)) = sin0, 5x + 2 = 0, 5x = – 2,x = …

Ответ: a) 2,5, б) – 0,4.

Пример 8. Решите уравнение arctg (4x – 1) =

Решение: tg(arctg (4x – 1)) = tg 4x – 1 = 1, 4x = 2, x = …

Ответ: 0,5.

Пример 9. Решить неравенство sin(t) 1/2.

Решение: Рисуем единичную окружность. Так как sin(t) по определению - это координата y, отмечаем на оси Оу точку у = 1/2. Проводим через неё прямую, параллельную оси Ох. В местах пересечения прямой с графиком единичной окружности отмечаем точки Pt1 и Pt2. Соединяем двум отрезками начало координат с точками Pt1 и Pt2

Решением данного неравенства будут все точки единичной окружности расположенные выше данных точек. Другими словами решением будет являться дуга l. Теперь необходимо указать условия, при которых произвольная точка будет принадлежать дуге l.hello_html_m203b787d.jpg

Pt1 лежит в правой полуокружности, её ордината равна 1/2,

тогда t1= arcsin(1/2) = π/6. Для описания точки Pt1 можно записать следующую формулу: t2 = π – arcsin(1/2) = 7π/6. В итоге получаем

для t следующее неравенство:/6 t 7π/6, Мы сохраняем знаки неравенств. А так как функция синус функция периодичная, значит решения будут повторяться через каждые 2π. Это условие добавляем к полученному неравенству для t и записываем ответ.

Ответ: /6+2πn t 7π/6 + 2πn, при любом целом n.

Пример 10. Решить неравенство cos(t) <1/2.

Решение: Нарисуем единичную окружность. Так как согласно определению cos(t) это координата х, отмечаем на грфике на оси Ох точку x = 1/2.
Проводим через эту точку прямую, параллельную оси Оу. В местах пересечения прямой с графиком единичной окружности отмечаем точки P
t1 и Pt2. Соединяем двум отрезками начало координат с точками Pt1 и Pt2.
hello_html_mecb7ec3.jpg

Решениями будут все точки единичной окружности, которые принадлежать

дуге l. Найдем точки t1 и t2t1 = arccos(1/2) = π/3 ,

t2 = 2π arccos(1/2) = 2π/3 = 5π/3.

Получили неравенство для t: π/3 < t < 5π/3.

Так как косинус - это функция периодичная, то решения будут повторяться через каждые 2π. Это условие добавляем к полученному неравенству для t и записываем ответ.hello_html_m174f9f6b.jpg

Ответ: π/3+2πn π/3+2πn, для любого целого n.

Пример 11. Решить неравенство tg(t) 1.

Решение: Период тангенса равняется π. Найдем решения, которые принадлежат промежутку (-π/2; π/2) правая полуокружность. Далее воспользовавшись периодичностью тангенса, запишем все решения данного неравенства. Нарисуем единичную окружность и отметим на ней линию тангенсов.

Если t будет являться решение неравенства, то ордината точки Т = tg(t) должна быть меньше или равна 1. Множество таких точек будет составлять луч АТ. Множество точек Pt, которые будут соответствовать точкам этого луча – дуга l. Причем, точка P(-π/2) не принадлежит этой дуге. Найдем условие, при котором некоторая точка Pt будет принадлежать дуге l. t1 = arctg(1) = π/4.  Получаем неравенство /2 t/4. 

Учитывая период тангенса записываем ответ.

Ответ: /2 + πnt /4 + πn, для любого целого n.

Пример 12. Решить неравенство:   sin x > 0.

Решение: В пределах одного оборота единичного радиуса это неравенство справедливо

при 0 < x < hello_html_3fac242e.gif. Теперь необходимо добавить период синуса  2hello_html_3fac242e.gif n :hello_html_m12869a62.png

0 + 2πn х π + 2πn, 2πn х π + 2πn, при любом целом n.hello_html_6e343a75.png

Ответ: 2πn х π+ 2πn, при любом целом n.

Пример 13. а) Решить неравенство:   sin x > 0.5 .

Решение: π/6 + 2πn < х < 5π/6 + 2πn, для любого целого n.

б) Решить неравенство cosх > /2.

Решение: /4 + 2πn х π/4 + 2πn, для любого целого n.( по рис.)

Пример 14. Решить неравенство cos (x/4 – 1) ≤ (/2).

Решение: Обозначим x/4 – 1 = у. Решая неравенство cos у ≤ (/2), находим 
3π/4 + 2πn ≤ у ≤ 5π/4 + 2πn, n Z.

Заменяя у = x/4 – 1, получаем 3π/4 + 2πn ≤ x/4 – 1 ≤ 5π/4 + 2πn, откуда 
1 + 3π/4 + 2πn ≤ x/4 ≤ 1 + 5π/4 + 2πn, 4 + 3π + 8 πn ≤ х ≤ 4 + 5π + 8 πn, n Z.

Ответ: 4 + 3π + 8 πn ≤ х ≤ 4 + 5π + 8 πn, n Z.


2)Решить задание ( по примерам):

  1. Решите уравнение sin4xcos2x = 0.

  2. Решите уравнение (2 sin x ) hello_html_m61765ba1.gif (tg x – ) = 0.

  3. Решите уравнение (ctg x ) hello_html_m61765ba1.gif (2sin +) = 0.

  4. Решите уравнение cos 4xcos3x + sin4xsin3x =

  5. Решите уравнение 2cos(х + π/4) = –

  6. Решите уравнение sin( 2х + π/3) = 0.

  7. Решите уравнение a) arccos б)arcsin (5x + 4) = 0.

  8. Решите уравнение arctg (3x – 1) =

  9. Решить неравенство cos x ≤ - .

  10. Решить неравенство sin x > - .

  11. Решить неравенство sin x ≥ .

  12. Решить неравенство ctg x < - .

  13. Решить неравенство tg x > 1.

  14. Решить неравенство tg x ≤ 1.

  15. Решить неравенство ctg x ≥ - .

  16. Решить неравенство сtg x ≤ 1.

3)Решить задание :

  1. Решите уравнения:

  1. Решите уравнения:

  1. Решите уравнения:

  1. Решите уравнения:

  1. Решите уравнения:

  2. Решите уравнения:

  1. Решите уравнения:

  2. Решите уравнения:

  3. Решите уравнения:


  1. Решите уравнения:

  2. Решите уравнения:

  1. Решите уравнения:

  1. Решите неравенства:

а) ,б) , в) .

  1. Решите неравенства:

а) ;б) ;в) ;

  1. Найдите какой-либо корень уравнения , удовлетворяющий неравенству .

  2. Решите неравенство: .

  3. Решите неравенство:

  1. Решите неравенства:

а) б) в) .

  1. Решите неравенства:

а)

  1. Определите все а, при каждом из которых неравенство

4sinx + 3cosxа имеет хотя бы одно решение.




ПЗ № 24. Обратные тригонометрические функции: арксинус, арккосинус, арктангенс.

Задание:

1) Перепишите и заполните пропуски:
Для выполнения заданий, связанных с обратными тригонометрическими функциями, нужно, во-первых, четко помнить определения этих понятий:


hello_html_mc5281e4.gif





,


hello_html_m3aca29e1.gif

Пример 1. Вычислить:

а) arccos (cos ), б) cos(arccos 0,4),в) arcsin (sin ),

г) sin(arcsin 0,6), д) sin (arccos 0,6),е)tg(arcsin 0,8).

Решение:

а) arccos (cos ) = , б) cos(arccos 0,4) = …,

в) arcsin (sin ) = …, г) sin(arcsin 0,6) = 0,6,

д) ,

е) .

Ответ:

а) arccos (cos ) = , б) cos(arccos 0,4) = 0,4,в) arcsin (sin ) = ,

г) sin(arcsin 0,6) = 0,6, д) , е) .

Пример 2. Вычислить cos(4arctg 5).

Решение:

Пусть α = arctg5, тогда tg α = 5. Требуется найти cos4α. Вычислим вначале cos2α, используя универсальную подстановку:

.


Тогда получаем, что:

.

Ответ: .

Пример 3. Вычислить  arcsin (sin 12).

Решение:

По условию задачи требуется найти угол, синус которого равен синусу угла в 12 радиан и который принадлежит промежутку  Заметим, что поэтому . Поскольку  , угол 12 - 4π является искомым углом: его синус равен sin 12, и он находится в области возможных значений арксинуса.

Ответ: arcsin (sin12) = 12 - 4π.

Пример 4. Вычислить  

Решение:

Введем два угла:  и . Оба они лежат в первой четверти, значит, все их тригонометрические функции положительны.

Мы знаем, что. Требуется найти синус суммы этих углов, а для этого нужно знать их синусы и косинусы.

Во-первых,

  .

Во-вторых, 

.

Следовательно,

Ответ:  .

Пример 5. Вычислить 

Решение:

Типичная ошибка в данном случае – это сразу же написать в ответ 4. Как мы указывали в предыдущем примере, для использования основных свойств аркфункций необходимо проверить соответствующие ограничения на их аргумент. Мы имеем дело со свойством:

при . Но 4> .

Главное на этом этапе решения не подумать, что указанное выражение не имеет смысла и его нельзя вычислить. Ведь четверку, которая является аргументом тангенса, мы можем уменьшить при помощи вычитания периода тангенса, и это не повлияет на значение выражения. Проделав такие действия, у нас появится шанс уменьшить аргумент так, чтобы он вошел в указанный диапазон.

 , т.к. < 1 поскольку > 3, следовательно, , т.к. .

Ответ: .


Пример 6. Вычислить sin (2 arcsin 0,6).

Решение:


Ответ: 0,96.

 Пример 7. Вычислить arccos xarcsin x = arccos .

Решение:

Учитывая, что arccos = и arcsin x + arccos x = , заменим в уравнении arcsin x выражением – arccos x, получим уравнение

arccos x – (– arccos x)= ,

2arccos x – = , 2arccos x = + = .

arccos x = , x = cos , x= 1/2 = …

Ответ: 0,5.

Пример 8. Решите уравнения:

а) 6arcsin (x2 – 6x+8,5) = π ;

б)  3arcsin2x – 10arcsinx + 3 = 0.    

Решение:

а) 6arcsin (x2 – 6x+8,5) = π  , arcsin(x2 – 6x+8,5) = ,

x2 – 6x+8,5 = 0,5; x2 – 6x+8 = 0,

D = 36 – 418 =….

, .

б) 3arcsin2x – 10arcsinx + 3 = 0.    arcsinx = у,

2 – 10у + 3 = 0, D = 100 – 433 = 64.

- не уд. усл.

.

arcsinx = 0,3, х= sin 0,3

Ответ: а) x1= 4,x2 = 2.б) х= sin 0,3

Пример 9. Вычислить: а) arcsin (-2), б) arccos

Решение:

а) Типичная ошибка в данном случае – это начать выносить минус и что-то упрощать. Первое, что необходимо заметить, это то, что аргумент арксинуса не входит в область определения.

Следовательно, данная запись не имеет значения, и вычислить арксинус нельзя.

б) Стандартная ошибка в данном случае заключается в том, что путают местами значения аргумента и функции и дают ответ 1/2. Это неверно! Конечно, возникает мысль, что в таблице косинусу  соответствует значение 1/2, но в таком случае перепутано то, что вычисляются аркфункции не от углов, а от значений тригонометрических функций. Т.е. arccos 1/2 =, а не arccos  = 1/2.

Кроме того, поскольку мы выяснили, что  является именно аргументом арккосинуса,

то необходимо проверить, чтобы он входил в область определения. Для этого вспомним, что >1,т.е. , а значит арккосинус не имеет смысла и вычислить его нельзя.

Кстати, например, выражение arccos  имеет смысл, т.к. , но поскольку значение косинуса, равное  не является табличным, то и вычислить арккосинус с помощью таблицы нельзя.

Ответ: Выражения не имеют смысла.

 Пример 10. Вычислить arcсtg х, если известно, что arctg х =  .

Решение:

Вспомним, по какой формуле связаны между собой указанные функции:


И выразим из нее то, что нам нужно:.

Ответ: .

2)А)Решить задание ( по примерам):

  1. Вычислить:

а) arccos (cos ), б) cos(arccos 0,25),в) arcsin (sin ),

г) sin(arcsin 0,45), д) sin (arccos 0,8),е)tg(arcsin 0,6).

  1. Вычислить cos(4arctg 3).

  2. Вычислить  arcsin (sin 6).

  3. Вычислить  

  4. Вычислить 

  5. Вычислить sin (2 arcsin 0,8).

  6. Вычислить arccos x – arcsin x = arccos .

  7. Решите уравнения:

а) 6arcsin (x2 – 7x+12,5) = π ;

б)  3arcsin2x – 11arcsinx + 6 = 0.    

  1. Вычислить: а) arcsin (hello_html_m7cdd195e.gif 4), б) arccos

  2. Вычислить arcсtg х, если известно, что arctg х =  .


Б) Построить таблицы:

Табличные значения обратных тригонометрических функций.

hello_html_4c5b70ac.jpg

hello_html_m6baf672f.jpg

В) Преобразование выражений.

(Перепишите и заполните пропуски)









3)Решить задание ( по примерам):


hello_html_m46bcf841.gif

hello_html_m3156b9af.gif



4)Решить задание :

  1. Вычислите значения: a) cos; б)sin; в) sin

  2. Упростите выражение: а) arctg+ arctg; б) 3arcsin+ arcsin.

  3. Упростите выражение: а) arccos б) - arcsin;

  4. Решите уравнение: а) arcsin x =; б) arcsin x = arcctg x; в) arccos(x+1) = arcctg x; г) 4arccos xhello_html_24574eef.gif4arccos x hello_html_24574eef.gif 4arcsinx+=0;д) arcsin(x +1) + arcos2x =0;

ПЗ № 25. Примеры зависимостей между переменными в реальных процессах из смежных дисциплин. Определение функций.

Задание:

1) Опорный конспект.

А) Примеры применения различных функций в жизни, технике, природе.

Определение. Числовой функцией с областью определения D называется соответствие, при котором каждому числу x из множества D сопоставляется по некоторому правилу единственное число y, зависящее от x. Принято называтьx независимой переменной или аргументом, а у — зависимой переменной или значением функции.

Записывают указанное соотношение между x и у в общем виде так: у = f (x) или у = F (x) и т. п.

График функции y = f (х) - это множество всех точек плоскости, координаты (х, у) которых удовлетворяют соотношению y = f(x).

Способы задания функции:

  1. аналитический (с помощью формулы);

  2. графический;

  3. табличный;

  4. словесный.

В наши дни без функций невозможно не только рассчитать космические траектории, работу ядерных реакторов, и бег океанской волны и закономерности развития циклона, но и экономично управлять производством, распределением ресурсов, организацией технологичных процессов, прогнозировать течение химических реакций или изменение численности различных взаимосвязанных в природе видов животных и растений, потому что все это – динамические процессы, которые описывает функция.

а) Линейная функция

Функция y = a x + b называется линейной потому, что ее график есть прямая линия. Характеристическим свойством линейной функции является изменение функции пропорционально изменению аргумента. Поэтому с помощью линейной функции описываются пропорциональные зависимости. Например, при равномерном движении с постоянной скоростью v пройденный путь s пропорционален времени t и выражается формулой s = v t, т.е. s – линейная функция t.hello_html_31dd52b1.png

Пример линейной функции дает зависимость между различными шкалами температур. Абсолютная температураТ (по Кельвину) связана с температурой tͦC на шкале Цельсия формулой t = T + 273 ͦ. Другой пример – напряжение в электрической цепи прямо пропорционально силе тока U = IR. Можно много приводить примеров линейных зависимостей в физике, химии. Рассмотрим задачу на линейное расширение тел.

Задача. При температуре 0оС рельс имеет длину l0= 12,5 м. при возрастании температуры происходит тепловое расширение рельса и его длина, выраженная в метрах, меняется по закону l(tо) = l0(1 + hello_html_m5cdf3cb6.giftо),где hello_html_m5cdf3cb6.gif= 1,2 ˖ 10–5коэффициент теплового расширения в градусах Цельсия в минус первой степени, tо – температура (в градусах Цельсия). При какой температуре рельс удлинится на 6 мм. Ответ выразить в градусах Цельсия.

Решение. Выразим из заданной формулы t: .

Заметим, ,

тогда hello_html_53e6f54e.gif

Ответ: 40.

б) Квадратичная функция

Графиком квадратичной функции

является парабола.

Хорошо известно, что траектория камня, брошенного под углом к горизонту, летящего футбольного мяча или артиллерийского снаряда будет параболой (при отсутствии сопротивления воздуха). Однако мало кто знает, что зона достижимости для пущенных нами камней вновь будет параболой. В данном случае мы говорим об огибающей кривой траекторией камней, выпущенных из данной точки под разными углами, но с одной и той же начальной скоростью. Если рассматривать такую огибающую в пространстве, то возникнет поверхность, образованная вращением этой параболы вокруг ее оси. Такая поверхность носит название параболоида вращения.

Задача. Выcота над землeй подброшенного вверх мяча меняетcя по закону hello_html_403f5e9b.png, где h — выcота в метрах, t — время в cекундах, прошедшее c момента броcка. Cколько cекунд мяч будет находитьcя на выcоте не менее трeх метров?

Решение. Решим неравенство,

, , t1 = 1,4, t2 = 0,2

0,2t1,4

t = 1,4 - 0,2 = 1,2

Ответ: 1,2

Парабола обладает оптическим свойством: все лучи, исходящие из источника света, находящегося в фокусе параболы, после отражения оказываются направленными параллельно ее оси. Это свойство параболы используется при изготовлении прожекторов, автомобильных фар, карманных фонариков, зеркала которых имеют вид параболоидов вращения. hello_html_m779371d5.png

Б)Функциональные зависимости в повседневной жизни

Пример 1. Рассмотрим деление праздничного торта между гостями. Отчего зависит количество порций?– от числа гостей. А от чего зависит вес порции? – тоже от числа гостей.

В первом случае, чем больше гостей, тем на большее количество порций мы должны разделить торт (рис. 1).hello_html_m9d34e61.png

Здесь наглядно можно представить прямую пропорциональную зависимость.hello_html_m70d6cf06.gif

Во втором случае, чем больше гостей, тем меньше вес порции. Здесь мы видим обратную пропорциональную зависимость (рис. 2).

Пример 2. Мы живём в век информационных технологий. Ежедневно мы получаем массу информации из различных источников: телевидения, радио, газет, журналов, и, конечно, из Интернета. Известно, что объём информации каждые пять лет увеличивается в два раза.hello_html_m64a6d6cd.png

Если построить график зависимости объёма информации от времени, то получим некоторую кривую, которая в математике называется экспонентой и является графиком показательной функции (рис. 3).

Пример 3.На голове человека растут волосы, которые регулярно стригут.hello_html_fde3c9e.png

График полученной зависимости (при условии, что стрижку делают регулярно) похож на функцию дробной части числа, смещённую на aединиц вверх: (рис. 4).


Пример 4. За время обучения в школе каждый год переходим в следующий класс.hello_html_4bc21f47.png

Такая зависимость сходна с функцией целой части числа на ограниченном промежутке (рис. 5).

Пример 5. По графикам оценить

множество значений каждой из

представленных функций.



hello_html_m321966bf.gif

Ответ:

hello_html_5687e039.gif

2)Решить задание:

1)

hello_html_20988e1e.jpg

2) Функция задана формулой f(х) = - 5х – 2. Найдите: а) f(0); б) f(2); в) f(- 3); г) f().

3)Известно, что g(х) = 12 – 4х. Найдите значение х, при котором: а) g(х) = 0; б) g(х) = 6; в) g(х) = - 8.

4)Функция задана формулой f(х) = . Найдите: а) f(0); б) f(2); в) f(- 3); г) f().

5)Постройте график функции у = х +3. При каких значениях х выполняется неравенство ?

6)С помощью формул описано изменение температуры воды в баке (в 0С) как функции времени t (в минутах):

2t + 20, если 0 ≤ t < 40,

p = 100, если 40 ≤ t < 60,

t + 140, если 60 ≤ t ≤ 150.

Найдите: р (20); р (40); р (50); р (60); р (90). Постройте график функции р = f (t). Какой физический смысл имеет рассматриваемый процесс в каждом из промежутков [0;40], [40;60], [60;150]?

7)Известно, что g(х) = 2х – 4. Найдите значение х, при котором: а) g(х) = 0; б) g(х) = 6; в) g(х) = - 8.

8)Зависимость расстояния s (в километрах) велосипедиста до базы от времени его движения t (в часах) задана следующим образом:

2t + 20, если 0 ≤ t < 40,

s = 100, если 40 ≤ t < 60,

t + 140, если 60 ≤ t ≤ 150.

Найдите: s (0); s (1); s (1,4); s (2). Постройте график функции s = f (t) (масштаб по оси t: 1 ед. – 6 клеточек; по оси s: 10 ед. – 4 клеточки). Опишите, как происходило движение велосипедиста.

9) По графикам оценить множество значений каждой из представленных функций.


hello_html_m5e890c1e.gif

10) Садово-огородные процессы тоже можно представить в виде функции и потроить график. К примеру, яблоко росло, зрело, потом его высушили. Постройте эту кусочную функцию.hello_html_m6473b701.gif

11) Графиком проиллюстрировать смысл пословицы «Каково жизнь проживешь, такую славу наживешь».

12) 1. По графику функции у = 2х2 найдите:

а) значение функции, если х = 10; х = – 12;

б) значение аргумента, если у = 4; у = – 5.

2. Постройте график функции у = – 0,5х2

13) Постройте график функции:

.

14) Вычислите координаты точек пересечения параболы и прямой:

.

15) Постройте график функции:

.


ПЗ № 26. Построение и чтение графиков функций. Исследование функции.

Задание:

1)Опорный конспект.

Построение и чтение графиков.

Построение и чтение графиков должно опираться на множество практических навыков.

Приведем некоторые из рекомендованных правил при графической обработке данных и чтении графиков:

- во всякой диаграмме графический образ, как основной элемент, для которого существуют и которому подчинены все остальные элементы, должен быть в центре внимания пользователя;

- композиция диаграммы должна подчиняться правильному соотношению ее частей (согласованию их размеров, толщины, формы и положения);

- график должен быть достаточно четким, но наиболее важные его элементы должны выделяться на общем фоне в соответствии с их значением;

- вертикальную шкалу для кривой независимо от ее назначения желательно выбирать так, чтобы на диаграмме оказалась нулевая отметка;

- нулевые линии шкал для кривой следует резко отграничивать от других координатных линий;

- когда шкала относится к датам, а представляемый период является неполным, лучше не выделять первые и последние ординаты, так как подобная диаграмма не отмечает начало или конец времени;

- для кривых, которые имеют шкалу, изображающую проценты, желательно выделить каким-то образом линию 100% или другие линии, используемые в качестве основы для сравнения, а также обязательно показывать 0%;

- кривые линии диаграммы должны резко отличаться от прямых линий;

- для кривых, характеризующих группы наблюдений, рекомендуется по возможности ясно указывать на диаграмме все кривые, представляющие отдельные наблюдения;

- горизонтальную шкалу для кривых следует писать слева направо, а вертикальную - снизу вверх;

- рекомендуется показывать достаточный минимум координатных линий для облегчения чтения диаграммы;

- при необходимости желательно включать в график цифровые данные или формулы;

- на графиках при резких колебаниях кривых закраска полос неудобна;

- использование изображения линейных величин с помощью площадей или объемов некорректно, т.к. их не удастся правильно истолковать;

- характер координатной сетки должен быть разный в зависимости от назначения графического образа.

- название графика располагают под ним, хотя иногда его можно писать выше диаграммы, если она не предназначена для печати, например, в настенных графиках;

- наименования следует формулировать возможно яснее и полнее. Если требуется, необходимо дополнительно вводить подзаголовки или пояснения;

- общая структура графиков должна предполагать чтение слева направо;

- чтение графика следует начинать с заголовка, сообщающего, какие сведения можно из него получить. Затем надо уяснить строение графического образа и изучить специфику его элементов: шкалу, масштабы, единицы измерения, легенды и т.п., что необходимо для определения сообщаемой информации по частным вопросам. Нужно начинать восприятие графического образа как общего целого, т. е. во взаимоотношениях элементов. Затем надо увидеть выражаемое содержание графика, ясно представлять, чему соответствуют те или иные изменения графического образа.hello_html_m188e6616.png

Примеры графиков.

1)Оценка ущерба из-за загрязнения окружающей среды при производстве электроэнергии (ленточная диаграмма сравнения с логарифмической шкалой).

2)По графику найдите:hello_html_m63b1a32.png

а)Какова область определения функции?

б)Назовите множество значений функции.

в)Назовите нули функции.

г) Назовите точки максимумов функции.

д)Назовите точки минимумов функции.


Ответ:

а) ,б) ,в) – 4, - 2,0,2,4 , г) – 3 ;1, д) – 1 ;3.

3) Построить график функции hello_html_4124e417.gif на основании результатов исследования функции.

Решение:

Для построения графика функции исследуем ее, придерживаясь общей схемы исследования.

  1. Нахождение области определения:

hello_html_m40b31e73.gif.

  1. Определение четности или нечетности:

hello_html_m6a4e5b1f.gif. Функция hello_html_4124e417.gif — четная.

Дальнейшее исследование будем проводить для hello_html_m4066d436.gif.

  1. Область изменения функции.

Если x = 1, то y = 0. Если hello_html_6087da64.gif. Следовательно, hello_html_402daa4e.gif.

hello_html_m2a046132.gif

  1. Пересечение с координатными осями.

Пересечений с осью 0y нет, т.к. x = 0 не входит в область определения функции.

  1. Выделение промежутков монотонности.

Для x1 > x2  ≥ 1 рассмотрим разность:

hello_html_62b3d6c1.gif.

При возрастании значений x от 1 до ∞ значения y возрастают.hello_html_m7fc1e073.gif

  1. Нахождение корней функции и промежутков знакопостоянства.

Если hello_html_191efdd0.gif при x = 1, hello_html_m77f109f8.gif при всех hello_html_156ca2f3.gif.

По результатам исследований строим график функции hello_html_4124e417.gifhello_html_m287b4e73.png

4) Построить графики следующих функций: 

Решение:

  1. Рассмотрим функцию. Она определена на всей оси 0x, четная. Ее график состоит из двух лучей, выходящих из начала координат и направленных по биссектрисам I и II координатных углов.hello_html_4a52d6cc.png

  2. Обратимся к функции hello_html_96be002.gif. Она определена на всей оси 0x, четная. На рисунке приведен ее график, причем он построен из двух половинок y = 2x , при x ≥ 0 и y = 2-x , при x < 0.

5) По графику выполните задание:

hello_html_m46e816e6.pnghello_html_m3316d847.png

hello_html_535ea5db.png

Ответ: 4,4,24;3,8;6;2,5.

6)

hello_html_m2930b060.jpg

7) По графику выполните задание:

hello_html_48efe2fd.png

hello_html_c75446b.pnghello_html_m72dfc30b.pnghello_html_142e8802.pnghello_html_2cc50b8.png

2)Решить задание:

  1. По графику функции найдите:

а)Какова область определения функции?

б)Назовите множество значений функции.

в)Назовите нули функции.

г) Назовите точки максимумов функции.

д)Назовите точки минимумов функции.

  1. Функция задана формулой . Найдите значение , при котором () =0.

  2. Определите при каких значениях существует функция .

  3. Постройте график квадратичной функции и запишите ее свойства (область определения функции, область значений функции, нули функции, промежутки знакопостоянства, промежутки монотонности):

  4. По графику выполните задание:

hello_html_m3575ae9b.pnghello_html_m5d77f20a.png

hello_html_m60d1b10c.pnghello_html_3ec17e75.png

  1. Определите при каких значениях существует функция .hello_html_m18d33162.png

  2. По графику функции найдите:

а) множество значений функции;

б) значение аргумента при у (– 3; 2];

в) промежутки знакопостоянства функции;

г) точки экстремума функции.

  1. Функция задана формулой . Найдите (-5).hello_html_404ccfeb.gif

  2. Функция задана формулой . Найдите (-3).

  3. Функция задана формулой . Найдите значение , при котором () =0.

  4. Постройте график функции у = 2х + 3 .

  5. По графику функции у = найдите:

а) ;б) значение аргумента, если у = 3; у = 1,5.hello_html_mbde74e8.png

  1. Проходит ли график функции у = через точки

А(13; 196 ); В(7; 49); С(–10; 100).

  1. На рисунке изображены графики функций y = f(x) и y = g(x), заданных на промежутке [-9; 8]. Укажите те значения х, при которых выполняется неравенство f(x) < g(x).



ПЗ № 27. Свойства линейной, квадратичной, кусочно-линейной и дробно-линейной функций.

Задание:

1)Опорный конспект.

Переменная величина y называется функцией переменной величины x, если каждому значению x соответствует определенное значение y.

Множество всех тех значений, которые принимает аргумент x функции y = f (x) , называется областью определения этой функции.

Множество всех тех значений, которые принимает сама функция y = f (x) , называется областью значений (изменения) этой функции.

Функция y = f (x)   называется четной, если при всех значений x из области определения этой функции f (- x) = f (x).

Функция y = f (x)   называется нечетной, если при всех значений x из области определения этой функции f (- x) = - f (x).

Функция y = f (x)   называется возрастающей (убывающей)  на данном промежутке, если при произвольных двух различных значениях аргумента, из данного промежутка, большему значению аргумента соответствует большее (меньшее) значение функции.

Функция y = f (x)   называется периодической, с периодом T, где T ≠ 0, если значение функции не изменяется при прибавлении числа T к любому допустимому значению аргумента:

 f ( x +T) = f (x).

Функция y = f (x)   называется ограниченной, если можно указать такое положительное число M, что hello_html_m66ebaed3.gifдля всех значений x из области определения функции. Если же точка M не существует, то функция называется неограниченной.

Графиком функции y = f (x)   называется множество всех точек плоскости,

координаты которых (x, f (x)). Функцию вида y = ax2 + bx + c называют квадратичной. Графиком квадратичной функции является кривая, называемая параболой.

Точку с координатами (- b/2a, - (b2 – 4ac) / 4a) называют вершиной параболы.

Соответствие между элементами двух множеств X и Y, при котором каждому элементу множества X сопоставляется не более одного элемента Y, называется функцией.

Отсюда следует, что понятие функции имеет три главных компонента:

множество X (которое называется областью определения функции);

множество Y (которое называется областью значений функции);

закон соответствия (который иногда называется функциональной зависимостью).

При этом закон соответствия может быть задан любым способом: таблицей, графиком, формулой или как-то иначе, например, при помощи словесного описания.

Если функцию задают формулой, то при этом фактически указывают область определения функции и закон соответствия (область значений функции не указывается явно, так как она устанавливается, исходя из данной формулы).

Областью определения функции, заданной явной аналитической формулой, считают множество всех тех значений аргумента, для которых все указанные в формуле операции выполнимы.

Опишем далее способы построения графиков функций.

1 способ «по точкам». Вытекает из определения графика функции. Он является длинным и недостаточно надежным. Применяется в школьном курсе математики при первоначальном знакомстве с простейшими функциями.

2 способ «путем сдвига графиков основных функций или сдвига осей координат». Чтобы построить график функции y = f (x)  + c можно или график функции y = f (x)   сдвинуть вдоль оси 0y на c единиц в сторону, совпадающую со знаком c , или перенести параллельно ось 0y в сторону, противоположную знаку c.

Чтобы построить график функции y = f (x + b) , можно или график функции y = f (x)   вдоль оси 0x на b единиц в сторону, противоположную знаку b, или перенести параллельно ось 0y в сторону, совпадающую со знаком b.

3 способ, «путем симметричного отображения относительно осей координат». Чтобы построить график функции y = - f (x)   , можно построить изображение, симметричное графику функции y = f (x)   относительно оси абсцисс.

Чтобы построить график функции y = f ( - x) , можно построить изображение, симметричное графику функции y = f (x)   относительно оси ординат.

4 способ, «путем деформирования графиков основных функций». Чтобы построить график функции y =a f (x)   при a > 0, можно график исходной функции растянуть (сжать) вдоль оси ординат, если a > 1 (0 < a < 1).

Чтобы построить график функции y = f (bx)    при b > 0, можно график исходной функции растянуть (сжать) вдоль оси абсцисс, если b > 1 (0 < b < 1).

5 способ. «Способы построения графиков функций, аналитическое выражение которых содержит знак абсолютной величины».

  1. Функция четная. Чтобы построить ее график, достаточно построить для всех неотрицательных значении аргумента график функции y = f (x), а затем достроить его левую часть, симметричную правой относительно оси ординат.

  2. Рассмотри далее, как строить функцию. Можно данную функцию рассматривать как совокупность двух функций:  .

  3. Чтобы построить график функции, достаточно построить график функции y = f (x)  и ту часть графика, которая расположена в нижней полуплоскости, симметрично отразить относительно оси абсцисс.

  4. Вспомним, как строится функция.

Функция  четная. Построить для всех неотрицательных значений аргумента график функции y = f (x)   , затем его симметрично отразить относительно оси ординат, и, наконец, ту часть полученного графика, которая расположена в нижней полуплоскости, симметрично отразить относительно оси абсцисс.

6 способ. «Кусочно-линейная функция».

Графиком кусочно-линейной функции является ломаная линия. Для построения графика находят уравнения звеньев ломаной.

Пример 1. Построить график функции hello_html_m13626efb.png 

Решение:

Дана функция hello_html_m13626efb.png. Преобразуем hello_html_m331da3eb.png

  1. Область определения: hello_html_5b28aadc.gif

  2. Область значений: hello_html_m64d87c78.gif

  3. Четность, нечетность: и ни четная, ни нечетная hello_html_m493e4791.gif

  4. Монотонность: убывает во всей области определения hello_html_m247f5f00.gif.

  5. Пересечение с осями 0x и 0yhello_html_m6dc57d6.gif

  6. Промежутки знакопостоянства:

hello_html_m3b988a6f.gif,hello_html_509010ce.gif.hello_html_m64191655.png

  1. Поведение функции вблизи точек разрыва и при hello_html_m263828a.gif:

hello_html_m64b6c4e7.gif

По результатам решения строим график.

Комментарий. При построении графика функции следует найти точки, в которых он пересекает оси координат, а также выяснить поведение функции при x, стремящемся к hello_html_m4087d48.gif в случае, когда ее область определения не ограничена. Необходимо также исследовать поведение функции вблизи тех точек, в которых она не определена.

Пример 2.Примеры функций.

hello_html_m7e89d1ab.jpghello_html_587b5025.png


2)Решить задание:

  1. С помощью графиков определите, сколько решений имеет система уравнений:

  2. Постройте график функции у = х2 + х – 6.

Используя график, решите неравенство х2 + х – 6 < 0.

  1. Постройте график функции у = 2х2 найдите:

а) значение функции, если х = 10; х = – 12;hello_html_42f57f2d.png

б) значение аргумента, если у = 4; у = – 5.

  1. Постройте график у = найдите:

а) значение y, если х = 10; х = – 8;

б) значение x, если у = 11;у = – 7.

  1. По графику функции у = 0,5х + 3 найдите:

а) значение функции, если х = 10; х = – 12;

б) значение аргумента, если у = 4; у = – 5.

  1. Постройте график функции у = – 2х

  2. Решите графически систему уравнений:

  1. Постройте график функции у = 0,5х2 найдите:а) значение функции, если х = 10; х = – 12;

б) значение аргумента, если у = 4; у = – 5.

  1. Постройте график функции:

.

  1. Постройте график функции и запишите ее свойства (область определения, область значений, нули функции, промежутки знакопостоянства, промежутки монотонности):

  1. Функция задана формулой . Найдите (-3).

  2. Функция задана формулой . Найдите значение , при котором () =0.

  3. Вычислите координаты точек пересечения параболы и прямой:

.

  1. Найдите область значения функции .

  2. Функция задана формулой . Найдите (-5).

  1. Для каждой функции, заданной формулой, укажите график.

1) у = х – 1 2) у = – х + 1 3) у = х2 – 1

hello_html_m7e6c4af8.gifhello_html_6cdb33ee.gifhello_html_65e33a5c.gif

а) б) в)



  1. Постройте график функции: .

  2. Функция задана формулой f(х) = - 2х –12. Найдите: а) f(0); б) f(2); в) f(- 3); г) f().

  3. Известно, что g(х) = 12 – 6х. Найдите значение х, при котором:

а) g(х) = 0; б) g(х) = 6; в) g(х) = - 8.

  1. Функция задана формулой f(х) = . Найдите: а) f(0); б) f(2); в) f(- 3); г) f().

  2. Постройте график функции у = х +2. При каких значениях х выполняется неравенство ?

  3. С помощью формул описано изменение температуры воды в баке (в 0С) как функции времени t (в минутах):

2t + 20, если 0 ≤ t < 40,

p = 100, если 40 ≤ t < 60,

t + 140, если 60 ≤ t ≤ 150.

Найдите: р (20); р (40); р (50); р (60); р (90). Постройте график функции р = f (t). Какой физический смысл имеет рассматриваемый процесс в каждом из промежутков [0;40], [40;60], [60;150]?

  1. Известно, что g(х) = 2х – 6. Найдите значение х, при котором: а) g(х) = 0; б) g(х) = 6; в) g(х) = - 8.

  2. Постройте график функции и запишите ее свойства (область определения, область значений, нули функции, промежутки знакопостоянства, промежутки монотонности):

  1. Постройте график функции у = найдите:

а) значение у, если х = 10; х = – 8; б) значение х, если у = 11; у = – 7.

hello_html_5f6884f1.png


hello_html_5f6884f1.pnghello_html_5f6884f1.png

ПЗ № 28. Непрерывные и периодические функции. Свойства и графики синуса, косинуса, тангенса и котангенса.

Задание:

1)Перепишите и заполните пропуски (используя графики):

hello_html_2271e06c.gifhello_html_ma7f3704.gif


Пример 1.Определить возрастает или убывает функция: а) y = cos x при ,hello_html_m70210242.png

б) y = sin x при ,

в) у = tg x при .

Решение: а) убывает,…,б) …, возрастает, в) возрастает.

Ответ: а) …, возрастает, б) убывает, …, в) ...

Пример 2.Нацдите х, при котором функция пересекает ось ох: а) y = cos x при ,

б) y = sin x при ,в) у = tg x при .

Решение: а) ,б) ,в) . Ответ: а) ,б) ,в) .

Пример 3. Определить принимает положительные или отрицательные значения функция:

а) y = cos x при ,б) б) y = sin x при ,в) у = tg x при .

Решение: а) положительные при , отрицательные при ,

б) положительные при , отрицательные при ,

в) положительные при , отрицательные при .

Ответ: а) положительные при , отрицательные при ,

б) положительные при , отрицательные при ,

в) положительные при , отрицательные при .

Пример 4.Сравнить а) и , б) и .

Решение: а) <, (0< < < ),б) >.Ответ: : а) <,б) >.

Пример 5.Построить график функции по таблице:

а) y = 2cos x , б) y = 6sinx .

х


0




х


0




у

0

2

0

2

0

у

6

0

6

6

0



Решение:

а) б)

hello_html_mab2a1ac.jpghello_html_m5dd79f1d.jpg

2)Решить задание ( по примерам):

  1. Определить возрастает или убывает функция: а) y = cos x при ,

б) y = sin x при ,в) у = tg x при.

  1. Нацдите х, при котором функция пересекает ось ох: а) y = cos x при ,

б) y = sin x при,в) у = tg x при .

  1. Определить принимает положительные или отрицательные значения функция:

а) y = cos x при ,б) б) y = sin x при ,в) у = tg x при.

  1. Построить график функции по таблице:

а) y = 6cos x . б) y = 4sinx .

x


0




y

0

6

0

6

0

x

0





y

0

4

0

4

0


  1. Сравнить а) и , б) и .

3)Решить задание :

  1. Построить график функции y = ctg x ; запишите свойства этой функции, используя свойства функции y = tg x, и то что эти функции взаимо обратны.

  2. Сравнить числа: а) и, б) tg 2,3 и tg 3, в) и, г) tg 1 и tg 1,5.

  3. Построить график функции по таблице: y = sin 4x .


x

0





y

0

1

0

1

0


  1. Построить график функции по таблице: y = cos 4x.


x

0





y

1

0

1

0

1


  1. Построить график функции по таблице: y = tg 2x .


x

0





y

0

1


1

0


  1. Построить график функции:

а) y = sin 2x , б) y = 2sin x , в) y = cos 2x , г) y = 5sin x , д) y = 4cos x,е) y = 2сtg x .

  1. Построить график функции:

а) y = sin 4x, б) y = cos 4x, в) y = tg 2x, г) y = 2sin x, д) y = 2cos x, е) y = 2tg x.

ПЗ № 29. Обратные функции и их графики. Обратные тригонометрические функции.

Задание:

1)А)Опорный конспект.

Определение обратной функции.

Пусть функция hello_html_m94574d3.gif строго монотонная (возрастающая или убывающая) и непрерывная на области определения hello_html_m5c67524f.gif, область значений этой функции hello_html_m1ceea025.gif, тогда на интервале hello_html_m4d9ff88d.gif определена непрерывная строго монотонная функция hello_html_6b07b91.gif с областью значений hello_html_m2f63b715.gif, которая является обратной дляhello_html_m94574d3.gif.

Другими словами, об обратной функции hello_html_6b07b91.gif для функции hello_html_m94574d3.gif на конкретном промежутке имеет смысл говорить, если на этом интервале hello_html_m94574d3.gif либо возрастает, либо убывает.

Функции f и g называют взаимно обратными.

Пример1. Найти функцию обратную для hello_html_2dd2d5b0.gif.

Решение.

Областью определения и областью значений этой функции является все множество действительных чисел. Выразим x через y (другими словами, решим уравнение hello_html_2dd2d5b0.gif относительно x ).

hello_html_m6133d6d7.gif - это и есть обратная функция, правда здесь y – аргумент, а x – функция этого аргумента. Чтобы не нарушать привычки в обозначениях (это не имеет принципиального значения), переставив буквы x и y , будем писать hello_html_10fbd097.gif.hello_html_4ce77a33.gif

Таким образом, hello_html_2dd2d5b0.gif и hello_html_10fbd097.gif - взаимно обратные функции.

Приведем графическую иллюстрацию взаимно обратных линейных функций.

Очевидно, что графики симметричны относительно прямой y=x (биссектрисы первого и третьего квадрантов).

Пример2. Найти функцию обратную для hello_html_38039a0f.gif.

Решение.

Областью определения этой функции является все множество действительных чисел, областью значений является интервал hello_html_4027ae8.gif. Выразим x через y (другими словами, решим уравнение hello_html_38039a0f.gif относительно x). hello_html_m3d98c9db.gif - это и есть обратная функция. Переставив буквы x и y , имеем hello_html_5b975f78.gif.hello_html_m61499e94.png

Таким образом, hello_html_38039a0f.gif и hello_html_5b975f78.gif - показательная и логарифмическая функции есть взаимно обратные функции на области определения.

График взаимно обратных показательной и логарифмической функций.
Перечислим свойства взаимно обратных функций hello_html_m94574d3.gif и hello_html_6b07b91.gif.

hello_html_m1bccfa33.gif и hello_html_m57c27ae5.gif.

Из первого свойства видно, что область определения функции hello_html_m94574d3.gifсовпадает с областью значений функции hello_html_6b07b91.gif и наоборот.

Графики взаимно обратных функций симметричны относительно прямой y=x.

Если hello_html_m94574d3.gif возрастает, то и hello_html_6b07b91.gif возрастает, если hello_html_m94574d3.gif убывает, то и hello_html_6b07b91.gif убывает.

Примеры нахождения взаимнообратных функций.

1)Для степенной функции hello_html_m3aecfc2f.gif при hello_html_m3f0c2e4.gif обратной является также степенная функция hello_html_2c09c577.gif Если заменить буквы, то получим пару взаимно обратных функций hello_html_m3aecfc2f.gif и hello_html_2c09c577.gif

Графики для положительных а и отрицательных а.
hello_html_2cbbb011.gif

2) Взаимно обратные показательная и логарифмическая функции hello_html_m4ab91685.gif и hello_html_2adc860c.gif, графики.

Подразумеваем, что а положительное и не равное единице число.

Графики для hello_html_m10ba216.gif и для hello_html_6b08bef4.gif
hello_html_3ffd9e74.gif

3) Взаимно обратные тригонометрические и обратные тригонометрические функции.

а)График главной ветви синуса и арксинуса (светлая область).
hello_html_m1494567d.gif hello_html_47303997.pngy = arcsin x

б)График главной ветви косинуса и арккосинуса (светлая область).
hello_html_m21877848.gif hello_html_m5cdd44d0.png y = arccos x


в) График главной ветви тангенса и арктангенса (светлая область).
hello_html_m4f9321da.gif hello_html_m50d8df33.png y = arctg x


г) График главной ветви котангенса и арккотангенса (светлая область).
hello_html_6b553bd4.gif hello_html_67f26bab.png y=arcctgx

Если Вам потребуются обратные функции для ветвей тригонометрических функций, отличных от главных, то соответствующую обратную тригонометрическую функцию нужно будет сдвинуть вдоль оси ординат на необходимое количество периодов.hello_html_m27c17baf.gif

Например, если Вам потребуется обратная функция для ветви тангенса на промежутке hello_html_m9b7177e.gif (эта ветвь получается из главной ветви сдвигом на величину hello_html_7d78f2fc.gifвдоль оси ох ), то ей будет являться ветвь арктангенса, сдвинутая вдоль оси oy на hello_html_7d78f2fc.gif.

Б) Построить таблицы:

Табличные значения обратных тригонометрических функций.

hello_html_4c5b70ac.jpg

hello_html_m6baf672f.jpg

В) Преобразование выражений. (Перепишите и заполните пропуски)





4) Вычислить без калькулятора


2)Решить задание ( по примерам):



hello_html_m46bcf841.gif

3)Решить задание :

В)1. Найдите значение выражения: а) arcsin1; б) arccos; в) arctg(); г)arcctg0.

2. Найдите значение выражения: а) arcsin; б) arccos0; в) arctg; г) arcctg.

3. Построить графики функций:

а) y =2 arcsin x ,б) y = 3arccos x,в) y = 2arctg x



ПЗ № 30. Преобразования графика функции. Гармонические колебания. Прикладные задачи.

Задание:

1)Опорный конспект.

1.Преобразования графиков функций — это линейные преобразования функции y = f(x) или её аргумента x к виду y = af(kx + b) + m, а также преобразование с использованием модуля. Зная, как строить графики функции y = f(x), где y = kx + b, y = ax2, y = xn , y=k/x, 

y = sin x, y = cosx, y = tgx, y = ctgx, y=ax,y=logax можно построить график функции y = af(kx + b) + m.


Общий вид функции

Преобразования

y = f(x - b)

Параллельный перенос графика вдоль оси абсцисс на | b | единиц

  • вправо, если b > 0;

  • влево, если b < 0.

y = f(x + b)

  • влево, если b > 0;

  • вправо, если b < 0.

y = f(x) + m

Параллельный перенос графика вдоль оси ординат на | m | единиц

  • вверх, если m > 0,

  • вниз, если m < 0.


Отражение графика

y = f( - x)

Симметричное отражение графика относительно оси ординат.

y = - f(x)

Симметричное отражение графика относительно оси абсцисс.


Сжатие и растяжение графика

y = f(kx)

  • При k > 1 — сжатие графика к оси ординат в k раз,

  • при 0 < k < 1 — растяжение графика от оси ординат в k раз.

y = kf(x)

  • При k > 1 — растяжение графика от оси абсцисс в k раз,

  • при 0 < k < 1 — cжатие графика к оси абсцисс в k раз.


Преобразования графика с модулем

y = | f(x) |

  • При f(x) > 0 — график остаётся без изменений,

  • при f(x) < 0 — график симметрично отражается относительно оси абсцисс.

y = f( | x | )

  • При x≥0— график остаётся без изменений,

  • при x < 0 — график симметрично отражается относительно оси ординат.


hello_html_m1e9c802a.gif

Примеры построения функций.



hello_html_mf8eb205.pnghello_html_4df25cf6.png

hello_html_500049aa.pnghello_html_1aa59faf.png

hello_html_m623ad66f.pnghello_html_m6eb3bff8.png

hello_html_3a70f67b.pnghello_html_m1ba2c529.png

2.Тригонометрические функции используются для описания различных колебательных процессов: колебания груза, подвешенного на пружине, вокруг положения равновесие, закон изменения переменного тока в цепи, колебания маятника, распространение звуковых и цветовых волн и т.д.

Формулы  и  , с помощью которых описываются такие процессы, называются формулами гармонических колебаний. Положительная величина А называется амплитудойколебания, положительная величина w  частотой колебания, величина  начальной фазой колебания. Амплитуда характеризует размах колебания, частота – количество колебаний в единицу времени.

Построение графиков гармонических колебаний (гармоник) , производится в несколько этапов.

Рассмотрим алгоритм построения графика функции  : а) строим график функции  ; б) строим график функции  , сдвигая график функции  на |hello_html_1b4e4442.gif| единиц по осиОХ (если   , то сдвигаем влево, если  , то сдвигаем вправо); в) строим график функции  , сжимая его в w раз к оси OY;

г) строим график функции  , растягивая его в A раз от осиОХ.

Заметим, что функции  и  , описывающие гармонические колебания, являются периодическими с периодом   . Они ограничены сверху и снизу, их наибольшее и наименьшее значения равны  .

Пример 1.Постройте график гармонического колебания  .

Решение. Для этой гармоники амплитуда  , частота –  , начальная фаза –  .

Строим график функции  ; сдвигаем на  единиц по оси ОХ вправо; сжимаем график к оси OY в 2 раза; растягиваем от оси OX в 3 раза (рис. 38).


hello_html_m1f203bb.jpg

Пример 2.Постройте график гармонического колебания  .

Решение. Преобразуем формулу, раскрыв в аргументе косинуса скобки:  . Следовательно, для этой гармоники амплитуда  , частота –  , начальная фаза –  .

Строим график функции  ; сдвигаем график на  единиц по оси ОХ вправо; сжимаем график к оси OY в 2 раза; растягиваем от оси OX в 3 раза (рис. 39).


hello_html_m500cf9e4.jpg

Пример 3.Постройте график гармонического колебания  .

hello_html_41baac58.jpg 

Решение. Эта формула не задает гармоническое колебание, так как  . Применив формулу приведения , преобразуем формулу к виду:  . Следовательно, для этой гармоники амплитуда  , частота –  , начальная фаза –  .

Строим график функции  ; сдвигаем на  единиц по оси ОХ влево; сжимаем график к оси OY в 2 раза; растягиваем от оси OX в 3 раза (рис. 40).

3.

hello_html_m26d06696.gif

hello_html_126ef828.gif



2)Решить задание:

1.

hello_html_60058b54.png

2.

hello_html_m5af30e0a.png

3.

hello_html_m18f76492.png

4. hello_html_m52a0ab8f.png

5. hello_html_m52a0ab8f.png

6. hello_html_m52a0ab8f.png

7.

hello_html_c3deb3c.png

8. Построить графики функций:

hello_html_m25e403ff.png

9. Построить графики функций:1) у = 1/2 sin (3x) – 2, 2) y = 2 3x+1 – 4,

3) y = 2 (x – 1)2 – 3, 4) y = –3 log2(x + 1), .

10.

hello_html_m749d1356.png

ПЗ № 31. Показательные, логарифмические, тригонометрические уравнения и неравенства.

Задание:

1)Опорный конспект.

Графиком называется множество точек координатной плоскости, у которых значения x и y связаны некоторой зависимостью и каждому значению x соответствует единственное значение y.

Графический способ - один из самых удобных и наглядных способов представления и анализа информации.

На практике довольно часто оказывается полезным графический метод решения уравнений. Он заключается в следующем: для решения уравнений f(x)=0 строят график функции y=f(x) и находят абсциссы точек пересечения графика с осью Оx: эти абсциссы и являются корнями уравнения. Алгоритм решения уравнений графическим способом

Чтобы решить графически уравнение вида f(х) = g(х), нужно:

1.Построить в одной координатной плоскости графики функции:

у = f(х) и у = g(х).

2. Найти точки пересечения этих графиков.

3. Указать абсциссу каждой из этих пересечения.

4. Записать ответ.

Довольно просто решать графически систему уравнений, так как каждое уравнение системы на координатной плоскости представляет какую- то линию.

Построив графики этих уравнений и найдя координаты точек их пересечения (если они существуют), мы получим искомое решение.

Графическое решение неравенств, сводится к отысканию таких точек x, при которых один график лежит выше или ниже другого.

Примеры:

1. Решите уравнение

2. Решите уравнение

3. Решить уравнение hello_html_m49d202a5.gifhello_html_663ba92f.png

Решение: Построим графики функций hello_html_598b2242.gif и y = x


Графики функций не пересекаются, и, значит, уравнение не имеет корней (см. рисунок).

Ответ: корней нет.

4.Найти значение выражения хhello_html_7b9d0882.gif+ уhello_html_7b9d0882.gif,если (х;у) является решением системы уравнений. hello_html_m7beea7c6.gifhello_html_m4aa16645.jpg

Решение:

hello_html_6938f234.gifhello_html_3fee811a.gif-параллельный перенос на 1 единицу влево.

hello_html_3133347a.gif - параллельный перенос на 2 единицы влево.

х= - 1, у=1

х+ у=0.


Ответ: 0.

5. Решите неравенство hello_html_m6d811b21.gif>12 - 1,5х. №6. Решите неравенство hello_html_m342748d5.gif. Oтвет: х>0.

Ответ: х>2. hello_html_m34a0a2e3.jpg

hello_html_m62d29737.jpg

















7. Решить уравнение  sinx + cosx=1. Построим графики функций y=sinx u y=1-cosx.(рисунок 5) Из графика видно, что уравнение имеет 2 решения: х=2πп,где пЄZ и х=π/2+2πk,где kЄZ.


hello_html_6222e497.jpg

8.Решить уравнение: 3x = (х-1) 2 + 3

Решение: применяем функциональный метод решения уравнений:

hello_html_78cdd215.gif

т.к. данная система имеет единственное решение, то методом подбора находим х=1hello_html_1db23044.jpg

Ответ: 1.


9.Решить неравенство: сos x hello_html_m62c2f6f2.gif1 + 3x

Решение:



hello_html_58548c10.gif

Ответ: (hello_html_m293ab4b8.gif ; hello_html_7ea8f9fd.gif).hello_html_m5828edc.png

10. Решить уравнение hello_html_m6e244885.png

В нашем случае функция hello_html_m81b4bc5.pngвозрастает при х>0, а функция y = 3 – x убывает при всех значениях х, в том числе и при х>0, значит, уравнение hello_html_m3f92800e.pngимеет не более одного корня. Заметим, что при х = 2 уравнение обращается в верное равенство, так как hello_html_m47d6d725.png.

Ответ: 2 .

2)Решить задание:

1)Есть ли корень у уравнения и если есть, то положительный он или отрицательный?

а) hello_html_2e3d017c.gif; б) hello_html_m152ef3cd.gif, в) 6х =1/6, г) hello_html_71df238a.gif.hello_html_m1b1b0d93.pnghello_html_m2db28fe1.pnghello_html_66f33c29.png

hello_html_5dceb135.png

2) Решить графическим методом уравнение  .

3) Решите графическим методом уравнения:

а) б).



4)На рисунке изображен график функции y=f(x). Найдите количество целых корней уравнения f(x)= 0. hello_html_m209c52f.gif

1) 1 2) 6 3) 7 4) 8

5) На каком из рисунков изображен график функции ?

1) у 2) у 3) у 4) у



1 1 1



6) График какой функции изображен на рисунке?

1) у = 2х-1,5; 2) у = 2х – 2;

3) у = 2х – 3; 4) у = 2 – 2.

7)График какой функции изображен на рисунке?

hello_html_m352f9896.gif





1) у = sinx; 2) ; 3) ; 4) .

8) На рисунке изображен график функций y

y = f (x) и y = g (x), заданный на промежутке

[-5;6]. Укажите те значения х, для которых

выполняется неравенство g (x) f (x) 1


1) [-5; 0] 2) [-5; 2]

0 1 x

3) [-2; 2] 4) [2; 6]


9) На рисунке изображен график функции y=f(x). Найдите количество целых корней уравнения f(x)= 0.hello_html_3b033243.gif

1) 3 2) 4 3) 2 4) 1



10) На рисунке изображен график функции y=f(x). Найдите количество целых корней уравнения f(x)+2= 0.

hello_html_5555b426.gif

1) 3 2) 5 3) 4 4) 1

11) Решите графическим методом уравнения:

а) , б), в) cos x≤ 1+ 4x, г) 5x = (х - 1) 2 + 5.

12) Решить графическим методом уравнение  .


ПЗ № 32. Решение иррациональных уравнений.

Задание:

1) Перепишите и заполните пропуски:
Пример 1. Решить уравнениеhello_html_3be855b.gif Решение: Уединим радикал hello_html_m5a2fbf9f.gif Это уравнение равносильно системе hello_html_4e1d1e98.gif Решим уравнение (1): hello_html_6861a4c5.gif hello_html_m66272cef.gif hello_html_m211ee85b.gif hello_html_3fa90232.gif х = … Найденное значение hello_html_m727f5169.gif удовлетворяет условиям (2) и (3).

Ответ: –1. Пример 2. а) Найдите корень уравнения = 3 . Решение:

Возведем в квадрат правую и левую части уравнения: )2 = 32, 15 – 2х = 9, –2х = 9 – 15,

2х = – 6, х = ... Сделаем проверку. Для этого подставим число 3 в исходное уравнение: = 3, 3 = 3 – верно.

Ответ: 3.

б) Решить уравнение = .

Решение: = => ˂=> => => х = ...

Ответ: 1. Пример 3. Решить уравнение = х -7 .

Решение: = х -7 => => => => => х = ...

Ответ: 14.

Пример 4. Решите уравнение   = .

Решение:

  = => 7 х + х 2 2 = 2х 5 =>

5 – х = => 25 – 10х + х2 = х2 + 9х – 14 => 2 19х + 39 = 0,

D = (– 19)2 42 39= 361 – 312 = …, х1= (19 + 7) : 4 = …, х2 = (19 – 7) : 4 = …,

Проверка:  

а)  х1= 6,5,   = ,

  = –  неверное равенство.

б) х2 = 3,   – = ,   = , –  верное равенство.

Ответ: 3.

Пример 5. Решить уравнение hello_html_11c97566.gif

Решение:

Возводим в куб обе части уравнения hello_html_m5925937d.gif получим hello_html_36f8101a.gif

Учитывая, что выражение в скобках равно 1 (см. условие), получаем hello_html_m4386db45.gif hello_html_m523bd08c.gif hello_html_73392c.gif

Возводим в куб: hello_html_m30f89b75.gif hello_html_m522b6165.gif hello_html_m3348bac9.gif

Проверкой убеждаемся, что hello_html_m294b9fba.gif и hello_html_3a69ee1b.gif корни уравнения.

Ответ: 80, – 109.


2)Решить задание ( по примерам):

Решить уравнения:

  1. а) . б) .

  2. .

3) Решить задание:

  1. Решить уравнение:.

  2. Решить уравнение:.

  3. Решить уравнение:.

  4. Решить уравнение:.

  5. Решить уравнение: .

  6. Решить уравнение:3.

  7. Решить уравнение: .

  8. Решить уравнение:.

  9. Решить уравнение:.

  10. Решить уравнение:.

  11. Найдите корень уравнения:

  12. Найдите корень уравнения:

  13. Найдите корень уравнения:

Если уравнение имеет более одного корня, укажите меньший из них.

  1. Найдите корень уравнения:

  2. Найдите корень уравнения:

  3. Решить уравнение:

  4. Решить уравнение:

  5. Решить уравнение:

  6. Решить уравнение:

  7. Решить уравнение:

  8. Решить уравнение:

  9. Решить уравнение:

  10. Решить уравнение:

  11. Решить уравнение:

  12. Решить уравнение:


ПЗ № 33. Решение показательных уравнений.

Задание:

1) Перепишите и заполните пропуски:
Пример 1. а)Найдите корень уравнения .

Решение: Чтобы решить это уравнение, вспомним свойства степени и приведем правую и левую части уравнения к степени с основанием 5: ,

Если степени с равными основаниями равны, то равны их показатели. Приравняем показатели степеней: х – 7 = - 3, х = 7 – 3, х = ...

Ответ: 4 .

б)Найдите корень уравнения .

Решение: Представим правую и левую части уравнения в виде степени с основанием ,

Приравняем показатели степеней: – 3 (– 3+ х) = 9, 9 – 3х = …, – 3х = 0, х = ...

Ответ: 0.

Пример 2. Решите уравнение.

Разделим обе части уравнения на : .

Пустьm,m > 0 , тогда 2m2 – 3m – 5 = 0, D = 9 – 42(– 5) = 9 + 40 = …, m1 = (3 + 7) : 4 = …,

m2 = (3 – 7) : 4 = – 4 : 4 = …, – не удовл. условию m > 0 .
Если m = 2,5 , то
Ответ:  – 1.

Пример 3. Решите уравнение 49x 8∙7x + 7 = 0.

Решение: Обозначим получим уравнение относительно у: у2 – 8у + 7 = 0,

D = (– 8)2 41 7= 64 – 28 = …, у1= (8 + 6) : 2 = …, у2 = (8 – 6) : 2 = ... Получим, что и , отсюда х1 = …, х2 = ... Ответ: х1 = 1, х2 = 0.

Пример 4. а)Решить уравнение .
Решение:

Ответ: 3.

б) Решите уравнение 

Решение:

Ответ: 1.

в)Решите уравнение

Решение:

Ответ: 4.

Пример 5. Решите уравнение а) 2х+1 + 2х-1 + 2х = 28, б) 9х – 8∙3х – 9 = 0, в) 8∙4х – 6∙2х + 1 = 0.

Решение:

а) 2х+1 + 2х-1 + 2х = 28, 2х-1 ∙ (22 + 1 + 2) = 28, 2х-1∙7 = 28, 2х-1 = 4, 2х-1 = 22, х – 1 = 2, х = ...

Ответ: 3.

б) 9х – 8∙3х – 9 = 0, (3х)2 – 8∙3х -9 = 0, Обозначим 3х = t, где t >0, тогда t2 – 8t – 9 = 0,

D = (–8)2 41 (–9) = 64 + 36 = …, t 1= (8 + 10) : 2 = …, t 2 = (8 – 10) : 2 = ... . t1 = 9, t2 = – 1, Возвращаемся к замене: 3х = 9, х = …, 3х = – 1, корней нет.

Ответ: 2.

в) 8∙4х – 6∙2х + 1 = 0, 8∙(2х)2 – 6∙2х + 1 = 0, Обозначим 2х = t, где t >0, тогда 8 t2 – 6t + 1 = 0, D = (–6)2 41 8= 36 – 32 = …, t1= (6 + 2) : 16 = …, t2 = (6 – 2) : 16 = ... t1 =, t2 = Возвращаемся к замене: 2х = , х 1= …, 2х = , х 2= ...

Ответ: – 1, – 2.

2)Решить задание ( по примерам):

  1. а) Найдите корень уравнения .б) Найдите корень уравнения: .

  2. Решите уравнение .

  3. Решите уравнение 25x 6∙5x + 5 = 0.

  4. а)Решить уравнение: б)Решите уравнение 

в)Решите уравнение.

  1. Решите уравнение а) 3х+1 + 3х-1 + 3х = 117, б) 16х – 15∙4х – 16 = 0, в) 81х + 6∙9х + 9 = 0.

3)Решить задание :

  1. Решить уравнения: а) , б) в)

  2. Решите уравнение а) 4x 5∙2x + 4 = 0, б) 9x 4∙3x + 3 = 0.

  3. Решите уравнение  

  4. Найдите сумму корней уравнения :.

  5. Если - корень уравнения , то найдите значение выражения .

  6. Найдите произведение корней уравнения .

  7. Решите уравнение .

  8. Решите уравнение .

  9. Решите уравнение

  10. Решите уравнение 4х + 2х – 6 = 0;

  11. Решите уравнение 9х + 3х+1 = 4;

  12. Решите уравнение

  13. Решите уравнение .

  14. Решите уравнение: .

  15. Решите уравнение: 92х+1 – 9 = 72.

  16. Решите уравнение:

  17. Пусть х0 ─ наибольший корень уравнения . Найти 2х0 – 5.

  18. Решите уравнение: 23х+2 + 8х = 0,625.

  19. Пусть х0 ─ наименьший корень уравнения . Найти 3х0 + 2.

  20. Найти сумму корней уравнения: 4х – 40∙2х + 256 = 0.

  21. Решите уравнение:

  22. Решите уравнение :3∙ + 325 ∙ = 0.

  23. Решите уравнения:


  1. Решите уравнения:

  2. Решите уравнение: .

  3. Решите уравнения:


  1. Решите уравнение: .

  2. Решите уравнения:

  3. Найти корень (или сумму корней, если их несколько) уравнения: 7 · 8х+1 + 8х+3 = 71.

  4. Найти корень (или сумму корней, если их несколько) уравнения: 7 = 6 · 7х + 7.

  5. Решите уравнение: .

  6. Решите уравнение: .

ПЗ № 34. Решение логарифмических уравнений.

Задание:

1) Перепишите и заполните пропуски:
Пример 1. Решите уравнение Решение: Используем метод - решение логарифмических уравнений заменой.

ОДЗ: х > 0. Введем замену , чтобы записать исходное уравнение в виде стандартного квадратного уравнения. Тогда уравнение примет вид: у2 – 4у + 4 = 0, ( у – 2)2 = 0, у – 2 = 0, у = ... Вернемся к  х : . Тогда по определению логарифма получаем, что х = 32, х = … - уд.ОДЗ.

Ответ: 9.

Пример 2. Решите уравнение:.  

Решение: Вновь начнем решение с определения области допустимых значений уравнения. Она определяется следующей системой неравенств:


Воспользовавшись правилом сложения логарифмов, переходим к равносильному в области допустимых значений уравнению:

Основания логарифмов одинаковы, поэтому в области допустимых значений можно перейти к следующему квадратному уравнению:

(х + 2) (х + 3) = 1 х , х2 + 6х + 5 = 0, D = (6)2 41 5= 36 – 20 = …,

х1= ( 6 4) : 2 = , х2 = ( 4) : 2 = ...  

Первый корень не входит в область допустимых значений уравнения, второй — входит.

Ответ: x = -1.

Пример 3. Решите уравнение:

Решение: Найдем ОДЗ по определению логарифма. ОДЗ:

.

Перепишем исходное уравнение, используя свойства суммы логарифмов и логарифма степени. Получим следующее уравнение:

Приравняем подлогарифмические выражения:

(3х ) (х) = ,

Найдем корни полученного квадратного уравнения:

D = (92)2 41 () = 8464 + 8436 = …,

х1= (92 + 130) : 6 = 222 : 6 = …, х2 = (92 130) : 6 = .

Учитывая ОДЗ, корнем исходного логарифмического уравнения будет только х = ...

Ответ: х = 37.

Пример 4. Решите уравнение:

Решение: В область допустимых значений входят только те x, при которых выражение, находящееся под знаком логарифма, больше нуля. Эти значения определяются следующей системой неравенств:


С учетом того, что получаем промежуток, определяющий область допустимых значений данного логарифмического уравнения:.

На основании теоремы 1, все условия которой здесь выполнены, переходим к следующему равносильному квадратичному уравнению:

  D = (5)2 41 () = 25 + 56 = …, х1= (5 + 9) : 2 = …, х2 = (5 9) : 2 =

В область допустимых значений входит только первый корень. Ответ: x = 7.

Пример 5. Решите уравнение:

Решение: Используем метод - решение логарифмических уравнений, переходя к одному основанию. ОДЗ: 

К логарифму по основанию x (второе слагаемое) вначале применим свойство логарифма степени, а затем по формуле замены основания логарифма приведем его к основанию 2:



Так как  то


Введем замену  тогда уравнение примет вид: у2 – 5у + 4 = 0.

Найдем корни полученного квадратного уравнения:

D = (5)2 41 = 25 = …, y1= (5 + 3) : 2 = …, y2 = (5 3) : 2 = ...

Вернемся к x, используя определения логарифма:

x = x = …, x = , x = …, Оба значения принадлежат ОДЗ.

Ответ: 16 и 2.

Пример 6. Решите уравнение:

Решение: Область допустимых значений уравнения определяется здесь легко: x > 0.

Используем подстановку: Уравнение принимает вид: 3у2 + 5у = 0,

D = (5)2 43 () = 25 + 24 = …, у1= (5 + 7) : 6 = 1/3, у2 = (5 7) : 6 =

Вернемся к x, используя определения логарифма:

x = , x = , x =... Оба ответа входят в область допустимых значений уравнения, поскольку являются положительными числами.

Ответ: и 4.

2)Решить задание ( по примерам):

  1. Решите уравнение

  2. Решите уравнение .

  3. Решите уравнение

  4. Решите уравнение:

  5. Решите уравнение:

  6. Решите уравнение:

3)Решить задание :

  1. Решите уравнение .

  2. Решите уравнение .

  3. Решите уравнение:

  4. Решите уравнение:

  5. Решите уравнение:

  6. Решите уравнение:

  7. Если - корень уравнения , то найдите значение выражения .

  8. Найдите произведение корней уравнения .

  9. Найдите сумму корней уравнения .

  10. Найдите больший корень уравнения .

  11. Решите уравнение:

  12. Решите уравнение:

  13. Решите уравнение:

  14. Решите уравнение:

  15. Найдите сумму корней уравнения .

  16. Найдите сумму корней уравнения .

  17. Найдите произведение корней уравнения .

  18. Если - корень уравнения , то найдите значение выражения .

  19. Решите уравнение: а) ,б).

  20. Решите уравнение: .

  21. Решите уравнение: .

  22. Решите уравнение:

ПЗ № 35. Корни уравнений. Равносильность уравнений. Преобразование уравнений. Основные приемы решения уравнений.

1)Опорный конспект.

Уравнением с одной переменной x называется выражение f(x) = g(x), содержащее переменную величину x и знак равенства.

Число a называется корнем (или решением) уравнения f(x) = g(x), если при подстановке этого числа в уравнение получается верное числовое равенство.

Решение уравнения – это процесс, состоящий в основном в замене заданного уравнения другим уравнением, ему равносильным. Такая замена называется тождественным преобразованием.

Основные тождественные преобразования:

Замена одного выражения другим, тождественно равным ему. Например,уравнение 

( 3x+ 2 ) 2 = 15x+10 можно заменить следующим равносильным: 9x2 + 12x +4 = 15x + 10

Перенос членов уравнения из одной стороны в другую с обратными знаками. Так, в предыдущем уравнении мы можем перенести все его члены из правой части в левую со знаком « – »: 

9x2 + 12x + – 15x – 10 = 0, после чего получим:9x2 – 3x – 6 = 0 .

Умножение или деление обеих частей уравнения на одно и то же выражение (число), отличное от нуляЭто очень важно, так как новое уравнение может не быть равносильным предыдущему, если выражение, на которое мы умножаем или делим, может быть равно нулю. Уравнение x – 1 = 0 имеет единственный корень x = 1. Умножив обе его части на x – 3 , мы получим уравнение

( x – 1 )( x – 3 ) = 0, у которого два корня: x = 1 и = 3. Последнее значение не является корнем заданного уравнения x – 1 = 0. Это так называемый посторонний корень. И наоборот, деление может привести кпотере корня. Так, если ( x – 1 )( x – 3 ) = 0 является исходным уравнением, то корень x = 3 будет потерян при делении обеих частей уравнения на x – 3 .

Можно возвести обе части уравнения в нечетную степень или извлечьиз обеих частей уравнения корень нечетной степени. Необходимо помнить, что: а) возведение в четную степень может привести к приобретению посторонних корней; б) неправильное извлечение корня четной степени может привести к потере корней.

Уравнение 7= 35 имеет единственный корень x = 5 . Возведя обе части этого уравнения в квадрат, получим уравнение: 49x2 = 1225 , имеющее два корня: = 5 и = – 5. Последнее значение является посторонним корнем.

Неправильное извлечение квадратного корня из обеих частей уравнения 49x 2 = 1225 даёт в результате 7= 35,и мы теряем корень = – 5.Правильное извлечение квадратного корня приводит к
уравнению: | 7
| = 35, а следовательно, к двум случаям: 1) 7= 35, тогда x = 5 ;

 2) – 7= 35, тогда x = – 5. Следовательно, при правильном извлечении квадратного корня мы не теряем корней уравнения.

ОДЗ (областью допустимых значений) уравнения называется множество тех значений неизвестной, при которых определены его правая и левая части.

Очевидно, что вне ОДЗ решений не существует, однако не все числа, входящие в ОДЗ, служат решениями уравнения. Уравнение можно решить и не находя ОДЗ. С другой стороны, верно найденное ОДЗ и последующий отбор корней с его помощью не может гарантировать отсутствие ошибок.

Из этих правил следует, что с помощью стандартных приёмов и методов решения уравнений, а именно:

  • преобразования (раскрытие скобок, освобождение от знаменателя, приведение подобных членов, возведение уравнения в нечетную натуральную степень и т. д.),

  • разложения на множители (формально этот приём относится к преобразованиям, но, так как он довольно часто встречается самостоятельно, мы его выделяем особо),

  • введения вспомогательных неизвестных,

уравнение (1) может быть сведено к более простому и, самое главное, равносильному уравнению f1 (x) = g1 (x).

Решение уравнений

1) Если показательное уравнение сводится к виду ax = ab (1) где a> 0 и a ≠1, то оно имеет единственный корень х = b.

2) Иногда, чтобы привести показательное уравнение к виду (1), необходимо в левой части уравнения вынести за скобки общий множитель а х, например:

и т. д.

Или разделить обе части уравнения на выражение, не равное нулю, к примеру:

и т. д..

  1. некоторые показательные уравнения заменой а х = t сводятся к квадратным.

Надо помнить, что t > 0, так как показательная функция не может принимать отрицательные значения.

Чаще всего при решении логарифмического уравнения его приводят к виду

loga (f(x)) = log a (g(x)), тогда f(x) = g(x).

Решив полученное уравнение, следует сделать проверку корней, чтобы исходное уравнение не потеряло смысл.

Примеры: Решить следующие уравнения :

1) 9 х – 73 х = - 12;2) , 3) 4) 81 х – 29 х = 8 3х;

5) log5 (x2 - 10) = log 5 9x; 6) log 7 (x2 + 6x)=1;

7) log 0,6 (x + 3) + log 0,6 (x - 3) = log 0,6 ( 2x - 1) ; 8 ) 25x5x = 625.

Решение:

1) 9 х – 73 х = - 12; 3 х = tt2 - 7t + 12=0t1 = 3,t 2= 43x = 3;3x = 4

x1 = 1; x 2= log 3 4

Ответ. х1 = 1;. x 2= log 3 4.

2) .

Ответ: x1=5; x2= - 1

3) 4x=t

hello_html_1b781d56.png

Ответ. x = log 44,4.

4) 27 х – 29 х = 8 3х; 3x (9x - 23x - 8)=0, ,3x = t > 0t2- 2 t - 8=0t=4 u t= - 2 тогда 3x = 4x = log 34

Ответ. x = log 34.

5) log5 (x2 - 10) = log 5 9x; x2 - 9x - 10=0 x1=10 u x2=-1 x > 0

Ответ. x = 10

6) log 7 (x2 + 6x)=1; (x2 + 6x)=7x2 + 6x - 7=0x1 = - 7 u x2 = 1

Ответ. x1= - 7; x 2= 1.

  1. log 0,6 (x + 3) + log 0,6 (x - 3) = log 0,6 ( 2x - 1) log 0,6 (x + 3) (x - 3) = log 0,6 ( 2x - 1)

х2 – 9 = 2х - 1 х2 – 9 - 2х + 1=0 х 2 – 2х - 8 =0х1 = 4 и х2 = - 2

ОДЗ: х+3 > 0, x – 3 > 0, 2x -1 > 0x > - 3, x > 3,x > 1/2 x > 3 х1 = 4 и х2 = - 2.

Ответ. х1 = 4.

8 ) 25x5x = 625 , 53x = 62553x =543x=4x=4/3. Ответ. х= 4/3.

9)Решить уравнение log2 (x – 5) + log2 (x +2) = 3

Решение: Используем свойство логарифмов. Представим число (3) как логарифм

по основанию 2:

log2( x-5)(x + 2 ) = log2 8 (x-5)(x+2) = 8 x2 – 3x – 10 = 8 .

x2 – 3x - 18 = 0; x1 =- 3; x2 = 6.

Выполнив проверку, убеждаемся, что при x = - 3 log2 (x – 5) и

log2 (x+2) не имеют смысла

Ответ. х = 6.

10) Решить уравнение: .

Решение:

Используя определение логарифма, и учитывая область определения, получим


Ответ: 21.

11) Решить уравнение: .

Решение:


Ответ: 1.

12) Решить уравнение: .

Решение:

Решая полученное квадратное уравнение заменой , находим


Ответ: 2, 8.

2)А)Решить задание :

1.Решить следующие уравнения

1) 4 х+3 + 4 х =260; 2) ; 3) 4) 36 х – 218 х = 8 9х;

5) log3 (x2 + 6) = log3 5x; 6) log12 (x2 – x)=1; 7) log20,3 (x+1) – 4 log 0,3 (x+1) + 3 =0; 8) 9x3x = 81.

2. Решите уравнение: а) hello_html_m210f189a.gif; б) hello_html_3e7e6ab7.gif; в) hello_html_m1fc53f6a.gif.

3. Решите уравнение: а) hello_html_m46c4ac43.gif; б) hello_html_m62035bd7.gif; в) hello_html_m2653f2da.gif.

4. Решить уравнение: .

5. Решить уравнение: .

6. Решить уравнение: .

7. Решить уравнение: .

8. Решить уравнение: .

9. Решить уравнение: .

10. Решить уравнение: .

В) ТЕСТ.

1.Найдите сумму корней уравнения х3 +2х2 -9х –18 = 0.

1) -2 2) -8 3) 2 4) 8

2. Найдите сумму корней уравнения .

1) 1,5 2) 8 3) 8,5 4) 6,5

3. Решите уравнение .

1) 4 2) 12 3) 2 4) 8

4. Найдите сумму корней уравнения .

1) 1 2) 3 3) 5 4) 6

5. Решите уравнение .


6. Сколько корней имеет уравнение: х4+9х2+4=0.

1) 2 2) ни одного 3) 4 4) 1

7. Решите уравнение .

1) 1 2) 2 3) 3 4) 8

8. Найдите сумму корней уравнения .

1) 2) 3) 15 4)

9. Решите уравнение .

1) 100 2) 1 3) 0,1 4) 10

10. Решите уравнение .


11. Сколько корней имеет уравнение

1) 4 2) 2 3) 1 4) ни одного

12. Укажите промежуток, которому принадлежит корень уравнения

1) (-;0) 2) (0; 5) 3) (5; 50) 4) (50;100).

13. Решите уравнение .

1) 2) 3) 3 4)

14. Найдите сумму корней уравнения .

1) -1,25 2) -3,25 3) -1 4) 1

С) ТЕСТ.

1. Решите уравнение: 2х = 32.

1) 5; 2) 16; 3) 2; 4) 0.

2. Решите уравнение: 3х – 1 = 27.

1) 2; 2) 4; 3) 10; 4) 8.

3. Решите уравнение: log 3 х = 3.

1) 3; 2) 1; 3) 27; 4) –3 .

4. Укажите промежуток, которому принадлежит корень уравнения:

log 5 х + log 5 9 = 3

1) [ – 10; – 15] ; 2) (5; 10); 3) (10; 13); 4) [ 10; 15]

5. Укажите промежуток, которому принадлежит корень уравнения:

√ х + 1 = 1

1) [ 2; 5] ; 2) (– 2; – 1); 3) [– 1; 1]; 4) [ 1; 2]

6. Решите уравнение: log 7 (2 х – 3) = log 7 (х – 2).

1) 5; 2) решений нет; 3) 1; 4) 3 (1/3).

7. Найдите сумму корней уравнения: (1/3) +5 (1/3)х – 24 = 0

1) –1; 2) 11; 3) –5; 4) 1 .

8.Укажите промежуток, которому принадлежит корень уравнения: log 5 (3х – 1) = 3

1) (15; 20) ; 2) [ 0; 2]; 3) (20; 40); 4) [ 40; 50]

9. Решите уравнение: 5х+1 + 5х + 5х – 1 = 31.

1) решений нет; 2) 0; 3) 1; 4) 5 .

10. Решите уравнение: : log 5 (2 х – 3) + log 5 6 = log 5 18.

1) 0; 2) 3; 3) 2; 4) 1/3.

11. Найдите сумму корней уравнения: log22 х – 4 log 2 х= 12.

1) 64,25; 2) 8; 3) 4; 4) 60.

12. Укажите промежуток, которому принадлежит корень уравнения: log 5 (3х + 1) = 2

1) (–∞; 0) ; 2) [ 0; 7]; 3) (2; 3); 4) (- 1/3; +∞)

13. Решите уравнение: 3х+2 + 3 х + 1 + 3х = 39

1) –1; 2) 0; 3) 1; 4) 3 .

14. Найдите сумму корней уравнения: 3 – 43х + 3 = 0.

1) 0; 2) 1; 3) 3; 4) 2.

15. Решите уравнение lg(5х+11)–lg(1/2) = lg13

1) –0,9; 2) 2,5; 3) 10/9; 4) 10.

16. Решите уравнение: .

1) –3 ; 2) 3; 3) 5; 4)5/3.


ПЗ № 36. Решение систем уравнений. Использование свойств и графиков функций для решения уравнений и неравенств.

Задание:

1)Перепишите и заполните пропуски:
Пример 1. Решить систему уравнений:  .

Решение: ,

Выразим у через х из (2) -го уравнения системы и подставим это значение в (1) -ое уравнение системы. Решаем (2) -ое уравнение полученной системы: 2х+2x+2=10, применяем формулу: ax+y=ax∙ay.

2x+2x∙22=10, вынесем общий множитель 2х за скобки: 2х · (1+22)=10 или 2х∙5=10, отсюда 2х=2.

2х=21, отсюда х=... Возвращаемся к системе уравнений. у = х + 1 = 1 + 1 = ...

Ответ: (1; 2).

Пример 2. Решить систему уравнений:  .

Решение: Представляем левую и правую части (1) -го уравнения в виде степеней с основанием 2, а правую часть (2) -го уравнения как нулевую степень числа 5. Если равны две степени с одинаковыми основаниями, то равны и показатели этих степеней — приравниваем показатели степеней с основаниями 2 и показатели степеней с основаниями 5. Получившуюся систему линейных уравнений с двумя переменными решаем методом сложения.

,

Находим х = … и это значение подставляем вместо х во второе уравнение системы, находим у.

2(2 + у) = 7, 2 + у = 3,5 ; у = … Ответ: (2; 1,5).

Пример 3. Решить систему уравнений: .

Решение: ,

Сделаем замену , .

Выразим через . Подставим во 2 уравнение. Решим уравнение с переменной .

. По теореме Виета ,


Возвращаемся к х, у. , х = 2, у = 1. Ответ: (2; 1).

Пример 4. Решить систему уравнений:

Решение: ,

Подставим из 2 уравнения у в 1, решим с переменной х.

х· (4х 15) = 4, 4х2 15х 4 = 0. D = 152 4 = 225 64 = .


По свойству логарифмов х > 0, y > 0, поэтому х = 0,25. Найдем у: у = 4 0,25 15 = 1 + 15 = …

Ответ: (0,25;16).

Пример 5. а)Решить систему уравнений:

Решение: ,

Решаем способом сложения: 2 = 10, = 5, х = …, у = х21 = 25 21 = …

Ответ: (25,4).

б) Решить систему уравнений:

Решение: ,

Из 1 уравнения выразим х и подставим во 2: 2х = 5 у, х = 2,5 0,5у,

, , ,

, D = ()2 4 = 9 352 = …


О.Д.З. : , у2 не уд. О.Д.З., поэтому

Ответ: (3;).

Пример 6. Решить уравнение hello_html_m51b291f3.png

ОДЗ этого уравнения состоит из всех x, одновременно удовлетворяющих условиям hello_html_2eb3b384.png. Это значит, что ОДЗ есть пустое множество. Этим решение уравнения и завершается, т. к. установлено, то ни одно число не может являться решением, т.е. уравнение не имеет корней.

Ответ: решений нет.

Пример 7. Решить неравенство hello_html_m3821e4c8.png

Нахождение ОДЗ неравенства есть трудная задача, поэтому перейдем к равносильной ему системе неравенствhello_html_5d6879e5.pnghello_html_m29a2435e.png.

Третье неравенство имеет решение hello_html_m53fb9541.png. Первое и второе неравенство справедливо лишь для x из промежутка hello_html_m31dbfcc5.png. Поэтому этот промежуток является множеством решений системы.

Ответ: hello_html_m31dbfcc5.png.

Пример 8. Решить неравенство hello_html_29bb9126.png. Существует стандартный прием решения: возведение в квадрат (при условии hello_html_53296a34.pnghello_html_m23cdad48.png0). Мы рассмотрим решение данного неравенства с использованием свойства монотонности. Функция, расположенная в левой части неравенства, монотонно возрастает, в правой части - убывает. Из этого следует, что уравнение hello_html_3124ddfd.png имеет не более одного решения, причем если x0 – решение этого уравнения, то при hello_html_24195d59.png будет hello_html_23029354.png, а решением данного неравенства будет hello_html_4c5db010.png. Значение hello_html_450402f6.png легко подбирается: hello_html_me29bdd5.png.

Ответ: hello_html_m7c58eed1.png.

Пример 9. Решить уравнение hello_html_6f9b3b8f.png

Данное уравнение имеет очевидное решение hello_html_32a3376d.png. Докажем, что других решений нет. Поделим обе части на hello_html_8475fac.png, получим hello_html_m1d3f68cc.png.Левая часть представляет собой монотонно убывающую функцию. Правая часть функция постоянная. Следовательно, каждое свое значение она принимает один раз, то есть данное уравнение имеет единственное решение.Ответ: hello_html_32a3376d.png.

Пример 10.а)

hello_html_2d44b68f.jpg

б)

hello_html_m16d16eed.jpg

2)Решить задание ( по примерам):

  1. Решить систему уравнений:

  2. Решить систему уравнений: 

  3. Решить систему уравнений:

  4. Решить систему уравнений:

  5. а)Решить систему уравнений:

б) Решить систему уравнений:

3)А)Решить задание:

  1. Найти значение выражения ,если и - решение системы уравнений

а) ,б) в) г) ,д)

  1. Найти значение выражения ,если и - решение системы уравнений

а)б) в) г) ,д)

  1. Решить систему уравнений: 


а) б) в)

г)д)

  1. Решите систему уравнений , решение принадлежит отрезку [0;2].


  1. Решить систему уравнений:

 а)б)

В) ТЕСТ. 1-в.

1. Какое неравенство не существует при х = - 6:hello_html_m9cafd1f.gif

1) ;

2) ;

3) ?

2. Укажите промежуток, которому принадлежат корни уравнения :

1) (2;3); 2) ; 3) (-0,5;0); 4) (1;2).

3. Выберите решение неравенства :hello_html_4ec559a2.gifhello_html_m44224fa1.gif

1) ;

2) ;

3) ;

4) .

4. Какой рисунок соответствует уравнению

? (укажите номер рисунка).

5. Укажите приближенное значение корня уравнения :

1) -2; 2) 0,4; 3) -0,3.

2-в.

1. Укажите промежуток, которому принадлежат корни уравнения :hello_html_7a8b474.gif

1) (2;3); 2) ; 3) (0,5;1,5); 4) (0;0,5).

2. Какой рисунок соответствует уравнению ? (укажите номер рисунка).

3. Какое неравенство не существует при х = - 2:

1) ;

2) ;

3) ?hello_html_m7559fa1.gifhello_html_3271fc9f.gif

4. Выберите решение неравенства :

1) ;

2) ;

3) ;

4) .


5. Укажите значение наибольшего корня уравнения :

1) 0,15; 2) 0; 3)1.

ПЗ № 37. Числовая последовательность, способы ее задания, вычисления членов последовательности. Предел последовательности. Бесконечно убывающая геометрическая прогрессия.

Задание:

1)Опорный конспект.

ЧИСЛОВАЯ ПОСЛЕДОВАТЕЛЬНОСТЬ  функция вида y = f(x), x О N, где N– множество натуральных чисел (или функция натурального аргумента), обозначается y = f(n) или y1, y2,…, yn,…. Значения y1, y2, y3,… называют соответственно первым, вторым, третьим, … членами последовательности.

Например, для функции y = n2 можно записать:

y1 = 12 = 1; y2 = 22 = 4; y3 = 32 = 9;…yn = n2;…

Способы задания последовательностей. Последовательности можно задавать различными способами, среди которых особенно важны три: аналитический, описательный и рекуррентный.

1. Последовательность задана аналитически, если задана формула ее n-го члена: yn = f(n).

Пример. yn = 2n – 1  последовательность нечетных чисел: 1, 3, 5, 7, 9, …

2. Описательный способ задания числовой последовательности состоит в том, что объясняется, из каких элементов строится последовательность.

Пример 1. «Все члены последовательности равны 1». Это значит, речь идет о стационарной последовательности 1, 1, 1, …, 1, ….

Пример 2. «Последовательность состоит из всех простых чисел в порядке возрастания». Таким образом, задана последовательность 2, 3, 5, 7, 11, …. При таком способе задания последовательности в данном примере трудно ответить, чему равен, скажем, 1000-й элемент последовательности.

3. Рекуррентный способ задания последовательности состоит в том, что указывается правило, позволяющее вычислить n-й член последовательности, если известны ее предыдущие члены. Название рекуррентный способ происходит от латинского слова recurrere – возвращаться. Чаще всего в таких случаях указывают формулу, позволяющую выразить n-й член последовательности через предыдущие, и задают 1–2 начальных члена последовательности.

Пример 1. y1 = 3; yn = yn–1 + 4, если n = 2, 3, 4,….

Здесь y1 = 3; y2 = 3 + 4 = 7; y3 = 7 + 4 = 11; ….

Можно видеть, что полученную в этом примере последовательность может быть задана и аналитически: yn = 4n – 1.

Пример 2. y1 = 1; y2 = 1; yn = yn–2 + yn–1 , если n = 3, 4,….

Здесь: y1 = 1; y2 = 1; y3 = 1 + 1 = 2; y4 = 1 + 2 = 3; y5 = 2 + 3 = 5; y6 = 3 + 5 = 8;

Арифметическая прогрессия.

Числовую последовательность, каждый член которой, начиная со второго, равен сумме предыдущего члена и одного и того же числа d, называют арифметической прогрессией, а число d – разностью арифметической прогрессии.

Таким образом, арифметическая прогрессия – это числовая последовательность {an}, заданная рекуррентно соотношениями a1 = a, an = an–1 + d (n = 2, 3, 4, …) (a и d – заданные числа).

Пример 1. 1, 3, 5, 7, 9, 11, … – возрастающая арифметическая прогрессия, у которой a1 = 1, d = 2.

Пример 2. 20, 17, 14, 11, 8, 5, 2, –1, –4,… – убывающая арифметическая прогрессия,

у которой a1 = 20, d = –3.

Нетрудно найти явное (формульное) выражение an через n. Величина очередного элемента возрастает на d по сравнению с предыдущим, таким образом, величина n элемента возрастет на величину (n – 1)d по сравнению с первым членом арифметической прогрессии, т.е.

an = a1 + d(n – 1). Это формула n-го члена арифметической прогрессии.

hello_html_m442ca9c.gif. Это формула суммы n членов арифметической прогрессии.

Пример3. При каком значении x числа 3x + 2, 5x – 4 и 11x + 12 образуют конечную арифметическую прогрессию?

Согласно характеристическому свойству, заданные выражения должны удовлетворять соотношению

5x – 4 = ((3x + 2) + (11x + 12))/2.

Решение этого уравнения дает x = –5,5. При этом значении x заданные выражения 3x + 2, 5x – 4 и 11x + 12 принимают, соответственно, значения –14,5, –31,5, –48,5. Это – арифметическая прогрессия, ее разность равна –17.

Геометрическая прогрессия.

Числовую последовательность, все члены которой отличны от нуля и каждый член которой, начиная со второго, получается из предыдущего члена умножением на одно и то же число q, называют геометрической прогрессией, а число q – знаменателем геометрической прогрессии.

Таким образом, геометрическая прогрессия – это числовая последовательность {bn}, заданная рекуррентно соотношениями b1 = b, bn = bn–1 q (n = 2, 3, 4…). (b и q – заданные числа, b ≠ 0, q ≠ 0).

Пример 1. 2, 6, 18, 54, … – возрастающая геометрическая прогрессия b = 2, q = 3.

Пример 2. 2, –2, 2, –2, …  геометрическая прогрессия b = 2, q = –1.

Пример 3. 8, 8, 8, 8, …  геометрическая прогрессия b = 8, q = 1.

Геометрическая прогрессия является возрастающей последовательностью, еслиb1 > 0, q > 1, и убывающей, если b1 > 0, 0 < q < 1.

Одно из очевидных свойств геометрической прогрессии состоит в том, что если последовательность является геометрической прогрессией, то и последовательность квадратов, т.е.

b12, b22, b32, …, bn2,… является геометрической прогрессией, первый член которой равен b12, а знаменатель – q2.

Формула n-го члена геометрической прогрессии имеет вид bn = b1qn–1.

Формула суммы первых n членов прогрессии

 hello_html_470610d0.gif, или hello_html_m6e7205ae.gif, hello_html_763ec99f.gif (в случае hello_html_m61e84691.gif, hello_html_1c16e6b4.gif).

Если геометрическая прогрессия бесконечно убывающая (hello_html_m474ab65f.gif), то ее сумма вычисляется по формуле  hello_html_m3403f07c.gif.

Обращение периодической десятичной дроби в обыкновенную. Предположим, мы хотим обратить периодическую десятичную дробь 0,(3) в обыкновенную. Рассмотрим эту десятичную дробь в следующем виде:

hello_html_m3f36da04.gif

Это бесконечно убывающая геометрическая прогрессия, первый член которой равен 3/10, а разность q = 1/10. В соответствии с выше приведенной формулой эта сумма равна:

hello_html_m22101018.gif

Таким образом, 0,(3) = 1/3.

1)

Бесконечно убывающая геометрическая прогрессия, т.к Sп=<1.

. . . х-1=3. х=4.

2) 1+2х+4х+…+(2х)+…=3,4-1,2х <0,5

Бесконечно убывающая геометрическая прогрессия, т.к <0,5. .

Бесконечно убывающая геометрическая прогрессия (|q|<1). hello_html_m522d392.png.


Пример 1. Найдите сумму всех целых чисел, начиная от 30 и до 80 включительно.

    Решение:

        Сумма всех целых чисел от 30 и до 80 включительно представляет собой сумму членов арифметической прогрессии, где а1 = 30, разность d = 1, а количество членов n = 51. 

        hello_html_m2c236891.png. Ответ: 2805.

Пример 2. Решите уравнение 22∙24∙26∙...∙22n=(0,125)-10 .

    Решение:

        22∙24∙26∙...∙22n=22+4+6+...+2n.  hello_html_m638d9306.png.

        Получили: 22+4+6+...+2n=230 hello_html_3e9ff5b7.png2+4+6+...+2n=30.

        В левой части равенства сумма n членов арифметической прогрессии, где а1 =2, а d = 2. Согласно формуле суммы членов арифметической прогрессии имеем:

        hello_html_m512755f7.png.

        n1 = – 6 не удовлетворяет условию т.к. n отрицательным быть не может.

        n2 =5 удовлетворяет условию. Ответ: n = 5.

Пример 3. В геометрической прогрессии пятый член равен 2, восьмой равен 16. Найти сумму первых десяти членов.

    Решение:Пусть (bn) – геометрическая прогрессия. Известно, что b5 = 2. По формуле n – го члена геометрической прогрессии      b5 = b1 q4 = 2. Аналогично b8 = b1 q7 = 16. Поделив второе равенство на первое, получим q3 = 8, т.е. q = 2. Подставив q = 2 в первое равенство, найдём b1:

        hello_html_6ff45943.png. Согласно формуле hello_html_m65da4e87.pngполучим: hello_html_3f7115ea.png.

Ответ: hello_html_aae12bf.png.

Пример 4. Записать число 0,6222… обыкновенной дробью.

    Решение:

        Запишем число 0,6222…в виде суммы: 0,6222…= 0,6 + 0,02 + 0,002 + 0,0002 + …,

        где 0,02 + 0,002 + 0,0002 + … - сумма членов бесконечно убывающей геометрической прогрессии, которая находится по формуле hello_html_m5ab5ce1b.png, где b1 = 0,02; q = 0,1.

        hello_html_549eecc.png.

Ответ: 28/45 .

2)Решить задание:

  1. Найти произведение третьего и четвёртого членов арифметической прогрессии,

если первый член равен 3, а второй равен – 2.

    2) Между числами – 8,8 и 2 вставьте пять чисел так, чтобы получилась арифметическая прогрессия.

  1. Третий член арифметической прогрессии в три раза меньше шестого, а сумма второго и

пятого равна 16. Определите первый член прогрессии

    4) Найдите знаменатель геометрической прогрессии, если b1=2, b5=162.

    5) В геометрической прогрессии q=0,5, b6=1/32 , найти b1.

    6) Найдите третий член геометрической прогрессии, если её знаменатель равен -2,

а седьмой член 16.

    7) В геометрической прогрессии b1+b2+b3=31, b1+b3=26. Найти b7.

    8) Какое наибольшее число последовательных нечётных чисел, начиная с 1, можно сложить,

чтобы получившаяся сумма осталась меньше 400?

    9) В геометрической прогрессии b2>b1 в два раза, а b=64. Найти b1 .

    10) Найдите сумму первых пяти членов последовательности, общий член которой выражается

формулой hello_html_284c9d.png.

    11) Сумма членов арифметической прогрессии с третьего по одиннадцатый включительно

равна 27. Найти номер члена прогрессии равного 3.

    12) Сумма бесконечной убывающей геометрической прогрессии равна 32, а сумма пяти её

первых членов равна 31. Найдите первый член прогрессии.

    13) Найдите сумму всех чётных натуральных трёхзначных чисел, делящихся на 3.

    14) Вычислить сумму: hello_html_m52b7aa09.png.

    15) Найдите сумму всех положительных членов арифметической прогрессии 10,3; 8,5; ….

    16) В арифметической прогрессии a10=-23 . Найти a3+a17.

    17) В арифметической прогрессии a5+a9=-20. Найти a7.

    18) Произведение девяти первых членов геометрической прогрессии равно hello_html_m6523f58.png.

Какой член геометрической прогрессии можно найти на основании этой информации? Чему он равен?

    19) Решите уравнение hello_html_m14c9512b.png.

    20) Найдите суммуhello_html_m19749d63.png.

    21) Три различных числа a1, a2, a3 в указанном порядке образуют арифметическую прогрессию,

а числа 2a3-a1, a2+a3-a1, a1  в указанном порядке образуют геометрическую прогрессию.

Найти знаменатель геометрической прогрессии.

3)Решить задание:

  1. Пусть – арифметическая прогрессия с разностью d и Sn – сумма n первых членов. Найти:

1. a13, если a5=2; a40=142.

2. a1+a20, если a3+a18=50.

3. n, если a1=3; a2=5; Sn=360.

4. a1 и d, если a17+a20=35; a16a21=150.

5. a1 и d, если Sn=2n2-3n.

6. Сумму всех натуральных трехзначных чисел, не делящихся на 3.

7. Первых 100 натуральных чисел, каждое из которых при делении на 5 дает в остатке 2.

  1. Пусть – геометрическая прогрессия со знаменателем q и Sn – суммой первых n членов. Найти:

8. b6, если b5=36, b7=114.

9. q, если b1=10, b2+b3=60.

10. b13, если b11=25, b15=400.

11. b1 и q, если b1+b2+b3=62, b12+b22+b32=2604.

12. S6, если b1=–2, b6=–486.

13. n, если b1=9, bn=, Sn=25.

14. Какому условию удовлетворяют три числа a1, a2, a3, которые одновременно являются последовательными членами как геометрической, так и арифметической прогрессий?

15. Решить уравнение: .

16. По преданию, индийский шах позволил изобретателю шахматной игры самому назначить себе награду. Изобретатель просил, чтобы ему за первую клетку шахматной доски было дано 1 зерно, за вторую – 2, за третью – 4. В общем случае, за каждую следующую клетку в 2 раза больше, чем за предыдущую. Узнать, сколькими цифрами изображается число зерен, предназначенное изобретателю; найти это число.

17.Три числа образуют геометрическую прогрессию. Если второе число увеличить на 2, то прогрессия станет арифметической, а если после этого последнее число увеличить на 9, то прогрессия станет геометрической. Найти эти числа.

18.Решить уравнение .

19.Найти a1 и d, если a11=6; a16=8,5.

20. Может ли число 75 быть членом геометрической прогрессии , у которой b1=4 и q=?

21.Найти количество всех трехзначных натуральных чисел, делящихся на 7.

22.Доказать, что последовательность с общим членом является арифметической прогрессией.



ПЗ № 38. Производная: механический и геометрический смысл производной. Уравнение касательной в общем виде.

Задание:

1)Опорный конспект.

Механический смысл производной. Рассмотрим простейший случай: движение материальной точки вдоль координатной оси, причём закон движения задан:  координата  x  движущейся точки – известная функция  x ( t ) времени  t. В течение интервала времени от  t0  до  t0 + hello_html_7bc674bb.gif  точка перемещается на расстояние:  x ( t0 + hello_html_7bc674bb.gif ) - x ( t0 ) = hello_html_m1ebb9fce.gif, а её средняя скорость равна:  va = hello_html_m1ebb9fce.gif / hello_html_7bc674bb.gif . При  hello_html_7bc674bb.gif hello_html_2c63fb57.gif 0  значение средней скорости стремится к определённой величине, которая называется мгновенной скоростью  v ( t0 )  материальной точки в момент времени  t0 . Но по определению производной мы имеем:

hello_html_75ac67c4.gif

отсюда,  v t0 ) = x’ t0 ) , т.e. скорость – это производная координаты по времени. В этом и состоит  механический смысл производной.Аналогично, ускорение – это производная скорости по времени:  a = v’ t ).

Пример. Точка движется прямолинейно по закону S (t)= 2t3 – 0,5t2 + 3t (S – путь в метрах, t – время в секундах). Вычислить скорость движения точки в момент времени t=1с.

Решение: vt ) = s ’ t ) = 6t2 – t + 3, v(1) = 6 – 1 + 3 = 8.

Геометрический смысл производной заключается в том, что численно производная функции в данной точке равна тангенсу угла, образованного касательной, проведенной через эту точку к данной кривой, и положительным направлением оси Ох:

hello_html_m2921bfa1.jpg

 Уравнение касательной. 

y =  f ( x0 ) +  f ’( x0 ) · ( x – x0  ) .


2)Перепишите и заполните пропуски:

Пример 1. Найти угловой коэффициент касательной к графику функции y = f(x) в точке с

абсциссой х: а) y(x) = x³, x = 1, б) y(x) = ln x, x = 1, в) y(x) = 3x² 4x, x = 2,

г) y(x) = х3 + 7x² 5x+3, x = 3, д) y(x) = ех, x = ln 7, e) y(x) = 7sinx, x = 0,ж) y(x) = е, x = ln 4.

Решение: угловой коэффициент k равен производной от функции в точке, т.е. k = y (x0) ,

найдем производные и вычислим их в точке x0

a)   бв)  

г)

д) е ln 7= …,е) 7cos x, 7 cos 0 = 7 1 = …,

ж) е3 ln 4 = 343 = 364 = …

Ответ: a)3, б)1, в)8,г) 64,д) 7,е)7,ж) 192.

Пример 2. а) Найти угловой коэффициент k, если α = arctg 6, α = - arctg 8.

б) Найти α,если y(x) = х3, x = 2.

Решение: а) k = tgα = tg k = tgα = tg k = tgα = tg

k = tgα = tg

б)

Ответ: а)1, ,6,- 8, б) arctg 4.

Пример 3. Дана функция y = x3. Составить уравнение касательной к графику этой функции в точке x0 = 2.
Решение: Уравнение касательной: y = f  (x0) · (x − x0) + f(x0). Точка x0 = 2 нам дана, а вот значения f (x0) и f (x0) придется вычислять.

Для начала найдем значение функции. Тут все легко: f (x0) = f (2) = 23 = …;
Теперь найдем производную: f  (x) = (x3) = 3x2;
Подставляем в производную x0 = 2: f   (x0) = f  (2) = 3 · 22 = 34 = …;
Итого получаем: y = 12 · (x − 2) + 8 = 12x − 24 + 8 = 12x − 16. 
Это и есть уравнение касательной.

Ответ: y = 12x − 16. 
Пример 4. Составить уравнение касательной к графику функции f (x) = 2sin x + 5 в точке x0 = π/2.

Решение: f (x0) = f (π/2) = 2sin (π/2) + 5 = 2 + 5 = …; f  (x) = (2sin x + 5) = 2cos x;
f  (x0) = f  (π/2) = 2cos (π/2) = 0;

Уравнение касательной: y = 0 · (x − π/2) + 7  y = ...hello_html_44c9db7b.png

Ответ: y = 7.

Пример 5. Составьте уравнение касательной к графику функции  

в точке M(3; – 2).

Решение: Точка M(3; – 2) является точкой касания, так как

1. a = 3 – абсцисса точки касания.2. f(3) = – 2. 3. f '(x) = x2 – 4, f '(3) = 9 4 = …
y = – 2 + 5(x – 3), y = …x – 17 – уравнение касательной.

Ответ: y = 5x – 17.

Пример 6. Напишите уравнения всех касательных к графику функции y = – x2 – 4x + 2, проходящих через точку M(– 3; 6).

Решение: Точка M(– 3; 6) не является точкой касания, так как f(– 3)  6 (рис. 2).hello_html_m126a8a75.png

1. a – абсцисса точки касания.
2. f(a) = – a
2 – 4a + 2.
3. f '(x) = – 2x – 4, f '(a) = – 2a – 4.
4. y = – a2 – 4a + 2 – 2(a + 2)(x – a) – уравнение касательной.

Касательная проходит через точку M(– 3; 6), следовательно, ее координаты удовлетворяют уравнению касательной.

6 = – a2 – 4a + 2 – 2(a + 2)(– 3 – a),
a
2 + 6a + 8 = 0 , D = 62 41 8 = 36 32 = …,

а1= (6 2) : 2 = 8 : 2 = …, а2 = (6 2) : 2 = 4 : 2 = …,

Если a = – 4, то уравнение касательной имеет вид y = 4x + 18.

Если a = – 2, то уравнение касательной имеет вид y = 6.

Ответ: y = 4x + 18 или y = 6.
Пример 7. Напишите уравнения всех касательных к графику функции y = x3 – 3x2 + 3, параллельных прямой y = 9x + 1.

Решение: 1. a – абсцисса точки касания. 2. f(a) = a3 – 3a2 + 3.3. f '(x) = 3x2 – 6x, f '(a) = 3a2 – 6a.

Но, с другой стороны, f '(a) = 9 (условие параллельности). Значит, надо решить уравнение 3a2 – 6a = 9. 3a2 – 6a 9 = 0, hello_html_5a070ee0.png

D = (6)2 43 () = 36 108 = …, а1= (6 12) : 6 = 18 : 6 = …,

а2 = (6 12) : 6 = 6 : 6 = …,

Его корни a = – 1, a = 3 (рис. 3).

4. 1сл.) a = – 1; f(– 1) = – 1– 3 + 3 = …;  f '(– 1) = 3 + 6 = …;

 y = – 1 + 9(x + 1); y = 9x + 8 – уравнение касательной;

2сл.) a = 3; f(3) = 27–27 + 3 = …; f '(3) = 27 – 18 = …;
y = 3 + 9(x – 3); y = 9x – 24 – уравнение касательной.

Ответ: y = 9x + 8 и y = 9x – 24.

Пример 8. Напишите уравнение касательной к графику функции y = 0,5x2 – 3x + 1, проходящей под углом 45° к прямой y = 0 (рис. 4).hello_html_m32119c54.png

Решение: Из условия '(a) = tg 45°, найдем a:  a – 3 = 1 ,a = 3 + 1 = ...

1. a = 4 – абсцисса точки касания.
2. f(4) = 8 – 12 + 1 = ...
3. f '(4) = 4 – 3 = ...
4. y = – 3 + 1(x – 4). y = x – 7 – уравнение касательной.

Ответ: y = x – 7.

Пример 9. На параболе у = х2 взяты две точки с абсциссами 1 и 3. Через эти точки проведена прямая. В какой точке параболы касательная будет параллельна проведенной прямой?

Решение: у = х2 , (1;1), (3;9). Найдем уравнение прямой .

4х – 4 = у – 1. у = 4х – 3.

Прямые параллельны, если их угловые коэффициенты равны.

- угловой коэффициент касательной в точке с абсциссой х0.

 0 = 4. х0 = ... ,

Ответ: в точке (2;4) касательная параллельна заданной прямой.

Пример 10. При каких b и c прямые y = x и y = – 2x являются касательными к графику

функции y = x2 + bx + c?

Решение: Пусть t – абсцисса точки касания прямой y = x с параболой y = x2 + bx + c;

p – абсцисса точки касания прямой y = – 2x с параболой y = x2 + bx + c.

Тогда уравнение касательной y = x примет вид y = (2t + b)x + c – t2, а уравнение

касательной y = – 2x примет вид y = (2p + b)x + c – p2. Составим и решим систему уравнений:

;


2t = 1,5; t = 0,75;

p = – t = …,

c = = = …,

b = 1 – 2t = 1 – 2 0,75 = 1– 1,5 = …

Ответ: b = – 0,5; c = 0,562 5.

3)Решить задание ( по примерам):

  1. Найти угловой коэффициент касательной к графику функции y = f(x) в точке с

абсциссой х: а) y(x) = x4, x = 1, б) y(x) = ln x, x = 2, в) y(x) = 3x² - 4x, x = 4,

г) y(x) = х3 + 7x² - 5x+3, x =5, д) y(x) = ех, x = ln 8, e) y(x) = 9sinx, x = 0,ж) y(x) = е, x = ln 6.

  1. а) Найти угловой коэффициент k, если α = arctg 9, α = - arctg 11.

б) Найти α,если y(x) = х3, x = 4.

  1. Дана функция y = x3. Составить уравнение касательной к графику этой функции в точке x0 = 1.

  2. Составить уравнение касательной к графику функции f (x) = 4sin x + 5 в точке x0 = π/2.

  3. Составьте уравнение касательной к графику функции  в точке M(3; – 1).

  4. Напишите уравнения всех касательных к графику функции y = – x2 – 4x + 2, проходящих через точку M(– 3; 9).

  5. Напишите уравнения всех касательных к графику функции y = x3 – 3x2 + 3, параллельных

прямой y = 24x + 1.

  1. Напишите уравнение касательной к графику функции y = 0,5x2 – 5x + 1, проходящей
    под углом 45° к прямой y = 0 .

  2. На параболе у=х2 взяты две точки с абсциссами 1 и 2. Через эти точки проведена прямая.
    В какой точке параболы касательная будет параллельна проведенной прямой?

  3. При каких b и c прямые y = x и y = – 2x являются касательными к графику
    функции y = x2 + 2bx + c?

  4. Точка движется прямолинейно по закону S (t)= t5t4 + 6 (S – путь в метрах, t – время в секундах). Вычислить скорость движения точки в момент времени t=2с.

  5. Точка движется прямолинейно по закону S (t)= 2t3 – 0,5t2 + 3t (S – путь в метрах, t – время в секундах). Вычислить скорость движения точки в момент времени t= 2с.

  6. Какая из приведенных зависимостей описывает равноускоренное движение:

а) x = 4 + 2t; б) v = 5; в) x = 8 - 2t - 4t2; г) x = 10 + 5t2.

  1. Точка движется вдоль оси х согласно закону х = 8 – 2t – 4t2 . Определите начальную скорость и ускорение . Запишите уравнение для скорости.

4)Решить задание:

  1. Напишите уравнения касательных, проведенных к графику функции y = 2x2 – 4x + 3 в точках пересечения графика с прямой y = x + 3.

  2. При каких значениях a касательная, проведенная к графику функции y = x2 – ax в точке графика с абсциссой x0 = 1, проходит через точку M(2; 3)?

  3. Найдите все общие точки графика функции y = 3x – x3 и касательной, проведенной к этому графику через точку P(0; 16).

  4. На кривой y = x2 – x + 1 найдите точку, в которой касательная к графику параллельна
    прямой y – 3x + 1 = 0.

  5. Найдите угол q между касательными к графику функции y = x3 – 4x2 + 3x + 1, проведенными в точках с абсциссами 0 и 1.

  6. Прямая y = 2x + 7 и парабола y = x2 – 1 пересекаются в точках M и N. Найдите точку K пересечения прямых, касающихся параболы в точках M и N.

  7. При каких значениях b касательная, проведенная к графику функции y = bx3 – 2x2 – 4 в точке с абсциссой x0 = 2, проходит через точку M(1; 8)?

  8. Найти угол между касательными к графику функции , проведенными в точках с абсциссами 1 и 2.

  9. Является ли прямая у = х – 1 касательной к кривой у = х3 – 2х + 1?

  10. Найдите уравнение касательной к графику функции  в точке с абсциссой .

  11. К графику функции у = 3(х + 2) проведены две параллельные касательные, одна из которых проходит через точку графика с абсциссой х0 = – 1. Найдите абсциссу точки, в которой
    другая касательная касается графика данной функции.

  12. Напишите уравнение касательной к графику функции f(x) = x2 – 4x + 5, если эта касательная проходит через точку (0; 4) и абсцисса точки касания положительна.

  13. Напишите уравнение касательной к графику функции f(x) = x2 + 3x + 5, если эта касательная проходит через точку (0; 1) и абсцисса точки касания отрицательна.

  14. Найдите уравнение параболы f(x) = ax2 + bx + 1 касающейся прямой у = 7х + 2
    в точке М (1; 5).

  15. К графику функции провести касательную так, чтобы она была параллельна прямой у = 4х – 5.

  16. Из точки (0; 1) провести касательную к графику функции.

  17. Составить уравнение касательной к графику функции  в точке с абсциссой.

  18. Составить уравнение касательной к графику функции  в точке с абсциссой 

  19. Составить уравнение касательной к графику функции
      в точке с абсциссой  .

  20. Составить уравнение касательной к графику функции > 0, отсекающей от осей координат треугольник, площадь которого равна .

Задача. Пусть X = 2 + 4t2 - sin2πt. Найти: а) мгновенную скорость, б) ускорение, если t = 0,5 c.

Задача. Высота снежка, брошенного вертикально вверх со скоростью U0 с начальной высоты h0, меняется по закону h =h0+U0t-gt2/2, где g =10м/c – ускорение силы тяжести. Покажите, что энергия камня Е= mv2/ 2 + mgh, где m масса снежка, не зависит от времени.

Задача. Количество электричества, протекающее через проводник, задаѐтся формулой

q(t) = t+4/t. В какой момент времени ток в цепи равен нулю?

ПЗ № 39. Правила и формулы дифференцирования, таблица производных элементарных функций.

Задание:

1)Опорный конспект.

Пример 1. Найти производную функции  y = .

Решение: По свойству дифференцирования произведения,

hello_html_m667262fc.png.

Используя формулу для нахождения производной показательной и степенной функций, получим: hello_html_m33ce9c10.png , hello_html_m7a155e45.png

Для нахождения производной использовались правила дифференцирования и таблица производных функций. Ответ: hello_html_m7259a439.png .

Пример 2. Найти производную функции  y = .

Решение: Воспользуемся правилом дифференцирования частного:

hello_html_m6c0be978.png.

Производная суммы/разности равна сумме/разности производных и константу можно выносить за знак производной, поэтому имеем:

hello_html_m6d9e2531.png,

hello_html_m4661b9f4.png,

hello_html_5aeb11a1.png, hello_html_303e1e97.png , hello_html_47c000ce.png .

Ответ: hello_html_6873cbb0.png .

Пример 3. Найти производную функции y =   .

Решение: По правилу дифференцирования частного:

hello_html_m2652334c.png,

Далее воспользуемся формулами из таблицы производных - формулам для производных степенной и тригонометрических функций, а также учитываем тот факт, что производная суммы равна сумме производных:

hello_html_6bfbb3b0.png,

hello_html_4f5a6440.png,

hello_html_345a5152.png, hello_html_3dd92d7d.png .

Ответ: hello_html_5f22ddde.png .

Пример 4. Найти производную функции  hello_html_mb62401c.png .

Решение: По свойству дифференцирования частного получаем:

hello_html_m1bbacdfd.png,

Далее пользуясь формулами для производных логарифмической и степенной функции, получим:

hello_html_m7e5e4019.png, hello_html_7fc6a7d9.png , hello_html_m68b092d7.png .Ответ: hello_html_ecc0376.png .

Пример 5. а) Найти производную функции  .

Решение:

Примените таблицу основных производных и формулы производных линейной комбинации и отношения функций.



Ответ:  .

б) Вычислить производную функции y = cos ln ().

Решение: Примените таблицу основных производных и формулу производной сложной функции.

y / = sin ln (3x2 ) (ln (3x2)) / = sin ln (3x2 ) / =

= sin ln (3x2 ) .

Ответ:  sin ln (3x2 ) .

Задание: Найти производную функции: 

1) y = . 2) y = . 3) y = . 4) .5) а) . б) y = cos ln (2x2).

2)Перепишите и заполните пропуски:

Пример 1. Найдите производные функций: а) y = ex x7 ,б) у=3ех+cos2x, в) у = ехsinx,

г) у= ln2x ,д) , е) , ж)

Решение: а) б) в) = ех cosx; г) ,

д)е)ж)

Ответ: а)б) в) = ех cosx; г) ,д)

е)ж)

Пример 2. Вычислите значение производной функции:

а) у= в точке , б) у=ех sinx + x2 в точке ,

в) у = cos2x + 4x в точке ,г) в точке .

Решение: а)


б)

в)

г) Ответ: а)10,5; б)1;в)4; г)2.

Пример 3. Найдите производные функций: а) б)

в) г) д)
Решение: а) у  (x) = (x 2 + sin x) = (x 2) + (sin x) = …x + cos x;
б)
у  (x) = (x 3 · cos x) = (x 3) · cos x + x 3 · (cos x) = …x 2 · cos x + x 3· (− sin x) =

= x 2 · (3cos x  x · sin x),

в) у  (x) = ((x 2 + 7x − 7) · e x ) = (x 2 + 7x − 7) · e x + (x 2 + 7x − 7) · (e x ) = (2x + 7) · e x +

+(x 2 + 7x − 7) · e x = e x · (2x + 7 + x 2 + 7x−7) = (x 2 + …x) · e x = x(x + …) · e x .

г)
д)

По традиции, разложим числитель на множители — это значительно упростит ответ:


Ответ: а) у  (x) = 2x + cos x; б) у  (x) = x 2 · (3cos x  x · sin x), в) у  (x) = x(x + 9) · e x ,

г) д)

Пример 4. Найти производные функций:  f(x) = e 2x + 3g(x) = sin (x 2 + ln x).
Решение: Заметим, что если в функции f(x) вместо выражения 2x + 3 будет просто x, то получится элементарная функция f(x) = e x . Поэтому делаем замену: пусть 2x + 3 = tf(x) = f(t) = e t . Ищем производную сложной функции по формуле:

f  (x) = f  (t) · t  = (e t ) · t  = e t · t . Выполняем обратную замену: t = 2x + 3. Получим:

f  (x) = e t · t  = e 2x + 3 · (2x + 3) = e 2x + 3 · 2 = … · e 2x + 3

Теперь разберемся с функцией g(x). Очевидно, надо заменить x 2 + ln x = t. Имеем:

g  (x) = g  (t) · t  = (sin t) · t  = cos t · t . Обратная замена: t = x 2 + ln x. Тогда:

g  (x) = cos (x 2 + ln x) · (x 2 + ln x) = cos (x 2 + ln x) · (…x + 1/x).

Ответ: f  (x) = 2 · e 2x + 3; g  (x) = (2x + 1/x) · cos (x 2 + ln x).
Пример 5. Найти производную функции :а)б)
Решение: а)

б)
Ответ: а) б)

2)Решить задание ( по примерам):

  1. Найдите производные функций: а) y = 2ex –3x7 ,б) у=5ех+cos3x, в) у = ехcosx,

г) у= – ln4х, д) , е) , ж)

  1. Вычислите значение производной функции:

а) у= в точке , б) у=2ех sinx +3 x2 в точке ,

в) у = cos2x + 8x в точке ,г) в точке .

  1. Найдите производные функций: а) б)

в) г) д)

  1. Найти производные функций:  f(x) = e 4x + 3; g(x) = sin (2x 2 + ln x).

  2. Найти производные функций : а)б)

3)А)Решить задание:

  1. Найдите производную функции y = e -x 2x7 , у= 4х3+ е .

  2. Найдите производную функции у = x2 + sinx в точке х0 =.
    Найдите производную функции у = sinх ex – 9x3 в точке xo=0.

  3. Найдите значение производной функции у = 5cos x – 7x в точке хо = 0 .

  4. Вычислите значение производной функции y = ln(2x+11)+ 5x в точке хо= 5.

  5. Найдите производную функции: а) б)

В) ТЕСТ.

ЧастьА.

А1. Найдите производную функции y = e -x -2x7 . 1) y´= - e-x -14x6; 2) y´= - e-x –; 3) y´= -e-x –2x6; 4) y´= e-x -14x6.

А2. Найдите производную функции у=4х3+ е .

1) у´=12х2 ; 2) у´=12х2 – е ; 3) у´=х4 - е ; 4) у´=12х2 – хе -х-1.

А3. Найдите производную функции у = x2 + sinx в точке х0 =.

1) 2 -1; 2) 2 + 1; 3) 2 -1; 4) 2. А4. Вычислите значение производной функции в точке хо=2. 1) 10; 2) 12; 3) 8; 4) 6.

А5. Найдите производную функции у = sinх ex – 9x3 в точке xo=0. 1) 0; 2) -1; 3) 1; 4) -9.

А6. Найдите значение производной функции у = 5cos x – 7x в точке хо = 0 . 1) -14; 2) -7;3) -9; 4) -2.

А7. Найдите производную функции .

1) 4х – 6+; 2) (2х - 3)2+; 3) 8х – 12 +; 4) 4х – 6 - . А8. Вычислите значение производной функции в точке хо= 4.

1) 21; 2) 24; 3) 0; 4) 3,5.

А9. Вычислите значение производной функции y = ln(2x+11)+ 5x в точке хо= -5. 1) 7; 2) -25; 3) 6; 4) 1. А10. Вычислите значение производной функции в точке хо= .

1) 1; 2) 2; 3) 0; 4) 4.

Часть В.

В1.Найдите производную функции:

1) ; 2) ;3) ; 4) ;

В2. К графику функции проведена касательная через точку с абсциссой . Вычислите тангенс угла наклона этой касательной к оси абсцисс.

ПЗ № 40. Исследование функции с помощью производной.

Задание:

1)Перепишите и заполните пропуски:

Пример 1. Исследовать и построить график функции:

Решение:

  1. D (f) = R, т.к. f -многочлен.

  2. Выясняем, является ли функция f четной или нечетной. - функция ни четная, ни нечетная.

  3. Функция непериодическая.

  4. Находим точки пересечения графика с осями координат:

а) с осью ОХ: у=0 получаем точки (0;0), (3;0)

б) с осью ОУ: х=0 получаем точки (0;0)

  1. Найдем производную функции:

  2. Найдем критические точки: , т.е. ,х = … или х = ...

Отмечаем эти точки 0 и 2 на числовой прямой, и определяем знак производной в каждом промежутке. −  +  

6(−  1) −  3(−  1)2 = −  6 −  3 = −  9 < 0

0 2 х

Значит, в промежутках и функция убывает и (0;2) – функция возрастает.

х = 0 - точка минимума, т.к. производная меняет знак с минуса на плюс.

Вычислим уmin=

х = 2 – точка максимума, т.к. производная меняет знак с плюса на минус.

Вычислим уmax= .

7.Составляем таблицу для внесения всех данных

x


0

(0;2)

2



− 

0

+

2

− 

f(x)


0


4




min


max


8. Строим график функции.

hello_html_6ac426ef.png


Пример 2. Сколько корней имеет уравнение: x4   4x3   9 = 0?

Решение: р (x) = x4   4x3   9, D(р) = ( hello_html_8781bd9.gif; hello_html_8781bd9.gif).

р ' (x) = 4 x 3 12x 2 = 4 x 2 3) = 0, x1 = 0; 1 петля; x2 = …, р ' (4) = 4 hello_html_m61765ba1.gif16 hello_html_m61765ba1.gif1 > 0






р(x) убывает на интервале (hello_html_8781bd9.gif ; 3]; р (x) возрастает на [3; +hello_html_8781bd9.gif).

x = 3 – min, р min= р (3) = 34   4 hello_html_m61765ba1.gif 33   9 = 81 4hello_html_m61765ba1.gif27 – 9 = 81   117= − < 0, в точке x = 0 график имеет точки перегиба (то есть меняет выпуклость), f(0) = 0   0  9 = ...

Строим эскиз графика

hello_html_331651eb.png

График пересекает ось 0Х в двух точках x1 и x2, следовательно, многочлен, а значит и данное уравнение имеет два корня.

Ответ: два.

Пример 3. Исследовать функцию f(x)= 3x5 3 + 2 и построим ее график.

Решение: 1.D (f ) = R, так как f (x) - многочлен.

2.Функция f не является ни четной, ни нечетной, так как

f (− x) = 3(− x)5 5(− x)3 + 2 = − 3x 5 + 5х3 + 2= −  ( 3x5 − 5х3 − 2) hello_html_382283fb.gif f(x)

3.Найдем координаты точек пересечения графика с осями координат:

а) с осью 0Х, для этого решим уравнение: 3x5 3 + 2 = 0.

Методом подбора можно найти один из корней (x = 1). Другие корни могут быть найдены только приближенно. Поэтому для данной функции остальные точки пересечения графика с осью абсцисс и промежутки знакопостоянства находить не будем.

б) с осью 0У: f(0) = 3hello_html_m61765ba1.gif05  5hello_html_m61765ba1.gif03 + 2 = …

Точка А (0; 2) - точка пересечения графика функции с осью 0У.

Отметили, что промежутки знакопостоянства не будем находить.

4.Найдем промежутки возрастания и убывания функции : а ) f '(x)= 15x4  15х2 = 15х2 hello_html_m61765ba1.gif2   1)

D (f ') = R, поэтому критических точек которых f '(x)не существует, нет.

б) f '(x) = 0, если х2hello_html_m61765ba1.gif2 1)=0 <=> x1 = 0 ; 1 петля ; x2  1= 0, x2 =  1, х2 = …, х3 = …

в) Получим три критические точки, они разбивают координатную прямую на четыре промежутка. Определим знак производной на этих промежутках: f ' (4) = 15 hello_html_m61765ba1.gif16 hello_html_m61765ba1.gif15 > 0






Так как функция непрерывна в точках – 1; 0; 1, то f   возрастает на (– hello_html_8781bd9.gif; – 1] и [1; +hello_html_8781bd9.gif);

f  убывает на [– 1; 0] и [0; 1].

5.Найдем точки экстремума функции и вычислим значения функции в этих точках.

x = − 1 - точка max, f (− 1) = 3hello_html_m61765ba1.gif (− 1) 5  5hello_html_m61765ba1.gif (− 1) 3 + 2 = − 3+ 5 + 2 = 7 – 3 = … ;

x = 1 - точка min, f (1) = 3hello_html_m61765ba1.gif15  5hello_html_m61765ba1.gif13 + 2 = 3− 5 + 2 = 5 – 5 = ...

Полученные результаты занесем в таблицу и построим график .


x


1

(− 1;0)

0

(0;1)

1



+

0

0

0

+

f(x)


4


2


0




max



min



Пример 4. Исследовать и построить график функции:hello_html_m46464d67.png

Решение: проведем исследование функции:

  1. Функция определена и непрерывна на всей числовой

прямой, D (f ) = R .
,

значит, данная функция является четной, ее график симметричен относительно оси ординат.
Очевидно, что функция непериодическая.

  1. Точки пересечения графика с координатными осями, интервалы знакопостоянства функции.
    График функции проходит через начало координат.

 на всей области определения.

  1. Возрастание, убывание, экстремумы функции.



х = 0
 – критическая точка. Определим знаки :
hello_html_5a5cd5d3.jpg

 возрастает на  и убывает на.
В точке х= 0 функция достигает минимума: .

  1. Найдем дополнительные точки и выполним чертёж:


х

0,5

1,5

2

2,5

3

4

5

6


0,08

0,43

0,57

0,68

0,75

0,84

0,89

0,93


hello_html_m1f9f2eb8.jpg


Пример 5. Сколько корней имеет уравнение: ?

Решение: Рассмотрим функцию р(x) =

1) Найдем область определения функции D(р) = (−hello_html_8781bd9.gif; hello_html_8781bd9.gif).

2) Найдем производную р' (x) = x 3 − 3x 2 – x + 3.

3) Найдем критические точки и промежутки возрастания и убывания функции:

р' (x) = 0 <=> x 3 − 3x 2 – х + 3= 0 <=> x 2 hello_html_m61765ba1.gif (х − 3) − (х − 3) = 0 <=> (х − 3) hello_html_m61765ba1.gif ( x 2 − 1) = 0 <=>

х1=3, х2= 1, х3= − 1. Получим три критические точки, они разбивают координатную прямую на четыре промежутка. Определим знак производной на этих промежутках:

р ' (4) = 1 ·15 > 0

hello_html_47ff2293.png




р(x) возрастает на интервалах [1; 1] и [3; +hello_html_8781bd9.gif);

р(x) убывает на (hello_html_8781bd9.gif ; 1] и [1; 3].

4) Найдем точки экстремума и экстремумы функции:

х = − 1 min р min= 1/4 + 1 − 1/2 − 3= 0,25 + 1 – 0,5 – 3 = 1,25 – 3,5 = − … < 0,

x = 1 max р max= 1/4 − 1 − 1/2 + 3 = 0,25 – 1 – 0,5 + 3 = 2 – 0,25=… > 0,

х = 3 min р min = 81/4 − 27 − 9/2 + 9 = 20,25 – 27 – 4,5 + 9 = 29,25 – 31,5 =

= − … < 0.

Строим эскиз графика.

Из рисунка видно, что многочлен имеет 4 корня, следовательно, уравнение имеет 4 решения.

Ответ: уравнение имеет 4 решения.

2)Решить задание ( по примерам):

  1. Исследуйте функцию и постройте ее график.

  2. Сколько корней имеет уравнение: x4 − 4x3 + 9 = 0?

  3. Исследовать функцию f(x)= 3x5 3 + 6 и построим ее график.

  4. Исследовать и построить график функции:.

  5. Сколько корней имеет уравнение: ?

3)Решить задание:

  1. Исследуйте функцию y = 1/3x3 − 3x2 + 8x и постройте ее график.

  2. Сколько корней имеет уравнение: x2 − x3/3− 1= 0?

  3. Исследовать и построить график функции:

а) y = 3x4 − 4x3 − 12x2 + 10; б) y = в) f (х) = x4 − 2х2.

  1. Исследуйте функцию и постройте ее график: f (x) = x4 −2х2 −3.

  2. Найти число корней уравнения: 2x 3 − 3x 2 − 12х − 11= 0.

  3. Исследуйте функцию и постройте ее график: а)б)

  4. Сколько корней имеет уравнение: а)б) в)

  5. Определите промежутки монотонности и экстремумы функции 

  6. Докажите, что функция f(x) = 4x — 3 sin x возрастает на всей числовой прямой.

  7. Исследуйте функцию f(x) = x4 + 4x2  5 и постройте ее график.

  8. Определите промежутки монотонности и экстремумы функции 

  9. Докажите, что функция f(x) = 5 cosx 7x убывает на всей числовой прямой.

  10. Исследуйте функцию f(x) = x4 + 8x2 − 9 и постройте ее график.

  11. Определите промежутки монотонности и экстремумы функции  .

  12. Докажите, что уравнение х5 + 2х3 + 8x + cos 3x = 0 имеет ровно один корень.

  13. Дана функция  
    а) постройте график функции 
    f(х);
    б) сколько корней имеет уравнение 
    f(х) = а?

  14. Дана функция 
    а) постройте график функции 
    f(x); б) сколько корней имеет уравнение f(х) = а?

  15. При каком наибольшем значении параметра а функция  убывает на всей числовой прямой?

  16. Докажите, что уравнение х5 + 4х3 + 7x sin2x = 0 имеет ровно один корень.

  17. При каком наибольшем значении параметра а функция  возрастает на всей числовой прямой?

  18. При каких значениях а функция f(х) = 8ac – a sin 6 7 sin 5х возрастает на всей числовой оси и не имеет стационарных точек?

  19. Проведите исследование и постройте график функции 

  20. Исследуйте функцию f(x) = х3 – 3x2 + 2 и постройте ее график.

  21. Исследуйте функцию f(х) = х3 3х + 2 и постройте ее график.

  22. При каких значениях параметра а функция f(x) = a sin 7x + 8 ax + sin 4 5x убывает на всей числовой оси и не имеет стационарных точек?


ПЗ № 41. Нахождение наибольшего, наименьшего значения и экстремальных значений функции.

Задание:

1)Перепишите и заполните пропуски:

Пример 1. Найти точку максимума функции y= x3 3x2 24x 5.

Решение: Требуется найти критическую точку, в которой знак производной меняется с плюса на минус. Область определения функции: hello_html_4fb18fd6.gif Найдем критические точки функции:



- Критические точки.

Исследуем знак производной на интервалах, разделенных критическими точками:

х

- 4 2

max min

Ответ: x = ...

Пример 2. Найдите точки экстремума функции и определите их характер y= x 4 8x2.

Решение: y = x 4 8x2 , D(y) = R , y = (x 4 8x2) = 4x 3 – 16x, y = 0,

4x 3 – 16x = 0, 4x(x2 4) = 0, 4x(x2) (x2) = 0,

x1= 0 или х2=0 или х2=0

х2 = … х3 =…

х1= 0, х2 = 2, х3 = 2 – это стационарные точки.


2 0 2 х

Функция убывает на ( ;2, на 0; 2. Функция возрастает на 2; 0, на 2; +).

х3 = …, х2 = … – это точки минимума. х1= … – это точка максимума.

Ответ: х3 = 2, х2 = 2– это точки минимума, х1= 0 – это точка максимума.

Пример 3. Найдите точки экстремума функции и определите их характер: y= x2 6x1.

Решение: y = x2 6x1, D(y) =R,

y = ( x3 x2 6x1) = x25x6 = (х3)(х2) , y = 0, x 2 5x6 = 0,

x1 = 3, x2 = 2 – это стационарные точки.

х

2 3

Функция возрастает на ( ; 2, на 3; +).Функция убывает на 2; 3.

x2 = … – это точка максимума, х1 = ... – это точка минимума.

Ответ: х2 = 2 – это точка максимума, х1 = 3 – это точка минимума.

Пример 4. Найдите точки экстремума функции и определите их характер.

y= 2x5 5x4 10x3 3.

Решение: y = 2x5 5x4 10x3 3, D(y) = R, y = (2x5 5x4 10x3 3) = 25x4 54x3 103x2 = =…x4 …x3 …x2 = 10х2 (х1)(х3), y = 0 , 10x4 20x3 30x2 = 0, 10x2 (x2 + 2x 3) = 0,

x 2 = 0 или х2 2х3=0, х1 = 0, х2 = 1, х3 = 3 – это стационарные точки.


3 0 1 х

Функция возрастает на ( ; 3, на 1; ). Функция убывает на 3; 1.

х3 = – это точка максимума. х2 = … – это точка минимума.

Ответ: х3 = 3 – это точка максимума, х2 = 1 – это точка минимума.

Пример 5. Найти наибольшее и наименьшее значения функции у(x) = 2x3 12x2 18x  3  на отрезке [– 1;2] .

Решение: 1) Вычислим значения функции в критических точках, принадлежащих данному отрезку:

 

Полученное квадратное уравнение имеет два действительных корня:

х1= 1, х2 = 3 – критические точки. Первая критическая точка принадлежит данному

отрезку: х1= 1 . А вот вторая – нет: х2= 1 , поэтому про неё сразу забываем.

Вычислим значение функции в нужной точке:

2)Вычислим значения функции  на концах отрезка:


3) Дело сделано, среди чисел выбираем наибольшее и наименьшее.

Ответ:  

Пример 6. Число 24 представьте в виде суммы двух неотрицательных слагаемых так, чтобы сумма квадратов этих чисел была наименьшей.

Решение: Пусть х – первое слагаемое, тогда (24-х) – второе слагаемое. Сумма квадратов этих чисел По условию задачи Рассмотрим функцию Она на интервале (0;24) непрерывна и дифференцируема. Найдем критические точки.

Это значение единственное, поэтому первое число – 12, второе – 12. Ответ: 24=12+12.
Пример 7. Найдите размеры участка прямоугольной формы, имеющего наибольшую площадь, если его периметр равен 200 м.

Решение: A B

x

D C Так как функция S(x) непрерывная на всей числовой прямой, b

то будем искать ее наибольшее значение на отрезке .

Значит, наибольшей будет площадь участка 2500 м2, а стороны участка равны 50 м и 50 м.

Ответ: 50 м и 50 м.

2)Решить задание ( по примерам):

  1. Найти точку максимума функции y = x3 6x2 15x 3.

  2. Найдите точки экстремума функции а) y = x 4 2x2 , б) y = x2 4x3 ,
    в) y = 2x
    5 10x4 40x3 5 и определите их характер.

  3. Найти наибольшее и наименьшее значения функции f(x) = x3 3x2 – 72x  90 на отрезке [– 4;5] .

  4. Число 12 представьте в виде суммы двух неотрицательных слагаемых так, чтобы сумма квадратов этих чисел была наименьшей.

  5. Найдите размеры участка прямоугольной формы, имеющего наибольшую площадь, если его периметр равен 120 м.

3)Решить задание : 1.Найдите точки экстремума функции

2. Найти наибольшее и наименьшее значения функции f(x) = x4 4x2  8 на отрезке [– 1;2] .

  1. Найти максимальное и минимальное значения функции f(x) = на отрезке [– 8;0] .

  2. Найти наибольшее и наименьшее значения функции f(x) = 2x312x2 – 30x 9 на отрезке [– 4;2] .

  3. Найти наибольшее и наименьшее значение функции  на отрезке [1;4];

  4. Найдите наибольшее значение функции   на отрезке .

  5. а) Найдите наибольшее и наименьшее значения функции

на отрезке .

б )Площадь прямоугольника равна 25 см2. Найдите наименьший возможный периметр этого прямоугольника.

ПЗ № 42. Интеграл и первообразная. Теорема Ньютона—Лейбница.

Задание:

1)Перепишите и заполните пропуски:

Пример 1. Найти общий вид первообразных данных функций :

1) f(x) = x2  cosx;   2) f(x) = 3; 3) f(x) = 10 sinx;  4) f(x) = 2sin 4x; 5) f(x) = 5x4x2  ; 6) f(x) = (3x – 1)2; 7) f(x) =. .

Решение: 1) F(x) = x3/3 – sinx  C; 2) F(x) =3x  C; 3) F(x) = 10  C; 

4) F(x) = 1/2 cos…x + C;5) F(x) =   x3/3 – 2;  6) F(x) = (3x – 1)3/… + C; 7) F(x) = /… + C.

Ответ: 1) F(x) = x3/3 – sinx  C; 2) F(x) =3x  C; 3) F(x) = 10cosx  C; 

4) F(x) = 1/2 cos 4x + C;5) F(x) = x5  x3/3 – 2;  6) F(x) = (3x – 1)3/9 + C; 7) F(x) =  /3 + C.

Пример 2. Для функции  f(x) = 4x + 1/x2 найти первообразную, график которой проходит через точку M(– 1; 4).

Решение: F(x) = 2x2– 1/x + C. , F(x) = 2()2– 1/() + C = 2 C = 4, 4 = 3 + C, C = …

Ответ: F(x) = 2х2 – 1/х + 1.

Пример 3. Докажите , что функция F(x) является первообразной для функции f(x).

a) f(x) = 2x; F(x) = x2 , б) f(x) = – sin x; F(x) = сos x ,
в) f(x) = 6x2 + 4; F(x) = 2x3 + 4x, г) f(x) = 1/cos2 x; F(x) = tg x .

Решение: a) f(x) = 2x; F(x) = x2 , F (x)= (x2) = … = f(x);

б) f(x) = – sin x; F(x) = сos x , F (x)= (cos x) = – … = f(x);

в) f(x) = 6x2 4; F(x) = 2x3 4x, F (x)= (2x3 4x) = …x2 4 = f(x);

г) f(x) = 1/cos2 x; F(x) = tg x , F (x)= (tg x) = 1/cos2 x= f(x).

Ответ: F(x) является первообразной для f(x). Пример 4. Найдите первообразные функций: a) f(x) = x4 x2 x ; б) g(u) = ;

в) h(x) = (x3 + 1)2 ; г) v(x) = cos (5x ). Решение: Для нахождения первообразных функций воспользуемся таблицей первообразных. а) x5/5 - одна из первообразных функции х4; x3/3 - одна из первообразных функции х2; x2/2 - одна из первообразных функции х; х - одна из первообразных функции 1. По правилу 1 нахождения первообразных F(x) =   - первообразная функции f(х);

б) функцию g(u) запишем в виде g(u) =  u - 1/3 u3/2.

3/2u2/3 - одна из первообразных функции u-1/3; 2/5u5/2 - одна из первообразных функции u3/2; 

G(x) = 3/2u2/3  2/5u5/2  - первообразная функции g(u);

в) h(x) = (x3 + 1)2 = x6 2x3 .

x7/7 - одна из первообразных функции х6; x4/4 - одна из первообразных функции х3; х - одна из первообразных функции 1.

По правилам 1 и 2 нахождения первообразных H(x) = …  1/2x4C - первообразная функции h(х);

г) v(x) = cos (5x ) , sinu - одна из первообразных функции cosu; V(x) = … sin(5x)  C - первообразная функции v(х).

Ответ: a) F(x) =   ; б) G(x) = 3/2u2/3  2/5u5/2  в) H(x) = 1/7x7  1/2x4xC; г) V(x) = 1/5 sin(5x)  C.

Пример 5. Найдите первообразную: a) g(x) = (47x)5 ; б) g(x)= x 2; в) t(x) = (5+ 2x)3.

Решение: a) G (x) = ; б) G(x) = ; в) T(x) = .


Ответ: a) G (x) = ; б) G(x) = ; в) T(x) = .

Пример 6. Вычислить интеграл: а) ,

б) ,

в)

г) д) е)

ж)

2)Решить задание ( по примерам):

  1. Найти общий вид первообразных данных функций :

1) f(x) = 4x3  2cosx;   2) f(x) = 5; 3) f(x) = 5 sinx;  4) f(x) = 3sin 4x;
5)
 f(x) = 6x5x4  ; 6) f(x) = (4x – 1)2; 7) f(x) =. .

  1. Для функции  f(x) = 6x + 1/x2 найти первообразную, график которой проходит через точку M(– 1; 8).

  2. Докажите , что функция F(x) является первообразной для функции f(x).

а)f(x) = 4x; F(x) = 2x2 , б) f(x) = – 2sin x; F(x) = 2сos x ,

в) f(x) = 6x2 7; F(x) = 2x3 7x, г) f(x) = 1/sin2 x; F(x) = ctg x .

  1. Найдите первообразные функций:
    a) f(x) = 10x4 4x2 2x ; б) g(u) = ; в) h(x) = (x3 3)2 ; г) v(x) = cos (7x ).

  2. Найдите первообразную: a) g(x)= (45x)5 ; б) g(x)= 2x- 2; в) t(x) = (54x)5.

3)Решить задание:

  1. Найдите первообразную функции f(x) = 4x3– 3x2 , график которой проходит через

точку M(–1; 2).

  1. Для функции f(х) = еx найти первообразную, график которой проходит через точку М(0; 2).

  2. Найти общий вид первообразной для функции f:

а)f(x) = 1,б) f(x) = х+1,в) f(x) = х9 + 3, г) f(x) =3х32 ,д) f(x) = 5х + ,

е) f(x) = 1+ , ж) f(x) = 4 + 2cosx, з) f(x) =sin2x + x, и) f(x) = sinx + cosx.

  1. При каких значениях k и С функции kcosx + x - 4 является первообразной функции 3sinх + 1? 

  2. При каком значении k и С функция   является первообразной функции  ?

  3. Найдите одну из первообразных функции f:

а) f(x) = 2х5 ,б) f(x) = 3х3+2х1, в) f(x) = 3 cosx 4 sinx ,г) f(x) = (х+1)4,

  1. Докажите, что функция F является первообразной для функции f на множестве R:

а) F(x) = 4хх3 , f(x) = 4 2, б) F(x) = 0,5 sinx, f(x) = cosx.

  1. Найдите первообразные функций: а) f(x) =,б)f(x) = 7 sinx, в) f(x) =г)f(x) =12cosx.

  2. При каком значении к и С функция является первообразной функции 5cosх + 2х?

  3. Вычислить интеграл: а) , б) ,в) , г) , д) , е) , ж)

ПЗ № 43. Применение интеграла к вычислению физических величин и площадей.

Задание:

1)Опорный конспект.

1. ВЫЧИСЛЕНИЕ ПУТИ, ПРОЙДЕННОГО ТОЧКОЙ

Путь, пройденный точкой при неравномерном движении по прямой с переменной скоростью hello_html_m14eaee5.gifза промежуток времени от hello_html_m52f1cc8d.gif до hello_html_m19189cb0.gif вычисляется по формуле hello_html_m12db380c.gif.

Примеры:

1. Скорость движения точки hello_html_dc36de3.gif м/с. Найти путь, пройденный точкой за 4-ю секунду.

Решение: согласно условию, hello_html_65824c05.gif. Следовательно, hello_html_4274c13b.gif

2. Два тела начали двигаться одновременно из одной точки в одном направлении по прямой. Первое тело движется со скоростью hello_html_m7ae6071a.gif м/с, второе — со скоростью v = (4t+5) м/с. На каком расстоянии друг от друга они окажутся через 5 с?

Решение: очевидно, что искомая величина есть разность расстояний, пройденных первым и вторым телом за 5 с:

hello_html_m41c22c75.gif

3. Тело брошено с поверхности земли вертикально вверх со скоростью и = (39,2—9,8^) м/с. Найти наибольшую высоту подъема тела.

Решение: тело достигнет наибольшей высоты подъема в такой момент времени t, когда v = 0, т.е. 39,29,8t = 0, откуда I 4 с. По формуле (1) на ходим

hello_html_129eae47.gif

2. ВЫЧИСЛЕНИЕ РАБОТЫ СИЛЫ

Работа, произведенная переменной силой f(х) при перемещении по оси Ох материальной точки от х = адо х=b, находится по формуле hello_html_95bb59.gifПри решении задач на вычисление работы силы часто используется закон Г у к а: F=kx, (3) где F — сила Н; х—абсолютное удлинение пружины, м, вызванное силой F, а k —коэффициент пропорциональности, Н/м.

Пример:

1. Пружина в спокойном состоянии имеет длину 0,2 м. Сила в 50 Н растягивает пружину на 0,01 м. Какую работу надо совершить, чтобы растянуть ее от 0,22 до 0,32 м?

Решение: используя равенство (3), имеем 50=0,01k, т. е. kК = 5000 Н/м. Находим пределы интегрирования: а = 0,22 — 0,2 = 0,02 (м), b=0,32— 0,2 = 0,12(м). Теперь по формуле (2) получим

hello_html_m396fccd3.gif

3. ВЫЧИСЛЕНИЕ РАБОТЫ, ПРОИЗВОДИМОЙ ПРИ ПОДНЯТИИ ГРУЗА

Задача. Цилиндрическая цистерна с радиусом основания 0,5 м и высотой 2 м заполнена водой. Вычислить работу, которую необходимо произвести, чтобы выкачать воду из цистерны.

Решение: выделим на глубине х горизонтальный слой высотой dх (рис.). Работа А, которую надо произвести, чтобы поднять слой воды весом Р на высоту х, равна Рх.

Изменение глубины х на малую величину dх вызовет изменение объема V на величину dV пr2 dх и изменение веса Р на величину * dР = 9807 r2 dх; при этом совершаемая работа А изменится на величину dА=9807пr2 хdх. Проинтегрировав это равенство при изменении x от 0 до Н, получим

hello_html_78e3df37.gif

4. ВЫЧИСЛЕНИЕ СИЛЫ ДАВЛЕНИЯ ЖИДКОСТИ

Значение силы Р давления жидкости на горизонтальную площадку зависит от глубины погружения х этой площадки, т. е. от расстояния площадки до поверхности жидкости.

Сила давления (Н) на горизонтальную площадку вычисляется по формуле Р =9807 hello_html_m358815e3.gifS x,

где hello_html_m358815e3.gif — плотность жидкости, кг/м3; S — площадь площадки, м2х - глубина погружения

площадки, м.

Если площадка, испытывающая давление жидкости, не горизонтальна, то давление на нее различно на разных глубинах, следовательно, сила давления на площадку есть функция глубины ее погружения Р (х).


Физические приложения интеграла

1. Реши задачи.

Вычислите массу участка стержня от hello_html_m2acd972a.gif, если его линейная плотность задается формулой hello_html_m51532c9b.gif
  1. Вычислите работу за промежуток времени [4;9 ], если мощность вычисляется по формуле hello_html_630bc344.gif.

  1. Вычислите количество электричества, протекшего по проводнику за промежуток времени [ 2;3 ], если сила тока задается формулой hello_html_m4b9cde72.gif

  1. Вычислите работу по переносу единичной массы, совершенную силойhello_html_2919d23.gif [ -1;2].


2)Перепишите и заполните пропуски:


Пример 1. а)Вычислить площадь фигуры, ограниченной линиями у = х2 2, у = 0, х = 2, х = 1.

Решение: Выполним чертеж (обратите внимание, что уравнение у = 0  задает ось  ОХ): Штриховать криволинейную трапецию я не буду, здесь очевидно, о какой площади идет речь. Решение продолжается так:hello_html_m6ea081fc.png

На отрезке[– 2;1]    график функции у = х2 2  расположен над осью ОХ, поэтому:


Ответ: S = 9 eд2.


б)Вычислить площадь фигуры, ограниченной линиями у =  , х = 1  и координатными осями.

Решение: Выполним чертеж: Если криволинейная трапеция расположена под осью OX (или, по крайней мере, не выше данной оси), то её площадь можно найти по формуле: S =  .
В данном случае:
hello_html_m521974c9.png

Ответ: 

Пример 2.а)Найти площадь плоской фигуры, ограниченной линиями  у = 2х , у =  .hello_html_m34e0aa0e.png

Решение: Сначала нужно выполнить чертеж. Найдем точки пересечения параболы у = 2х   и прямой у =   .

Решаем уравнение: =  , 3х = 0, х(3) = 0,

х1 = …, х2 = ...

Значит, нижний предел интегрирования а = 0, верхний предел интегрирования b = 3 .

x = a ,x = b , можно найти по формуле: S = .

Искомая фигура ограничена параболой y = 2х   сверху и прямой у =    снизу.
На отрезке
[0;3]  2х  , по соответствующей формуле

Ответ: S = 4,5 eд2.  . б)Вычислить площадь фигуры, ограниченной линиями , y = x  , y = 0  , x = 3 .

Решение: Сначала выполним чертеж: Площадь фигуры считается с помощью двух определенных интегралов. Действительно:hello_html_m2509795.jpg

1) На отрезке [– 1;1]  над осью OX расположен график прямой y = x   ;

2) На отрезке [1;3]   над осью OX  расположен график гиперболы

Совершенно очевидно, что площади можно (и нужно) приплюсовать, поэтому:

Ответ: .hello_html_dfee4aa.png

Пример 3.a) Вычислить площадь фигуры, ограниченной линиями

 ,2x  .

Решение: Представим уравнения в виде и выполним поточечный чертеж:
Из чертежа видно, что верхний предел у нас «хороший»:
  b = 1.
Найдем точки пересечения прямой
    и параболы
Для этого решаем уравнение:
3x2 = 2x 3x2 2x

D = 4 12 = …, = 4, x1 = , x2 = ... Действительно,a = .

На отрезке по соответствующей формуле: Ответ: .hello_html_3f5273e5.png

б)Найти площадь фигуры, ограниченной линиями y =  , y = 2x  .

Решение: Выполним чертеж:
На отрезке по соответствующей формуле:


Ответ: S = 10 eд2.  .


Пример 4.Найти площадь фигуры, ограниченной линиями x , xy = 3 .hello_html_m2e7c3126.png

Решение: Выполним чертеж . На отрезке  , по соответствующей формуле:
Ответ:  .
hello_html_78b890fb.jpg


Пример 5.a) Найти площадь фигуры, ограниченной параболой у = х2 +10 и касательными к этой параболе, проведёнными из точки (0;1). 

Решение: Неизвестна абсцисса точки касания х = а. Чтобы её найти, составим уравнение касательной:  y = f (x0) .

Имеем f(x) = x2 f (x) = 2x;значит, f(a) = a2 f (a) = 2a; уравнение касательной имеет вид:

y = a2 2 a(x ) = a2 2 ax ;

Уравнение касательной y = (1)

По условию касательная должна проходить через точку (0;1), то есть координаты точки (0;1) должны удовлетворять уравнению (1):

1 = 2a0 ; , a1 = a2 = ...

Подставим найденные значения в уравнение (1):


Если a =  то y = 9 10 Если a = 3 , то y =  .

Получили два уравнения касательных y =  . Параболы y = х2 + 10 они касаются в точках А(3;19) и В(3;19).

Найдём площадь фигуры DACB: SDACB = 2SDCB ,


hello_html_m157678ba.gif






SDACB = 2 9 = ...

Ответ: 18.

б) Вычислить площадь фигуры, ограниченной линиями hello_html_35881468.jpg

у = 4/x, y = х, х = 4.

Решение: SABC = SMBAD SMBCD;

SMBAD = 1/2(MB ) MD = = 1/2 (2 ) 2 = 6;




hello_html_450e27c6.gif


Ответ: 6 – 4ln2.



2)Решить задание ( по примерам):

  1. а)Вычислить площадь фигуры, ограниченной линиями .

б)Вычислить площадь фигуры, ограниченной линиями и координатными осями.

  1. а)Найти площадь плоской фигуры, ограниченной линиями  .

б)Вычислить площадь фигуры, ограниченной линиями.

  1. a) Вычислить площадь фигуры, ограниченной линиями .

б)Найти площадь фигуры, ограниченной линиями y =  , y = 2x  .

  1. a)Найти площадь фигуры, ограниченной линиями  .

б) В каком отношении парабола делит площадь четырёхугольника, вершины которого находятся в точках с координатами (0;0); (2;0); (0;6); (2;6)? 

  1. a)Найти площадь фигуры, ограниченной параболой и касательными к этой параболе, проведёнными из точки (0;1). 

б) Вычислить площадь фигуры, ограниченной линиями .

3)Решить задание:

  1. a)Вычислить площадь фигуры, ограниченной линиями .

б)Вычислить площадь фигуры, ограниченной линиями и координатными осями.

  1. а)Найти площадь плоской фигуры, ограниченной линиями  .

б)Вычислить площадь фигуры, ограниченной линиями.

  1. a)Вычислить площадь фигуры, ограниченной линиями.

б)Найти площадь фигуры, ограниченной линиями .

  1. a)Найти площадь фигуры, ограниченной линиями.

б) В каком отношении парабола делит площадь четырёхугольника, вершины которого находятся в точках с координатами (0;0); (2;0); (0;6); (2;6)? 

  1. a)Найти площадь фигуры, ограниченной параболой и касательными к этой параболе, проведёнными из точки (0;1). 

б) Вычислить площадь фигуры, ограниченной линиями .

  1. Найти площадь фигуры, ограниченной функцией и осями координат.

  2. Найти площадь фигуры, ограниченной функциями и касательной к этой параболе, проведенной в точке (1/2;3/4).

  3. Найти площадь фигуры, ограниченной линиями.

  4. Найти площадь фигуры, ограниченной линиями.

  5. Найти площадь фигуры, ограниченной линиями.

  6. Найти площадь фигуры, ограниченной линиями.

  7. Найти площадь фигуры, ограниченной линиями..

  8. Найти площадь фигуры, ограниченной линиями .

  9. Найти площадь фигуры, ограниченной линиями.

  10. Найти площадь фигуры, ограниченную линиями .

  11. Найти площадь фигуры, ограниченной линиям .

  12. Вычислите площадь фигуры, ограниченной параболой и осью абсцисс.

  13. Вычислите площадь фигуры, ограниченной прямыми , .

  14. Вычислите площадь фигуры, ограниченной прямой и параболой .

  15. Вычислите площадь фигуры, ограниченной линиями и .





























Просмотрено: 0%
Просмотрено: 0%
Скачать материал
Скачать материал "ПЗ по специальности Технология машиностроения,1 курс(2016-2017 уч.г.)"

Получите профессию

Бухгалтер

за 6 месяцев

Пройти курс

Рабочие листы
к вашим урокам

Скачать

Получите профессию

Интернет-маркетолог

за 6 месяцев

Пройти курс

Рабочие листы
к вашим урокам

Скачать

Скачать материал

Найдите материал к любому уроку, указав свой предмет (категорию), класс, учебник и тему:

6 660 880 материалов в базе

Скачать материал

Другие материалы

Вам будут интересны эти курсы:

Оставьте свой комментарий

Авторизуйтесь, чтобы задавать вопросы.

  • Скачать материал
    • 03.04.2017 2766
    • RAR 26.5 мбайт
    • Оцените материал:
  • Настоящий материал опубликован пользователем Зайцева Светлана Егоровна. Инфоурок является информационным посредником и предоставляет пользователям возможность размещать на сайте методические материалы. Всю ответственность за опубликованные материалы, содержащиеся в них сведения, а также за соблюдение авторских прав несут пользователи, загрузившие материал на сайт

    Если Вы считаете, что материал нарушает авторские права либо по каким-то другим причинам должен быть удален с сайта, Вы можете оставить жалобу на материал.

    Удалить материал
  • Автор материала

    Зайцева Светлана Егоровна
    Зайцева Светлана Егоровна
    • На сайте: 8 лет и 2 месяца
    • Подписчики: 0
    • Всего просмотров: 343920
    • Всего материалов: 170

Ваша скидка на курсы

40%
Скидка для нового слушателя. Войдите на сайт, чтобы применить скидку к любому курсу
Курсы со скидкой

Курс профессиональной переподготовки

Технолог-калькулятор общественного питания

Технолог-калькулятор общественного питания

500/1000 ч.

Подать заявку О курсе

Курс повышения квалификации

Аспекты преподавания самостоятельного учебного курса «Вероятность и статистика» в условиях реализации ФГОС ООО

36 ч. — 180 ч.

от 1700 руб. от 850 руб.
Подать заявку О курсе
  • Сейчас обучается 281 человек из 65 регионов
  • Этот курс уже прошли 985 человек

Курс повышения квалификации

Педагогическое проектирование как средство оптимизации труда учителя математики в условиях ФГОС второго поколения

36/72 ч.

от 1700 руб. от 850 руб.
Подать заявку О курсе
  • Сейчас обучается 85 человек из 35 регионов
  • Этот курс уже прошли 1 415 человек

Курс повышения квалификации

Формирование умений и навыков самостоятельной работы у обучающихся 5-9 классов на уроках математики в соответствии с требованиями ФГОС

36 ч. — 144 ч.

от 1700 руб. от 850 руб.
Подать заявку О курсе
  • Сейчас обучается 96 человек из 39 регионов
  • Этот курс уже прошли 452 человека

Мини-курс

Эффективная самопрезентация

4 ч.

780 руб. 390 руб.
Подать заявку О курсе
  • Сейчас обучается 55 человек из 31 региона
  • Этот курс уже прошли 32 человека

Мини-курс

Стратегия продаж и продуктовая линейка: успех в современном бизнесе

2 ч.

780 руб. 390 руб.
Подать заявку О курсе

Мини-курс

Интерактивные материалы на печатной основе

4 ч.

780 руб. 390 руб.
Подать заявку О курсе
  • Сейчас обучается 57 человек из 25 регионов
  • Этот курс уже прошли 30 человек
Сейчас в эфире

Арт-педагогика как метод профилактики детских неврозов

Перейти к трансляции